You are on page 1of 368

R.GLADKOVAand N.

KUT YLOVSKAYA

Selected
Questions and Problems
in Physics

MIR PUBLISHERS MOSCOW


Star (
-hart
SELECTED
QUESTIONS AND PROBLEMS
IN PHYSICS
P. A . FjraflKOBa, H . I I . KyTUJiOBCKaH

CSopHHK aaAan n BonpocoB no <|)H3HKe

«Bucuiafl uiKOJia» Mockbs


R.GLADKOVAand N. KUT YLOVSKAYA

Selected
Questions and Problems
in Physics

Mir Publishers
Moscow
Translated from Russian by
Natalia Wadhwa

First published 1989


Revised from the 1986 Russian edition

Ha aHZAiiucKOM nsune

Printed in the Union of Soviet Socialist Republics

ISBN 5-03-000908-6 (g) ManaTejitcTBo «Buciua«i WKOJia»,


1986
(g) English translation, Mir Publishers,
1989
Contents

Preface 7
General Methodical Instructions 8

Chapter I. Fundamentals of MolecularPhysics and Thermodynamics


§ 1. Molecular Kinetic Theory. Motion of Molecules, Their
Size and Mass 10
§ 2. Velocities of Molecules. Basic Equation in the Kinetic
Theory of Gases 19
§ 3. Equation of State for an Ideal Gas. Isothermal, Isocho-
ric, and Isobaric Processes 24
§ 4. The Change in the Internal Energy During Heat Trans­
fer and Due to Mechanical Work 37
§ 5. Properties of Vapours 50
§ 6. Properties of Liquids 62
§ 7. Properties of Solids. Deformations 68
§ 8. Thermal Expansion of Bodies 78

Chapter II. Fundamentals of Electrodynamics


§ 9. Electric Field 87
§ 10. Electric Current in Metals. Ohm’s Law. Electric Re­
sistance 106
§ 11. Work, Power, and the Thermal Effect of Current 132
§ 12. Electric Current in Electrolytes 141
§ 13. Electric Current in Gases and inVacuum 148
§ 14. Electric Current in Semiconductors 153
§ 15. Electromagnetism 155
§ 16. Electromagnetic Induction 169

Chapter III. Oscillations .and Waves


§ 17. Mechanical Vibrations and Waves. Sound and Ultrasound 178
§ 18. Alternating Current 195
§ 19. Electromagnetic Oscillations and Waves 211

Chapter IV. Optics. Fundamentals of the Special Theory of Relativity


§ 20. Geometrical Optics 217
§ 21. Photometry 243
6 Contents

§ 22. Phenomena Explained by the Wave Properties of Radia­


tion. Interference. Diffraction 251
§ 23. Radiation and Spectra 260
§ 24. Phenomena Explained by the Quantum Properties of
Radiation. Photoelectric Effect 263
§ 25. Fundamentals of the Special Theory of Relativity 269

Chapter V. Physics of Atomic Nucleus


§ 26. Structure of Atomic Nucleus. Atomic Energy and Its
Application 276

Chapter VI. General Remarks on Astronomy


§ 27. Fundamentals of Astronomy 288
Appendices 297
Answers 330
Preface

This collection of questions and problems in physics is in­


tended for the students of correspondence courses and evening-
classes in intermediate colleges and is in accord with the
existing curriculum.
The purpose of this book is to teach the students how to
solve problems in physics. This should stimulate corre­
spondence course students to work independently, encourage
them to accumulate an adequate theoretical background,
and develop in them the requisite aptitude for practical
activity in various branches of the economy.
Each section begins with a brief description of the basic
theoretical concepts, laws, and formulas. This provides the
maximum possible help to correspondence course students in
solving problems. A large number of problems are supplied
with detailed solutions and an analysis of the results, while
in some cases different approaches are used to solve the same
problem so that the student can discover the most rational
form of independent study.
The theoretical material is presented in a lucid form, and
most problems are of medium complexity. However, each
section contains tougher problems as well. Their solution
requires a broader range of theoretical data, and will fa­
cilitate a deeper understanding of the physics course.
In keeping with the existing curriculum, problems in
astronomy have also been included in the collection. Their
solution requires the use of a star chart, which is printed
on the fly-leaf.
The authors are grateful to A. L. Kosorukov, a researcher
at the Institute of Applied Mathematics of the USSR Acad­
emy of Sciences, for the help in compiling the problems.

R. Gladkova
N. Kutylovskaya
General Methodical Instructions

Before starting a new chapter in this course, go through


the contents of the textbook for this chapter, and write down
the topics in a separate notebook. Having carefully read
the list of topics, write down the numbers of the relevant
sections in the margin.
While studying the material in a section, it is necessary
to read through the entire section without stopping at the
difficult parts. During the second reading, think about the
meaning of each sentence, and write down the derivation of
the formulas, the definitions of the physical quantities and
their units, as well as the formulation of the relevant laws of
physics. Use the textbook whenever necessary. Your study
of the section should be completed by revising the material,
repeating the examples and explanations, and making notes
of the derivations and diagrams. If you can reproduce the
entire material without looking at either the textbook or
your notes, it is safe to assume that you have grasped the
subject.
If difficulties are encountered while reading the theoreti­
cal material, and if it is not possible to overcome these d if­
ficulties independently, you must ask your teacher and get
oral or written consultation before revising the main and
supplementary material.
It is not possible to get a firm grip over theoretical mate­
rial without solving problems. Moreover, the solution of
problems helps when memorizing laws and formulas. Hence
attention must be paid to the solution of problems.
Before solving a problem, read the conditions and write
them in your notebook without abbreviations after under­
standing the essence of the problem. Using the standard
General Methodical Instructions 9

notation for physical quantities, write down the given quan­


tities in SI units in the notebook. After this, write down
the quantities to be determined. Use the tables in the Appen­
dices for the extra quantities required to solve the problem.
Having understood the physical phenomenon and using the
physical laws applicable to a current problem, write down
the necessary formulas to express the required quantity.
In other words, solve the problem in the general form ana­
lytically. Numerical values should be substituted in the
final expression together with the units of the quantities.
The units of quantities should be simplified first, followed
by the numerical values.
Obey the rules for operations with numbers. Whenever
possible, use mathematical tables. All calculations should
be made with a pocket calculator or a slide rule. The cor­
rectness of the solution can be verified by comparing the
result with the actual values of the quantities. Explanatory
notes and figures should accompany each solution. The an­
swers must be based on the laws studied before solving the
problem, and should reveal the essence of the phenomenon
involved. Solutions of typical problems and detailed expla­
nations are included to help students tackle the problems.
While studying the course on physics, students must pass
tests to determine the level of their understanding of a top­
ic, and also do practical work to strengthen their theoreti­
cal background as well as to acquire a practical knowledge
of the way equipment is handled.
At the end of the course, correspondence course students
have to pass examinations, where they must demonstrate a
sound knowledge of theory, the formulation of laws, formu­
las, and units of physical quantities. They must also learn
how to identify problems in everyday practice on the basis
of their knowledge of physical laws, and also how to solve
physical problems.
Chapter I

Fundamentals of Molecular Physics


and Thermodynamics

§ 1. MOLECULAR KINETIC THEORY. MOTION OF MOLECULES,


THEIR SIZE AND MASS

Basic Concepts and Formulas


The molecular kinetic theory explains the structure and prop­
erties of bodies through the motion and interaction of the
small particles constituting them. The theory is based on
three postulates:
1. All substances consist of very small particles, viz.
molecules.1 Molecules, in turn, consist of still smaller par­
ticles, viz. atoms, which themselves are made up of pro­
tons, electrons, and neutrons.
2. Molecules are in a state of perpetual random motion
known as thermal motion. As a result, every molecule pos­
sesses a kinetic energy.
3. Molecules of a substance interact with one another.
The forces of interaction (attractive and repulsive) are
electrical in origin. They depend both on the nature of the
molecules and on their separation. Consequently, molecules
possess a potential energy.
The internal energy of a body is defined in molecular phys­
ics as the sum of the kinetic energies of all the molecules
and the potential energies of their interaction.
The number of molecules in any body is extremely large,
while their size and mass are very small.
The SI unit of the amount of substance, viz. the mole, is
a base unit. The mole is the amount of substance containing
the same number of structural elements as the number of
12C atoms in a mass of 0.012 kg.
A mole of any substance contains the same number of
molecules. This number is known as the Avogadro constant

1 A molecule is the smallest particle of a substance that retains


its chemical properties.
§ 1. Molecular Kinetic Theory 11

Na. The mass of a molecule is


m0 = M/N a,
where M is the molar mass.
The number of molecules in 1 m3 of a substance is deter­
mined from the formula
n = N Ap/M,
where p is the density of the substance.
The number of molecules in a given mass of a substance is
N = mNJM,
where m is the mass of the substance and mIM = v is the
number of moles.
A gas whose molecules do not attract one another is
called an ideal gas.
Under normal conditions (T0 = 273 K and p 0 =
101325 Pa), the molar volumes of all ideal gases are Ihe
same: Vm = 22.4 x lO -3 m3/mol.
The Loschmidt number N h is the number of gas molecules
in 1 m3 of a substance under normal conditions:
JVl = N J V m,
where Pm is the molar volume defined as
Vm = Ml Po.
The mean distance I between molecules is defined as
j= yvjN ,
where V/N is the elementary volume per molecule.
The mean free path \ is the mean distance covered by a
molecule between two successive collisions:
X = viz,
where v is the arithmetic mean velocity of the molecules
and z is the average number of collisions per second:
z = Y 2 ndettvn0.
Here d eit is the effective diameter of a molecule and n0 is
the number of molecules in 1 m3 of a substance.
12 Ch. I. Fundamentals of Molecular Physics

Worked Problems
Problem 1. Determine the amount of substance (in moles)
contained in (a) 1 kg of mercury, and (b) 5.6 dm3 of oxygen
under normal conditions.
Given: m, = 1 kg is the mass of mercury, V0 = 5.6 dm3 =
5.6 X 10~3m3 is the volume of oxygen under normal condi­
tions. From tables, we find the molar mass of mercury,
M, = 200.6 X 10~3 kg/mol, the molar mass of oxygen,
M 2 = 32 X 10~3 kg/mol, and the density of oxygen under
normal conditions, pn = 1.43 kg/m3.
Find: the amount of substance v, in 1 kg of mercury and
the amount of substance, v2, in 5.6 dm3 of oxygen under
normal conditions.
Solution. The amount of substance in 1 kg of mercury
can be determined as follows:
v, = mJNI,, 1 kg = 4.98 mol.
Vl 200.6 x 10"3 kg/mol
In order to determine the amount of substance in 5.6 dm3
of oxygen, we calculate the mass m2 of oxygen: m2 = p0F0.
This gives
PqF0 1.43 kg/m3 X 5.6 X 10~3 m3 = 0.25 mol.
v2 Mt ’ V 2 32 x 10~3 kg/mol
Answer. One kg of the mercury contains about 5 mol, and
5.6 dm3 of the oxygen contain about 0.25 mol.
Problem 2. Calculate the number of molecules contained
in 0.5 kg of oxygen and in 5.0 cm3 of carbon dioxide under
normal conditions.
Given: mt = 0.5 kg is the mass of oxygen, V0 = 5.0 cm3 =
5.0 X 10~6 m3 is the volume of carbon dioxide under normal
conditions. From tables, we find the molar mass of oxygen,
M l = 32 x 10' 3 kg/mol, and the molar mass of carbon
dioxide, M t = 44 X 10' 3 kg/mol (it is equal to the sum of
the molar masses of carbon and oxygen), the Avogadro con­
stant N x = 6.022 X 1023 mol"1, the molar volume Fm =
22.4 x 10"3 m3/mol, and the Loschmidt number N L =
2.68 X 1025 m"3.
Find: the number N 1 of molecules in 0.5 kg of oxygen and
the number N 2 of molecules in 5.0 cm3 of carbon dioxide
under normal conditions.
§ 1. Mblecular Kinetic Theory 13

Solution. The amount of substance in moles is v = m J M l .


Knowing that one mole contains N \ molecules, we deter­
mine the number of molecules in 0.5 kg of oxygen:
/V, = v/VA --= miN J M
0 5 kg
yvt =, 6.022 .< 1023 mol"' ~ 9.4 x 102*.
32 X 10~3 kg/mol
In order to find the number of carbon dioxide molecules,
N 2, contained in 5.0 cm3 under normal conditions, we use
the relation
N 2 = A AF0/k m,
where N \ / V m is the number of molecules in 1 m3. This
gives
6.022 x IQ23 m ol-1*
yVo 22.4 XlO-3 m3-mol-1 5 X 10-« n-3 - 1.34 X 102°.
Remark. The second part of the problem can be solved
quite easily if we consider that 1 m3 of a gas contains /VL
molecules under normal conditions. Then N 2 = jVLlro =
2.68 X lO25 m -3 X 5.0 x 10~6 m3 = 1.34 x 1020.
Answer. The number of molecules in 0.5 kg of oxygen is
approximately 9.4 X 1024, while 5.0 cm3 of carbon dioxide
contain 1.34 X 1020 molecules.
Problem 3. The molar mass of oxygen is 32 X 10~3 kg/mol.
Determine the molar mass of air if the densities of oxygen and
air under normal conditions are 1.43 and 1.29 kg/m3 re­
spectively.
Given: the molar mass of oxygen, = 32 X 10~3 kg/mol,
the density of oxygen, p01 = 1.43 kg/m3, and the density of
air, p02 = 1.29 kg/m3.
Find: the molar mass AI2 of air.
Solution. The molar mass M can be expressed in terms of
the mass m0 of a molecule and the Avogadro constant N A:
M = m0N A.
The density p0 of a gas under normal conditions is deter­
mined from the relation p0 = m0A L, where /VL is the
Loschmidt number.
Let us divide termwise the expressions for molar mass and
density: M/p0 = N J N L. But the ratio yVA/yVL of the two
constants is a constant. Consequently, the molar masses of
14 Ch. I. FuncTfumentals of Molecular Physics

two gases are directly proportional to their densities under


normal conditions:
Mi/poi — -^2^Po2 = • • • = N a/N l .
This gives
•^72 = Po2-^l/Pol’
M z = i 43 x 32 x 10-3 kg/mol ~ 29 x 10-3 kg/mol.
Answer. The molar mass of air is approximately 29 X
10-3 kg/mol.
Problem 4. Determine the mass of an acetylene molecule
C2H2 and the density of acetylene under normal conditions.
Given: the chemical formula of acetylene is C2H2. From
tables, we find that the molar masses of carbon and hydrogen
are Me, = 24 X 10-3 kg/mol and M H, = 2 X 10-3 kg/mol
respectively. The Loschmidt number is N h = 2.68 X
1025 m -3 and the Avogadro constant is N A = 6.022 X
1023 mol-1.
Find: the mass m0 of an acetylene molecule and the density
p0 of acetylene under normal conditions.
Solution. The mass of an acetylene molecule can be deter­
mined from the relation
m0 - M/Na.
The acetylene molecule C2H2 consists of carbon and hydro­
gen. Therefore, the molar mass of acetylene can be deter­
mined from the molar masses of its components: M = M c , +
Mh, = 26 X 10-3 kg/mol.
Remark. The molar mass of any chemical compound, for
example, ammonia NH3, can be determined in this way:
M = AfN + 3A / h = 17 x 10-3 kg/mol.
For an acetylene molecule, we have
26 X 10-3 kg/mol
m0 6.022 x 1023 mol-1 = 4.32 X 10-29 kg-
Knowing the mass m0 of a molecule and the number of
molecules in 1 m3 (equal to /VL), we can determine the den­
sity p0 of acetylene under normal conditions:
p0 = m0N h, p0 = 4.32 x 10-26 kg x 2.68 x 102B m -3
~ 1.16 kg/m3.
§ 1. Molecular Kinetic Theory______________ 15

Answer. The mass of an acetylene molecule is approximate­


ly 4.3 x 10"29 kg. The density of acetylene under normal
conditions is 1.16 kg/m3.
Problem 5. Calculate the approximate size of a water
molecule assuming that molecules are
spherical and touch each other.
Given: the chemical formula of water H H D E
is H20 . From tables, we find the den­
sity of water, p = 1 x 103 kg/m3, at
room temperature and the molar mass
of water, M = 18 x 10-3 kg/mol (see xX XJ ?\
Problem 4). The Avogadro constant >Y
is N k = 6.022 x 1023 m ol-1. s
Find: the volume V and the diame­
Fig. 1
ter d of a water molecule.
Solution. Knowing the density and the molar mass of
water, we can determine its molar volume Vm:

Vm = Mlp, Vm = -1-81Xx \ V 4 g/m301 = 1-8 X 10~5 m3/11101-


One mole of any substance is known to contain N A mole­
cules. Consequently, the volume of a water molecule can be
determined by dividing the molar volume by the Avogadro
constant:
1.8 x 10~5 m3/mol
F= V ~ 0.3 x 10~28 m3.
6.022 x 1023 mol-1
It was stated in the problem that water molecules are
spherical and closely packed (Fig. 1), i.e. the gaps between
the molecules are negligibly small. Hence we can assume
that the elementary volume V taken from the molar volume
Fm will contain one molecule whose diameter is approxi­
mately equal to the edge of the cube:
d = yv,
d = y 0.3 x 10"28 m3 ~ 3 X 10"10 m.
In the general form, the solution is
d = 3/ V = y V J N a --= / M/pNA.
Answer. The volume of a water molecule is approximately
3 X 10-29 m3, and the diameter of a molecule is about
3 x 10-10 m.
16 Ch. I. Fundamentals of Molecular Physics

Problem 6 . Calculate the mean distance between the cen­


tres of molecules of an ideal gas under normal conditions.
Solution. The molar volume under normal conditions is
Fm = 22.4 X 10' 3 m3/mol. One mole of any substance con­
tains N a = 6.022 X 1023 molecules. If the molecules are
assumed to be distributed uniformly over the entire vol­
ume, the volume per molecule is V = Vm/NA.
This volume can be regarded as a cube whose edge is
equal to the mean distance d between the molecules (see
Fig. 1):

Answer. The mean distance between gas molecules under


normal conditions is approximately 3.3 X 10-9 m.
Problem 7. The arithmetic mean velocity of nitrogen mol­
ecules under normal conditions is 453 m/s. Find the mean
free path, the mean free time, and the mean momentum
of a molecule if it undergoes 7.55 X 109 collisions per
second.
Given: the arithmetic mean velocity v = 453 m/s of a
nitrogen molecule under normal conditions, the average
number of collisions of the molecule per second, z = 7.55 X
109 s -1. From tables, we find the molar mass of nitrogen,
M = 28 X 10~3 kg/mol, and the Avogadro constant, N \ =
6.022 X 1023 m ol-1.
Find: the mean free path k, the mean free time t, and
the mean momentum p of a molecule.
Solution. As a result of the random motion, the values of
k, t, and p for molecules do not remain constant. Therefore,
we must determine the mean values of these quantities:

v y 453 m/s
= 6.0 x 10 9 m.

k — _ 6.0 x 10"8 ni
= 1.3 X io-*« S
7 ’ 1 ~ 453 m/s

In order to determine the momentum of a nitrogen mole­


cule, we must find its mass: m0 = MINA.
§ 1. Molecular Kinelic Theory 17

The mean momentum p of the molecule can be calcu­


lated thus
n mav = (M/NA) v,
28 X IQ '3 kg/mol ,r o
/' m/s ~ 2.11 x 10 23 kg-m/s.
6.022 x 10“ m ol'1
Answer. The mean free path and the mean free time are
ll.0 X 10-8 m and 1.3 X 10-10 s respectively. The mean
momentum of a nitrogen molecule is 2.11 X 10“23 kg-m/s.

Questions and Problems


1.1. What experimental facts clearly confirm the random
nature of molecule motion and the relationship between the
intensity of this motion and the temperature?
1.2. Why is diffusion in liquids much slower than it is in
gases?
1.3. What physical process occurs when the surface of a
solid is painted?
1.4. Common salt placed in water is uniformly distribut­
ed over the entire volume a certain time after being added.
Fxplain this phenomenon.
1.5. Why do gauge blocks
(Johansson blocks) stick together
when their end faces are brought
in contact (Fig. 2)?
1.6. What phenomenon is re- Fig. 2
sponsible for gluing solids?
1.7. One of the largest nuggets of gold, with a mass of
62.3 kg, was found at the mouth of the Amazon river. How
much substance is contained in it?
1.8. Determine the mass of 1 kmol of carbon, nitrogen,
and helium.
1.9. What is the mass of 50 mol of oxygen?
1.10. How many molecules are contained in 32 kg of
oxygen and in 2 g of hydrogen?
1.11. What is the volume occupied by 7 x 1028 molecules
of carbon dioxide under normal conditions?
1.12. Determine the amount of substance contained in 6 g
of carbon dioxide. How many molecules constitute this mass?
1.13. What is the volume occupied by 0.6 X 1023 atoms
of graphite? The density of graphite is known.
2-0530
18 Ch. I. Fundamentals of Molecular Physics

1.14. The density of brass is 8500 kg/m3. What does this


mean?
1.15. The density of aluminium is 2.7 X 103 kg/m3. How
much substance is contained in 1 m3 of aluminium?
1.16. Determine the mass of a molecule and of an atom
of oxygen, nitrogen, and helium.
1.17. Calculate the molar mass of methane and the mass
of a methane molecule CH4.
1.18. A l-|xm layer of silver is deposited on the surface of
a metal mirror. How many silver atoms would be in a sur­
face layer with area 25 cm2?
1.19. A grain of common salt having a mass of 3 X 10“3 g
is dissolved in 10 1 of water and uniformly distributed over
the entire volume. How many molecules of salt are contained
in 5 cm3 of the solution?
1.20. What is the ratio between the masses of a 10~8-g
dust particle and an air molecule? The molar mass M of
air is 29 X 10-3 kg/mol. i
1.21. Comparing the densities of air and hydrogen, find
the ratio between the masses of their molecules.
1.22. Determine the mass of a propane molecule CSH„
and the density of propane under normal conditions.
1.23. Calculate the mass of a butane molecule C4H,0 if
its density under normal conditions is 2.67 kg/m3.
1.24. Given the density of hydrogen under normal condi­
tions, determine its molar mass.
1.25. A drop of mineral oil with a mass of 0.023 mg is
poured on the surface of water and forms a film 60 cm2 in
area. Assuming that the molecules in the film are arranged
in one row, determine the size of a molecule.
1.26. Determine the approximate mass and size of a mol­
ecule of carbon disulfide CS2 assuming that the molecules
are closely packed and spherical.
1.27. What will be the length of a chain of the molecules
contained in 1 mg of water and closely arranged in a row?
How many times can such a chain be wound around the
globe along the equator if the equator is 4 X 107-m long
and the diameter of a molecule is 2.69 X 10~10 m?
1.28. The densities of hydrogen and methane under nor­
mal conditions are 0.09 and 0.72 kg/m3 respectively. Find
the molar mass of methane if the molar mass of hydrogen is
2 X 10~3 kg/mol.
§ 2. Velocities of Molecules 19

1.29. Determine the fraction of volume occupied by the


Kits molecules in a vessel containing a gas under normal
conditions. The diameter of a molecule should be assumed
to be 10~10 m.
1.30. The mean velocity of a carbon dioxide molecule is
,'U>2 m/s, and the average number of collisions of a molecule
por second is 9 X 10B. Calculate the mean free path.
1.31. The mean free path of molecules in a high vacuum
Is about 5000 km. What is the average number of collisions
of gas molecules per second if the mean molecular velocity is
!>(>0 m/s?
1.32. The maximum altitude of the orbit of the Vostok
spaceship is about 327 km above the ground. The mean free
patli of gas molecules at such an altitude is about 5 km. What
would be the velocity of molecules if the number of colli­
sions per second were approximately 0.11 s"1?
1.33. The mean velocity of an oxygen molecule under
normal conditions is 425.1 m/s. Determine the mean free
path of the molecule if it undergoes on the average 6.57 X
10° collisions per second.
1.34. Determine the mean free path of air molecules un-
dor normal conditions. The effective diameter of the mole­
cules should be taken to be 3 X 10-10 m.
1.35. Determine the mean free path of helium atoms
under the conditions such that the number of atoms per
unit volume (number density) is 3.2 X 1021 m -3, and the
affective diameter of helium atoms is 1.9 X 10~10 m.
1.36. A nitrogen molecule moves under normal condi­
tions at a mean velocity of 454 m/s. Determine the mean mo­
mentum of the molecule.
1.37. A carbon dioxide molecule having a momentum of
2.7 x 10-23 kg-m/s undergoes 9.5 X 109 collisions per sec­
ond. Determine the mean distance covered by the mole­
cule between collisions.§
§ 2. VELOCITIES OF MOLECULES. BASIC EQUATION
IN THE KINETIC THEORY OF GASES
Basic Concepts and Formulas
In their thermal motion, the molecules of monatomic gases
undergo translation. Molecules consisting of several atoms
are in translatory as well as in rotary motion.
20 Ch. I. Fundamentals of Molecular Physics

Brownian movement and molecular collisions point to a


permanent change both in magnitude and in direction of
molecular velocities. For this reason, the properties of gases
are studied by using the statistical approach, which makes
it possible to calculate the mean values of velocities of
molecules, their energy, and other parameters.
The arithmetic mean velocity of all the molecules is
— t-'i -r- v„ -[ ' VK / 8HT
V = — --- s-i.
N M.t 16 V t?
where M is the molar mass of the gas, R = 8.31 J/(mol-K)
is the molar gas constant, and N is the number of the mole­
cules.
The root-mean-square velocity of molecules is
- , / H hT ^ , _o , / H r
V M ~ 1-73 K M ‘
The most probable velocity of molecules is

The basic equation in the kinetic theory of gases establishes


a relation between the pressure of gas molecules and the
kinetic energy of their translatory motion:

___“ _ r - _ "‘o'-rms
P—3 — 3 no 2 '
where n0 is the number of molecules per m3 (number densi­
ty) and Ex is the mean kinetic energy of a molecule.
In an isochoric process, the pressure of a gas is proportion­
al to its thermodynamic temperature: Pi/p2 — TX!T2, and

E* = \ k T .
This gives
p = n0kT,
where k = R/NA = 1.38 x 10~a3 J/K is the Boltzmann
constant.
The mean values of the kinetic energy of translatory mo­
tion of molecules of different gases at the same temperature
§ 2. Velocities of Molecules 21

uro the same:


m#i*,rm sl m02vZms2
2 2
('.nusequently,
1,'rm si / m02
yrmS2 moi

Worked Problems
Problem 8 . Determine the mean value of the kinetic ener­
gy and the root-mean-square velocity of helium molecules
under normal conditions.
Given: the pressure and temperature of helium under nor-
inn 1 conditions are p 0 = 1.013 X 106 Pa and T0 = 273 K
respectively. From tables, we find the molar mass of heli­
um, M = 4 X 10-3 kg/mol, the Boltzmann constant k =
1.38 X 10-23 J/K, the Avogadro constant N A = 6.02 X
| 0M mol-1, and the density of helium under normal condi­
tions, p0 = 0.18 kg/m3.
Find: the mean kinetic energy of a helium molecule, E*,
and the root-mean-square velocity of helium molecules, vrm8.
Solution. We express the mean kinetic energy of a helium
molecule in terms of the temperature: Ey = (3/2) kT0. For
monatomic gases like helium, this will be the total kinetic
energy of molecules:
E = - x 1.38 x 10-23 J/K x 273 K ~ 5.65 x 1 0 J.

Since E - m0ufmS/2, we have yrms= ^ 2E/m0, where m0


is the mass of a helium molecule, which can be expressed
us the ratio of the molar mass and the Avogadro constant:
m0 = M/Na.
Finally, we obtain

v rms l / 2 x 5.65 x 10-21 J X 6.02 x 1023 mol"1


~ 1300 m/s.
V 4 x 10"3 kg/mol
Answer. The mean kinetic energy of a molecule is 5.65 X
10"21 J, the root-mean-square velocity is about 1300 m/s.
22 Ch. I. Fundamentals of Molecular Physics

Remark. The root-mean-square velocity can also be found


from the formula urms = j /3 p 0/p„.
Problem 9. Calculate the number of air molecules in a
room of size 6 x 4 x 2.5 m3 at a temperature of 27°C and
a pressure of 99.8 kPa.
Given: V = 60 m3 is the volume of air in the room, T =
300 K is the air temperature, p = 99.8 x 103 Pa is the air
pressure. We know the Boltzmann constant k = 1.38 X
10-23 J/K from tables.
Find: the number N of air molecules.
Solution. The number N of air molecules in the room
can be determined from their number density (their num­
ber in 1 m3) and the volume of the air:
N = naV.
In order to find n0, we shall use the basic equation p =
n0kT in the kinetic theory of gases, whence n0 = plkT.
This gives

Substituting the numerical values, we get


99.8 x 10» pa x 60 m3
N = 1.38 X10-*3 J/K X 300 K = 1-45 x 1027.

Answer. The room contains 1.45 X 1027 air molecules.

Questions and Problems


2.1. Determine the arithmetic mean and the root-mean-
square velocity of air and oxygen molecules at a temperature
of 300 K.
2.2. Helium and neon have the same temperature. Mole­
cules of which of the gases have a higher mean kinetic ener­
gy?
2.3. At what temperature is the root-mean-square veloci­
ty of oxygen molecules equal to 500 m/s?
2.4. What is the ratio between the root-mean-square ve­
locities of helium and neon molecules at the same tempera­
ture?
2.5. The critical temperature of hydrogen is 32 K, while
the hydrogen in the Sun’s atmosphere is at about 6000 K.
§ 2. Velocities of Molecules 23

Dnlurmine the root-mean-square velocities of hydrogen


mnlcc,ules at these two temperatures.
2.6. Calculate the mean kinetic energy of hydrogen mole­
cules at the temperatures mentioned in Problem 2.5.
2.7. What is the temperature of a monatomic gas if the
menu kinetic energy of its molecules is 0.8 x 10”19 J?
2.8. Determine the mean kinetic energy of translatory
motion of all the neon molecules in 1 mol and in 1 kg at
KMK) K.
2.9. What must be the temperature of hydrogen for its
molecules to have the same root-mean-square velocities as
Hint of helium at 580 K?
2.10. One cubic metre of a gas contains 2.4 X 1010 mole­
cules at 27°C. Determine the gas pressure. What is the term
applied to such a degree of rarefaction?
2.11. How many molecules are there in 0.5 m3 of a gas at
•MK) K and a pressure of 120 kPa?
2.12. Determine the mean kinetic energy of monatomic
gas molecules at 310 K. How many molecules are there in
I m3 of such a gas at 0.4 MPa?
2.13. Determine the pressure of nitrogen in an ampoule if
the number density of the molecules in it at 0°C is 3.5 X
10u .
2.14. Find the root-mean-square velocity and the mean
kinetic energy of helium molecules at 20°C.
2.15. The pressure of the rarefied air between the walls of
a Dewar flask is 1.33 X 10“2 Pa at 0°C. How many mole­
cules does a cubic centimetre contain?
2.16. A vessel whose volume is 2 m3 contains 2.4 kg of a
gas. What is the gas pressure if the root-mean-square veloc­
ity of its molecules is 500 m/s?
2.17. How many air molecules are contained in a 4 X
5 X 3 m3 room at 20°C under a pressure of 90 kPa?
2.18. What is the increment in the mean kinetic energy of
translation of molecules in a monatomic gas heated from
0°C to 373 K?
2.19. A gas has leaked from a 5-1 cylinder because of a
faulty valve. As a result, the gas pressure dropped by
2.9 kPa. The temperature in the cylinder remained unchanged
at 17°C. How many molecules escaped from the cylinder?
2.20. After an electric heater had been switched on, the
air temperature in the room was raised from 17 to 22°C at a
24 Ch. I. Fundamentals of Molecular Physics

constant pressure. By what amount (in percent) was the


number of air molecules in the room reduced?
2.21. Hydrogen leaked from a 1-m3 cylinder because of a
damaged valve. Initially, when the hydrogen was under a
pressure of 5 MPa, its temperature was 280 K. After some
time, the temperature rose to 290 K at the same pressure.
How many hydrogen molecules escaped from the cylinder?
What was the decrease in hydrogen mass?
2.22. The root-mean-square velocity of thermal motion of
gas molecules was increased by a factor of two. Determine
the change in the thermodynamic temperature of the gas and
in the kinetic energy of thermal motion of molecules if the
gas was monatomic.

§ 3. EQUATION OF STATE FOR AN IDEAL GAS. ISOTHERMAL,


ISOCIIORIC, AND ISORARIC PROCESSES

Basic Concepts and Formulas


The physical quantities characterizing the state of a body
are known as parameters. In order to characterize the state
of a gas, three parameters are needed: pressure p, volume V,
and temperature T. The equation relating the three param­
eters is known as the equation of state for an ideal gas.
For a fixed mass of a gas, the equation of state has the
form
pV . pjl'i pzV2
-— = const, or r / =J1\ '

If the gas was first under normal conditions (p0, F0, T0)
and then changed to a state for which the parameters became
p, V, and T, the equation of state has the form
P0V0IT0 = pV/T.
It should be noted that p 0 and T0 are known and equal to
1.013 X 105 Pa and 273 K. Therefore, in order to determine
V0 it is sufficient to know the values of p, V, and T.
The equation of state of a gas is applicable to isothermal,
isobaric, and isochoric processes. Indeed, by cancelling out
the parameter which is constant for a given process, we ob­
tain:
§ 3. Equation of State for an Ideal Gas 25

for an isothermal process


p1Vl = P 2 V2 for T = const, m = const,
for an isobaric process
V,/T1 = V2/T 2 or VJV2 = T J T 2 for p = const, m = const,
imd for an isochoric process
J*
Pi _ P2 or — = -~r- for V = const, m = const.
T1 T2 '
In the general case when the mass m is known or is to be
determined, the equation of state of an ideal gas is

where M is the molar mass of the gas and R = 8.314 J/


(inol-K) is the molar gas constant.
Using this equation, we can determine the density of a
gas as a function of its temperature and pressure. Dividing
both sides of the equation by volume V and substituting
density p for m/V, we obtain

For a fixed mass of a gas, MIR is constant. Consequently,


the gas density is directly proportional to pressure and in­
versely proportional to thermodynamic temperature.
Dalton’s law. If a vessel contains a mixture of several
gases which do not react chemically, the pressure of the gas
mixture is equal to the sum of the partial pressures of each
gas taken separately, i.e.
P = Pi + P2 + Ps + • • •,
where p lt p 2, p 3, . . . are the partial pressures, i.e. the
pressures exerted by each gas separately as if it alone were
occupying the entire vessel.

Worked Problems
Problem 10. A cylinder contains a gas at a temperature
of 17°C and a pressure of 1.0 MPa. What will be the change
in the pressure if the temperature is lowered to —23°C?
26 Ch. I. Fundamentals of Molecular Physics

Given: = 17°C and p l = 1.0 MPa are respectively the


temperature and pressure of the gas in the first state, and
t2 = —23°C is the gas temperature in the second state.
Find: the change in pressure, Ap, in the cylinder as a re­
sult of the transition of the gas from the first to the second
state.
Solution. We use the equation of state for the gas, p i Vl/ T 1=
P i V J T 2. Since the change in gas pressure occurs with
decreasing temperature but at a constant volume (an iso-
choric process), we have Vx = V2 and p j p 2 = T J T 2.
Hence p 2 = P\T2/T 1 and
Ap = Pi — p 2-
We shall write the parameters for the first and second states
of the gas separately. The parameters of the first state are
Pi = 1.0 X 10® Pa, Tx = 290 K, and the parameters of the
second state are T2 = 250 K, p 2 = ?
Let us determine the gas pressure p2 after the temperature
has been lowered and calculate the pressure difference Ap:

p2 = - |— - x i.O x 10® Pa = 0.86 x 10® Pa,

A p = (1.0 — 0.86) x 10® Pa = 0.14 x 10® Pa.


Answer. The gas pressure in the cylinder dropped by
0.14 MPa.
Problem 11. A 20-1 vessel is filled with air at a pressure
of 0.4 MPa and connected to another vessel from which all the
air has been pumped out. The pressure in the two vessels
equalizes at 1.0 X 105 Pa. Determine the volume of the
second vessel, assuming that the process is isothermal.
Given: p 1 = 0.4 MPa = 0.4 x 10® Pa and V! = 20 1 =
0.02 m3 are respectively the pressure and the volume of the
gas in the first state and p2 = 1.0 x 105 Pa is the gas pres­
sure after the second vessel has been connected to the first
one.
Find: volume V of the second vessel.
Solution. An isothermal process occurs at a constant tem-
petature (Tt = T2) and obeys Boyle’s law, which is forma­
lized by the equation pV = const, or = p2P2. After
the second vessel is connected to the first, the gas occupies
the volume V2 = Vl -\- V. Consequently, pl Vl = p2 (Pi +
§ 3. Equation of State for an Ideal Gas 27

V), or
i ^ = V + Vl
Pi
whence
PiVi 0.4 x 106 Pa x 0.02 m3
V V 1.0 X 105 Pa
- 0.02 m3
Pi
= 0.06 m3.
Answer. The volume of the second vessel is 60 1.
Problem 12. What was the temperature of a gas if as a
result of isobaric heating through 1 K its volume increased
hy 0.0035 of the initial value?
Given: AT = 1 K is the change in the gas temperature
and AP = 0.0035 Vx is the increment in the gas volume.
Find: Tu the initial temperature of the gas.
Solution. As a result of heating, the gas temperature has
increased by 1 K, hence T2 = 7 \ + AT. The gas volume
has increased thereby from to V2, i.e. V2 = V1 AP.
Since the process occurs at a constant pressure (px = p 2),
the equation of state can be transformed as follows:

or Vi T\ ± l T (Charles’ law).
Ti T2 Ti
Let us solve this equation for Tt : V1Tl + Vx AT =
P,?1! AVTX, whence
AT Vt X l K
AP ’ Ti 0.0035Pj
= 286 K.

Answer. The gas temperature before heating was 286 K.


Problem 13. Two cylinders having volumes 3 1 and 7 1 are
filled respectively with oxygen at a pressure of 200 kPa
and nitrogen at a pressure of 300 kPa at the same tempera­
ture. The cylinders are connected and after a certain time
each will contain a gas mixture at the same temperature.
Determine the pressure of the gas mixture in the cylinders.
Given: F10I = 3 1 and p10x = 200 kPa are the volume
and the pressure of the oxygen prior to the connection of
the cylinders, Flnu = 7 1 and plnit = 300 kPa are the
volume and the pressure of the nitrogen prior to the con­
nection of the cylinders.
Find: the pressure p of the gas mixture.
28 Ch. I. Fundamentals of Molecular Physics

Solution. According to Dalton’s law, the pressure of a gas


mixture is equal to the sum of the partial pressures of the
gases: p = p 2ox + P2nit, where p'20x and />2nit are the
partial pressures of the oxygen and the nitrogen.
After the cylinders have been connected, each gas occupies
a volume equal to the sum of the volumes of the cylinders:
Vinx — P 2nit — Flox “I" Flnit-
Let us write the parameters of the two states for the oxy­
gen and nitrogen in SI units:
for oxygen
state 1 state 2
F10x = 3 X 10-3 m3, F2ox = 3 X 10-3 m3
+ 7 X 10-3 m3 = 1 x 10“2 m3.
Piox = 2 X 10s Pa, P2ox = ?
for nitrogen
state 1 state 2
F,mt = 7 X 10-3 m3, FJnit = 1 X 10“a m3,
Pin It = 3 X 10® Pa, P2nit = ?
The process is isothermal, hence T = const.
In order to find p 2ox and P2mti we use Boyle’s law
P lO I ^ l O I — P 2 o x ^ 2 o x j / h n l t ^ l n l t — P 2 n itF 2 n it f ° r each gSS
separately:
PioxPiox 2 x 10s Pa x 3 X10 3 m3 n c; \s 4ns i>.»
V'2m = ---------1X 10-2 m3-------- = a b X W Fa
3 X 10B Pa X 7 X 10"3 m3
P in ltF m lt
P2nit = = 2.1 X 10s Pa.
^2nit 1 X 10“2 m3
Finally, we obtain
p = 0 . 6 x 10® Pa + 2.1 X 10B Pa = 2.7 x 10s Pa.
Answer. The pressure of the gas mixture in the cylinders
is 270 kPa.
Problem 14. The air in a balloon at a temperature of
20°C and a pressure of 99.75 kPa has a volume of 2.5 1.
When the balloon is immersed in water at a temperature of
58C, the air pressure in it increases to 2 X 105 Pa. What is
the change in the volume of the air in the balloon?
§ 3. Equation of State for an Ideal Gas 29

Given: t l = 20°C, p 1 = 99.75 kPa, and F, = 2.5 1 are the


temperature, pressure, and volume of the air before the
balloon is immersed in water, t2 = 5°C and p2 = 2 X 105 Pa
are the temperature and pressure of the air after the immer­
sion of the balloon in water.
Find: the change AF in the volume of the air in the bal­
loon.
Solution. Before the balloon is immersed in water, the
state of the air in it is characterized by the parameters
/>n Vj, and Tx. After immersion, the parameters are p 2,
V2, and T2.
Let us write the parameters of the gas in states 1 and 2
separately, expressing them in SI units:
state 1 state 2
p i = 9.975 x 10* Pa, p 2 = 2 X 10s Pa,
F, = 2.5 X 10-3 m3, V2 = ?
Tx = 293 K, T2 = 278 K.
As the air changes from state 1 to state 2, all three param­
eters change. Consequently, we must use the equation of
state to determine the final volume V2:
PlVl _ PlV 2
T, T2
whence

v2 P1 V1T2
T1P2 '
9.975 X 104 Pa x 2.5 x 10"3 m3 X 278 K
= 1.2 X 10"3 m3.
293 K X 2 x 105 Pa

Calculations show that the volume of the air in the balloon


changes as a result of its immersion in water by
AV = V t - V2,
AF = 2.5 X 10-3 ms - 1.2 x 10' 3 m3 = 1.3 X lO' 3 ma.
Answer. The change in the volume of the air is 1.3 X
10-3 ra3 = 1.3 1.
Problem 15. A cylinder having a volume of 0.6 m8 con­
tains oxygen at a temperature of 27°C. A pressure gauge on
30 Ch. I. Fundamentals of Molecular Physics

the cylinder indicates 11.7 MPa of excess pressure.2 Reduce


the volume of the oxygen to normal conditions and deter­
mine its mass.
Given: Vx = 0.6 m3 is the volume and T1 = 27°C is the
temperature of the oxygen in the cylinder, p g = 11.7 MPa
is the reading of the pressure gauge, T0 = 273 K, p 0 =
1.013 X 105 Pa are the temperature and pressure under nor­
mal conditions. From tables, we take the density p0 =
I. 43 kg/m3 of oxygen under normal conditions, the molar
mass of oxygen, M = 32 X 10“3 kg/mol, and the molar gas
constant R = 8.31 J/(mol-K).
Find: the volume F0 of the oxygen under normal condi­
tions and the mass m of the oxygen in the cylinder.
Solution. From the reading of the pressure gauge, we de­
termine the gas pressure in the cylinder: p 1 = p g + p0 =
I I . 7 X 10a Pa + 0.1013 X 10s Pa = 11.8 X 106 Pa. Re­
ducing the volume to normal conditions means determining
the volume the gas would occupy at a temperature of 273 K
and a pressure of 1.013 x 108 Pa.
We shall write the parameters of the oxygen in SI units
for the two states:
state 1 state 2
p t = 11.8 X 106 Pa, p0 = 1.013 X 105 Pa,
Fj = 0.6 m3, T0 = 273 K,
Tt = 300 K, V0 = ?
We solve the problem by using the equation of state for
an ideal gas:
PiVi _ p0V0
T! T0
whence
PiViTq
V0 = T1P0
T/ 11.8 X 10® Pa X 0.6 m3 X 273 K co c i
V ° = ----- 300 K X 1.013 x 105 Pa------= 63‘6 ™

2 The excess pressure indicated on a pressure gauge is the difference


between the pressure of the gas in the cylinder and atmospheric pres­
sure.
§ 3. Equation of State for an Ideal Gas 31

From V0 and p0, we can determine the mass of the oxygen:


m = p0V0, m = 1.43 kg/m3 X 63.6 m3 = 91 kg.
Answer. The volume of the oxygen under normal condi­
tions is 63.6 m3, the mass is approximately 91 kg.
Remark. The problem can be solved by using the equation
/),F, = ^ R T U from which we first determine the mass
in = ~ti I j . We can And the gas volume from the density
of the oxygen under normal conditions:
V0 = ml p0.
Problem 16. A cylinder contains acetylene at 27°C under
a pressure of 4.05 MPa. What will the pressure in the cylin­
der be after half the mass of the gas has been used up if the
temperature has fallen thereby to 12°C?
Given: T1 = 300 K and = 4.05 X 10a Pa are the in i­
tial temperature and pressure of the gas in the cylinder,
T2 = 285 K is the temperature of the remaining gas, m2 =
0.5 mx is the mass of the consumed gas.
Find: the pressure p 2 of the gas remaining in the cylinder.
Solution. In this problem, the temperature, pressure, and
mass of the gas change. Therefore, it is best to use the equa­
tions of state for the two cases in the form
= 9 - /?7\ and p2\ \ = ^ RTV

Dividing the Arst equation by the second termwise and can­


celling out mlt M, R, and the unknown volume Vit we
obtain
Pi T l
p2 O.STi ’
w lienee
0-5piTj
/V - Ti '
0.5 x 4.05 XlO6 P a x 285 K
P2 = = 192.4 x 104 Pa
300 K
~ 1.92 x 10® Pa.
Answer. The pressure of the acetylene remaining in the
cylinder is approximately 1.92 MPa.
32 Ch. I. Fundamentals of Molecular Physics

Problem 17. Determine the density of hydrogen at a tem­


perature of 17°C and a pressure of 204 kPa.
Given: T = 290 K and p = 2.04 X 105 Pa are the tem­
perature and pressure for which the density of hydrogen has
to be determined. From tables, we obtain the molar mass of
hydrogen, M = 2 X 10~3 kg/mol, and the molar gas con­
stant R = 8 .3 1 J/(mol-K).
Find: the hydrogen density p.
Solution. We write the equation of state pV = RT.
Dividing both sides by V, we obtain p = - ^ - ~ - , where
RT
m/V is the density. Then p = p , whence
pM 2.04 X 10s Pa X 2 X 10~3 kg/mol ^ _
P RT ’ 8.31 J/(mol-K) X 290 K ~ U 1 / k g / m 3.
Answer. The density of hydrogen is 0.17 kg/m3.
Problem 18. Plot the graphs for an isothermal, isobaric,
and isochoric process in p-V coordinates.
Solution. An isothermal process occurs at a constant tem­
perature T = const and obeys Boyle’s law. According to

p.MPo
Isobar
0.2

0.1 -
,
0 0.2 O.h Vm}

Fig. 4

this law, the pressure of a given mass of a gas varies in in­


verse proportion to its volume. Such a dependence is repre­
sented by a hyperbola, which is known in physics as an
isotherm. In order to plot the graph, it is sufficient to keep
the product pV constant, say, pV = 1.2. Ascribing arbit­
rary values 2, 4, 6 , . . . to the volume, we calculate the
corresponding pressures: 0.6, 0.3, 0.2, . . . . Having chosen
a scale, we plot the graph (Fig. 3)
§ 3. Equation of State for an Ideal Gas 33

The volume of a gas in an isobaric process varies with tem­


perature at constant pressure p = const. The process is
graphically represented by a straight line parallel to the
K-axis. For an isochoric process, the pressure varies at a
constant volume V = const. This de­
pendence is represented by a straight
line parallel to the p-axis (Fig. 4).
Problem 19. Which of the two iso­
therms (Fig. 5) plotted for the same
mass of a gas corresponds to a higher
temperature?
Solution. We construct the isochor
for a certain volume Vt . The isochor
intersects the isotherm corresponding
to the temperature Tx at a pressurep u
and the isochor intersects the isotherm
for T2 at a pressure p 2. Since the same volume corresponds
to states 1 and 2, we can write p 1/T 1 = p 2IT2 from Gay-
l.ussac’s law. Since p 2 > p lt T2 > Tx. Consequently, the
upper isotherm corresponds to the higher temperature.

(Questions and Problems

3.1. A vessel contains 10.2 1 of a gas under normal condi­


tions. What volume will the gas occupy at a temperature of
i()°C and a pressure of 1 MPa?
3.2. A gas occupies a volume of 4 1 at a temperature of
.r>0°C and a pressure of 196 kPa. At what pressure will this
gas occupy a volume of 16 1 if heated to 20°C?
3.3. At what temperature do 4.0 m3 of a gas produce a
pressure of 150 kPa if the same mass of the gas under normal
renditions has a volume of 5 m3?
3.4. A 45-1 cylinder contains oxygen at a temperature of
:!7"C. and a pressure of 1.52 MPa. What volume would the
gns occupy under normal conditions?
3.5. Compressed oxygen for welding is stored in 20-1 cyl­
inders at a pressure of 9.8 MPa and a temperature of 290 K.
IIeduce the volume of oxygen to normal conditions.
3.6. A 6-1 cylinder contains 0.1 kg of a gas at a temper-
el lire of 300 K and a pressure of 9.44 X 106 Pa. Determine
l lie molar mass and identify the gas.
i nil :ii)
34 Ch. I. Fundamentals of Molecular Physics

3.7. What amount of gas (in moles) is contained in a 10-1


cylinder at a pressure of 0.29 MPa and a temperature of
17°C?
3.8. Determine the mass of carbon dioxide stored in a
cylinder having a volume of 40 1 at a temperature of 13°C
and a pressure of 2.7 MPa.
3.9. Determine the amount of gas (in moles) occupying a
volume of 25 1 at a pressure of 1.4 X 105 Pa and a tempera­
ture of 300 K.
3.10. A mountain-climber takes in 5 g of air with each
breath under normal conditions. What volume of air must
he inhale in the mountains where the atmospheric pressure is
lower than that at sealevel and is 79.8 kPa at a temperature
of —13°C?
3.11. Under normal conditions neon occupies a volume of
12.4 1. How many times will the pressure of the gas increase
if it is placed in a vessel having a volume of 5.6 1 at a tem­
perature of 318 K?
3.12. An ideal gas occupies a volume of 2 1 at a pressure
of 1.33 kPa and a temperature of 15°C. What will be the
pressure if the temperature is doubled, while the volume de­
creases by 0.25 of the initial value?
3.13. A 40-1 cylinder contains 64 g of oxygen under a
pressure of 213 kPa. Determine the temperature of the gas.
3.14. 42 g of acetylene are kept in a 20-1 cylinder at a
temperature of 17°G. Determine the amount of substance
in the gas and its pressure.
3.15. A 40-1 cylinder contains 1.98 kg of carbon dioxide
at 0°G. When the temperature is increased through 48 K,
the cylinder explodes. At what pressure does the explosion
occur?
3.16. Calculate the molar mass of butane if 2 1 of the gas
have a mass of 4.2 g at a temperature of 15°C and a pressure
pf 87 kPa. Calculate the number of gas molecules in 1 m3.
3.17. At the beginning of the compression stroke in a die­
sel engine, the temperature of the air was 40°C and the pres­
sure was 78.4 kPa. As a result of compression, the volume
was reduced by a factor of 15, and the pressure increased
to 3.5 MPa. Determine the temperature of the air at the
end of the compression stroke.
3.18. The volume of 265 g of a gas at a temperature of
273 K and a pressure of 5 MPa is 60 1. What gas is it?
§ 3. Equation of State for an Ideal Gas 35

3.19. Determine the mass of air in a room of 6 X 5 X


■I in3 at a temperature of 293 K and a pressure of 1.04 X
|0» |*a. The molar mass M of air is 29 x 10"3 kg/mol.
3.20. What will the final temperature of a gas mixture in
an internal combustion engine be if it occupies a volume of
40 dm3 in the cylinder at a temperature of 50°C under nor­
mal atmospheric pressure and is compressed by the piston
In a volume of 5 dm3 and a pressure of 15.2 X 106 Pa?
3.21. A cylinder contains a gas at a temperature of 7°C
and a pressure of 91.2 MPa. What will be the pressure after
0,25 of the mass of the gas has flown out of the cylinder and
Iho temperature has risen to 27°C?
3.22. A closed vessel of 2-m3 volume contains 1 kg of
nitrogen and 1.5 kg of oxygen. Determine the pressure of
Mm mixture in the vessel if the temperature of the mixture
u 17°C.
3.23. Determine the density of oxygen at a temperature
of 47°C and a pressure of 105 Pa.
3.24. The pressure of air 10 km above the surface of the
Karth is about 30.6 kPa and the temperature is 230 K. De­
termine the density of the air, the number density of the
molecules, and their root-mean-square velocity at this alti-
lltdo.
3.25. 7 g of a gas contained in a cylinder at 27°C produce
n pressure of 4.9 X 104 Pa. 4 g of hydrogen at 60°C produce a
pressure of 43.5 x 104 Pa in the same volume. Determine the
molar mass of the unknown gas and identify it.
3.26. What will be the increase in the mass of air in a
room if the atmospheric pressure changes from 9.84 X 104
lo 10.1 x 104 Pa and the air temperature remains unchanged
mid equal to 273 K? The size of the room is 4 X 5 X 2.5 m3.
3.27. A vessel containing 10 1 of air under a pressure of
1 MPa is connected to a 4-1 empty vessel. Determine the
linal air pressure in the vessels, assuming that the process is
Isothermal.
3.28. A vessel contains a gas under a pressure of 5 X
106 Pa. What will the gas pressure be if 3/5 of the mass of
the gas has flown out, the temperature being maintained
constant?
3.29. Plot the graphs of an isothermal process in V-T
mid p-T coordinates.
3.30. The bladder of a football having a volume of 2.5 1
;i*
36 Ch. I. Fundamentals of Molecular Physics

must be inflated to a pressure of 300 kPa. The pump takes


in 0.14 1 of air under normal atmospheric pressure during
one stroke. How many strokes are required if the bladder
was initially empty?
3.31. Given two equations of isothermal processes for
the same mass of a gas: p l Vl = 6 and p 2V2 = 9. Plot the
graphs in p-V coordinates and answer the following ques­
tions: (1) in which units is the product pV expressed? (2)
which of the isotherms will be further from the coordinate
axes and why?
3.32. Dry atmospheric air consists of oxygen, nitrogen,
and argon. Disregarding other components whose concen­
trations are very small, determine the masses of these gases

in 1 m3 of atmospheric air under normal conditions if their


partial pressures are 2.1 X 104 Pa for oxygen, 7.8 X 104 Pa
for nitrogen, and 103 Pa for argon.
3.33. Represent an isobaric process graphically in V-T,
p-V, and p-T coordinates.
3.34. Figure 6 shows two isobars plotted for the same
mass of a gas. Which of the two isobaric processes occurs at
a higher pressure and why?
3.35. A gas is heated isobarically from 7^ to T2 (Fig. 7).
What changes occur in the gas?
3.36. What volume w ill a gas occupy at 348 K if its
volume at 35°C is 7.5 1? The process is isobaric.
3.37. A gas occupies a volume of 10 1 at 27°C. To what
temperature should it be cooled isobarically to reduce its
volume by 0.25 of the initial value?
3.38. Through how many degrees Kelvin should the tem­
perature of a gas be increased in an isobaric process to in-
§ 4. The Change in the Internal Energy 37

i roase its volume by a factor of 1.3 in comparison with the


volume occupied by it at 0°C?
3.39. A certain mass of a gas is taken through a closed
cycle 1-2-3-1 in which the state of the gas changes isobaric-
nlly-isochorically-isothermally. Represent these processes in
V T and p-V coordinates.
3.40. A balloon of volume 4500 m3 is filled with helium
nl 290 K. The mass of the envelope is 677 kg. Determine
the lifting force of the balloon at a temperature of 27°C,
assuming that the atmospheric pressure remains unchanged
imd is 102 kPa.

§ 4. THE CHANGE IN THE INTERNAL ENERGY DURING HEAT


TRANSFER AND DUE TO MECHANICAL WORK

Itasic Concepts and Formulas


When several bodies having different temperatures come
into contact, heat transfer takes place, as a result of which
the temperature of the bodies equalizes, and thermal equilib­
rium sets in. The internal energy of the bodies being heated
increases at the expense of the energy given away by the
bodies with higher temperatures. It has been established
that the change in the internal energy of a body is propor­
tional to its mass and to the change in its temperature:
AU = Q = cm AT, where Q is the amount of heat (a mea­
sure of the change in the internal energy) expressed in joules
(J), and c is the proportionality factor known as the specific
heat of a substance. For example, the specific heat of alu­
minium is 920 J/(kg-K), which means that the internal
energy of 1 kg of aluminium increases by 920 J when its
temperature is raised through 1 K.
In order to determine a specific heat, the heat balance
equation is employed. This equation is constructed by estab­
lishing the processes in which the energy is released (during
cooling or as a result of fuel combustion) and the processes
in which energy is absorbed (as a result of heating).
For example, a body of mass ml and specific heat cx has a
temperature 7\ and is exchanging heat with a body of
mass m2, which has a specific heat c2 and a temperature T2.
It is known that T, > T2. Then the first body gives away
38 Ch. I. Fundamentals of Molecular Physics

the following amount of heat:


C?glv = Cl f n i (^1 0 )-
The second body receives as a result of the heat transfer the
following amount of heat:
Q rcc = C2m 2 ( 0 — T 2) ,

where 0 is the final temperature of the two bodies. Since


energy is conserved, we can write @glv + (>rec = 0- This
gives
cIm1 ( 7 1! — © ) = c2mi ( 0 — T2).
This is the heat balance equation for the system of two
bodies.
A body can be heated at the expense of the energy released
during fuel combustion. The amount of heat liberated is pro­
portional to the mass of the burnt fuel and depends on which
fuel it is:
Q = gm„
where q is the specific heat of combustion of the fuel in joules
per kilogram (J/kg) and mf is the mass of the burnt fuel.
Not all the heat liberated during the combustion of a fuel
is usefully spent on heating. A fraction of the heat dissi­
pates. This is taken into account by introducing the efficiency
of a heater:
_ ^useful
(?spent
If (ireful = cm and (?spent = we have
cm ST
11= ---------
qmj
The internal energy of a body or a system of bodies may
change when a mechanical work is done. The mechanical
energy of a body or a system of bodies can be transformed
completely into internal energy, i.e. be spent on heating:
Q = A£ k + A £ p,
where A a n d A-Ep are the changes in the kinetic and po­
tential energies of the body.
The first law of thermodynamics (the law of energy conser­
vation). The amount of heat transferred to a closed system
§ 4. The Change in the Internal Energy 39

is spent on increasing its internal energy and on the mechan­


ical work done against external forces:
Q = AU + A,
where Q is the amount of heat transferred to the system,
AU is the change in its internal energy, and A is the work
done by the system.
For isochoric processes, V = const, and hence
A = p AV = 0, Q = AU,
i.e. the heat transferred to a gas is completely used to in­
crease its internal energy.
In isobaric processes, p = const, and hence
Q = AU + A,
i.e. the heat transferred to a gas is spent on increasing its
internal energy and on the work done to expand the gas.
For isothermal processes, T = const, and hence the in­
ternal energy does not change. Therefore, Q = A. The heat
supplied to a gas during an isothermal process is completely
spent on the mechanical work done by the gas.
Adiabatic processes occur in the absence of any heat
exchange with the surroundings, i.e. Q = 0. Therefore, the
lirst law of thermodynamics in this case will have the form
Q = AU + A, or A = - A U ,
i.e. an adiabatically expanding gas does work at the expense
of the change in its internal energy, being cooled thereby.

Worked Problems
Problem 20. In order to determine the specific heat of
copper, a copper cylinder of mass 0.5 kg is heated to 100°C
mid then placed in an aluminium calorimeter of mass 40 g,
containing 300 g of water at a temperature of 15°C. As a
result of heat exchange, the calorimeter stabilizes at tem­
perature 26°C. What is the specific heat of copper in this
experiment? Compare the obtained result with the tabulat­
ed value and determine the absolute and the relative error.
Given: mc = 0.5 kg is the mass of the copper cylinder,
'/’ = 373 K is the initial temperature of the cylinder, ma =
0.04 kg is the mass of the calorimeter, row = 0.3 kg is the
40 Ch. 1. Fundamentals oi Molecular Phygio

in ass of the water, Tx = 288 K is the initial tempera tore of


the water and calorimeter, and 8 = 299 K is the Anal tem-
erature of the water, calorimeter, and cylinder. Prom ta­
C les, we lake the specific heal of aluminium, c , -=880 J/(kg- K),
and the specific heal of water, cw — 4187 J/(kg-K ).
The tabulated specific heal of copper is ctat, — 380 J/(kg K).
Find: the specific heat of copper ce, the absolute error Ac
and the relative error A o f the measurement.
Solution. As a result of heat exchange, the temperature of
all the bodies in the calorimeter equalizes. The healed cy l­
inder gives off heat ^ (iT and is cooled from T to 8:
£ ijv = ccmc ( T — 8).
The calorimeter and water receive heal, and their tem ­
perature rises from T t to 0 :
Q t « *= (© — Pi) 4- (8 Pi),
or
Q ttc. ” -1- f»mw) <8 — P,).
Proceeding from the energy conservation law, we equate
the heal given off by the copper cylinder and the heal re­
ceived by the calorimeter and water: Q ,\* ~ Qr*<- Conse­
quently. ccm t (T — 8 ) = {emmk T f . i n , ) ( 8 — T,). We ob­
tained the heal balance equation from which we deter­
mine cr :
(c««n» -7- ) —r,)
c mci r - e i
Substituting the numerical values of the known quanti­
ties, we calculate the specific heat for copper:
_ (8fl0J/)hfl K )x0.04he + 'ilft7 J/(hg K) X n-1 kg) >.(2TO K - 288K)
1= 0.5 kg (373 K — 2W K)
i t 384 J/(kff-K ).
The absolute error is
Ae " cc f l«b,
A« «= 384 J/(kg*K) — 380 J /(k g - K )c t 4 J/(kg*K).
The relative error is
4 J/< k « K )
-^ X 100*4 X 100% is 1.1%.
cu t 380 J / ( k g K)
5 4. The CliiDge in tho internal Energy 41

Answer. The specific heal of copper is 384 J/(k gK ), the


.ihsolute error is approximately 4 J/(kg- K), and the relative
error is about 1.1%.
Problem 21.150 g of water is contained in a brass ealorim-
(>ler ol mass 0.2 kg at a temperature of 12°C. Find the
nnal temperature in the calorimeter after an iron weight
having a mass of 0.5 kg and heated to 100X is immersed in
filter. Plot the temperature versus the amount of heat graph
lor this hent transfer process,
Giien: mw = 150 g ■= 0,15 kg is the mass of water in the
colorimeter, >nca, *=■ 0.2 kg is the mass of tlio calorimeter,
t \ ■= 285 K is the Initial temperature of the water and
cnlnrimeter, m, — 0 ..r> kg is the mass of the weight, and
/'i — 373 K is the initial temperature of the weight. From
lable?, we taka the specific heat of iron, fj — 4G0 J/(kg-K),
Ilie specific heat of water, 4187 J/(kg-K), and the specific
Lcal of brass, fh *= 380 ]/(kg-K).
bind’ the final temperature 0 in the calorimeter.
Solution. As a result of heat exchange in the calorimeter,
(lie internal energy of the weight decreases, while 1he inter­
nal energy of the calorimeter and water increases. Tho amount
of heat is a measure of the change in Ihe internal energy.
The amount of heat Q, = c ^ i, (7't — 6 ) liberated d u r ­
ing the cooling of the weight is absorbed by heating the
water (£JW = ev m w (0 — 7'u)) and the calorimeter f()r(,| =>
'Vni^cji (0 7 **))■
The heal balance equation is

C|i/it(7’| — 0 ) — cwmv (0 — TV) h <Y*\>ncni (0 Tw),


We shall solve this equation for the unknown temperature
6 in the calorimeter at thermal equilibrium. For this pur­
pose, we open the parentheses;
<'ifn ,T , — c ^ i© — cwu/y,0 — cwmw7’w
C Crn;rTlr„|0 Ccn III,-ri i T n ■
All the Larms containing H can be transferred to the right-
hand side of the equation;
-j- Cyin^Tff -f- c( i |/7?(,B|7'w c wmw0 % CjoiiH
-r c.jimt.i©-
42 Ch. I. Fundamentals of Molecular Physics

We can now write the expression for 0 in the general form:


c\m \T i -{- ( C\\'Tnw- + Ccal,rlcal) T \v
0:
c ln ll r c v ,m w t C c a lm c a l

Substituting the numerical values, we obtain


460 x 0.5 X 373 + (4190 x 0.15 + 380 X 0.2) 285
0 —
4190 x 0.15 + 460 X 0.5 + 380 X 0.2
J - k f r l -K -l -k £ -K
~ 307 K, 0 = 34°C.
J ■kg- 1 -K ~ 1 -k p

The graph of t = f (Q) is represented in Fig. 8. The


straight line AB shows the change in the temperature of the
weight (the temperature decreases
from 100 to 34°C since the pro­
cess is accompanied by the liber­
ation of heat). Straight line BC
shows the change in the temper­
ature of the calorimeter and w a­
ter (the temperature increases
from 12 to 34°C, the process in­
volves the absorption of heat).
Straight lines CE and CD show
the change in the temperature of
the water and the calorimeter
respectively. Even though the temperature difference is the
same in both cases (22 Iv), the lines CD and CE have differ­
ent slopes. This is because the water absorbs more heat
than the calorimeter.
Answer. The equilibrium temperature in the calorimeter
is about 307 K.
Problem 22. Determine the efficiency of a melting furnace
in which 70 kg of A-l grade coal was burnt to heat 0.5 t of
aluminium from 282 K to the melting point.
Given: ma = 0.5 t = 500 kg is the mass of the alumini­
um, Tl = 282 K is the initial temperature of the alumin­
ium, mcoaI = 70 kg is the mass of the coal. From tables,
we obtain the melting point of aluminium Tm = 932 K,
the specific heat of aluminium ca = 880 J/(kg-K), and the
specific heat of combustion of coal q = 2.05 X 107 J/kg.
Find: the efficiency r| of the furnace.
Solution. The amount of heat (?us(.fui required to heat
the aluminium from Tl to its melting point can be deter-
§ 4. The Change in the Internal Energy 43

mined from the formula


(^useful cama. (^m ^l)-
The amount of heat (Tpent liberated as a result of the
combustion of the coal is given by
C ?spent = ? t t l c o a I-

The efficiency of the furnace is defined as the ratio of the


iimount of heat spent in heating the aluminium to the
iimount of heat obtained as a result of the fuel combustion:
^ 1)
11= X 100%, or (] = ca frt-a i
(/u se fu l
x 100%.
Aspect 9m c o a l

Substituting the numerical values, we obtain


880 J/ (kg-K) x 500 kg X 650 K
100% ~ 20%.
2.05 x 107 J / kgx70 kg
Answer. The efficiency of the furnace is about 20%.
Problem 23. The engine of a scooter develops a power of
3.31 kW at a speed of 58 km/h. How far will the scooter go
l>v consuming 3.2 1 of petrol if the efficiency of the engine
is 20%?
Given: P = 3.31 kW = 3310 W is the power developed
by the engine, v — 58 km/h = 16.1 m/s is the speed of the
scooter, V = 3.2 1 = 3.2 X 10-3 m3 is the volume of con­
sumed petrol, and t] = 20% is the efficiency of the engine.
From tables, we obtain the specific heat of combustion of
petrol, g = 4.6 X 107 J/kg, and the density of petrol,
[) = 700 kg/m3.
Find: the distance s covered by the scooter.
Solution. The energy Q = qm liberated by burning the
petrol is spent on the work done to move the scooter and the
driver. Only 20% of the energy received from the fuel will
lie spent in doing mechanical work. Using the law of energy
conservation, we can write t\Q = A, or t|qm = Pt. We e x ­
press the time required for the motion in terms of the distance-
covered and the velocity: t = s/v. Then y\qm = Ps/v, whence
Tlq m v
s = p~-
In order to determine the mass of the petrol, we express
it in terms of density and volume (m = pU) and substitute
44 Ch. I. Fundamentals of Molecular Physics

it into the expression for the distance:


W Vv
P '
Substituting the numerical values, we obtain
0.2 X 4.6 X 10’ J/kg X 700 kg/m3 x 3.2 X 10"3 in3 X 1G.1 m/s
'S— 3.31 X 103 W
= 105 in = 100 km.
Answer. There is enough petrol to cover 100 km.
Problem 24. A freely falling steel ball hits the ground with
a velocity of 41 m/s. After the impact it bounces to a height
of 1.6 m. Determine the change in the temperature of the
ball during the impact, assuming that only the internal ener­
gy of the ball changed as a result of the impact against the
ground.
Given: v = 41 m/s is the velocity of the ball when it hits
the ground, and k = 1.6 m is the height to which the ball
rises after the impact. From tables, we obtain the specific
heat of steel, c = 460 J/(kg-K), and the free fall acceler­
ation g = 9.8 m/s2.
Find: the change in the ball’s temperature, AT, as a re­
sult of impact.
Solution. The kinetic energy acquired by the ball by the
moment of impact is E k = mv2/2. A fraction Q of this ener­
gy is spent on increasing the internal energy of the ball,
i.e. on heating it, while the remaining energy is spent on
the ascent, i.e. converted into the potential energy of the
ball Ep = mgh. According to the energy conservation law,
we can write £ k = Q -f Ep.
Since Q = c m A T , we can write cm AT -{-mgh, or
—mgh —cm AT. We eliminate m from both sides:
^2
—g---- gh = cAT, and solve this equation for AT:
-2 gh
AT = 2c
a t _ ( 4 1 m/s)'2 — 2 x 9.8 m /s2 X 1.6 m . q
K.
~~ 2 x 460 J / ( k g K) ~
Answer. The temperature of the ball has increased through
about 1.8 K.
§ 4 The Change in the Internal Energy 45

Problem 25. The volume of 80 g of oxygen at an initial


temperature of 300 K increases by a factor of 1.5 as a result
of isobaric expansion. Determine the amount of heat spent
heating the oxygen, the work done during its expansion,
and the change in its internal energy.
Given: m = 80 g = 0.08 kg is the mass of the oxygen,
7, = 300 K is its initial temperature, V2 = 1.5F, is the
final volume of the gas. From tables, we find the molar
mass of oxygen, M = 32 x 10-3 kg/mol, the molar gas con­
stant i? = 8.31 J/(mol-K), and the specific heat of oxy­
gen at constant pressure cp = 0.92 X 103 J/(kg-K).
Find: the amount of spent heat Q, the work A done during
the isobaric expansion, and the change in the internal ener­
gy, AU, of the gas.
Solution. The amount of heat required to heat the gas is
Q = Cj,m AT,
where AT = T2 — Tv
The temperature T2 can be found from Charles’ law, which
governs the isobaric process: V2IVX = T2/ T,. Since V2 =
I. 5Vj by hypothesis, T2 = 1.5 T1 = 1.5 X 300 K =
450 K.
After substituting the numerical values, we obtain
Q = 0.92 x 103 J/(kg-K) x 0.08 kg x 150 K
= 11 040 J = 11.04 kJ.
In an isobaric process, A = p AV. Applying the equation
of state, we can write
A = p AV = — R AT,
1 M

A = 32 x^io-1‘kg/mol * 8.31 J/ (mol • K) x 150 K


= 3116 J = 3.12 kJ.
The first law of thermodynamics for isobaric processes
has the form Q = AU + A, and hence
AU = Q - A, AU = 11.04 kJ - 3.12 kJ = 7.92 kJ.
Answer. The amount of heat spent heating oxygen is
I I . 04 kJ, 3.12 kJ is spent on the expansion work, and
7.92 kJ on increasing the internal energy.
46 Ch. I. Fundamentals of Molecular Physics

Questions and Problems

4.1. How much energy is required to raise the tempera­


ture of 1 kg of tin through I K ?
4.2. The temperature of a 1-kg copper weight has
decreased from 293 K to 19°C. What is the decrease in its
internal energy due to cooling?
4.3. Two metal bars, one aluminium and one nickel,
with the same mass, have their temperatures lowered through
1 K. Which bar liberated a larger amount of heat?
What is the ratio of the amounts of liberated heat?
4.4. 1 kg of water and 1 kg of steel are heated through
1 K. What is the change in their internal energy?
4.5. Three cylinders having the same volume are made of
lead, copper, and aluminium. Which of them has the largest
heat capacity?
4.6. A copper weight and an iron weight of the same mass
were dropped from the same height to the ground. Which of
the weights had the higher temperature after the impact?
Does the answer depend on the mass of the weights?
4.7. Gases have larger specific heats at constant pressure
than at constant volume. How can this be explained?
4.8. How much energy will be liberated as a result of the
complete combustion of 1 kg of Donetsk coal?
4.9. How much fuel oil has to be burnt to obtain 4 X
107 J of heat?
4.10. How much heat will be liberated as a result of the
complete combustion of 5 m3 of natural gas?
4.11. How much heat must be spent in heating a copper
plate of mass 180 g through 15°C?
4.12 . 200 1 of water are drawn at 283 K for a bath. Hqw
much boiling water has to be added to elevate the water
temperature to 37°C?
4.13. To fill an aquarium, 20 1 of water at 15°C are
mixed with 2 1 of water at 70°C. Determine the water
temperature in the aquarium.
4.14. What will be the change in the internal energy of
1 1 of mercury as a result of heating it from 283 K to 50°C?
The change in the mercury’s density with temperature
should be neglected.
4.15. 8 1 of water at 20°C were poured into an aluminium
§ 4. The Change, in the Internal Energy 47

kettle whose mass is 1.5 kg. How much heat is required to


bring the water to boil?
4.16. 240 g of water at 288 K were poured into an alu­
minium calorimeter of mass 40 g. After a lead bar having a
mass of 100 g and heated to 100°C had been placed into the
calorimeter with water, a temperature of 289 K was estab­
lished. Write the heat balance equation and determine the
specific heat of lead.
4.17. In order to cool a copper component having a tem ­
perature of 373 K, it was placed into 420 g of water at a
temperature of 15°C. Determine the mass of the componenl
if the water gets heated to 18°C as a result of the heat trans­
fer.
4.18. Calculate the specific heat of brass if 334.4 J of
heat are required to heat a brass weight having a mass of
200 g from 285 K to 289.4 K.
4.19. The amount of heat required to raise the tempera­
ture of 200 g of mercury through 58.8°C is the same as that
required to raise the temperature of 50 g of water through
7°C. Using these data, determine the specific heat of mer­
cury.
4.20. Determine the temperature of a furnace in which
0.5 t of steel was hardened if 175 MJ of heat were spent to
heat the steel from 20°C to the hardening temperature.
4.21. A steel component having a mass of 0.3 kg was
heated to a high temperature and then hardened in 3 kg
of machine oil having a temperature of 283 K. Determine
the initial temperature of the component if its final tem ­
perature at thermal equilibrium was 303 K.
4.22. A metal cylinder having a mass of 146 g and heated
to 100°C is immersed in a brass calorimeter having a mass of
128 g and containing 240 g of water at 8.5°C. The tempera­
ture established as a result of heat transfer was 283 K . De­
termine the specific heat of the metal of which the cylinder
is made and identify the metal.
4.23. To heat 3 1 of water from 20 to 100°C on a gas
burner, 0.06 m3 of natural gas was burnt. Determine the
efficiency of the burner.
4.24. Determine the amount of coke required to heat
1.5 t of scrap iron from 20°C to its melting point, if the
efficiency of the cupola is 60%.
4.25. A bullet having a mass of 9 g and shot from a gun
48__________ Ch. I. Fundamentals of Molecular Physics

acquires a velocity of 800 m/s. Determine the mass of the


powder charge if the efficiency of the shot is 24%.
4.26. What is the change in the temperature of 2.0 m3 of
water in a boiler if 25 kg of Ekibastuz coal have been
burnt in the furnace whose efficiency is 50 %?
4.27. A tumbler contains 250 g of water at 80°G. What
will the decrease in the water temperature be if a silver
spoon having a mass of 50 g and a temperature of 293 K
is immersed in it?
4.28. A blast furnace consumes 2200 m3 of air per min­
ute. The air is heated in Cowper stoves by burning the blast­
furnace gas. Determine the volume of the gas burnt daily to
heat air from 273 K to 1200CC if the energy losses amount
to 30%.
4.29. A train having a mass of 2 X 10° kg and moving at
a velocity of 54 km/h comes to a halt. How much heat is
liberated in the brakes?
4.30. A lead bullet flies at a velocity of 300 m/s. What
will the change in its temperature be when it is stopped ab­
ruptly? Assume that 50% of its energy is spent heating the
bullet.
4.31. To what height should it be possible to lift a load
whose mass is 0.5 t if the entire energy given away by a
brass weight of 10 kg upon cooling by 100 K were spent for
lifting?
4.32. What will be the change in the temperature of wa­
ter falling from 120 m if 60% of its potential energy is spent
to heat it.
4.33. Determine the change in the temperature of water
falling from 96 m onto the blades of the turbine in the Bratsk
hydroelectric plant, assuming that 50% of the energy of the
falling water is spent on increasing the water’s internal
energy.
4.34. The striker of a hammer whose mass is 104 kg freely
falls from a height of 2.5 m onto a forged iron piece whose
mass is 200 kg. How many blows does the hammer strike if
the forged piece is heated through 20 K? Assume that 30%
of the energy of the hammer is spent on heating.
4.35. A worker pins together wooden struts by butting a
500-g iron nail into them with a 3-kg hammer moving at
12 m/s before the impact. Assuming that the entire energy
§ 4. The Change in the Internal Energy 49

of the hammer is spent on heating the nail, determine the


change in its temperature after 20 blows.
4.36. Two identical copper balls have received the same
amount of energy, as a result of which the first ball, remain­
ing at rest, is heated through 40 K, and the second is set in
motion without heating. Determine the velocity of the
second ball.
4.37. A lead bullet passes through a wooden wall so that
its velocity is 400 m/s at the moment of impact against the
wall and 100 m/s when it emerges from it. Determine the
change in the temperature of the bullet assuming that 40%
of the mechanical energy spent by the bullet piercing the
wall is converted into heat.
4.38. Determine the efficiency of a tractor engine which
consumes 292 g of diesel fuel per kilowatt per hour.
4.39. The engine of a crawler tractor develops a power of
73.6 kW by consuming 285 g of diesel fuel per kilowatt per
hour. Determine the efficiency of the engine.
4.40. A locomotive driven by an internal combustion en­
gine has an efficiency of 25% at a power of 3 MW. Deter­
mine the consumption of the diesel fuel per hour during
operation at maximum power.
4.41. Modern cars consume on the average 330 g of petrol
per hour per kilowatt of developed power. Determine the ef­
ficiency of a car engine.
4.42. Determine the power developed by the engine of a
“Zaporozhets" car if it consumes 74 g of petrol per kilometre
ill a velocity of 60 km/h. The efficiency of the engine is 30%.
4.43. The capacity of the fuel tank of a car is 60 1. How
fur can the car go on the fuel contained in a full tank at a
constant speed if the mass of the loaded car is 1800 kg and
Ilie efficiency of the engine is 20 %? The drag coefficient is
0.04.
4.44. A volume of a gas has increased by 0.02 m3 as a re­
sult of heating, while its internal energy has increased by
1280 J. How much heat was supplied to the gas if the process
occurred at a constant pressure of 1.5 X 106 Pa?
4.45. 3 m3 of air is stored under a pressure of 2 X 10s Pa
at 0°C. Determine the work done by the air during isobaric
heating through 12 K.
4.46. How much work is done to heat 160 g of oxygen
through 20 K at constant pressure?
ns3n
50 Ch. I. Fundamentals of Molecular Physics

4.47. 580 g of air is heated isobarically through 10 K.


How much heat is supplied to the air? What work is done in
the process?
4.48. 2 kg of air are contained in a cylinder with a piston
at 289 K. What work will be done by it during isobaric
heating to 373 K?
4.49. 3.47 kg of a gas does 144 kJ of work when isobari­
cally heated through 159 K. Determine the molar mass of
the gas and identify it.
4.50. 22 g of carbon dioxide are contained in a cylinder
under a heavy piston. What work is done by the gas as a
result of heating it from 17 to 117°C?
4.51. What work is done by an ideal gas filling a rubber
balloon when heated from 10 to 70°C? The initial volume of
the balloon is 5 1 and the atmospheric pressure is 105 Pa.
The elasticity of the balloon should be neglected.
4.52. A gas occupying a volume of 30 1 at a pressure of
1.2 X 105 Pa is isobarically heated from 300 to 450 K. De­
termine the work done by the gas.
4.53. What work is done by 1 mol of an ideal gas when
heated isobarically through I K ? Does this work depend on
the pressure and initial temperature?
4.54. Oxygen is contained in a cylinder under a piston.
Determine the mass of the oxygen if the work done to heat it
from 273 to 473 K is 16 kJ. Friction should be neglected.
4.55. Can the temperature of a gas be changed without
any heat exchange with the environment?
4.56. The volume of a gas can be reduced either by isother­
mal or by adiabatic compression. In which case will the
change in pressure be higher?§

§ 5. PROPERTIES OF VAPOURS

Basic Concepts and Formulas


The conversion of a liquid to vapour is known as evapora­
tion, and the reverse process is called condensation. Vapour
is formed as the result of either the evaporation or boiling of
a liquid. During evaporation, vapour is only formed at the
free surface of the liquid, which is possible at any temper­
ature. During boiling, vapour is formed in the bulk of the
liquid (vaporization).
§ 5. Properties of Vapours 51

Every liquid boils at a certain temperature known as its


boiling point. Evapbration is always accompanied by an
absorption of energy. Vaporization during boiling is accom­
panied by absorption of energy from outside. Evaporation
in curs due to a decrease in the internal energy of the evapo­
rating liquid (which is cooled thereby).
The specific latent heat of vaporization r is the amount of
boat required for the vaporization of 1 kg of a liquid at a
constant temperature (boiling point):

Q = rm, r = Q/rn.

The specific latent heat of vaporization is measured in


joules per kilogram (J/kg).
The specific latent heat of vaporization depends on the
pressure and temperature. It decreases with increasing pres­
sure.
The problems on vaporization are solved by using the heat
balance equation. Since vaporization during boiling occurs
at a certain temperature, while writing the heat balance
equation one should take into account not only Q = rm,
but also Qy = cm (T b — Tj), where T b is the boiling point
of a liquid.
One should distinguish between the processes of evapora­
tion and condensation: heat is absorbed in the former case
and liberated in the latter case.
Special attention should be paid to the physical meaning
of the specific heat of vaporization. For example, the specific
latent heat of vaporization of ammonia is 1.37 X 10® J/kg.
This means that the conversion of 1 kg of ammonia into va­
pour at the boiling point (—33.4°C) requires 1.37 X 10® J
of energy (the same amount of energy is liberated during con­
densation). If the temperature of ammonia is below the boil­
ing point, an additional amount of heat is required to heat
the ammonia to this temperature.
Vapour is called saturated if its pressure and density are
tbe maximum possible at a given temperature. Unsaturated
vapours have properties close to those of gases and hence
obey all the laws for ideal gases.
An unsaturated vapour can be converted into a saturated
vapour either by reducing its volume or by decreasing the
temperature.
52 Ch. I. Fundamentals of Molecular Physics

The humidity of air is characterized by the presence of


vapour in the atmosphere. The absolute humidity pa is a
quantity equal to the density of the water vapour in air, or
to its pressure p a. The absolute humidity is measured in
kilograms per cubic metre (kg/m3).
The relative humidity B is equal to the ratio of the abso­
lute humidity pa (or the pressure pa of the water vapour in
air) to the density ps (or pressure p s) of the saturated vapour
at a given temperature:

fl = -H“-100%, or B = 100%.
Ps Ps
The values of density ps or pressure p s of saturated vapours
are given in Table 6.
When the temperature of air drops to the dew point, the
relative humidity becomes 100%.

Worked Problems
Problem 26. How much heat is required to raise the tem­
perature of 200 g of water from 10°C to the boiling point and
to vaporize 10% of it? Assume that there are no energy los­
ses.
Given: mw = 0.2 kg is the mass of the cold water, t =
10°C, or T = 283 K, is the temperature of the cold water,
msteam — 0.1 mw is the mass of steam. From tables, we
find the specific heat of water, cw = 4187 J/(kg-K) ~
4190 J/(kg-K), the boiling point of water, t b = 100°C, or
T h = 373 K, and the specific latent heat of vaporization of
water, r = 2.26 x 10a J/kg.
Find: the amount of heat Q (spent energy).
Solution. Since the water temperature is below the boiling
point, it must be heated from T to T b, for which the amount
Qx = cwmw (7’b — T) of heat is required. The amount of
heat required to vaporize the water is Q2 = mstfamr, or
Q2 = 0.1 mwr.
The process is known to occur without energy losses.
Consequently, the total amount of heat that must be spent is
Q — Qi + (?2i
Qx = 4190 J/(kg-K) x 0.2 kg x 90 K = 75420 J
= 75.42 kJ,
§ 5. Properties of Vapours 53

Q„ = 0.1 x 0.2 kg x 2.26 x 106 J/kg = 4.52 x 104 J


= 45.2 kJ,
Q = 75.42 kJ + 45.2 kJ = 120.62 kJ.
This process can be represented graphically (Fig. 9). We
shall plot the amount of spent heat along the abscissa axis.
For the sake of simplicity, we
round off the values of Qx and
Q2 to integers: 75 and 45 kJ. The
temperature in degrees Celsius
(this is more convenient for cho­
osing a scale) is plotted along the
ordinate axis. When analyzing
the graph, we pay attention to
the fact that vaporization occurs
at a constant temperature (in this U.kJ
case, at 100°C). Consequently, the
water must be heated to this tem­ Fijr. 9
perature. The intercepts OA and
AB on the abscissa correspond to the values Qx and Q2.
Answer. About 121 kJ of energy is required to heat the
water and vaporize part of it.
Problem 27. How much charcoal must be burnt in order to
heat 6 t of water taken at 283 K to boiling and vaporize 1 t
of it? The efficiency of the boiler is 70%.
Given: mv = 6 X 103 kg is the mass of water in the boiler,
T = 283 K is the initial temperature of the water, T b =
373 K is the boiling point of water, ms(eam = 103 kg is
the mass of the steam, and q = 70% = 0.7 is the efficiency
of the boiler. From tables, we take the specific heat of wa­
ter, c = 4187 J/(kg-K) ~ 4190 J/(kg-K), the specific latent
heat of vaporization of water, r = 2.26 x 10a J/kg, and
specific heat of combustion of charcoal, q = 3.1 X 107 J/kg.
Find: the mass mc of charcoal.
Solution. The energy Q = qmc liberated during the com­
bustion of the charcoal is spent on heating the water to boil­
ing point, = cmw (Tb — T), and on vaporizing 1 t of
the water, Q2 = rmsteam.
From the energy conservation law and given that only
70% of energy liberated by burning the charcoal is spent on
heating and vaporizing the water, we write the heat balance
equation q@ = Qx + Q2, or qqme = cmw (Tb — T) +
54 Ch. I. Fundamentals of Molecular Physics

cmw (T’b — r ) ~f~rm steam


g tc& D11 W hG n C0 TYirQ
^19
Substituting the numerical values, we obtain
_ 4190 J/ (kg-K) X 6 X 103 kgx90 K+ 2.26 X 10" J/kg X 10® kg
0.7 X 3.1 X 107 J/kg ~
= 208.
Answer. About 208 kg of charcoal are spent.
Problem 28. 200 kg of steam at a temperature of 373 K are
passed through 4 t of water at a temperature of 293 K. To
what temperature will the water be heated? Energy losses
should be neglected. Graph the function t = / (iQ).
Given: mw = 4 x 103 kg is the mass of the water, TV =
293 K is the temperature of the water, /nsteam = 200 kg is
the mass of the steam, T BUam = 373 K is the temperature of
the steam. From tables, we take the specific heat of water,
cw = 4187 J/(kg-K) ~ 4190 J/(kg-K), and the specific la­
tent heat of vaporization of water, r = 2.26 X 10® J/kg.
Find: the temperature 9 at the end of the process.
Solution. We have here a heat exchange: steam at its boil
ing point (which equals its condensation temperature) gives
to water the amount of heat Qj = rmBfeam and is converted
into water at the same temperature (during condensation, as
in boiling, the temperature remains constant). The water
obtained from the steam is cooled from Ft, to 0 , liberating
the amount of heat Q2 = cwmsteam (Tsteam — 0 ). Since the
final temperature 0 is the same for all components, the in­
ternal energy of the cold water increases by Q = cwmw X
(0 - TV).
According to the energy conservation law, we can write
“t” Qt = Q» or rffig|eam -f cwm6[eam (7Veam 0)
= Cy/Wiyf ( 0 TV).
We transform the heat balance equation removing the pa­
rentheses and gathering the terms containing the unknown
temperature on the right-hand side:
r u isteam ^ w^-st e a m T steam V C-^TTlw T w = Cwm w0
4" cwTOsteam® ' (^w^w V ^w^steam)
whence
q __ rTO8team~f~cw (mstenm^'steam~4~ w)
Cyf ('nw ~f'm$team)
§ 5. Properties of Vapours 55

S u b s titu tin g th e num erical v alu es, we d eterm in e 8 :


2.26x10° J/kg x 200 kg
— 4190 J/(kg.K)(4xl03 kg+200 kg)
4190 J/(kg-K) (200 kg x 373 K+ 4 x 103 kg x 293 K) . . , n
' 4190 J/(kg-K)(4xl03 kg+ 200 kg) ~
or 8 = 49°C.
Figure 10 shows the temperature versus the amount of
heat, t = / (@). The temperature remains unchanged over
segment AB (the evolution of heat during condensation
occurs due to a decrease in the poten­
tial energy of interaction between
the molecules). The temperature of
condensate falls from Tsteam to 0
over segment BC (cooling the con­
densate liberates heat). The temper­
ature of the cold water increases
from T to 0 over segment CD (the
process involves the absorption
of heat).
Answer. The final temperature 0
is approximately 49°C.
Problem 29. 1.5 1 of water at 20°C are poured into an alu­
minium pot whose mass is 600 g and put on an electric hot
plate whose efficiency is 75%. In 35 min the water boils and
20% of it is converted into steam. What is the power of the
hot plate?
Given: wia = 0.6 kg is the mass of the aluminium pot,
7 = 1,5 1 = 1.5 x 10~3 m3 is the volume of the water,
t0 = 20°C is the initial temperature of the water and the
pot, r) = 75% = 0.75 is the efficiency of the hot plate,
t = 35 X 60 s is the duration of the process, and msteam =
0.2mw is the mass of the steam formed. From tables, we
lake the boiling point of water, t b = 100°C, the specific
heat of aluminium, ca = 880 J/(kg-K), the density of water,
o = 103 kg/m3, the specific heat of water, cw ~ 4190
J/(kg*K), and the specific latent heat of vaporization for
water, r = 2.26 x 106 J/kg.
Find: the power P of the hot plate.
Solution. The amount of heat required to heat the water in
the pot and to convert part of it into steam (which w ill be
56 Ch. I. Fundamentals of Molecular Physics

regarded as the useful heat @usefui) *s


^ u s e fu l = c a m a (^ b ^ o ) “ 1“ cwmw ( ^ b ^ o ) “ I- ^ s t e a m * " *
The amount of heat liberated by the hot plate will be re­
garded as the spent heat @8pent.
Proceeding from the energy conservation law and taking
into account the efficiency of the hot plate, we can write
U nspent = (^useful- H e n c e R epent = ^ u s e fu l^ 1! ’
The required power of the hot plate can be determined by
dividing the spent energy by the time:
n _ (eama cwmw) Ob—*o)-j-0-2mwr
“ Tlf
The unknown mass of the water can be determined from
the formula for density: p = mw/V :
mw = pP = 103 kg/m3 x 1.5 x 10-3 m3 = 1.5 kg.
Substituting the numerical values, we obtain
p _ [880 J/(kg-K)X 0.6 kg + 4190 J/(kg K) x 1.5 kg] X 80 K.
r 0.75 x 35 x 60 s
i 0 . 2 x 1 . 5 kg x 2 . 26 x 10a J/kg ^ t c a \ y
-1" 0.75 X 35 x 60s ~ W
Answer. The power of the hot plate is approximately
780 W.
Problem 30. The relative humidity of air in a 5 X 4 X
3 m8 room at 20°C is 70%. Determine the dew point. How
much moisture will be liberated from the air in the form of
dew if its temperature is reduced to 11°C? What will be the
relative humidity of the air?
Given: V = 60 m3 is the volume of the air in the room,
= 20°C is the initial temperature of the air, B 1 = 70% =
0.7 is the relative humidity of the air, and t2 = 11°C is the
final temperature of the air. From tables, we take the satu­
rated vapour density pfl = 17.3 x 10~3 kg/m3 for tx, and
the saturated vapour density pf2 = 10 x 10~3 kg/m3 for t2.
Find: the dew point td, the mass Am. of vapour that has
condensed, and the relative humidity of the air, B2, after
the dew has precipitated.
Solution. The temperature at which the water vapour in
the air becomes saturated is known as the dew point. In or­
der ‘o determine this temperature, we must know the abso-
§ 5. Properties of Vapours 57

lute humidity of the air, pa. Using the formula B t = pa/psl


for relative humidity, we obtain
(.:l = 5 lPsl = 0.7 x 17.3 X 10~3 kg/m3 = 12.1 x 10-3 kg/m3.
Using Table 6, we find that a vapour having a density of
12.1 X 10“3 kg/m3 is saturated at 14°C. Consequently, the
dew point is 14°C.
In order to answer the second question, we must use the
same table. The saturated vapour density at 11°C, i.e. the
maximum density, is p s2 = 10 X 10“3 kg/m3. Before the
dew precipitates, the mass of the air in the room is m1 =
paV, and after that, m2 = ps2V. Consequently, the mass of
the vapour precipitated in the form of dew is
Am = m1 — m2, Am = paV — pS2V = V (pa — ps2),
Am = 60 m3 (12.1 X 10"3 kg/m3 — 10 X 10‘3 kg/m3)
= 126 x 10-3 kg.
Since the absolute humidity at 11°C is the maximum ad­
missible density at this temperature, the relative humidity
is B 2 = 100%.
Answer. The dew point is 14°C, the mass of the condensed
vapour is 126 g, and the relative humidity is 100%.
Problem 31. The relative humidity of air at a temperature
of 16°C is 54%. What is the reading of the wet-bulb thermo­
meter of a psychrometer? What is the absolute humidity of
the air?
Given: B = 54% = 0.54 is the relative humidity of the
air and t = 16°C is the air temperature. From tables, we
take psl6 = 13.6 X 10~3 kg/m3.
Find: the reading of the wet-bulb thermometer and the
absolute humidity of the air, pa.
Solution. We shall use Table 20. In the first column, we
find 16°C (the reading of the dry-bulb thermometer). From
the same row, we take a relative humidity of 54%. It lies
in the column for which the temperature difference between
the readings of the dry-bulb and the wet-bulb thermometers
is 5°C. Consequently, t — tw = 5°C, whence
tw =. t - 5°C, *w = 16°C - 5°C = 11°C.
In order to find the absolute humidity, we write B =
Pa/Psia from which
Pa = 7?psl6,
pa = 0.54 X 13.6 X 10~3 k g/m 3 = 7.3 X 10~3 kg/m 3.
58 Ch. I. Fundamentals of Molecular Physics

Answer. The reading of the wet-bulb thermometer is 11°C


and the absolute humidity is approximately 7.3 X 10-3 kg/m3.
Problem 32. At a temperature of 30°C, the partial pressure
of water vapour in air is 4.1 kPa. Determine the absolute
humidity of the air.
Given: T = 303 K is the air temperature and p = 4.1
kPa = 4.1 X 103 Pa is the partial pressure of the water va­
pour. From tables, we take the molar mass of water vapour,
M = 18 x 10-3 kg/mol, and the molar gas constant R =
8.31 J/(mol- K).
Find: the absolute humidity of the air, pa.
Solution. Since unsaturated vapours obey the gas laws,
we can use the equation of state pV = ^ R T to determine
the density of the water vapour in air. From this equation,
we express the ratio of the vapour mass to its volume, i.e.
the absolute humidity of the air:
m ____ pM
- y " — Pa - -ftf-.

Substituting the numerical values, we obtain


4.1 X 103 Pa X 18 X 10-3 kg/mol
Pa= ' = 0.029 kg/in3.
8.31 J/(mol K) 303 K
Answer. 1 m3 of air contains about 29 g of water vapour.

Questions and Problems


5.1. Can water be made boil without heating it?
5.2. Equal masses of water and steam are at 100°C. Are
their internal energies equal?
5.3. In the south, drinking water in summer is stored in
vessels made of porous clay. Why?
5.4. At a pressure below 0.1 MPa (lower than atmospher­
ic pressure), water boils at a temperature lower than 100°C.
Explain why the steam temperature in a boiler, where a
pressure gauge indicates a pressure of 0.07 MPa, is higher
than 100°C.
5.5. A hermetically sealed vessel contains water and wa­
ter vapour. How will the number density of the water vapour
molecules change as a result of heating?
§ 5. Properties of Vapours 5!

5.6. Why does a mist appear in lowlands after a hot sum­


mer day?
5.7. When petroleum is distilled, first petrol is liberated,
followed by naphtha, kerosene, and other components. How
can this be explained?
5.8. Which puts out a fire more quickly, boiling or cold
water?
5.9. Is the temperature of the boiling water in a deep ves­
sel the same at the surface and at the bottom?
5.10. When a gas is released from a cylinder, dew or even
frost is formed at the valve. What is the reason behind this
phenomenon?
5.11. It is easier to tolerate hot weather in Central Asia
than at medium latitudes (at the same air temperature).
Why?
5.12. Water and ether have different specific latent heats
of vaporization. Is this always true?
5.13. How much heat is liberated during the condensation
of 1 g of petrol vapour having a temperature of 150°C?
5.14. In which case is more energy liberated: during the
condensation of 1 kg of water vapour or 1 kg of mercury
vapour? What is the ratio of the amounts of heat? The va­
pours are both at their boiling points.
5.15. How much heat is required to vaporize 5 kg of wa­
ter at 373 K? How much extra heat is needed if the tem­
perature of the water is 0°C?
5.16. How much heat is required to vaporize 10 g of
turpentine having a temperature of 100°C?
5.17. 100 g of steam at 100°C condense into water at 20°C.
How much heat is liberated in the process?
5.18. 9.9 X 105 J of heat are spent to heat 2.24 1 of wa­
ter taken at 19°C. The water is heated to 100°C and some of
it is converted into steam. Determine the mass of the steam.
5.19. What temperature becomes established in a bath
containing 80 1 of water at 20°C after 2.2 kg of steam at
373 K is added? The energy taken to heat the bath itself
should be neglected.
5.20. Stripped steam at 100°C is mixed with 2 t of water
whose temperature is 293 K. How much steam is required
to raise the temperature of the water to 309 K?
5.21. 0.3 kg of steam at 100°C are mixed with 6 kg of
water. After the steam condenses, the temperature of the
00 Ch. I. Fundamentals of Molecular Physics

water rises to 40°C. Determine the initial temperature of


the water.
5.22. 150 g of steam at 373 K are mixed with 1.65 1 of
water whose temperature is 20°C. Assuming that heat ex­
change occurs without energy dissipation, determine the
final temperature.
5.23. A copper body at 993 K is immersed in 1.75 kg of
water at 291 K. As a result, the water is heated to 100°C,
and 75 g of it are vaporized. Determine the mass of the
body, neglecting energy losses. Plot the graph of T = / ((?).
5.24. The temperature in a refrigerator is maintained by
vaporizing a refrigerant freon-12. What will be the change
in the internal energy of the air in the refrigerator chamber
if 50 g of freon-12 are vaporized? The freon is at its boiling
point.
5.25. 21 g of dry water vapour at 100°C is let into a cop­
per calorimeter having a mass of 200 g and containing 400 g
of water at 283 K. As a result, the temperature of the water
in the calorimeter rises to 40°C. Determine the specific la­
tent heat of vaporization of the water and compare the result
with the tabulated value. Calculate the absolute and rela­
tive errors of measurement.
5.26. A 5-kg iron weight at 500°C is placed in a boiler
containing 101 of water at 20°C. Some of the water is vapor­
ized, and the final temperature in the boiler is 25°C. The
heat spent during the heat exchange to heat the boiler is
21.85 kJ. Determine the mass of the steam formed.
5.27. A 3-kg steel bar at 450°C is immersed in a vessel
containing 3 1 of water at 20°C. As a result, 70 g of the water
are vaporized, and the temperature in the vessel becomes
50°C. Determine the heat losses during the heat exchange.
5.28. 2 1 of water are heated in a 600-g aluminium teapot
from 20 to 100°C. 50 g of the water are converted into steam.
How much natural gas was burnt if the burner efficiency is
60%?
5.29. 1.2 1 of water are heated from 283 to 373 K on an
electric hot plate. 3% of the water is converted into steam.
How long does the heating last if the power of the hot plate
is 800 W and its efficiency is 65 %?
5.30. 48 1 of water at 277 K were poured in a still. Deter­
mine the amount of wood burnt in the furnace to obtain 20 1
of distilled water. The efficiency ot the still is 15%.
§ 5. Properties of Vapours HI

5.31. Water boils at 470.4 K under a pressure of 1.47 MPa


(see Table 8). How much Donetsk coal must be burnt in the
furnace of a boiler to obtain 50 kg of steam under these
conditions? The initial temperature of the water is 10°C,
and the efficiency of the boiler is 80%.
5.32. A 1.2-kg aluminium pot containing 2 1 of water at
15°C is heated on a gas burner. The water in the pot is
heated to 373 K, and 200 g of it are vaporized. What is the
efficiency of the burner if 0.1 m3 of natural gas was burnt?
5.33. 1 m3 of air at 15°C contains 10 g of water vapour.
Determine the absolute and relative humidities of the air.
5.34. The temperature of air is 30°C, the relative humid-
ity is 64%. Determine the absolute humidity and the dew
point.
5.35. The relative humidity of air at 25°C is 75%. How
much water vapour is contained in a cubic metre of the air?
5.36. The absolute humidity of air at 5°C is 5.2 X
10~3 kg/m3. To what temperature should this air be cooled
for dew to start to precipitate?
5.37. The relative humidity of air at 18°C is 50% . At what
temperature will dew precipitate?
5.38. How much water vapour is contained in 1 1 of air
at 17°C if the dew point is 10°C? Determine the relative
humidity of the air.
5.39. The relative humidity of air in a room at 17°C is
70%. To what temperature has the window glass been
cooled if it is covered with moisture?
5.40. The temperature of air is 22°C and its dew point is
10°C. Determine the absolute and relative humidities of
the air.
5.41. In which case will moisture be felt more: if 1 m3
of air contains 10 g of water vapour at 25°C or 3.8 g at 4°C?
5.42. Fog was observed in the morning, when the air
temperature was 12°C. What was the temperature during
the night if the relative humidity of air did not change and
was 70 %?
5.43. The dry-bulb thermometer of a psychrometer
shows 21°C, while the reading of the wet-bulb thermometer
is 16°C. What is the relative humidity of air and how much
water vapour is contained in 1 m3 of the air?
5.44. The air temperature in a room is 23°C and its rela­
tive humidity is 55%. What is the reading of the wet-bulb
02 Ch. I. Fundamentals of Molecular Physics

thermometer of a psychrometer? What is the dew


point?
5.45. The relative humidity of air in the halls of a mu­
seum is 65%. The difference between the readings of the
wet-bulb and dry-bulb thermometers is 4°C. What is the
temperature in the halls? How will the relative humidity
change if the psychrometric difference decreases?
5.46. A cylinder contains air at 15°C. The relative hu­
midity of the air is 63%. After the air was dried by calci­
um chloride, the mass of the cylinder decreased by 3.243 g.
Determine the volume of the cylinder.
5.47. The relative humidity of air at 12°C is 78%. What
will the change in the relative humidity be if the tempera­
ture increases to 18°C?
5.48. The temperature in a room of size 8 X 5 X 2.5 m3
is 20°C and the relative humidity is 70%. How much water
vapour is contained in the room? How much vapour is con­
densed as a result of a temperature drop to 10°C?
5.49. The relative humidity of air in a room of volume
30 m3 is 60% at 20°C. Determine the saturated vapour pres­
sure at this temperature if the mass of water evaporated
in the room is 310 g.
5.50. The temperature of air in a room is 27°C and the
partial pressure of water vapour in it is 1.7 kPa. Determine
the absolute and relative humidities of the air.§

§ 6. PROPERTIES OF LIQUIDS

Basic Concepts and Formulas


Substances in the liquid state have constant volumes, are
fluid, and hence acquire the shape of the vessel in which
they are contained.
Since the molecules of a liquid are packed more densely
than gas molecules, the intermolecular interaction forces
are significant. The range of these forces does not exceed
10 nm.
The molecules in the surface layer of a liquid experience
the action of forces whose resultant is directed inside the
liquid. In order to bring a molecule from the bulk to the
surface of a liquid, work must be done. Therefore, the
molecules of the surface layer have excess potential energy,
§ 6. Properties of Liquids 63

which is responsible for the stretched state of the sur­


face layer.
Surface tension a is a quantity defined as the ratio of the
work required for increasing the surface area to the incre­
ment of this area:

The unit of surface tension is the joule per metre squared


(J/m2). Surface tension can also be defined as the ratio of
the force F acting in the surface (force of surface tension) to
the length I of the boundary of the liquid surface:
a = F/l.
In this case, a is expressed in newtons per metre (N/m).
A liquid is termed wetting if the forces of intermolecular
interaction between a solid and the liquid are stronger than
the forces acting between the liquid molecules. The meniscus
(curved surface of the liquid) is concave for wetting liquids,
and the wetting angle 0 (i.e. the angle between the meniscus
and the surface of the solid) is acute. For nonwetting liquids,
the meniscus is convex, and the wetting angle 6 is obtuse.
The curved surface of a liquid produces an additional
pressure (called Laplacian pressure)
2a
A rt= ± y ,
where R is the radius of the spherical surface. The additional
pressure in capillaries causes wetting liquids to rise and
nonwetting liquids to fall by height
2a
cos 0.
pgR
For complete wetting, we have 0 = 0 , cos 0 = 1, and
2a
h:
PgR '

Worked Problem*
Problem 33. A frame in the form of equilateral triangle
with sides 4 cm long is carefully placed on the surface of
water. What force keeps the frame on the surface? What
04 Ch. I. Fundamentals of Molecular Physics

force must be applied to separate the frame from the water


surface? The mass of the frame is 2 g.
Given: r = 4 X 10“2 m is the length of a side of the frame,
and m = 2 X 10~3 kg is the mass of the frame. From tables,
we take the surface tension of water, a = 0.072 N/m, and
the free fall acceleration g — 9.81 m/s2.
Find: the force F, keeping the frame on the surface and
the force F2 required to separate the frame from the water
surface.
Solution: The frame is kept on the water surface by the
force of surface tension, F1, which can be determined from
the formula a = FJl:
F, = ol,

where I is the length of the inner and outer boundaries of


the liquid surface, the length being equal to twice the per­
imeter of the triangle: 1 = 2 x 3 r. This gives
Fl = 6or, f j = 6 x 0.072 N/m x 4 x 10~2 m
= 1 .7 3 x 10~2 N.

In order to separate the frame from the water surface, \\e


must overcome the force of gravity acting on the frame in
addition to the force of surface tension, i.e.
F2 = F , + mg,
F2 = 1.73 x 10- 2 N + 2-10- 3 kg x 9.81 m/s2
= 3.7 x 10- 2 N.

Answer. The force keeping the frame on the surface of wa­


ter is 1.73 x 10"2 N. The force required to separate the
frame from the surface is 3.7 X 10-2 N.
Problem 34. The surface tension of water was determined
in a laboratory by using the drop weight method. 100 drops
were released from a burette the inner diameter of whose
opening is 1.8 mm. The mass of the droplets was 3.78 g.
Using these results, determine the surface tension of the
water and, comparing it with the tabulated value, calculate
the relative error of the measurements.
Given: d0 = 1.8 X 10“3 m is th e 1inner diameter of the
burette opening, n = 100 is the number of drops, and
§ 6. Properties of Liquids 65

m = 3.78 X 10~3 kg is the mass of 100 drops, From tables,


we take the free fall acceleration g = 9.81 m/s2 and the
tabulated value of the surface tension for
water, crt = 0.072 N/m.
Find: the surface tension a of water and
the relative error 60 of measurement.
Solution. Water drips from a burette when
its valve is not open completely. The dia­
meter dn of the neck of a drop immediately
before separation is smaller than the diam­
eter d0 of the burette opening (Fig. 11).
This ratio is usually dn/d0 = 0.9. The drop
separates only when the force of gravity
f
becomes equal to or slightly larger than
the force of surface tension: myg = al,
where ml = m/n and I = ndn = 0.9nd0. Fig. 11
Consequently,
m g
o= 0 . 9nd0n ’
3.78 X 10~3 kg x 9.8 m/s8
a = 0.9 x 3.14 x 1.8 X10-3 m x 100
= 0.0728 N/m = 7 2 .8 x lO’3 N/m.
The absolute error A„ is equal to the difference a — txt:
A„ = 72.8 x 10-3 N/m - 72.0 x 10- 3 N/m
= 0.8 x lO - 3 N/m.
The relative error is equal to the ratio of the absolute error
to the tabulated value of the surface tension:
0.8 x 10~3 N/m
^ = — 100%, 6„ = 72.0 x 10~3 N/m 100%~ 1.1%.
0t
Answer. The surface tension of water is 72.8 X 10-3 N/m,
and the relative error of measurement is about 1.1 %.
Problem 35. To what height will water rise in a
capillary whose inner diameter is 3.0 mm? What will
be the height qf mercury column in the same capillary?
The capillaries are made of glass.
Given: d = 3.0 x 10' 3 m is the inner diameter of the capil­
lary. From tables, we take the surface tension of water,
a t = 0.072 N/m, the surface tension of mercury, a 2 =
0.47 N/m, the density of water, p, = 103 kg/m3, the density
5-0530
66 Ch. I. Fundamentals of Molecular Physics

of mercury, p2 = 1.36 x 104 kg/m3, and the free fall ac­


celeration g = 9.81 m/s2.
Find: the heights hx and of the water and mercury col­
umns in the capillaries.
Solution. Water is a liquid that wets glass, and hence it
has a concave meniscus. For wetting liquids, the Laplacian
pressure is directed upwards, and equals p L = 2air for
complete wetting. The Laplacian pressure causes water to
rise in the capillary until the hydrostatic and Laplacian
pressures equalize: p L = Ph» Ph = Pgh, 2a Jr = ptgfc,, where
r = d/2. This gives
2at
*i = Pig r '
_________ 2 x 0.072 N/m__________
K = 103 kg/m3 x 9.81 m/s* x 1.5 x 10-3 m ~ 9.8 X 10~3 m.
Mercury is a liquid which does not wet glass. Therefore,
the Laplacian pressure is directed downwards, into the bulk
of the liquid. Hence the mercury is forced down the capil­
lary.
The height to which a nonwetting liquid is forced down is
determined by using the same formula
h2 = 2q2
P2gr '
h2 —
_________ 2 x 0.47 N/m_________
~ 4.7 x 10~3 m.
1.36 x 104 kg/m3 x 9.81 m/s* X1.5 X10' m
Answer. The water rises in the capillary by about 9.8 mm,
while the mercury is lowered by 4.7 mm.

Questions and Problems


6.1. Touch the surface of water between two floating
matches with a piece of soap. Repeat the experiment with a
piece of sugar. Why do the matches move apart in the first
case and approach each other in the second case? How do
sugar and soap affect the surface tension of water?
6.2. Small pellets are made by pouring molten lead into
a vessel containing water. Why do the pellets become
spherical?
6.3. A capillary tube is immersed in a vessel of hot water.
W ill the level of the water in the capillary be affected hy
a fall in the temperature of the water?
§ 6. Properties of Liquids 67

6.4. Why can oily spots not be washed away by water?


6.5. Why can tin solder copper and not aluminium?
6 .6. Why is an area to be soldered thoroughly cleaned
from grease, dirt, and oxides?
6.7. What liquid can be poured into a tumbler to above
the brim?
6.8 . Why are the foundations of buildings covered with
tar paper?
6.9. Why does caked soil during a drought dry more than
plowed soil?
6.10. It is expedient to break up the soil between rows of
crops regularly. Why is this type of cultivation sometimes
called “dry irrigation"?
6.11. A hollow metal cube with a capacity of 1 dm3 is
filled with kerosene. Determine the force of the surface
tension.
6.12. A cylindrical tumbler 9 cm in height contains
250 cm3 of milk. What is the force of surface tension?
6.13. Water and petrol are poured into a test tube with
a diameter of 1.5 cm so that the height of the petrol column
is 10 cm. Determine the force of surface
tension and the force of the pressure Scm
exerted by the petrol on the water. The
curvature of the surface of the petrol
should be neglected.
6.14. A match 4 cm long floats on
the surface of water whose temperature
is 20°C. If castor oil is poured to
one side of the match, it starts mov­
ing. Determine the force acting on
the match and the direction of its mo­ t ig- 12

tion.
6.15. A wire ring having a radius of 6.0 cm is brought in
contact with the surface of blue vitriol. The mass of the
ring is 5 g. What force should be applied to separate the
ring from the surface of the solution?
6.16. Determine the potential energy of the surface layer
of water having an area of 20 cm2.
6.17. A soap film is formed on a wire frame with a mov­
able side AB = 10 cm (Fig. 12). What work must be done to
stretch the film by moving AB by 5 cm? The friction be­
tween AB and the frame should be neglected.
5*
68 Ch. I. Fundamentals of Molecular Physics

6.18. 532 drops of castor oil were dripped from a pipette


with a tip diameter of 1.2 mm into a measuring glass. The
volume of castor oil was found to be 7 cm3. Determine the
surface tension of castor oil.
6.19. Calculate the surface tension of a liquid and identi­
fy it if a force of 0.035 N is required to separate a square
frame with a side of 8.75 cm from the liquid surface. The
mass of the frame is 2 g.
6.20. How high can water rise in a capillary tube with
an inner diameter of 10~3 m?
6.21. Determine the mass of alcohol rising in a capillary
tube immersed in a vessel containing alcohol. The inner di­
ameter of the capillary is 0.4 mm. The surface tension of
ethyl alcohol should be taken to be 0.02 N/m.
6.22. Determine the mass of mercury forced down a capil­
lary having an inner diameter of 0.1 mm and immersed in
mercury.
6.23. 100 drops of pure water and the same number of
drops of water mixed with ether are poured into two test
tubes. In which test tube will the liquid level be higher?
6.24. A liquid rises 4.25 cm up a capillary with an inner
diameter of 0.6 mm. Determine the density of the liquid
if its surface tension is 0.071 N/m.
6.25. Determine the difference between the levels of
mercury in two communicating capillaries having inner di­
ameters of 1 and 2 mm.

§ 7. PROPERTIES OF SOLIDS. DEFORMATIONS

Basic Concepts and Formulas


All solids are elastic3 and retain their volume and shape.
Solids are characterized by the long-range order in the
arrangement of the particles of which they are composed.
These particles are atoms, molecules, or ions.
The crystalline structure of a solid is a result of the or­
dered arrangement of its particles. The physical properties
of a crystal depend on the orientation of the symmetry axis
in the crystal (anisotropy).
Under the action of external forces, solids are deformed.
The deformations disappearing after the forces stop acting on*
* Amorphous bodies are treated as supercooled liauids.
§ 7. Properties of Solids. Deformations 69

a body are known as elastic. There are a large number of


different deformations, but extension (compression) and
shear are important.
Extension is characterized by an absolute deformation AI:
AI = I — l0
and a relative deformation (strain)

Mechanical stress o is defined as the ratio of the internal


force emerging in a body as a result of deformation to the
cross-sectional area of the body: o = F/S. The unit of
mechanical stress is the pascal (Pa).
Hooke’s law establishes a relation between elastic deforma­
tions and the internal forces. The mechanical stress o is
directly proportional to the strain e :
7 17 &
a = fee, or a = E -r-,
to
where E is Young’s modulus. Young’s modulus is measured
in the same units as stress, viz. in pascals.
The elastic lim it is the maximum stress emerging in
a material for which Hooke’s law remains in force.
The safety factor is defined as the ratio of the maximum
(ultimate) stress ou of a construction to the admissible
stress aad:

For an elastic deformation, the potential energy E p of


a body is equal to the work done during the deformation
(extension or compression) of a body:

EP= ~ y ~ ~ (At)2-
When a solid changes its state of aggregation (when it
melts), the separation between the particles in the crystal
lattice increases, and the lattice is destroyed. The potential
energy of the interaction between molecules (particles)
increases.
To melt 1 kg of a solid at its melting point, a certain
amount of heat X is required, which is known as the specific
70 Ch. I. Fundamentals of Molecular Physics

latent heat of fusion:


k = Q/m.
The specific latent heat of fusion is measured in joules per
kilogram (J/kg).
In order to melt a crystalline substance, heat must be spent
to heat it to its melting point and to convert it into a liquid:
Q = cm (Tm — T) -f km.

Worked Problems
Problem 36. Determine the elongation of a copper rod hav­
ing a length of 6 m and a cross-sectional area of 0.4 cm2
under the action of a force of 2 kN.
Given: I = 6 m is the length of the rod, S = 0.4 x
10-4 m* is its cross-sectional area, and F = 2 X 10s N is
the applied force. From tables, we take Young’s modulus
for copper E = 130 x 10® Pa.
Find: the elongation A/ of the rod.
Solution. We shall solve the problem using Hooke’s
law - j - = a, or -y - = - ^ j - , whence
2x 103 N X 6 m
130x10* Pa x 0.4 x 10-4 m2
~ 2.3 X 10~3 m.

Answer. The copper rod elongates by approximately


0.23 cm.
Problem 37. A chandelier of mass 250 kg is suspended on
an aluminium rod with an ultimate stress of 0.11 GPa.
What must the cross-sectional area of the rod be for the
safety factor to be 4? What is the strain in the rod?
Given: m = 250 kg is the mass of the chandelier, n = 4 is
the safety factor, and ou = 1.1 x 10® Pa is the ultimate
stress. From tables, we take the free fall acceleration g =
9.81 m/s* and Young’s modulus for aluminium, E = 7 X
1010 Pa.
Find: the cross-sectional area S of the rod and the strain e.
Solution. The deformation of the rod is caused by the force
of gravity G = mg. The cross-sectional area of the rod is
chosen depending on the mechanical stress a emerging in the
rod: a = G/S = mg/S, whence
S = mg/a.
§ 7. Properties of Solids. Deformations 71

Given the safety factor n and the ultimate stress ou, we


can determine the admissible stress: n = <ju/<t, a = a„/ra.
Finally,
S = J?Ln,
ou
250 kg x 9.81 m/s2 x 4 ^ n n jn-6 2
1.1 X 10* Pa —o-yxiu m.
The strain in the rod is
ou 1.1x10* Pa
e = nE 4 X 7 X 10l®Pa
~ 4 x 10“'*.
Answer. The cross-sectional area of the rod is 0.89 cm2
and the strain is about 4 X 10“4.
Problem 38. How much heat is required to convert 0.8 kg
of ice at —10°C into steam at ,
100°C? Plot the graph of t = / (Q). '‘
Given: m = 0.8 kg is the mass of 2
the ice, = —10°C is the initial
temperature of the ice and t2=100°C
is the temperature of the steam.
From tables, we take the melting
point for ice, t0 = 0°C, the boiling
point for water, = 100°C, the
specific heat for ice, = t0 —►
2090 J/(kg-K), the specific heat i, 0
for water, cw = 4187 J/(kg-K), the
specific latent heat of fusion for ice,
X,=3.35 X lO ^ /k g, and the spe­
cific latent heat of vaporization of water, r = 2.26 X
10# J/kg.
Find', the amount of heat Q required to convert the ice into
steam.
Solution. The required amount of heat Q is determined
by summing the amounts of heat Q = Qi + + Qi + Q 4
(Fig. 13), where is the heat required to heat the ice to tha
melting point,
Qi = cjn (t0 — *i),
<?, = 2090 J/(kg- K) x 0.8 kg x 10 K
= 16720 J = 16.72 kJ,
72 Ch. I. Fundamentals of Molecular Physics

is the amount of heat required to melt the ice,


Qz = Km, Q2 = 3.35 x 10s J/kg x 0.8 kg
= 2.6 x 10* J = 260 kJ,
Q3 is the amount of heat required to heat the water obtained
from the ice to the boiling point,
Q3 = cwm (t3 tq) ,
Q3 = 4187 J/(kg- K) x 0.8 kg x 100 K = 334960 J
~ 335 kJ,
and Qi is the amount of heat required to evaporate the water
Qt = rm, = 2.26 x 106 J/kg x 0.8 kg = 1.808
X 10® J ~ 1810 kJ.
The total amount of heat is
Q = 16.72 kJ + 260 kJ + 335 kJ + 1810 kJ
~ 2420 kJ.
Answer. In order to convert 0.8 kg of ice into steam, an
energy of 2420 kJ = 2.42 MJ is required.
Problem 39. A certain amount of steam at 100°C is intro­
duced into a vessel containing 0.5 kg of water and 20 g of
ice at 0°C. As a result, all the ice is melted, and the water is
heated to 19°C. Find the mass of the steam. The heat capac­
ity of the vessel should be neglected. Plot the graph of
* = / (Q)-
Given: mw = 0.5 kg is the mass of the water, m,
0.02 kg is the mass of the ice, t0 = 0°C is the initial tem­
perature of the water and ice, 0 = 19°G is the temperature
established in the vessel, and fstcam = 100°C is the temper­
ature of the steam. From tables, we take the specific latent
heat of vaporization (condensation) for water, r = 2.26 X
10® J/kg, the specific heat for water, c = 4187 J/(kg- K), and
the specific latent heat of fusion for ice, K = 3.35 x 105 J/kg.
Find: the mass msteaTn of the steam.
Solution. To solve the problem, we must write the heat
balance equation. The steam delivered to the vessel con­
taining water and ice condenses, liberating heat Qj =
m sieam* The water obtained from the steam is cooled
from fsieam = 100°C to 0, liberating heat Q2 = cms(eamX
(*stc«m — 9)- Thus, the heat given awayis(?glv = Qi + Qz-
§ 7. Properties of Solids. Deformations 73

The received heat @Tec is the sum of the heat spent to melt
the ice, Q3 = Xm„ and the heat spent to heat the cold water
and the water obtained from melting ice: Qt = c (ml + mw) x
(0 — t0). Consequently, Qrec = Q3 -f Qt .
According to the energy conservation law, @glv = Qrec-
Let us write the heat balance equation and determine
the mass of the steam:

* "^ stea m ^ ^ s t e a m (^steam ®) = “I” c(^ | “f" ^ w ) f® ^o) >


X.77i1+ c(m1+ mw)(0 — <„)
s,e a m r + e ((s t e a m - 0 )
3.35 x 10s J/kg x 0.02 kg + 4187 J/(kg-K) X 0.52 kgXl9K
msteam — 2.26 X 106 J/kg + 4187 J/(kg K) x 81 K
= 1.85 X lO-2 kg.
The graph of t = / ((?) is presented in Fig. 14. Over re­
g io n /, heat is liberated during the condensation of the
steam at constant temperature
(the potential energy of interaction
between the particles decreases).
In region 2, heat Qz is liberated
because the water obtained from
the steam cools (the kinetic energy
of the particles decreases). In region
3, heat Q3 is absorbed to melt the
ice (the potential energy of the in­
teraction between the particles in­
creases). In region 4, heat is ab­
sorbed by the cold water and the
ice water (the kinetic energy of the particles increases),
their temperature increasing to 0.
Answer. About 19 g of steam are required to melt the ice
and heat the water.
Problem 40. How much gray cast iron taken at 20°C can
be melted in a furnace with an efficiency of 2 0 % by burning
1.94 t of A-l grade coal?
Given: q = 20% = 0.2 is the efficiency of the furnace,
mc = 1.94 x 103 kg is the mass of coal spent, and tl = 20°C
is the initial temperature of the cast iron. From tables, we
take the specific heat of combustion of coal, q = 2.05 x
107 J/kg, the specific heat of cast iron, c = 550 J/(kg-K),
74 Ch. I. Fundamentals of Molecular Physics

the melting point for cast iron, t2 = 1150°C, and the specific
latent heat of fusion for cast iron, k = 9.7 x 104 J/kg.
Find: the mass ml of the molten cast iron.
Solution. The amount of heat Causerui required to heat the
cast iron and melt it can be determined from the formula
^useful = cmx(t2 — *i) + kmx. On the other hand, the
useful heat is just 20% of the spent heat, i.e. 20% of the
heat liberated by burning the coal: q = ^useful , where
vspent
$spent = (pUf.. Hence ^useful = q?mc-
We write the heat balance equation and determine the
mass of the cast iron:

m i[c(/2- / i ) + X] = T)qTOc, ml

TTl\ 0.2 X 2.05 x 107 J/kg X 1.94 X 103 kg , 4070 leer ~ 11 1 t


550 J/(kg-K) x 1130 K + 9.7 X 10JJ/kg

Answer. About 11.1 t of cast iron can be melted in the


furnace.
Problem 41. What must be the minimum velocity of an
iron meteorite for it to be completely evaporated in the
Earth’s atmosphere? Assume that the initial temperature of
the meteorite before it enters the atmosphere is 3 K and
that 50% of its kinetic energy is converted into internal
energy.
Given: T - 3 K is the initial temperature of the meteorite,
q = 5 0 % = 0.5. From tables, we take the specific heat of
iron in the solid state, c = 400 J/(kg-K), the melting point
of iron, Tm = 1803 K, the specific latent heat of fusion of
iron, k = 2.7 X 105 J/kg, the specific heat of iron in the
liquid state, cx = 830 J/(kg-K), the boiling point of iron,
Tb = 3323 K, and the specific latent heat of vaporization
for iron, r == 5.8 X 104 J/kg.
Find: the minimum velocity v of the meteorite at which it
enters the Earth’s atmosphere.
Solution. The meteorite is made of iron, so all extra data
needed are taken to be the tabulated values for iron.
When the meteorite enters the atmosphere, its kinetic en­
ergy is spent on heating, fusing, and evaporating it. The
problem states that these processes consume 50% of the ki-
§ 7. Properties of Solids. Deformations 75

netic energy of the meteorite. Therefore, we can write


0.5 = cm (Tm - T ) + km + cym (Tb - Tm) + rm.

Eliminating the mass of the meteorite from the equation


(we take it out of the parentheses on the right-hand side of
the equation and cancel it out), we obtain
v* = U c ( T m - T) + \ + cx (Tb - Tm) + rl.
We write the solution in the general form and then carry
out the calculations:
v = 2 ] /r c(Tm —71) + A,+ c, (Tjj — Tm) + r,
v = 2 Y (460 x 1800 -f 2.7 x 105+ 830 x 1520 + 5.8 x 104) m2/s2
~ 3100 m/s.
Answer. The minimum velocity of the meteorite must be
approximately 3.1 km/s.

Questions and Problems


7.1. Why should a tool being sharpended be periodically
cooled?
7.2. Two cubes, one cut from a single crystal and one
from glass, are placed in a vessel containing hot water.
Will they retain their shape?
7.3. The hardness of glass and tool steel is the same.
Why are cutting tools not made of glass?
7.4. Why is lead wire used for fuses rather than some
other metal?
7.5. Under what conditions can lead be melted in water?
7.6. Why does precipitation in summer have the form of
rain or hail and not snow?
7.7. Why is salt sprinkled over pavements covered with
ice in winter?
7.8. What is the diameter of a rod if the mechanical
stress caused in it by a force of 2 X 103 N is 160 MPa?
7.9. Tensile forces of 150 N each act on the two ends of an
iron wire of 2-m length and 1-mm2 cross-sectional area. De­
termine the absolute elongation and strain of the wire.
7.10. Determine the elongation of a 4-m long steel wire
76 Ch. I. Fundamentals of Molecular Physics

having a diameter of 2 mm and loaded with 70 kg. Deter­


mine the energy of elastic deformation of the wire. The mass
of the wire should be neglected.
7.11. What will be the difference in the elongation of
a wire if it is replaced by the same length of wire made of
the same material and with the same load but with twice
the diameter?
7.12. A wire having a cross-sectional area of 1 mm- and
a length of 1 m is elongated by 1 mm under a load of 200 N.
What will the elongation be for a wire made from the same
material, 3 mlong, with a cross-sectional area of 0.5 mm2,
and loaded by a force of 300 N?
7.13. A residual deformation appears in a copper wire
having a cross-sectional area of 1.5 mm2 when loaded by
a tensile force of 45 N. What is the elastic lim it of the
wire material?
7.14. A brass wire has a diameter of 1 mm and a length
of 3.6 m. Calculate Young’s modulus for brass if a 19-kg
load elongates the wire by 8 mm.
7.15. What must the minimum diameter of a steel rope
be for it to be able to hold a load of 2.45 X 104 N? The
ultimate stress for steel is 1.25 GPa.
7.16. What is the maximum load that can be borne by
a steel rope with a cross-sectional area of 12 mm2? The ulti­
mate stress should be taken as 785 MPa.
7.17. How many piles having a diameter of 10 cm each
are required to support a platform having a mass of 2 X
105 kg if the admissible compressive stress is 107 Pa?
7.18. Determine the cross-sectional area of the steel rope
of a crane uniformly lifting a 6-t load if the ultimate stress
of a material is 780 MPa and the safety factor is 10.
7.19. What must the minimum length of a steel rope sus­
pended at one end be for it to rupture at the point of sus­
pension under the force of gravity? The ultimate stress of
the rope is 320 MPa.
7.20. Under a 120-N force, a spring is elongated by 5 cm.
What is the potential energy of elastic deformation of the
spring?
7.21. What must the elongation be of an iron rod having
a length of 1.8 m, a cross-sectional area of 7.8 mm2, and a po­
tential energy of elastic deformation of 3.9 X 10~2 J?
7.22. A flask containing ice at 0°C is placed in a vessel
8 7. Properties of Solids. Deformations 77

containing water at the same temperature. W ill the ice


melt?
7.23. Why are alcohol and not mercury thermometers used
to measure temperatures in the north?
7.24. How much heat is required to melt 2.6 kg of lead
at 300 K?
7.25. How much heat must be spent to convert 125 g of
ice at 268 K into steam?
7.26. 0.15 kg of molten lead at its melting point are
poured into 300 g of water at 285 K. Determine the tempera­
ture at thermal equilibrium, neglecting heat losses.
7.27. Equal volumes of lead and tin are taken at their
melting points. What is the ratio between the amounts of
heat required to liquefy them? The temperature dependence
of their densities should be neglected.
7.28. To make shot, 50 kg of lead at its melting point are
sprayed in jets into water. Determine the final temperature
in 19 1 of water taken at 283 K. Plot the graph t = f {Q).
7.29. 58 g of molten lead at 357°C were poured into an
indentation in a piece of ice at 273 K. How much ice is
melted if the lead is cooled to 273 K? Plot the graph t — j(Q)-
7.30. An aluminium vessel whose mass is 240 g contains
360 g of water at 25°C. After 20 g of ice at its melting point
are placed in the water and melted, the temperature in the
vessel becomes 293 K. Using these data, determine the
specific latent heat of fusion for ice.
7.31. An aluminium bar at 520°C is placed on ice at
273 K. Determine the mass of the aluminium bar if 26 kg of
ice were melted to cool the bar.
7.32. In the far north, fresh water is obtained by melting
snow. How much firewood has to be burnt to melt 1500 kg of
snow at 263 K if the temperature of the obtained water is
5°C? The efficiency of the set-up is 30%.
7.33. The mass of fuel oil burnt in a smelting furnace con­
stitutes 12% of the mass of fused metal. Determine the
efficiency of the furnace in which 200 kg of liquid aluminium
are obtained in a smelting process at the melting point of
aluminium. The initial temperature of the metal is 283 K.
7.34. Determine the mass of metallurgical coke burnt to
smelt 1.5 t of scrap iron taken at 293 K. The efficiency of the
blast cupola is 60%.
7.35. A refrigerator operates for 5 h to produce ice at
78 Ch. L Fundamentals of Molecular Physics

271 K from 200 g of water taken at 15°C. Determine the


power of the refrigerator.
7.36. Calculate the efficiency of a smelting furnace in
which 1.1 t of Donetsk coal are burnt to obtain 7.2 t of white
cast iron. The initial temperature of the white cast iron
is 0°C.
7.37. In frictional welding, aluminium rods at 17°C are
pressed tightly together, and then one is rotated by a spindle.
As a result, the temperature of the aluminium where the
rods are in contact rises. What mass of the aluminium is
fused in 10 s if the force of the pressure is 1260 N at a rota­
tional speed of the spindle of 90 m/min?
7.38. A skidding car moves over a snow field. The skid­
ding consume 5 kW of power. How much snow will melt
under the wheels in 1 min if the temperature of snow is 0°C?
7.39. A lead bullet having a velocity of 450 m/s and
a temperature of 117°C strikes a steel beam. What fraction of
the bullet fuses if 40% of its kinetic energy before the im­
pact is spent on heating the bullet?
7.40. While making ice in a frige, the temperature of
water is reduced from 278 to 273 K in 5 min and it takes 1 h
20 min then for the water to be converted into ice. Using
these data, determine the specific latent heat of fusion for
ice assuming that the specific heat of water is 4190 J/(kg- K).
7.41. A 300-g lump of wetsnow is placed into 1.2 1 of water
#t 21°C. After all the snow has melted, the temperature of the
water drops to 6°C. How much water was contained in the
lump of snow? The specific heat of water should be taken
as 4190 J/(kg-K).

9 8. THERMAL EXPANSION OF BODIES

Basic Concepts and Formulas


Solids, liquids and gases all expand upon heating. Solids
expand much less than liquids and gases which expand
the most.
Solid bodies with one dominant dimension are character­
ized by the coefficient of linear expansion a.
If the length of a solid body at t0 = 0°C is l0 and be­
comes /( as a result of it being heated to a temperature t.
§ 8. Thermal Expansion o! Bodies 79

the coefficient of linear expansion is defined as


AI
a = In&t

where AI = lt — l0 and At ~ t — t0. The coefficient of


linear expansion is measured in reciprocal kelvins (K-1).
If the initial temperature is 0°C, At = t.
The length of a body at different temperatures can be de­
termined from the formula
h = (1 “1“ a 0 -

Since the value of a is very small, we can use the formula


li = h (1 + ocAt),

where At = t2 — tu and Z2 and lY are the lengths of the


body at these temperatures.
Solid three-dimensional bodies (like a cube or a sphere)
and liquids are characterized by a coefficient of volume
expansion fl:
A AE
^ V0Ai •
The coefficient of volume expansion is also measured in
reciprocal kelvins (K -1).
By analogy with linear expansion, the volume of a body
at any temperature can be expressed as follows:
Vt = V 0 (1 + P«).
For two nonzero temperatures, we can write
Vt = Vj (1 + pAt),
where At = t2 — tt.
The following relation can be established between the
coefficient of linear expansion and the coefficient of volume
expansion: p ~ 3a.
The densities of solids and liquids also change as a result
of heating. As the volume increases due to heating, the
density decreases:
o - P°
Pt 1 + fiAf •
80 Ch. I. Fundamentals of Molecular Physics

At constant pressure, the coefficients of volume expansion


for all gases are the same and equal to p = 0.00 361 K ' 1 =
273.15 ’

Worked Problems
Problem 42. A guitar is tuned in a room at 293 K , the length
of its steel string being 0.7 m. What will be the change in
the length of the string outdoors where the temperature is
263 K? What will the additional mechanical stress be?
What are the elastic force and the potential energy of the
elastically deformed string? The cross-sectional area of the
string is 0.85 mm*.
Given: lx = 0.7 m is the initial length of the string,
Tj = 293 K is the initial temperature, T2 = 263 K is the
temperature outdoors, and S = 8.5 X 10~7 m2 is the cross-
sectional area of the string. From tables, we take the coeffici­
ent of linear expansion for steel a = 1.2 x 10“5 K"1, and
Young’s modulus for steel E = 2.2 X 1011 Pa.
Find: the absolute decrease AI in the length of the string,
the additional mechanical stress a, the elastic force F, and
the potential energy Ep of elastically deformed string.
Solution. As the temperature decreases, the length of the
string decreases by Al. The value of AI can be determined
from the formula
AI = litxAT,
where A T = T2 — T1,
Al = 0.7 m x 1.2 x 10' 5 K - 1 ( - 3 0 K)
= — 2 .5 2 x l0 ~ 4m ~ — 0.252 mm.
The minus sign in front of Al in this case indicates that
the string has become shorter.
The measure of the stressed state of a deformed body is the
mechanical stress a, which can be determined using Hooke’s
law:
o = 2.2 xlO11 Pa x 2.52 x 10~4 m _ ? Q2 x 10? pa
11 0.7 m(
The elastic force F emerging during the compression
of the string is proportional to the absolute deformation:
§ 8. Thermal Expansion of Bodies 81

F= Al. Since = o, we have


*i *i
F = a S , F = 7 .9 2 x l0 7 Pa x 8 .5 x l0 ~ 7m2 ~ 67 ft'.
Knowing the elastic force and the decrease in the string
length, we can determine the energy of the elastic deforma­
tion of the string:
£ p= — E p= 6 7 N X 2 - 5 2 x l O- * m ^ 8 4 x 10- 3 J ,

Answer. As a result of the decrease tintemperature, the


string contracts by about 0.25 mm. The additional stress
emerging thereby is 7.92 x 107 Pa. The elastic force is
approximately 67 N, and the potential energy of the
elastic deformation is about 8.4 mJ.
Problem 43. A tank car contains 20 t of petrol at 298 K.
How much smaller will the volume of the petrol at the ter­
minal be if the ambient temperature is 248 K? The change
in the capacity of the tank car as a result of the change in
temperature should be neglected.
Given: 7\ = 298 K is the initial temperature of the pet­
rol, m = 20 t = 2 x 104 kg is the mass of the petrol, T2 =
248 K is the final temperature of the petrol. From tables,
we take the coefficient of volume expansion of petrol, p —
10~3 K"1, and the density of petrol at 273 K, p0 -= 7 X
102 kg/m3.
Find: the change AF in the volume of the petrol.
Solution. As the temperature falls, the volume of the pet­
rol decreases by AV = V2 — Vl , where Vx is the volume of
the petrol at 71,, and V2 is its volume at T2. The volumes Vl
and Vz can be found from the formulas
^1 = ^0 11 + P (T, - T0)], V2= V0 [1 + P (T2 - T0) l
The unknown volume F0 at T0 — 273 Kean be calculated
from the formula F0 = m/p0. Then
AF = [I + P (T2 - T0) - 1 - P (7\ - To)),
Po
or
^F ——jr~ (T-i —Ti),
Po
2 x 104 kg x 10~3 K-1(248 K-298 K) 1.43 m3.
AF 7 x 10® kg/m3
6-0530
82 Ch. I. Fundamentals oi Molecular Physics

The minus sign indicates that the volume has decreased as


a result of the fall in the temperature.
Answer. The volume of the petrol is reduced by 1.43 m3
due to the temperature drop.
Problem 44. A steel ingot has a volume of 2.8 dm3 at 0°C.
Determine its volume at 525°C. Find the density of steel at
this temperature. How much heat is spent to heat it?
Given: F0 = 2.8 x 10~3 m3 is the volume of the ingot at
0°C, t0 = 0°C is the initial temperature of the ingot, t =
525°C is the temperature of the hot ingot. From tables, we
take the coefficient of linear expansion for steel, a = 1.2 X
10-s K"1, the density of steel at 0°C, p„ = 7.8 X 103 kg/m3,
and the specific heat of steel, c = 460 J/(kg-K).
Find: the volume V of the hot ingot, the density p of steel
at 525°C, and the amount of heat Q spent to heat the ingot.
Solution. The volume of the ingot heated to 525°C can be
determined from the formula
V = VQ (1 + PAT),
where A T — t — t„.
Since the coefficient of volume expansion for solids is P —
3a (with a small error), we can find the volume:
V = 2.8 x 10-3 m3 (1 + 3 x 1.2 x 10 s K "1 x 525°C)
= 2.85 x 10~3 m3.
The density of steel decreases with increasing tempera­
ture. It can be determined from the formula
p = p 0/ ( l + P A 7’),
7.8 xlO3 kg/m*
= 7.65 X 103 kg/m3.
P— 1+ 3.6 x 10~6 K-1 x 525 K
Pay attention to the fact that the temperature difference
can be expressed either in degrees Celsius or in kelvins.
The heat spent to heat the ingot is Q = cm (t — t 0). Since
mass can be expressed in terms of density, m — p„F0 =
pF, we have
Q =■■ PoV0c (t — t0),
Q = 7 .8 x l0 3 kg/m3X 2 .8 x 10~3 m3x460 J/(kg-K) x525 K
= 5 .3 xlO 6 J.
Answer. The volume of the hot ingot is 2.85 x 10' 3 m3,
the density of steel at 525°C is 7.65 x 103 kg/m3, and the
amount of heat spent to heat it is 5.3 MJ.
§ 8. Thermal Expansion of Bodies S3

Remark. The volume of the hot ingot can be found in


a different way: first we can determine AT -- V(lfSA7\ and
then V = Vn -f AT.

Questions and Problems

8.1. Why is Invar used in clockwork mechanisms?


8.2. Why are the holes in the cover plates connecting
rail sections together oval?
8.3. The filaments fused into the glass parts of vacuum
tubes or incandescent lamps must have the same thermal
expansion coefficients as that of glass. Why? Can copper
wire be used?
8.4. A copper ring is heated in a bunsen flame. What will
be the change in the inner diameter of the ring?
8.5. How can a steel rod be extracted from a tight-
fitting brass sleeve?
8 .6. Prove that the coefficients of volume expansion for
solids are three times as large as the coefficients of linear
expansion. Is it an exact or approximate relation?
8.7. Cold water can be poured into a red-hot flask made
of quartz glass without breaking it. How can this be ex­
plained?
8 .8. Why should tanks for petrol, kerosene or petroleum
not be filled to the brim in summer time?
8.9. Dentists do not recommend that we eat very hoi
food. Why?
8.10. What will happen to a bimetallic strip4 when it is
heated? Where are such plates used?
8.11. A steel measuring tape 100 m long is used for geo­
desic surveying. How much will it be elongated if the tem­
perature increases by 10 K?
8.12. What is the maximum increase in temperature of
a 1-km long aluminium wire for it to be elongated by only
230 mm?
8.13. As a result of heating through 100 K, each metre of
a wire elongates by 0.4 mm. What is the material from
which the wire is made?

4 A bimetallic strip is made from two different metals rivettcd


together.
C*
84 Ch. I. Fundamentals of Molecular Physics

8.14. The Ostankino television tower in Moscow is made


of reinforced concrete and is 533 m high at 273 K. How high
will it be at -- 20°C and at — 20°C? Assume that a -- 1.2 X
10~5 K "1 for reinforced concrete.
8.15. An iron rod 60 cm long at 273 K is placed into
a furnace. As a result, it elongates by 6.5 mm. Determine
approximately the temperature in the furnace.
8.16. The diameter of an iron tyre at 0°C is 6 mm less than
the diameter of the wheel (equal to 1.2 m) on which it has to
be set. By how many kelvins should the temperature of the
tyre be raised to set it onto the wheel?
8.17. The inner diameter of a copper ring at 273 K is
5 cm. To what temperature should it be heated for it to
accommodate a ball 5.01 cm in diameter?
8.18. The transmission line between the Volzhskaya
hydroelectric station and Moscow is 1000 km long. The line is
made of steel-and-aluminium wire. What will be the differ­
ence in the length of the steel and aluminium components
if the temperature increases by 30 K?
8.19. The rod antenna of the Luna-20 probe, which is oper­
ating on the surface of the Moon, is 2 m long at noon when
the temperature at the Moon’s surface is 393 K. How long
will the antenna be at midnight when the temperature on
the Moon drops to 123 K? The antenna is made of brass.
8.20. A steel beam is fastened on two supports which pre­
vent it from expanding. The cross-sectional area of the beam
is 140 cm2. What will be the force of the pressure exerted by
the beam on the supports if the temperature increases by
20 K?
8.21. An aluminium rod whose cross-sectional area is
4 cm2 is tightly clamped between two jams. What must be
the increase in temperature for the force of pressure
exerted by the rod on the jams to be 9.7 kN?
8.22. What forces should be applied to the ends of a steel
rod of a cross-sectional area 10 cm2 to prevent it from elon­
gating when heated from 273 to 303 K?
8.23. Two rulers, one made from aluminium and one
from steel, have the same length of 1 m at 0°C. At what tem­
perature will the difference in their lengths be 5.5 mm?
8.24. How much heat is required to make a copper rod
whose cross-sectional area is 2 cm2 elongate by 0.1 mm
when heated?
8 8. Thermal Expansion of Bodies 35

8.25. A steel tyre is set on a wheel at 300 K. What force


will emerge when the tyre is cooled to 293 K if its cross-
sectional area is 20 cm2?
8.26. A cylinder is cut from quartz so that its axis is
parallel to the symmetry axis of the quartz crystal. At 18°C,
the radius of the cylinder base is 10 mm and its height is
50 mm. Determine the volume of the cylinder at 573 K. The
coefficient of linear expansion is 7.2 x 10~5 K "1 for the
longitudinal axis and 1.32 x 10"5 K ' 1 for the transverse
axis.
8.27. The coefficient of surface thermal expansion is twice
the coefficient of linear expansion. What must be stipulated
in this case?
8.28. A copper sheet having a size of 0.6 X 0.5 in2 is heat­
ed from 293 K to 600°C. Determine the area of the hot
sheet.
8.29. What is the increase in the temperature of an alu­
minium sheet if its area has increased by 3200 mm2 by
heating? The area of the sheet at 0°C is 1.5 m2.
8.30. A vessel whose volume is 5 1 is filled to the brim
with kerosene at 0°C, brought into a room where the temper­
ature is 18°C, and placed on a tray. How much kerosene (in
litres) will overflow if the thermal expansion of the vessel is
not taken into account? How much will overflow if it is
taken into account, the coefficient of volume expansion
for the vessel material being p = 3.6 X 10' 5 K '1?
8.31. A 2-1 aluminium kettle is filled with water at 4°C.
How much water (in litres) will overflow from the kettle if
it is heated to 353 K?
8.32. What is the density of tungsten at its melting point?
8.33. At what temperature is the density of concrete
2.19 x 10® kg/m3? Assume that the coefficient of linear
expansion for concrete is 1.2 X 10"5 K ”1.
8.34. The volume of a brass cylinder at 325 K is 425 cm3-
Determine the mass of the cylinder.
8.35. The mass of a copper bar is 10 kg. At what tempera­
ture will the bar have a volume of 1.125 dm3?
8.36. Alcohol at 273 K is poured into a 10-1 glass bottle.
Neglecting the thermal expansion of glass, calculate the vol­
ume and mass of alcohol that can be poured into it so that
it does not overflow at 50°C.
86 Ch. I. Fundamentals of Molecular Physics

8.37. Solve Problem 8.36 taking into account the expan­


sion of the glass bottle.
8.38. How much petrol can be poured at 273 K into a 60-
m3 iron tank so that it does not overflow when heated to
40°C? Determine the mass of the petrol.
8.39. What volume is occupied by water in a steam boiler
at 100°C if the mass of the water is 2000 kg?
8.40. What is the minimum capacity of the cooling sys­
tem of a transformer if the temperature of the oil in it does
not exceed 363 K during operation of the transformer? The
mass of the oil is 3000 kg.
8.41. A 500-cm3 flask made of quartz glass is filled with
mercury. Determine the coefficient of volume expansion for
mercury if 8.91 cm3 of the mercury overflows as a result of
heating the flask from 273 to 373 K.
8.42. How much heat is spent if the volume of a mass of
mercury increases when heated by 4.5 cm3? Heat losses
should be neglected.
Chapter II

Fundamentals of Electrodynamics

§ 9. ELECTRIC FIELD

Basic Concepts and Formulas


Under normal conditions, any physical body is electrically
neutral, i.e. contains equal numbers of elementary electric
charges of opposite signs. The unit of electric charge is the
coulomb (C). When a body is electrostatically charged, the
electric charges are redistributed so that the body has an
excess charge of one sign; it becomes electrically charrged.
For example, if a body has an excess number of electrons,
it is negatively charged.
The algebraic sum of the charges in an isolated system
remains constant (the law of charge conservation).
Two bodies bearing like charges repel each other, while
oppositely charged bodies attract each other. The force F
with which two point charges and Q2 interact is given
by Coulomb’s law
p _ 1 I Qi I I Qi I
— 4jre0 erJ
where r is the separation between the charges, e is the per­
m ittivity of the medium, and e0 is the electric constant.
It is good to remember that l/(4 n e0) = 9 X 109 m/F.
Electric charges interact through electric fields, which
are characterized by a vector quantity known as the electric
field strength E:

where is a test charge in the field.


The electric field strength produced by a point charge is
E = ~----- %,
4ne0 er2
where Q is the electric charge producing the field.
Ch. II. Fundamentals of Electrodynamics

When a charge moves in an electric lield, work is done.


Therefore, the energy characteristic of electric field is the
electric potential
A

The unit of potential is the volt (V).


The electric potential of the field produced by a point
charge or electrically charged sphere is

^sph :

The work A done in an electric field to move a charge


depends on the potential difference for the points between
which the charge Q moves:
A = Q (<Pi — < P a)-
It does not depend on the shape of the path along which
the point moves.
For a uniform field,
p fPi-«Ps _ V

where d is the separation between points 1 and 2 measured


along the field line. This formula gives the dimensions of
the electric field strength, viz. volt per metre (V/m).
When an isolated conductor is electrostatically charged,
its potential increases in proportion to the charge Q impart­
ed to it: Q = C<p. Here C is a proportionality factor
known as the capacitance of the conductor:

C = ^~.

The unit of capacitance is the farad (F).


The electric capacitance of an isolated sphere is
^"sph = 4 jT C (,er.

Electric energy can bestored in capacitors. The capacitance


of a parallel-plate capacitor is calculated by the formula
eneS
C=
§ 9. Electric Field 89

while the energy accumulated in it is given by

When capacitors are connected in parallel to form a bank,


the total capacitance is equal to the sum of the capacitances
of individual capacitors:
C = Cj + C2 + C3 + . . . .
The potential difference (voltage) across all the capacitors
is then the same.
When a bank is formed by series-connected capacitors, the
total capacitance is determined from the formula

Worked Problems
Problem 45. A conducting sphere bearing a charge of 1.8 x
10~8 C is brought in contact with two similar spheres, one of
which has a charge of — 0.3 X 10~8 C and the other is neu­
tral. Howwill the charge be distributed among the spheres?
What will be the force of interaction between two such
spheres in vacuum at a distance of 5 cm from each other?
Given: = 1.8 X 10~8 C, Q2 = — 0.3 X 10~8 C, and
(?3 = 0 are the charges of the spheres before they are
brought in contact, and r = 5 x 10~2 m is the distance at
which two spheres interact. From tables, we take the elec­
tric constant e0 = 8.85 X 10~12 F/m and the permittivity
of vacuum, e = 1.
Find: the electric charges Q[, Q't , and Q'a of the spheres
after they have been brought in contact and the force F of
electrostatic interaction between two spheres.
Solution. When the spheres are brought in contact, some of
the charges neutralize each other since they have opposite
signs. The remaining charge will be equally distributed
between the three spheres:
✓v <?i + <?2+ <?3 /v 1.8xl0"8 C —0.3x10“ BC
V 1 — XT2 V s 3 ' Y [ 3

= 0.5 X 10“8 C.
90 Ch. II. Fundamentals of Electrodynamics

The force of electrostatic interaction between two identical


charges Q\ and Q2' in vacuum can be determined from
Coulomb’s law:
1 (C',)2 N-m2
9 X 10® m/F = 9 X 10®
4ne0 r2 4ae0 C2
(0.5 xlO -9 C)=
F = 9 X 10“ N ■m2/C2 25x10-" m2 = 9 X 10"5 N.

Answer. The charge on each sphere alter they have been


brought in contact is 0.5 x 10-8 C and the force of their
interaction is 9 x 10~5 N.
Problem 46. Two charged metal
N spheres negligibly small in size are
placed in transformer oil where
{G they interact with a force of 2.5 X
10“4 N. Determine the separation
between the spheres if their charges
are 6 and 60 nC.
Given: — 6 x 10"° C and
Q2 — 6 X 10"8 C are the electric
charges on the spheres and F —
Fic. 15 2.5 < 10"4 N is the force of their
electrostatic interaction. From ta­
bles, we take the permittivity of transformer oil. e — 2.5.
Find: the distance r between the spheres.
Solution. We have point charges and hence Coulomb’s law
is applicable. Then the distance between the charges is

r QiQi

Substituting the numerical values, we obtain


_ - / 9x 10®m / F x 6 x 10 'X6x l 0-®C2
= 7.2 x 10-2 m.
T ~ V 2.5 X 2. 5x1in-" n
Answer. The separation between the spheres is 7.2 cm.
Problem 47. A 2-g sphere bearing a charge of 3 x 10-8 C
is suspended in air on a silk thread. Determine the tension
in the thread when another sphere bearing a like charge of
2.4 X 10"7 C is placed 10 cm beneath the first (Fig. 15a).
Given: m = 2 x 10~3 kg is the mass of the sphere, =
3 x 10-8 C is the electric charge of the sphere, r = 10-Im is
§ 9. Electric Field 91

the distance between the spheres, and Qt = 2.4 X 10' 7 C is


the electric charge of the second sphere. From tables, we
take the permittivity of air, e = 1, and the free fall accel­
eration g = 9.81 m/s2.
Find: the tension F t of the thread.
Solution. The sphere on the thread experiences three forces:
the force of gravity G = mg, the Coulomb force of repulsion
F— 7 QiQt
4jie„ e r2 ’
and the tension Ft in the thread (Fig. 156). Since the spheres

Fig. 16

are at rest, the equation for the forces acting on the first
sphere has the form Ft + F — G = 0. Hence
Ft = G — F, Ft = mg 1 gl<?2 /_ L _ 9 x 10e m/F) .
4ne0 er2 V4ne0
Substituting the numerical values, we obtain
Ft = 2 x 10-3 kg x 9.81 m/s2- 9 x 109m/F
w 3xl0- 9Cx2.4xl0”’ C
X 10~2m2 ~ 1.31 x 10~2 N.

Answer. The tension in the thread is approximately 1.3 X


lO-2 N.
Problem 48. Electric charges = 4.8 X 10-7 C, Qt =
Q3 = 1.6 x 10- 7 C, and = - 1.6 x lO"7 C (Fig. 16a)
are arranged in a circle of radius 2 cm at equal distances
from one another. Determine the held strength and potential
of the electric held formed by all the charges at the centre O
of the circle.
92 Ch. II. Fundamentals of Electrodynamics

Given: = 4.8 x 10- 7C, Q2 = Q3 = 1.6 x 10~7 C, and


= —1.6 xlO~7C are the charges producing the field, r =
2 x 10' 2 m is the radius of the circle. From tables, we take
the permittivity of vacuum, e ----- 1.
Find: the electric field strength E and the electric poten­
tial (p at point O.
Solution. Since the medium is not specified in the problem,
we solve it for vacuum. Each of the charges produces at
point 0 a field of strength Ex, E„, E3, and E4 respectively. The
electric field vector E at O is equal to the geometric sum of
the field strengths due to the individual charges:
E = Ej — E2 + Ea -f E4.
The electric field strength Ex due to charge Qt can be calcu­
lated by the formula

E < - T k & ( T k ’ -'3 y t a ‘ mlF) ’


£ ,1= 9 x 10® m/F 4x10 4 m-
^ = 10.8 x 10® V/m.

Since the charges Q2, Q3, and are equal in magnitude


and are at the same distance from point O, we can write
1 Qi
E 2 —E3 —£4 — 4ne0 n r 2 ’

1.6xl0~7 C
E o — E3= £’4= 9 X 109m/F 4xl0"4m2 = 3 .6 x 10® V/m.

In order to determine the resultant field strength, we first


add the vectors directed along the same straight line (Fig. 166)
E 1 = Ej - E 2 = 10.8 x 10" V/m - 3.6 X 10s V/m
= 7.2 x 10® V/m,
E„ = E 2 + E t = 3.6 x 10° V/m + 3.6 x 106 V/m
= 7.2 x 10° V/m.
The required resultant vector E can be found using the
parallelogram rule (Fig. 16c). In the case under considera­
tion, we can use the Pythagorean theorem since we have
a right-angled triangle:
E = V 2(7.2 X 10« V/m)? = 10.2 x 1Q« V/ip,
§ 9. Electric Field 93

The electric potential is a scalar quantity. Therefore, the


potential of the resultant field produced by the charges Qlt Q2,
Q3, and Q4 is equal to the algebraic (and not geometric as
for the field strength) sum of the potentials of the fields pro­
duced by all the charges:
4.8x 10~7 C
*■= 'P ,- 9 x l0 'm /F 2x l 0”2 m = 21.6 X 104 V,

= 9 X 10s m/F 42x^l 0~2^m = 7.2 x 104 V,


<p*= - 7.2 x 104 V.
The potential at point O is

= <Pl + ^2 + <P3 — <f>4-

Since and cp4 are equal in magnitude, we have

ip = ifi -f- q>2, = 21.6 x 104 V + 7.2 x 104 V


= 2.88 x 105V.
Answer. The field strength at the centre of the circle is ap­
proximately 107 V/m, and the field potential is 2.9 x 105 V.
Problem 49. Two point charges of 2.64 X 10-a and 3.3 X
10~9 G are in vacuum at a distance of 0.6 m from each other.
What work must be done to bring the charges closer to
25 cm?
Given: Qx = 2.64 x 10~8 C, Q2 = 3.3 X 10' 9 C are the
electric charges, = 0.6 m is the initial distance between
the charges and r2 = 0.25 m is the separation between the
charges after they are brought closer. From tables, we take
the electric constant e0 = 8.85 X 10~12 F/m and the permit­
tivity of vacuum, e = 1.
Find: the work A required to bring the charges closer to­
gether.
Solution. We assume that the charge Q1 produces an elec­
tric field and the charge Q2 is moved in the field. Then the
work of external forces done to bring the second charge closer
to the first is
A = Q2 (tpi — <Pa).
94 Ch. II. Fundamentals of Electrodynamics

where <pt and cp2 are the electric potentials of the points be
tween which charge Qz is moved:
Q. 2.64 x l ( F BC
<Pi 4jie0er 1 4ji8.85 X 10~la K/m X 0.6 m = 396 V,
Qi 2.64x10-* C
Vi 4jie0e r2 4n8.85xl0-12 F/mx0.25m 9 5 0 V .
We can now find the work:
A = 3.3 x 10-u C (396 V - 950 V) ~ - 1.83 x 10-° J.
The minus sign indicates that the work is done against the
field forces.
Answer. The work done to bring the charges closer is
1.83 x 10-6 J.
Problem 50. A dust particle with mass 10"7 g is suspended
between the plates of a parallel-plate air capacitor to which
a voltage of 500 V is applied (Fig. 17).
The separation between the plates is 5 cm.
k? Determine the electric charge on the dust
j particle.
'G Given: m — 10"10 kg is the mass of
——— “ the dust particle, U = 500 V is the
Fig. 17 voltage across the capacitor plates, and
d = 5 x 10~2 m is the distance between
the plates. From tables, we take the free fall acceleration
g = 9.8 m/s2.
Find: the charge Q of the dust particle.
Solution.The dust particle in the uniform field of the capac­
itor is acted upon by the force of gravity G = mg directed
downwards and by the force F = QE exerted by the electric
field in the upward direction. The particle is in equilibrium
provided that these forces are equal: G = F, or mg = QE.
Hence
Q = mg/E.
Using the well-known relation E = U/d between the
electric field strength and voltage, we obtain
n m g d f t 10“10 kgx9.8m /s 2x 5 x l 0 -2 m r
v — u > V — 500V
Answer. The charge on the dust particle is approximate­
ly 10- 13 C.
§ 9. Electric Field 95

Problem 51. A voltage of 90 V is applied to a parallel-


plate air capacitor with a plate area of 60 cm2. The charge on
the capacitor becomes 10"9 C. Determine the capacitance of
the capacitor, the energy stored in it, and the separation
between the plates.
Given: S = 6 x 10~3 m2 is the area of a plate, U = 90 V is
the voltage across the plates, and Q = 10~9 C is the charge
on the capacitor. From tables, we take the electric constant
e0 = 8.85 X 10' 12 F/m, and the permittivity of air e = 1.
Find: the capacitance C of the capacitor, the energy W
stored in the capacitor, and the separation d of the plates.
Solution. Using the formula C = Q/U, we determine the
capacitance of the capacitor:

C= ~ 1.1 x 10"“ F ~ 11 PF.


In order to determine the energy stored in the capacitor,
we can use any of the following three formulas:

W= 2 ’
W — CV*
2 ’
W= 2C

This gives
W = 10~9C2x90V = 4.5 x 10- 8 J.
The separation between the capacitor plates can be deter­
mined from the formula for the capacitance of a parallel-
plate capacitor, C -- ee0 S/d. Since e = 1, we obtain
C = £0 S/d. Hence
A 8 .8 5 x 1 0 -“ F / m x 6 x 10-3 m1
d= ft~ 1.1X10-“ F - o x i w m.
C ’

Answer. The capacitance of the capacitor is approximately


11 pF, its energy is 4.5 x 10"8 J, and the separation be­
tween the plates is about 5 mm.
Problem 52. Determine the capacitance of the capacitor
bank shown in Fig. 18a if Cx = 1.5 pF, C2 = 2 pF, C3 =
3 pF, Cx = 4 pF, and C5 = 2 pF. What energy is stored in
the bank if the voltage applied to it is 500 V?
Given: Cx = 1.5 pF, C2 = 2 pF, C3 = 3 pF, C4 = 4 pF,
and C5 = 2 pF are the capacitances of the capacitors, and
U = 500 V is the voltage across the bank.
96 Ch. II. Fundamentals of Electrodynamics

Find,-, the capacitance C of the bank and the energy W of


the electric field of the capacitor bank.
Solution. In the problem, it is convenient to express the
capacitances of the capacitors in microfarads. The capacitors
(°) C; Ci (b) Cf fc) c>

<j
T----- II ir^

.
— 11—

— II—
= cs = = c3

C'
ii
=

o
- C i

1
p
Fig. 18
C2, C3, and C4 are connected in series. They can be replaced
by a single capacitor having an equivalent capacitance
(Fig. 186). For series-connected capacitors, we have
_1_
Ci C2 + C, + C4 ’
J_ 1 1 1 Ci = 0.92 \iF.
Ci = T p F ■*' IT p F + 4 p F ’
The capacitors C5 and C, are connected in parallel. There­
fore, their equivalent capacitance is
= C5 + Ci, Cll = 2 pF + 0.92 pF = 2.92 pF ~ 3 pF.
As a result, we obtain the capacitors C4 and CM in series
(Fig. 18c).
The capacitance of the bank can be found from the formula

CiCn „ 1.5 p F x 3 pF
C= = 1 pF = 1 X IQ"6 F.
C, + C,, ' ° 1.5 pF + 3 pF
The energy stored in the bank can be determined from the
formula
cm w 1 XlO-6 FX25X101 V2
W 1.3 X 10'3J.

Answer. The capacitance of the bank is 1 pF and the ener­


gy is approximately 1.3 x 10"3 J.
Problem 53. An electron flies from point A to point B, the
potential difference between these points being 100 V.
§ 9. Electric Field 97

What is the velocity acquired by the electron at point B if


the velocity at point A is zero? The charge-to-mass ratio
for electrons is 1.76 x 10u C/kg.
Given: U = 100 V is the potential difference between
points A and B, elme = 1.76 x 1011 C/kg is the charge-to-
mass ratio for the electron.
Find: the velocity v of the electron at point B.
Solution. The work done by the field in displacing the
electron is A = eU. This work is converted into the kinetic
energy of the electron E k = mei^t2. From the energy con­
servation law, we have eU = mev2l2, whence

v = V 2 x 100 V x 1.76 x 10“ C/kg = 5.9 x 10«m/s.

Answer. The velocity of the electron at point B is ap­


proximately 5900 km/s.

Questions and Problems

Conservation of electric charge. Coulomb's law


9.1. A drop of oil has an electric charge of —3.2 x
10~19 C. Determine the number of excess electrons on the
drop.
9.2. Three electrons are missing from an electrostatically
charged body. Determine the magnitude and sign of the
charge on the body.
9.3. Can an electric charge of one sign be obtained as
a result of electrostatic charging by friction?
9.4. Under what conditions can a brass rod be charged?
9.5. An electric charge on a conducting sphere has to be
divided into three equal parts. How can this be done?
9.6. Why is a metal chain that reaches the ground fixed
to a lorry for transporting petrol?
9.7. Can a positive charge be obtained on an electroscope
using a negatively charged ebonite rod?
9.8. What will happen to the surface charge density on
a metal sheet rolled into a cylinder?
7-0530
98 Ch. II. Fundamentals of Electrodynamics

9.9. An elder ball is tied to a silk thread. What will hap­


pen when an electrostatically charged rod is brought
close to it?
9.10. Will the force of interaction between two point
charges change if the magnitude of each charge and the
separation between charges are halved?
9.11. Two identical conducting spheres bearing elec­
tric charges of 3.2 X 10“19 and —3.2 x 10~19 C are brought
in contact. What are the new values of charge on the
spheres? How many electrons have passed from one sphere
to another?
9.12. What is the force of interaction between two point
charges of 12 nC each separated in vacuum by 3 cm? By what
factor will the force of interaction be reduced if the charges
are placed in water?
9.13. Two identical point charges in glycerol 9.0 cm apart
interact with a force of 1.3 X 10~s N. Determine the mag­
nitudes of the charges.
9.14. What is the force of interaction between two 1-C
charges 1 m apart (a) in vacuum and (b) in kerosene?
9.15. At what distance would two 1-C charges interact
in vacuum with a force of 1 N?
9.16. Using Table 14, answer the following questions:
(1) What is the ratio of the forces of interaction between
electric charges in vacuum and in mica? (2) In what media
is the force of interaction between charges equal to half the
force of their interaction in vacuum? (3) To what medium
should charges be transferred from vacuum so that their
interaction is reduced by a factor of 81?
9.17. Will the electrostatic interaction force between two
charges change if they are set in ice instead of vacuum?
9.18. Two mercury drops on glass of mass 20 g each and
4.0 cm apart receive charges of —6.0 x 10”8 and 2.0 x
10~7 C. In what direction and with what acceleration will
they start to move? Will the motion be uniformly accelerated?
Gravitational forces should be neglected.
9.19. With what force is an electron moving in an orbit
of radius 5.0 x 10“u m attracted to a helium nucleus? The
charge of the nucleus is 3.2 X 10“le C. The orbit should
be assumed to be circular.
9.20. Two little spheres having a mass of 1 g each and
suspended in vacuum from the same point on silk threads
$ 9. Electric Field 99

have equal negative charges. They repel each other and move
apart to 12 cm, forming an angle of 22°. Determine the num­
ber of electrons on each ball and the tension in the threads.
Show the forces acting on the balls on a diagram. Will the
tension in the threads change if the charges interact in
zero-gravity?
9.21. Two point charges of 5.0 x 10-9 and 1.5 X 10"8 C
are 4.0 cm apart in vacuum. Determine the force with
which these charges will act on a third charge of l.O x 10"9C,
located at the midpoint of the line connecting the charges.
9.22. A charge of 1.57 X 10”8 C is transferred to a metal
sphere of radius 5 cm. What is the surface charge density on
the sphere?
9.23. The surface charge density of a conducting sphere
is 5 x 10'B C/m2. Determine the magnitude of the charge
on the sphere if its radius is 8 cm.
9.24. Two electric charges, one of which is twice the oth­
er, interact at a distance of 0.60 m in vacuum with a force
of 2.0 mN. Calculate the magnitudes of charges. At what
distance in kerosene will the interaction between the
charges be the same?
9.25. What is the ratio of the electrostatic force of repul­
sion between two electrons to their gravitational attraction?
9.26. Determine the perm ittivity of kerosene if two equal
like charges interact in vacuum with the same force at a dis­
tance of 0.283 m as in kerosene at 0.20 m. Assuming that
the force of interaction in kerosene is 3.0 X 10"2 N, deter­
mine the magnitudes of the charges.

Electric field strength


9.27. Do the electric field vector and the vector of the
force exerted on a charge by an electric field always have the
same directions?
9.28. Why is a metal cap sometimes put on a vacuum
tube?
9.29. Can electric charges be separated on (a) a conductor,
(b) a dielectric?
9.30. A cylindrical conductor is attached to a conical
conductor with the same base area. What can be said about
the electric field strength near various points on the surface
of the resultant conductor?
7*
100 Ch. II. Fundamentals of Electrodynamics

9.31. Why is a ball usually fixed to the rod of an electro­


scope?
9.32. Two electric charges of the same magnitude and
sign produce an electric field. What is the field strength at
the midpoint of the straight line connecting the conductors?
9.33. The electric field strength at a given point is
300 V/m. What does this mean?
9.34. Determine the electric field strength at a point
where a force of 5 mN acts on a charge of 0.7 X 10"® C.
9.35. What is the force exerted by an electric field on
a charge of 3.2 x 10~8 C at a point where the field strength
is 500 V/m?
9.36. The electric field strength near the surface of the
Earth before a stroke of lightning is 2 X 10s V/m. What is
the force exerted by the field on an electron?
9.37. An electric field is produced in vacuum by a point
charge of 7.5 X 10~8 C. Determine the field strength 15 cm
away from the charge.
9.38. A point charge of 2.2 nC produces a field with
a strength of 2.5 kV/m at 6.0 cm from the charge. Determine
the permittivity of the medium.
9.39. A vessel of kerosene contains a conducting sphere
with a negligibly small size and a charge of 8.0 X 10‘ 8 C.
Determine the electric field strength 5.0 cm from the charge.
What will be the change in the field strength at this point
if the kerosene is let out of the vessel?
9.40. The electric field strength 5.0 cm from a charge is
1.5 x 105 V/m. What is the electric field strength 10.0 cm
from the charge? Determine the magnitude of the charge.
9.41. How many excess electrons are contained on a dust
particle acted upon by a force of 2.4 X 10~10 N in an electric
field of strength 1.5 x 105 V/m?
9.42. An electric field is produced by a point charge.
What are the loci of the points where the magnitude of the
electric field strength is the same?
9.43. Will an electric field remain uniform if a conducting
sphere is placed in it?
9.44. Two identical point charges of 1.0 X 10"8 and
2.0 X 10"8 C are in vacuum 20 cm apart and produce
an electric field. Determine the field strength at the midpoint
of the line connecting the charges. What will the electric
field strength be if the charges are unlike?
§ 9. Electric Field 101

9.45. Two conducting spheres having diameters of 10.0


and 4.0 cm are 120.0 cm apart and bear charges of 3.0 x
10'® and 2.0 X 10~® C respectively. Determine the electric
held strength at the midpoint O of the straight line con­
necting the spheres (Fig. 19).
9.46. Two like electric charges of 7.0 X 10~8 C each are at
points A and B (Fig. 20). Determine the electric held

strength at point O which is the apex of the right angle A OB]


AO = BO = 5.0 cm.
9.47. Determine the electric held strength of the held at
a point three radii from th3 surface of a charged conducting
sphere. The surface charge density on the sphere is 1.6 X
10"7 C/m2.
9.48. A conducting sphere of radius R is electrostatically
charged to a surface density a. What will be the held strength
(a) at the centre of the sphere, (b) at half the radius of the
sphere, (c) on the surface of the sphere?
9.49. A drop bearing a charge of 2 X 10~8 C is in equi­
librium in a uniform electric held of strength 49 V/m. Deter­
mine the mass of the drop.
9.50. With what acceleration will a 10-g ball with charge
1.0 X 10“5C fall in the electric held of the Earth? The elec­
tric held strength near the surface of the Earth is 130 V/m.
9.51. A thin conducting ring of radius R has an electric
charge Q. Determine the electric held strength at the cen­
tre O of the ring and at point A (Fig. 21).
9.52. Why is the uniformity of the electric held of a
charged parallel-plate capacitor violated if an uncharged
metal sphere is placed between its plates?
102 Ch. II. Fundamentals of Electrodynamics

Electric potential. Potential difference. Work done in an


electric field
9.53. Considering the Earth to be a sphere of radius
6400 km, determine its electric charge and potential if the
electric field strength produced by the Earth near the surface
is 130 V/m.
9.54. The work done in displacing a 2.0 x 10~8-C charge
from infinity to a point in a field is 1.13 X 10~4 J. What is
the electric potential at this point?
9.55. Determine the potential difference between two
points in a field if a work of 3.0 x 10“5 J has to be done in
moving an 8.0 X 10~7-C charge between these points.
9.56. An electric field moves a positive charge of 3.0 X
10"7C between two points with potentials 200 V and 1200 V.
What is the work done by the field in this case?
9.57. An electric field is produced by a point charge of
4 x 10"8 C. What is the electric potential 6 cm away from
this charge? What work must be done against the field in
order to bring a positive charge of 1 C from infinity to
this point?
9.58. Two point electric charges of 1.0 X 10~5 and 6.0 X
10' 8 C are 20 cm apart in air. Determine the electric poten­
tial at the midpoint of the straight line connecting the
charges.
9.59. A 1.6 X 10~7-C charge is moved 3.0 cm along a field
line in a uniform electric field having a strength of 5.0 X
W V /m . Determine the work done and the potential differ­
ence for two points between which the charge is moved.
9.60. As an electron passes between two points in an
electric field, its velocity increases from 2.0 X 108 to 3.0 X
107 m/s. What is the potential difference between these
points? What is the increase in the kinetic energy of the
electron?
9.61. Two 8-nC charges are located at two apexes of
an equilateral triangle having a side of 6.0 cm. Determine the
field strength and the potential at the third apex.
9.62. Two conducting spheres of radii 2.0 and 3.0 cm are
charged to 30 and 40 V respectively. What will be the elec­
tric potential of the spheres after they have been connected
by a wire? Assume that the separation between the spheres
is large compared to their radii.
§ 0. Electric Field 103

9.63. An electron having a velocity of 6.0 X 107 m/s


flies into a parallel-plate air capacitor midway between the
plates. What must the voltage across the plates be for the
deviation of the electron to be maximum? The plates are
10.0 cm long and 3.0 cm apart.
9.64. An electron with a velocity of 1.6 x 10* m/s flies
into a uniform electric field of strength 90 V/m and moves
along a field line until it comes to a halt. How long and
how far does it fly in the field? Assume that the electron
mass is 9.0 x 10~sl kg.

Capacitance. Capacitors. The electrical energy in a


capacitor
9.65. Can the potential of a charged conductor be
changed without changing its charge?
9.66. A parallel-plate air capacitor of capacitance C is
immersed in a medium with a permittivity of 2. What will
the capacitance of the capacitor be?
9.67. The capacitance of a spherical conductor is propor­
tional to its radius. What must the radius of a conducting
sphere be for its capacitance in vacuum to be 1 F? What is
the ratio between the radius of such a sphere and the Earth’s
radius?
9.68. A parallel-plate air capacitor is connected to a con­
stant-voltage source. What will be the change in the capaci­
tance and the energy of the capacitor, the voltage across
it, and the charge on the capacitor plates if the separation
between the plates is reduced?
9.69. Determine the capacitance of an isolated conducting
sphere of radius 5.0 cm immersed in kerosene. Express the
answer in farads, microfarads, and picofarads.
9.70. Assuming the Earth to be a sphere of radius 6400 km,
determine its capacitance.
9.71. An isolated conducting sphere whose capacitance
is 5.0 pF is electrostatically charged to a potential of 1570 V.
Determine the radius of the sphere and the surface charge
density on it.
9.72. Why do electrolytic capacitors have a large capa­
citance?
9.73. Two conducting spheres having a charge of 1.0 X
104 Ch. II. Fundamentals of Electrodynamics

10"8 C each are arranged a long way apart. The capacitance


of the larger sphere is 2.2 X 10"11 F, and that of the smaller
sphere is 5.6 X 10~12 F. What are their potentials? What
will happen when they are connected by a conductor?
9.74. Calculate the capacitance of a parallel-plate capaci­
tor made from tin foil plates 15 cm2 in area with a mica
dielectric layer 0.8 mm thick. The permittivity of mica
is e = 6.
9v75. A parallel-plate air capacitor consists of two plates
100 cm2 each in area. When a charge of 6.0 X 10~9 C is trans­
ferred to one of the plates, the capacitor is charged to a vol­
tage of 120 V. Determine the separation between the plates.
9.76. The plates of a parallel-plate air capacitor have an
area of 62.3 cm2 each and their separation is 5 mm. Determine
the charge of the capacitor if the potential difference across
its plates is 60 V.
9.77. The separation between the plates of a parallel-
plate air capacitor is 0.1 cm and the area of each plate is
200 cm2. The potential difference across the plates is 600 V.
What charge is stored in the capacitor? What will be the
change in the voltage if the space between the plates is
filled with mica, whose permittivity is 6?
9.78. A parallel-plate air capacitor with a plate separa­
tion of 1.5 mm is charged to a voltage of 150 V. How far
apart should the plates he moved in order to increase the
voltage to 600 V?
9.79. Determine the capacitance of a capacitor in which
nine mica plates having a thickness of 0.12 mm and an area
of 12.56 cm2 are interleaved between tin foil plates. The
permittivity e of mica is 6.
9.80. Determine the energy of a parallel-plate paper ca­
pacitor with a plate area of 600 cm2. The charge on the
capacitor is 2 X 10“7 C and the dielectric is wax paper
2.0 mm thick.
9.81. Three capacitors whose capacitances are 4, 2, and
6 pF are connected to form a hank and to a constant-voltage
source of 200 V. Determine the capacitance and the energy
of the bank when the capacitors are connected (1) in series
and (2) in parallel.
9.82. Determine the capacitance of a capacitor bank
connected as shown in Fig. 22 if = 1.2 pF and C2 =
Cs = 0 .6 . p F .
§ 9. Electric Field 105

9.83. Determine the capacitance of a bank of capacitors


connected as shown in Fig. 23. The capacitance of all the
capacitors is the same and equal to 0.6 pF. Determine the
electric charge stored in the bank if a voltage of 100 V is
applied to it.
9.84. Capacitors are connected as shown in Fig. 24.
Given: Cx = C2 = 2 pF, C3 = C4 = Cs = 6 pF. Determine

G
Fig. 22 Fig. 23

the potential difference between points A and B if the


energy stored in the bank is 1.35 X 10-1 J.
9.85. An uncharged 100-pF capacitor is connected in
parallel to a 50-pF capacitor charged to a voltage of 300 V.
G
<-2
|— H----------I H — 11 —
8
G G
H I— IH
H I— I H H
G G G <■i o
Fig. 24 Fig. 25

What is the voltage across the capacitors? What, is the


charge distribution between them?
9.86. After a capacitor having an unknown capacitance
and charged to a voltage of 600 V is connected in parallel
to an uncharged 5-pF capacitor, the voltage across the bank
drops to 100 V. What is the capacitance of the first capacitor?
9.87. Two capacitors having different capacitances are
connected in parallel and to a constant-voltage source. Which
of the capacitors will accumulate more energy?
9.88. A pulsed flash for photography is obtained by
discharging a 800-pF capacitor for 2.8 ms at a voltage of
100 Cli. II. Fundamentals of Electrodynamics

300 V across the capacitor. Determine the energy of the


flash and the mean power.
9.89. Three capacitors are connected to make a bank as
shown in Fig. 25 and then to a 200-V constant-voltage
source. Determine the capacitance of capacitor and its
charge if the capacitance of the bank is C = 6 p,F and the
capacitances of the remaining capacitors are C2 = 2 p.F and
C3 = 3 pF.
P.90. The force with which the plates of a parallel-plate
capacitor attract each other is determined from the formula
F = Q 2/ ( 2 e 0eS). Is the energy required to move the plates
apart higher (1) when the capacitor remains connected to
the source or (2) when it has been charged and then before
the plates are moved apart disconnected from the source?
9.91. A spherical capacitor consists of two conducting
concentric hollow spheres. If the difference between the radii
of the spheres is small, the capacitance of the capacitor can
be calculated using the formula for a parallel-plate capaci­
tor. Write this formula in a general form and determine the
capacitance of an air capacitor formed by spheres of radii
5.00 and 4.95 cm.
9.92. Determine the volume energy density of the uniform
electric field in a mica capacitor charged to a voltage of
90 V. The separation between the plates is 1.0 mm, and the
permittivity of mica should be taken to be 6.§

§ 1C. ELECTRIC CURRENT IN METALS. OHM’S LAW.


ELECTRIC RESISTANCE

Basic Concepts and Formulas


An electric current is an ordered (directional) motion of
charged particles (in metals, these particles arefree electrons).
The current / in a conductor is the amount of electricity Q
flowing per second through a cross section of the conductor:
/ = Q/t, or / = envS,
where n is the number density of the charge carriers (e), v is
the mean velocity of the charges, and S the cross-sectional
area of the conductor. Current is a base quantity in SI, its
unit being the ampere (A). The direction of current is assumed
to be opposite to the direction of motion of electrons.
§ 10. Electric Current in Metals. Oliin's Law 107

In older to create a direct current in metals, an electric


field must act on the free electrons. The field must ensure
a constant potential difference across the ends of the conduc­
tor (circuit). In a current source, the extraneous forces pro­
duce an excess of electrons at the negative terminal and an
electron shortage at the positive terminal. In other words,
a potential difference is created. Each current source is
characterized by an electromotive force (emf) % which is
equal to the work done by extraneous forces in moving
a positive charge of 1 C along the circuit:

The emf unit is the volt (V).


Ohm’s law for a conductor establishes a relation between
the current flowing in it and the voltage across its ends:

' 4 ".
where the proportionality factor HR is called the electric
conductance and R the electric resistance of the conductor.
The unit of resistance is the ohm (£2).
The resistance of a conductor depends on its size, material,
and temperature:
R = P -J -, R t = R0( i- \ - a M ’),

where p is the resistivity in Q-m, and a is the temperature


resistance coefficient in K -1:

For metal conductors, a is positive. At temperatures


close to absolute zero, superconductivity is observed for
some conductors. This is a state in which the resistance
abruptly drops to zero.
Ohm’s law for a circuit establishes a relation between
the current, electromotive force, and the total resistance of
the circuit:

where R and r are the resistances of the external part of the


circuit and of the current source.
108 Cli. II. Fundamentals of Electrodynamics

The voltage U across the terminals of a current source


with a closed circuit is smaller than the emf by the voltage
drop within the source itself:
U = t - Ir.

Short circuiting is observed when the resistance of the


external part of the circuit is negligibly small, while the
current attains its maximum value. Using Ohm’s law for
a closed circuit, we can determine the current during short
circuiting:
^ sh C “ £ /> " •

Individual parts of a circuit (resistors) can be connected


in series or in parallel.
For a connection in series, the resistors are connected one
after another so that the current in all the subcircuits is the
same, while the total, or equivalent, resistance of the circuit
is given by
R*cr = T T ■ ■ • T~ Rn-
If Rx = R 2 = . . . = R n - R, we have
Rser Rn.
The voltage drop for a series connection is proportional
to the resistances:
Uj_ = J}±
Ut r2 •
For a connection in parallel, the voltage across all the
parallel branches is the same, while the current in a branch
is determined by its resistance:
iL -J h .

The total, or equivalent, resistance is then given by the


formula
+ _L
Ryar ^1 R% Rn
If i?! = R 2 = . . . = R n = R, we have
1 1 R_
n , Or R par — n
^par R
§ 10. Electric Current in Metals. Ohm’s Law 109

To measure the current, an ammeter is connected in series


to the circuit. The resistance of the instrument should he
very low. In order to change the value of a scale division on
the ammeter, a bypass (shunt) resistor R sh is connected in
parallel with the instrument. The resistance of the shunt is
lower than the resistance of the ammeter by a factor of n —1:

where n is the number by which the scale of the instrument


is multiplied (Fig. 26).
To measure voltage, a voltmeter is connected in parallel
to the region of the circuit across which the voltage is being

Fig. 20 Fig. 27

measured. The resistance of the voltmeter must be very high.


If the voltage to be measured exceeds the range of a voltme­
ter, a multiplying coil (series resistor) R s is connected in
series with the instrument (Fig. 27):
R s == R v (n — 1),
where n is the number by which the range of the voltmeter is
increased, and R v is the resistance of the voltmeter.
Sources of electric power (current sources) can be connect­
ed to form batteries. For a series connection, the positive
terminal of the first source is connected to the negative ter­
minal of the second source, and so on. The current in this
case is
, _ n%
fl + n r ’

where n is the number of identical current sources in the


battery.
For a parallel connection, all the positive terminals are
connected in one junction and the negative terminals form
110 Ch. II. Fundamentals of Electrodynamics

the other junction. The current in such a battery is

R -J- r / n

While solving problems in which Ohm’s law is applied


to branched circuits, it is necessary
(1) to choose arbitrarily the directions of the currents
and the direction of circumvention of subcircuits and indi­
cate them on a circuit diagram;
(2) to write equations for the currents at the junctions,
the number of equations being one less than the number of
junctions, the algebraic sum of currents at a junction is
always equal to zero if the currents flowing to a junction
and from it are taken with opposite signs; and
(3) to write the equations for all the closed subcircuits
considering that the algebraic sum of emf’s in any closed
subcircuit is equal to the algebraic sum of the voltage drops;
if the potential increases in the direction of circumvention
(we move from “minus” to “plus”), the emf is assumed to be
positive, otherwise, the emf is assumed to be negative.
The voltage drop is assumed to be positive if the direction
of the current coincides with the chosen direction of circum­
vention; otherwise, it is taken with the minus sign. The
total number of equations must be equal to the number of
unknowns.

Worked Problems
Problem 54. A voltage of 3.6 V is applied to the ends of
a steel conductor 20 m long. Determine the mean velocity
of ordered motion of the charge carriers in the conductor if
their number density is 4.0 x 1028 m~3.
Given: U = 3.6 V is the voltage drop across the conductor,
n = 4.0 x 1028 m~3 is the number density of mobile charge
carriers, and I = 20 m is the length of the conductor. From
tables, we find the electron charge e = 1.6 X 10"19 C and
the resistivity of steel, p = 1.2 X 10"7 Q-m.
Find: the mean velocity v of the charge carriers.
Solution. The mobile charge carriers in metals are free, or
conduction, electrons. Consequently, we must find the mean
velocity v of the ordered motion of free electrons.
§ 10. Electric Current in Metals. Ohm’s Law 111

In the electron theory of conductivity. Ohm’s law is for­


mulated as follows: / = envS, whence

The current can be determined from the formula / — UIR,


where R should be expressed in terms of the length and the
cross-sectional area: R = pl/S. Substituting the expression
for current into the formula for velocity, we obtain

- US - _____u_
p lenS ’ p len ’

3.6 V
— 1.2X10-7 Q -m x 2 0 m X l.6 x l0 -ls C x 4 .0 x l0 sem“3

~ 2.343 x 10"4 m/s.

Answer. The mean velocity of ordered motion of the free


electrons is approximately 0.23 mm/s.
Problem 55. A telegram is sent from Leningrad to Moscow
(the distance between the cities is 650 km) via a steel tele­
graph line carrying a current of 1.7
mA at a voltage of 150 V. The cross-
sectional area of the wire is 5.0 mm2.
Determine the voltage drop in the
wires and the voltage at the receiving
end (Fig. 28).
Given: I = 6.5 X 105 m is the dis­
tance between Leningrad and Moscow,
I = 1.7 xiO"3 A is the current in the
wires, C/j — 150 V is the voltage at the beginning of the
transmission line, and S = 5 x 10"" m2 is the cross-sec­
tional area of the wire. From tables, we find the resistivity
of steel, p = 1.2 x 10"7 Q-m.
Find: the voltage drop £/w in the wires and the voltage U2
at the end of the line.
Solution. The length of the wire forming the electric cir­
cuit (which must be closed) is twice the distance between
the cities.
112 Ch. II. Fundamentals of Electrodynamics

The voltage drop in the wires is f/w = //? w, where


= p Then

Uw - I p ~ ,

T1 1.7 x 10“3 A x 1.2xlO “7Q -m x 2 x 6 . 5 x 106 m


U w = ----------------------:----- 77—7:— 5-------------------- ~ 00 V.
w 5 x l 0 " “ m2

The voltage at the end of the transmission line is

U2 = Uj - Uw, U2 = 150 V - 53 V = 97 V.

Answer. The voltage drop in the wires is approximately


53 V, and the voltage at the end of the line is 97 V.
Problem 56. What is the change in the resistance of a tele­
graph line due to the change from winter temperatures to
summer temperatures if the line is made from a steel wire
having a cross-sectional area of 5.0 mm2? The temperature
changes from —30 to +30°C. The length of the wire at 0°C
is 200 km. The linear expansion of the wire should be neg­
lected.
Given: S = 5.0 X 10~6 m2 is the cross-sectional area of
the wire, = —30°C is the winter temperature, t2 =
+30°C is the summer temperature, and I = 2.0 X
105 m is the length of the wire. From tables, we find the resis­
tivity of steel at 0°C, p0 = 1.2 x 10"7 £2*m, and the tem­
perature resistance coefficient for steel, a = 0.004 K -1.
Find: the change AR in the resistance of the telegraph line
due to the seasonal change in temperature.
Solution. The resistance of the line at 0CC is R 0 = pnl/S,
while the resistances at t x and t2 are = p0 (1 + ati)
and R 2 -- p0 —(1 -f- a t 2). The change in the resistance is
given hy

AR = R 2 — R {, Afl = p0- |- ( l 4- at2— 1 —a<t) ==p0 — /,)■


Since the absolute values of the temperatures are equal
(|fil = | t2 |), we can write
§ 10. Electric Current in Metals. Ohm’s Law 113

AR -= p„-J- 2a<it

Afl = 1.2 X 10- 7Q- m X 2 x 0.004 K~i x 30 K

~ 1.2 kQ.
Answer. The resistance of the wire has increased by about
1.2 kS2 as a result of the change from winter to summer tem­
peratures.
Problem 57. Calculate the resistance of the circuit in
Fig. 29a.
Given: the resistances R l = 6Q, R 2 = 5Q, R 3 = 4Q ,
R 4 = 12 Q, and R 5 = 8 Q of the resistors.
Find: the total (equivalent) resistance R of the circuit.
Solution. Whil^ solving problems where loads are con­
nected in series and in parallel, it is expedient to replace

a branched circuit by equivalent circuits as shown in


Figs. 29b-e.
Since resistors R a and R t are connected in parallel, their
equivalent resistance is
^3-^4 4QX12Q
f l i - ^3 + ^4 Ri 3Q
412 + 12Q
(Fig. 296).
Resistors R t and R 2 are connected in series, and their
equivalent resistance is i?,, = R 2 + /?,, R tl = 5 Q +
3 S2 = 8 Q (Fig. 29c).
8-0530
114 Ch. II. Fundamentals of Electrodynamics

Resistors R tt and R s are connected in parallel, and hence


their equivalent resistance is
8 Qx8 Q
R
ffm /fn + /f5 * " HI 8 Q + 8 2 '= 4Q
(Fig. 29(f).
Resistance R lir can be determined in a different way:
R llt = R J 2 = 4 Q.
The required resistance R is
R = f?i -)- f?uj, f? = 6 f l - f 4 Q = 10 Q
(Fig. 29e).
Answer. The total resistance of the circuit is 10 {2.
Problem 58. When a current source with an emf of 4.2 V is
connected to a nickeline wire 10 m long and having a diam­
eter of 1.0 mm, the current in the circuit is 0.6 A. Deter­
mine the internal resistance of the current source.
Given: the emf % — 4.2 V of the current source, I =
10 m is the length of the nickeline wire, d = 1.0 X 10-3 m
is the diameter of the wire cross section, and I = 0.6 A is
the current in the circuit. From tables, we find the resistivity
of nickeline, p = 4.2 x 10~7 Q-m.
Find: the internal resistance r of the current source.
Solution. We shall solve the problem by using Ohm’s law
for a closed circuit:
I %
R + r'
whence

The external resistance R can be determined from the for­


mula R — pl/S. Considering that S = Juf2/4, we obtain
4p2 4 x 4 . 2 x l O “7 Q-mxlOm
R = 5.4 Q.
3.14x1.0x10-“ma
Then
r = :^U.b¥Ar - 5.40 = 1.60.
Answer. The internal resistance of the current source
is 1.6 Q.
Problem 59. A battery of cells with an emf of 3 V and an
internal resistance of 0.25 Q supplies power to a circuit
§ 10. Electric Current in Metala. Ohm’a Law 115

consisting of four resistors R t = 1.0 £2, R a = 3.0 £2, R 3 =


1.5 Q, and R t = 0.75 Q connected as shown in Fig. 30a.
Determine the current in the unbranched subcircuit and
the voltage drop in the battery.
Given: g = 3 V is the emf of the battery, r = 0.25 £2 is
the interna] resistance of the battery, and R t = 1.0 £2,
R a = 3.0 £2, i f , = 1.5 £2, and Jt4 = 0.75 £2 are the resist­
ances of the resistors.
Find: the current / in the circuit and the voltage drop
(/,„t in the battery.
Solution. We shall solve the problem with the help of
Ohm's law for a closed circuit: I = £/(/? + r). Therefore,
(a) M ^

we must determine the external resistance R. For simplicity,


we represent the circuit as shown in Fig. 305. It is clear
from the circuit diagram that the resistors R x and R 2 are
connected in parallel. Their equivalent resistance R t
can be found from the formula
D 1.0 Qx3.0 Q
0.75 Q.
" I — 1 .0 0 + 3 .0 0
The subcircuit with resistance R t and resistor R t are con­
nected in series. Their equivalent resistance R IX can be de­
termined as follows:
R jj — i?j + f?!, = 0.75 Q + 0.75 £2 — 1.5 £2.
Resistances R lt and R 3 can be replaced by an equivalent
resistance R which can be determined from the rules gov­
erning parallel connection of loads. Since these resistances
are equal, we have
R = R J 2 = 0.75 £2.
!•
116 Cfr- II. Fundamentals of Electrodynamics

The current in the unbranched circuit is


3.0 V
/ = 3 A.
n.75 ft-(-0.25 Q
The voltage drop in the current source can be determined
from Ohm’s law for a conductor:
f/lnt = /r, Uint = 3 A X 0.25 Q = 0.75 V.
Answer. The current in the unbranched circuit is 3 A and
the voltage drop in the current source is 0.75 V.
Problem 60. A galvanometer with an internal resistance of
19.8 Q can be used to measure currents up to 10 mA. What
must be done to extend its range to 1.0 A? How can it be
used to measure voltages up to 10 V?
Given: R g = 19.8 Q is the resistance of the galvanometer,
Ig = 0.01 A is the current through the galvanometer, / =
1.0 A is the current in the circuit, and U = 10 V is the
voltage to be measured.
Find: the resistance of the shunt i?sh and the resistance
R s of the series resistor.
Solution. If a galvanometer is used as an ammeter (in this
case it is connected in series to a circuit), a shunt (resistor)
R sh is connected in parallel to it as shown in Fig. 26. The
resistance of the shunt can be calculated by using the rules
of parallel connection:
_«g_ 7 sh 7 - A = n — 1.
A h —I A’ 7e
7?sh A ’ 7?sh

where n
Je
R g *> 19.8 Q
^sh r ’ /fsh = 99 0.2 Q.
If a galvanometer is used as a voltmeter (in this case it is
connected in parallel to the subcircuit in which the voltage
drop U has to be measured), a resistor R s is connected in
series to it as shown in Fig. 27.
The voltage drop U is distributed in proportion to the
resistances R g and R s:
U —Ug Rs U n Rs
~Wg i v tj^ - ' - r ^'

Rt —(n i) Rg' where n = V g = ~ r ^ '


§ 10. Electric Current in Metals. Ohm’s Law 117

10 V
0.01 AX 19.8 Q
=50,
= 49 X 19.8 Q ~ 970 J2.
Answer. In order to measure current, the galvanometer
must be shunted by a resistor with / f sh = 0.2 Q. To measure
voltage, a series resistor with re­
sistance R s ~ 970 Q should be
connected to the galvanometer.
Problem 61. How should two gal­
vanic cells having an emf of 1.45 V
each and an internal resistance of
0.4 Q each be connected in order to
obtain the maximum current when
the circuit is closed with an exter­
nal resistance of 0.65 Q?
Given: n = 2 is the number of
cells in the battery, g = 1.45 V is
the emf of a cell, r = 0.4 Q is the
internal resistance of a cell, and R 0.65 Q is the resist­
ance of the external circuit.
Find: I ser, the current for the series connection and the
current / par for the parallel connection of the cells.
Solution. In order to find out which is the best way to
connect the circuit, we determine the currents for the cells
in series and in parallel and compare them:
ing 2x1.45 V
= 2 A,
fl + nr 0.65 Q + 2x" .4 Q
g 1.45 V
= 1.7 A.
Par R-\-r!n ’ Par 0.65Q + 0 .4 Q/2

Answer. It is better to connect the cells in series.


Problem 62. The circuit shown in Fig. 31 contains three re­
sistors /?! = 100 Q, R 2 = 50 Q, and R 3 = 20 Q and. gal­
vanic cel]s w ithem f’sg ! = 2V and g2. The ammeter indi­
cates a current of 50 mA. Determine the currents in the re­
sistors and the emf of the second cell. The internal resist­
ance of the ammeter and of the cells should be neglected.
Given: Ri = 100 Q, R 2 = 50 £2, and R a = 20 Q are the
resistances of the resistors, g t = 2 V is the emf of the
first cell, / A = 0.05 A is the reading of the ammeter.
118 Ch. II. Fundamentals of Electrodynamics

Find: the currents / , , / 2, and J3 in the resistors and the


emf %2 of the second cell.
Solution. We choose the direction of the current arbitrarily
and indicate it by arrows on the circuit diagram. Since the
algebraic sum of the currents flowing to and from a junction
is zero (Kirchhoff’s first rule), we can w r ite /x — I 2 — I 3 =
0, whence
/. = A - h- (1)
The current indicated by the ammeter is / A = / 3.
Let us go around the circuit clockwise and indicate this
direction on the circuit diagram.
Since the algebraic sum of voltages in a closed circuit is
equal to the algebraic sum of emf's (Kirchhofl’s second rule),
we can write
for the circuit ABCDFA
- I J - I 2R2 = - I lt or I lRl + / 2fl2 = (2)
for the circuit AFMNA

IlRl + I 3R 3 = r ,. (3)
We determine I x from Eq.(l) and substitute it into Eq.(2):

1\ — I a + (^3 + ^*) + 1 2 ^ 2 — ^ 1-
This gives
1 - h R i T 2 V —0.05 A x 10112
i 1%— - 0 .0 2 A.
2 Bi-i-R 1i HIQ + 50!2

The minus sign indicates that the current / 2 flows in the op­
posite direction to that indicated on the diagram.
The current is
/ j = I a + I 2 = 0.05 A - 0.02 A = 0.03 A.
Substituting I x into Eq. (3), we obtain g 2:
t 2 = 0.03 A x 100 £2 + 0.05 A x 20 £2 = 4 V.
Answer. The currents in resistors R x, R 2, and R 3 are
0.03, —0.02, and 0.05 A respectively, the emf of the second
cell is 4 V.
§ 10. Electric Current in Metals. Ohm’s Law 119

Questions and Problems

Current. Resistance. Ohm's law for a conductor


10.1. The anode current in a vacuum tube is 12 mA. How
many electrons arrive at the anode per second?
10.2. How many electrons pass through the cross section of
the contact wire of a tram in 2 s if the current is 500 A?
10.3. Determine the current in the contact copper wire of
a trolleybus circuit if the number density of conduction
electrons in copper is 3 x 1023 cm '1, and the mean velocity
of their ordered motion is 0.25 mm/s. The cross-sectional
area of the wire is 85 mm2.
10.4. Determine the number density of conduction elec­
trons in copper if the current in a copper wire with a cross-
sectional area of 105 mm2 is 500 A, and the mean velocity of
the ordered motion of conduction electrons is 0.1 mm/s.
10.5. A voltage of 18 V is applied across the ends of a cop­
per wire 200 m long. Determine the mean velocity of the
ordered motion of electrons in the conductor if the number
density of conduction electrons in it is 3.0 X 1023 cm-3.
10.6. What is the electric field strength in an aluminium
wire of cross-sectional area 6 mm2 for a current of 9 A if the
potential difference between the ends of the wire is 21 V,
the mobility of conduction electrons in aluminium is 7 x
10~3 m2/(V-s), and the electron number density in it is
3.7 x 1022 cm-3? How long is the conductor?
10.7. The coil of a hot plate carries a current of 2.7 A
with an electric field strength of 3.1 V/m. Determine the
mobility of the conduction electrons if their number density
in the coil is 1023 cm"3, and the cross-sectional area of the
wire is 0.10 mm2.
10.8. Determine the current density in a conductor if
0.15 C of electricity pass per second through a cross-sectipnal
area of 6 mm2. Is the admissible current density important
when choosing the wire for a residential building?
10.9. A constant current of 3.6 mA is maintained when
gold plating an article 1.2 x 15 cm* in area. Determine
the current density.
10.10. What must the diameter of a copper wire be if it
is to carry a current of 1000 A and the admissible current
density is 2.5 A/mm2?
120 Ch. II. Fundamentals of Electrodynamics

10.11. Determine the current density in the excitation


winding of the traction motor of a locomotive if the cross-
sectional area of the winding is 4.7 x 25 mm2 and the
nominal current is 725 A.
10.12. What is the resistance of an aluminium wire 2 m
long and having a cross-sectional area of 1 mm2? What
will the resistance of the same length of wire with twice
the cross-sectional area be?
10.13. Two wires, one of nickeline and the other of nich-
rome, have the same length and diameter. Which of them
has a higher resistance? What is the ratio between their
resistances?
10.14. An iron wire and a tungsten wire of the same length
and diameter are connected in turn to an accumulator. In
which wire will the current be stronger? What is the ratio
of the currents?
10.15. A long conductor was cut into two halves which
were then twisted along the whole length. How does the
resistance of the wire change?
10.16. Determine the capacitance of a capacitor the volt­
age across whose plates is 1 V as a result of charging for 12 s
with a current of 10~a A.
10.17. An electric circuit whose conductance is 2.4 x
10‘2 S is connected to a d.c. source of 50 V. Determine the
current in the circuit and its resistance.
10.18. Determine the length of a manganin wire required
to make a potentiometer rated at a maximum resistance of
1500 Q if the diameter of the wire is 0.3 mm.
10.19. A potentiometer is made of 2.25 m of constantan
wire having a diameter of 0.15 mm. Determine the resistance
of the potentiometer.
10.20. A bundle of copper wire having a mass of 3.6 kg
has a resistance of 22.5 S2. Determine the length of the
wire in the bundle.
10.21. A coil made of 75 m of constantan wire whose di­
ameter is 0.1 mm is connected to a current source supplying
a voltage of 12 V. Determine the current in the coil.
10.22. The admissible current density of the wiring in
a room is 6 A/mm2. What is the voltage drop in the copper
wire if its length is 30 m?
10.23. Determine the voltage drop and the current density
in the winding of the magnetic pole of a locomotive motor.
§ 10. Electric Current in Metals. Ohm’s Law 121

The winding is made of copper strip having a cross-sectional


area of 28 x 4.7 mm2, the average length of a turn being
1.5 m, and the number of turns is 40. The current in the
winding is 352 A.
10.24. An electric appliance is 0.5 km from a current
source and is connected to it through a wire having a cross-
sectional area of 5 mm2. The appliance is moved 1 km further
away from the source. What should the cross-sectional area
of the wire be for the veltage drop to remain unchanged?
10.25. How much copper is required to manufacture the
contact wire of a tram circuit having a resistance of 0.2 Q if
the cross-sectional area of the wire is 85 mm2?
10.26. Determine the resistance of 1 km of tram steel rails
if the mass of the rails is 55 kg/m.
10.27. Two wires, one of copper and one of nichrome, of
the same length and diameter are heated through 5 K. The
resistance of which wire changes more? What is the ratio of
the resistances?
10.28. The series resistance connected to the ignition coil
of a “Moskvich” motor car is 1.35 Q at 293 K and 2 Q at
373 K. Determine the material of which it is made.
10.29. The starter rheostat of an electric train is made of
a cast iron plate and a ferro-aluminium high-resistance rib­
bon alloy. Its resistance is 4 Q at 298 K. What will the
change in its resistance be when the temperature is 723 K?
10.30. What must the temperature resistance coefficient
of the material used for potentiometers be?
10.31. The tungsten filament of an incandescent lamp has
a resistance of 20 £2 when cold. What will its operating re­
sistance be, i.e. when the filament is heated to 2100°C?
10.32. What is the increase in the temperature of the cop­
per winding of a d.c. motor armature as a result of pro­
longed operation during which the resistance has increased
from 2 to 2.2 £2?
10.33. An electric arc rated for a voltage of 45 V and
a current of 10 A is connected to a circuit at a voltage of
110 V. Determine the required resistance of the series re­
sistor if the resistance of the leads is 0.5 II.
10.34. What is the increase in the temperature of a poten­
tiometer made of iron wire if its resistance has doubled?
10.35. The voltage at an electric substation is 600 V
(Fig. 32). The two copper cables AB and CD connecting the
122 Ch. II. Fundamentals of Electrodynamics

substation with a trolleybus’ contacts are each 440 m long


and 400 mm2 in cross-sectional area. The contact wires are
made of copper and have a cross-sectional area of 85 mm2.
The current in the trolleybus motor is 200 A. Determine the
voltage across the current collectors of the trolley.
10.36. The experimental d.c. transmission line between
Kashira and Moscow operates at a voltage of 200 kV and
HOm
Substation
I--------1 Contacts
I

Fig. 32

is made of a single-core aluminium cable 112 km long. De­


termine the cross-sectional area of the cable if the voltage
drop in it is 3.1 % of the nominal value at a current of 150 A.
10.37. A d.c. transmission line under a voltage of 1500 kV
is 2500 km long. What will the voltage at the end of the
line be if it is made of an alu­
B, minium wire with a cross-
sectional area of 600 mm2? The
admissible current load in the
I---------& J wire is 1070 A.
10.38.
Fig. 33 ters in Fig. 33 will show a
higher reading? What will the
change in the resistance of the circuit be when bulb B j
fuses? Will the reading of ammeters and A 2 change in
this case?
10.39. The starter rheostat of a railway engine consists of
80 plates having a resistance of 0.05 Q each and connected
in series. Determine the resistance of the starter rheostat.
10.40. The armature winding of a traction motor consists
of 132 sections connected in series in two parallel branches.
The resistance of one section is 0.001 Q. Determine the
resistance of the whole winding.
§ 10. Electric Current in Metals. Ohm’s Law 123

10.41. Determine the total resistance, the current, and the


voltage distribution in a circuit (Fig. 34), if the applied
voltage is 220 V.
10.42. What series resistor should be connected to the
heating element of an electric iron having a resistance
R = 24 Q and rated at a voltage of 120 V in order to use
it at a voltage of 220 V?
10.43. A circuit is constructed as shown in Fig. 34 and
connected to a d.c. source of 120 V. It is known that R t =

Fig. 34 Fig. 35

6 Q, R t = 15 Q, and / = 5 A. Determine the total re­


sistance of the circuit and the resistance of resistor R s.
10.44. Figure 35 shows the excitation circuit for the trac­
tion motor of a trolleybus. The resistances of the sections of
the rheostatic controller are R t = 1040 Q, R 2 = 200 £2,
R a = 90 Q, and /?4 = 10 Q. The resistance of the excitation
winding is R ex = 160 Q. Determine the excitation current
with one of the switches 1, 2, or 3 closed and without the
switches.
10.45. The magnetic field in the motor of an electric loco­
motive is produced by four series-connected windings. De­
termine the resistance of the windings if the number of
turns in one of them is 39, the average length of a turn is
1.5 m, and the current in the windings is 352 A, the admis­
sible current density being 2.7 A/mm*.
10.46. How many equal pieces should a conductor of re­
sistance 4 Q be cut into and how should these pieces be con­
nected to obtain a resistance of 1 Q?
10.47. What resistor should be connected in parallel to
a resistor of 100 Q for their total resistance to be 20 Q?
10.48. Determine the total resistance of a lamp resistor,
the current in the circuit and through each lamp if the re-
124 Ch. 11. Fundamentals of Electrodynamics

sistor contains 10 parallel-connected lamps having a re­


sistance of 440 £2 each, and the voltage applied is
220 V,
10.49. Three resistors of 1, 2, and 16 £2 are connected in
parallel to a d.c. source. The current in the unbranched cir­
cuit is 5 A. Determine the total resistance of the circuit and
the current through each resistor.
10.50. When two conductors are connected in parallel,
their equivalent resistance is 0.72 £2. The equivalent re­
sistance for the same conduc­
tors connected in series is 3 £2.
Determine the resistance of
f each conductor.
* 10.51. Three incandescent
lamps having a resistance of
xs-in 400 £2 each are connected to a
circuit at a voltage of 220 V.
Fig- 36 Determine the current passing
through each lamp if they are connected in parallel and in
series. Determine the total current in each case.
10.52. Five resistors are connected as shown in Fig. 36.
They are connected to a 12-V d.c. source. Determine the
total resistance of the circuit and the current in resistors
R i, 7?3, and R x.
10.53. A circuit is formed as shown in Fig. 36. All the
resistors have the same resistance of 1.2 £2. The current
through the first resistor is 10 A. Determine the total re­
sistance, the voltage across the terminals of the circuit, and
the currents and voltages in each resistor.
10.54. The filament of an incandescent lamp is heated to
about 2200°C and fuses more often when it is being switched
on than when it is being switched off. Why? What conclu­
sions can be drawn from a comparison of the currents through
the lamp in the two cases? The material of the filament is
tungsten.
10.55. Two resistors R x = 12 £2 and R 2 are connected in
parallel to a current source. Determine the resistance R t
and the current through it if the total current is 4 A and
the voltage supplied to the resistors is 12 V.
10.56. Determine the voltage across the current col­
lectors of trolleybuses if the contact wire is made of copper
and has a cross-sectional area of 85 mm2. The lengths of the
§ 10. Electric Current in Metals. Ohm’s Law 125

wires and the currents are indicated in Fig. 37. The generator
voltage UBC is 575 V.
10.57. Determine the voltage drop across a railway track
if an electric locomotive is 800 m from the cable connecting

SOOA
o
500A
o-
SOOA

Fig. 38

the rails with the substation (Fig. 38). The cross-sectional


area of the cable is 72 cm2.
10.58. Figure 39 shows the circuit diagram of a starter
rheostat consisting of three sections. Determine the re-

L j r iz, 0,1
l_ t x j f T _ f

Fig. 40

sistance of the rheostat for the following three positions:


(a) switch 1 is closed, (b) switch 2 is'closed, (c) switches 1
and 4 are closed, (d) switches 2 and 4 are closed, (e) switches
1, 2, and 4 are closed, and (f) switches 2, 3, and 4 are closed.
10.59. Determine the resistance of two sections of a starter
rheostat if the resistance of a cast iron plate is 0.06 Q. The
first section contains 10 such plates and the second 12 plates.
The plates are connected as shown in Fig. 40.
120 Ch. II. Fundamentals of Electrodynamics

10.60. Determine the resistance of a starter rheostat con­


sisting of two series-connected sections. In the first section,
22 coils are connected in two parallel groups, each of which
contains 11 series-connected coils. In the second section,
45 coils are connected into
three parallel groups of 15 se­
ries-connected coils each. The
coils are made of ferro-alumin-
ium high-resistance alloy
wire 1.6 m long and 3 mm in di­
Fig. 41 ameter.
10.61.
of a tram car consists of two
parallel groups each containing five series-connected bulbs.
Determine the total current in the circuit and in the groups
if the resistance of each bulb is 220 Q and the voltage in the
circuit is 550 V.
10.62. Incandescent lamps, with a resistance of 440 Q
each, are connected to a circuit at a voltage of 220 V as

Fig. 42 Fig. 43

shown in Fig. 41. Determine the total resistance of the lamps


and the current and voltage in each lamp.
10.63. What will the change in the voltage and current in
the lamps (see Problem 10.62) be if one of the lamps fuses?
10.64. The armature winding in a locomotive motor con­
sists of 924 copper rods 1 m long each. The rods are distrib­
uted equally among four parallel branches (Fig. 42). De­
termine the resistance of the armature winding for a current
of 352 A, assuming that the current density is 5 A/mm!.
10.65. Figure 43 presents a circuit diagram. The voltage
UAB is 120 V. Determine the resistance of the circuit, the
unbranched current, and the current in each resistor.
§ 10. Electric Current in Metals. Ohm’s Law 127

10.66. Figure 44 shows a circuit diagram. The voltage


UAB is 229 V. Determine the total current and the current
in parallel branches.
10.67. A current up to 50 A is to be measured with an
ammeter whose scale is rated to 10 A. Determine the re-
X ,’ 129

sistance of the shunt if the resistance of the ammeter is


0.01 Q (see Fig. 26).
10.68. An 0.18-Q ammeter indicates a current of 6 A. The
ammeter has a shunt whose resistance is 0.02 £2. Determine
the current in the mains (see Fig. 26).
10.69. A voltmeter with a scale from 0 to 150 V has to be
used for measuring voltage from 0 to 250 V. Calculate the
resistance of a series resistor if the resistance of the volt­
meter is 600 £2 (Fig. 45).
10.70. A voltmeter rated for the maximum voltage of
20 V at a current of 8 mA has to be used for measuring a volt­
age of 100 V. What series resistance is required?
10.71. A voltmeter having an internal resistance of 1000 £2
is connected to a circuit with a constant voltage supply in
series with a series resistor R s and indicates 180 V. If another
identical series resistor is connected to the circuit, the volt­
meter indicates U2 = 150 V. Determine the resistance of the
series resistor R s and the voltage U in the circuit.
10.72. The maximum current that can be measured with
a milliammeter having an interval resistance R = 150 £2
is / = 10 mA. How long must a manganin wire of diameter
d = 0.1 mm be if it is used as the series resistor to convert
the instrument into a voltmeter with a scale from 0 to 10 V?
10.73. The internal resistance of a voltmeter is 300 £2.
A series resistor of 1200 £2 is connected to it in a measuring
circuit. What is the ratio of the scale factors of the volt­
meter with and without the series resistor?
128 Ch. II. Fundamentals of Electrodynamics

Ohm's law for a closed circuit. Connection of batteries


10.74. A voltmeter is connected to the terminals of a cur­
rent source. The external circuit is closed. What does the
voltmeter measure in this case?
10.75. A cell whose internal resistance is 0.4 Q is closed
through an external circuit having a resistance of 2.1 £2.
The current in the circuit is 0.6 A. Deter­
mine the emf of the cell and the voltage
drop across it.
10.76. Figure 46 shows a circuit diagram.
When the resistance of the external circuit
is 1.5 £2, the voltmeter indicates 1.65 V,
while for an external resistance of 3.6 £2 the
reading of the voltmeter is 1.8 V. Determine
the emf and the internal resistance of the cell.
10.77. A galvanic cell with an emf of 1.45 V and an inter­
nal resistance of 1.5 £2 is closed through an external re­
sistance of 3.5 £2. Determine the current in the circuit, the
voltage across the cell terminals, and the efficiency of the
cell in this circuit.
10.78. Determine the emf of a generator and the voltage
across its terminals if its resistance is 0.05 £2 and the exter­
nal resistance is 11.45 £2. The current in the circuit is 20 A.
10.79. Determine the current in a circuit if the emf of the
generator is 230 V, its resistance is 0.1 £2, and the resistance
of the external circuit is 22.9 £2.
10.80. A resistor of 2.2 £2 is connected to a generator with
an emf of 230 V. What is the resistance of the generator if
the voltage across its terminals is 220 V?
10.81. A galvanic battery with an emf of 15 V and an
internal resistance of 5 £2 is closed through a conductor
whose resistance is 10 £2. A 1-pF capacitor is connected to
the battery terminals. Determine the electric charge on the
capacitor.
10.82. A 2-pF capacitor and a 3-£2 resistor are connected
in parallel to a current source with an emf of 10 V and an
internal resistance of 1 £2. What is the electric charge on the
capacitor?
10.83. If a current source is closed through a 13-£2 re­
sistor, the current in the circuit is 0.8 A. If the source is
closed through an 8-£2 resistor, the current in the circuit is
§ 10. Electric Current in Metals. Ohm’s Law 129

1.2 A. Determine the emf and the internal resistance of the


current source.
10.84. A d.c. generator with an emf of 150 V and an inter­
nal resistance of 0.3 £2 supplies voltage to 20 incandescent
lamps having a resistance of 240 £2 each and connected in
parallel. The resistance of the leads is 2.7 £2. Determine the
voltage across the generator terminals and
across the lamps.
10.85. The emergency lamps of a tram
car are fed by an accumulator battery
having an emf of 48 V and an internal
resistance of 0.2 £2. Ten lamps having a
resistance of 39.5 £2 each are connected as
shown in Fig. 47. Determine the current
in each lamp and in the leads.
10.86. A circuit contains 20 parallel-
connected bulbs. The current through a
bulb is 1 A. The resistance of the wires
connecting the load with a generator is 0.2 £2. What must the
emf of the generator be for the voltage across the bulbs to
be 220 V? The internal resistance of the generator is 0.05 £2.
10.87. Three electric motors and ten parallel-connected
incandescent lamps are connected to a generator with emf

Fig. 48 Fig. 49

240 V and resistance 0.025 £2. A current of 50 A passes


through each motor while the current through each lamp
is 1 A. The resistance of the leads is 0.1 £2. Determine the
voltage across the generator terminals and across the loads.
10.88. A workshop receives electric power from a collective
farm’s electric power station. There are two electric motors
(M ) and four bulbs in the workshop connected as shown in
Fig. 48. The current through each motor is 10 A, and through
9-05S0
130 Ch. II. Fundamentals of Electrodynamics

each bulb 0.5 A. The distance I between the station and the
workshop is 0.5 km. The internal resistance of the generator
is 0.1 Q and the voltage across its terminals is 220 V. De­
termine the emf of the generator and the cross-sectional
area of the copper leads if the admissible voltage drop in
them is 8 %.
10.89. Determine the counter emf of a traction motor if
the resistance of its windings is 0.1 Q and the voltage in the
circuit is 550 V at a current of 150 A.
10.90. Determine the emf of a generator with an internal
resistance of 0.05 Q and the counter emf of a motor if the
current in the circuit is 100 A, the voltage across the gen­
erator terminals is 225 V, and the resistances of the motor
winding and the leads are 0.2 and 0.1 Q respec­
tively.
10.91. The circuit diagram of a d.c. motor is shown in
Fig. 49. The voltage in the circuit is U — 550 V, and the
current is / = 102 A. The resistance of the armature circuit
is R a = 0.1 Q, that of the parallel excitation winding
R ex = 150 Q, and that of the rheostatic controller R r =
125 Q. Determine the current in the parallel excitation
winding when the rheostatic controller is completely on, and
the counter emf if the electric motor is started without a
starter rheostat.
10.92. Four loads having a resistance of 10 Q each are con­
nected to an accumulator battery having an emf of 48 V
and an internal resistance of 0.25 £2. Determine the current
through the battery if the loads are connected (a) in series,
(b) in parallel, and (c) in two parallel branches containing
two series-connected loads each.
10.93. One of two cells has an emf of 1.45 V and an internal
resistance of 0.5 Q and supplies voltage to a circuit with an
efficiency of 90%, while the other cell has an emf of 2 V and
an internal resistance of 0.5 £2 and operates with an effic­
iency of 80% in an identical circuit. Determine the current
in the two circuits.
10.94. An accumulator with an emf of 1.45 V produces
a current of 0.5 A in a conductor whose resistance is 2.5 £2.
Determine the short-circuit current.
10.95. During a short circuit, the current from a source of
1.8 V is 6 A. What must the external resistance be for the
current to be 2 A?
§ 10. Electric Current in Metals. Ohm’s Law 131

10.96. Why should lead accumulators not be short cir­


cuited?
10.97. A galvanic cell is first connected to an external
resistance of 1.9 12 and the current in the circuit is 0.6 A.
Then it is connected to an external resistance of 2.4 12 and
the current becomes 0.5 A. What is the short-circuit current
for this cell?
10.98. Three galvanic cells, each having an emf of 1.5 V
and an internal resistance of 0.6 12, are connected in series
to a conductor having a resistance of 1.8 12. Determine the
current in the circuit. What will the current in the same
conductor be if the cells are connected in parallel?
10.99. Using the data in Problem 10.98, determine the
connection for which the short-circuit current is the
largest.
10.100. How should six galvanic cells, each having an emf
of 1.5 V and an internal resistance of 0.5 12, be connected in
parallel groups to obtain the maximum current when con­
nected to a 1.0-12 resistor?
10.101. How should 12 cells, each having an emf of 1.2 V
and an internal resistance of 0.6 12, be connected to produce
a current of 1.6 A in an external circuit whose resistance is
2.2 Q?
10.102. A “Krona” battery is the power source for tran­
sistor radios. It contains seven series-connected galvanic
cells. What is the emf of a cell if the emf of the battery is
9 V? What will the change in the emf of the battery be if the
constituent cells are connected in parallel?
10.103. Two alkaline accumulators, each having an emf of
1.5 V and an internal resistance of 0.3 12, are connected first
in series and then in parallel and used to supply voltage to
a circuit with a resistance of 6 12. Which connection produces
the higher current in the external circuit, i.e. which method
is more advantageous?
10.104. Solve Problem 10.103 for the case when the exter­
nal resistance of the circuit is 0.9 12. Using the results of the
two problems, find the ratio between the external and inter­
nal resistances for which connecting the current sources in
series is more advantageous.
10.105. A passenger carriage is lighted using a battery
containing 26 series-connected lead accumulators. The emf
of an accumulator is 2 V and the internal resistance is
9*
132 Ch. II. Fundamentals of Electrodynamics

0.004 Q. Determine the emf and the voltage across the ter­
minals of the battery if the current in the circuit is 20 A.
10.106. What is the reading of a voltmeter connected to
the terminals of a battery consisting of three series-connected
alkaline accumulators, each with an emf of 1.2 V and an
internal resistance of 0.3 Q? The external circuit consists

Fig. 50 Fig. 51
of a bulb with a resistance of 16 Q and 2-m long aluminium
leads with a cross-sectional area of 0.2 mm2. Determine the
voltage on the bulb.
10.107. An accumulator battery, having an emf of 22 V
and an internal resistance of 0.1 Q, is charged from a recti­
fier the voltage across whose terminals is 24 V. Determine
the resistance introduced into the circuit by a potentiometer
connected in series with the battery if the charging current
is 4 A.
10.108. Two accumulators, having emf’s of 1.3 and 1.8 V
and internal resistances of 0.1 and 0.15 Q respectively, are
connected in parallel. Determine the current in the circuit
and the voltage across the terminals (Fig. 50).
10.109. Determine the current through a resistor R = 2 Q
connected in a circuit as shown in Fig. 51. Given: t x =
2 V, rj = 0.50 Q, g 2 = 4.0 V, and r2 = 0.70 Q.§

§ 11. WORK, POWER, AND THE THERMAL EFFECT


OF CURRENT

Basic Concepts and Formulas


Electric energy is transformed into other kinds of energy so
that energy is conserved. The measure of the transformation
is the work done by electric current:
A = Q U = JUt,
§ 11. Work, Power, and the Thermal Effect of Current 133

If the current or voltage are substituted using Ohm’s law


for a conductor we obtain
A = I U t , l zRt = — f.
Power is the ratio of the work done by an electric current
during time t to the time t :
/» = - , p^w = r-R= ^-.
The unit of work is the joule (J). In electrical engineering
and everyday life, electrical work is measured in kilowatt-
hours (1 kWh = 3.6 MJ). The unit of power is the watt
(1 W = 1 J/s).
Joule’s law: the amount of heat liberated in a conductor
carrying a current is proportional to the square of the current,
the time of its passage, and the resistance of the conductor:
Q = P R t.
This formula is for series-connected loads. For a parallel
connection, the formula is

Worked Problems
Problem 63. An electric motor operating for 5 h is driven
at the mains voltage of 380 V and a current of 35 A. The
resistance of the motor winding is 0.5 Q. Determine the
amount of energy consumed, the amount of heat liberated
in the winding during the operation, and the mechanical
work done by the motor.
Given: U = 380 V is the voltage at the motor terminals,
/ = 35 A is the current, R = 0.5 Q is the resistance of the
motor winding, and t = 5 h = 5 X 3600 s is the operation
time.
Find: the energy A consumed by the motor, the amount
of heat Q liberated in the winding, and the mechanical
work A mech.
Solution. The energy consumed or the total work done by
the current can be determined from the formula
A = IU t, A = 35 A x 380 V X 5 X 3600 s
~ 2.4 x 10* J.
134_________ Ch. II. Fundamentals of Electrodynamics___________

The amount of heat liberated in the winding of the motor


can be determined from Joule’s law:
Q = P R t,
Q = (35 A)2 x 0.5 Q x 5 x 3600 s = 1.1 X 107 J.
The mechanical work done by the motor can be deter­
mined by subtracting the energy spent in heating the wind­
ing from the whole energy spent:
A mech — A
A mech = 2.4 X 10® J - 1.1 X 107 J = 2.29 X 10® J.
Answer. The energy spent by the motor is approximately
2.4 X 10® J, the amount of heat liberated in the winding is
11 MJ, and the mechanical work is 2.3 X 10® J.
Problem 64. A tower crane, whose efficiency is 70% , hoists
a load of 49 kN at a constant velocity of 0.55 m/s. Determine
the current in its electric motors if driven from a supply
voltage of 380 V.
Given: r\ = 10% is the efficiency of the crane, G = 4.9 X
104 N is the force of gravity acting on the load, v =
0.55 m/s is the velocity of hoisting, and U = 380 V is the
voltage in the circuit.
Find: the current I .
Solution. We write the formula for the efficiency as the
ratio of the useful power (Gv) and the power spent (IU):

V = -jjf ioo%-
Hence we obtain the current:
/ = - $ -« X W ,
T _ 4 . 9 x l ° 4 N xft.55 m /sX l00% _ ifu A
7no/0 x 3 8 0 V
Answer. The current is approximately 101 A.
Problem 65. Two resistors of 40 and 80 Q are connected in
parallel to a source of constant voltage. The amount of heat
liberated in the first resistor is 3.0 X 10® J. How much
heat is liberated in the second resistor over the same time
and in both resistors if they are connected in series?
Given: = 40 Q is the resistance of the first resistor,
R 2 = 80 Q is the resistance of the second resistor, and Ql =
§11. Work, Power, and the Thermal Effect ol Current 135

3.0 X 10* J is the amount of heat liberated in the first re­


sistor.
Find: the amount of heat Qa liberated in the second re­
sistor, and the amount of heat liberated in the series-con­
nected resistors.
Solution. We use Joule’s law

Qi =
0)
<?2 =
Dividing these equations termwise, we obtain QJQt —
R t/Ri, whence we get Qt :
n R iQ\ n 4 0 S lx 3 .0 x in ‘ I
<?* = 80 Q
1.5 X 10* J.
If the resistors are connected in series, we have
n = -V lL _ ( 2)

Expressing UH from (1) and substituting it into Eq. (2), we


obtain
<>i*i 3.0x10* J x 4 0 a
Q = 1.0 x 10s J.
120 a
Answer. The amount of heat liberated by the second re­
sistor is 1.5 X 10* J, and the heat liberated by the series-
connected resistors is 1.0 X 10* J.
Problem 66. The air in a room loses 293 MJ of heat per
day. Determine the diameter of a nichrome wire 10.2 m
long from which the coil of an electric heater is made such
that the temperature in the room is kept constant if supplied
with a voltage of 220 V.
Given: Q = 2.93 X 10* J is the amount of heat lost, t =
24 x 3600 s is the time during which the heat is lost, I =
10.2 m is the length of the nichrome wire, U = 220 V is
the supply voltage. From tables, we find the resistivity of
nichrome, p = 1.05 x 10'* Q-m.
Find: the diameter d of the wire.
Solution. Let us determine the amount of heat lost by the
air in the room per second, i.e. the power of heat losses
Pi = QH.
The power of the electric heater can be determined from
the formula P t = U*/R.
136 Ch. II. Fundamentals of Electrodynamics

In order to maintain the temperature in the room, the


power of the heater must be the same as the power of heat
losses: P x = P 2, Qlt = U2/R. Hence we can find the re­
sistance of the heater coil: R = UH/Q. It is well known that
R = pl/S, where S = nd2/4. Then R = 4pl/nd2.
Equating the expressions for R , we obtain — = ■4pI
nd*
Hence
t / 4<?pZ
“ V U *tn '
4 x 2 .9 3 x 1 0 ® J X l . 05x10-* Q- mX 10.2 m
220 V X 220 V X 24 X 3600 9 X 3.14
= 9.5 x 10“'* m.
Answer. The diameter of the wire is approximately 1 mm.
Problem 67. How long will it take to boil 1 1 of water from
15°C using an electric immersion heater whose resistance
is 25 Q and efficiency is 85 %? The applied voltage is 120 V.
Given: V = 10~3 m3 is the volume of the water, £, = 15°C
is the initial temperature of the water, tt = 100°C is the
boiling point of water, R = 25 Q is the resistance of the coil,
U = 120 V is the applied voltage, and q = 0.85 is the ef­
ficiency of the heater. From tables, we find the density of
water, p = 103 kg/m3, and the specific heat of water, c =
4187 J/(kg-K).
Find: the time t required for boiling water.
Solution. Problems involving a given efficiency should
be solved starting with the formula for efficiency tj =
, where QTec = cm (t„ — <,), and ()Klv = —5- t. This
Vglv ‘ n
gives q = R ■ Hence we can find the time: t —
c m (<2 — <,) R
T| U*
Considering that the mass of water can be
expressed in terms of density and volume, m pV, we
obtain
cpV (tt — R
t x\W »
4187 J/(kg-K) X 103 Kg/m3 x 10~3 m3 x 85 K x 25 fl = 72 q g
0 .8 5 x 1 2 0 V x 120 V
= 12 min.
Answer. It takes 12 min to boil the water.
§ 11. Work, Power, and the Thermal Effect of Current 137

Questions and Problems

11.1. How much electricity will pass through the cross


section of a conductor in 30 s if its resistance is 20 Q and the
voltage across it is 12 V? Determine the work done by the
electric current.
11.2. What energy is supplied by a generator to an exter­
nal circuit over 8 h if the readings of the ammeter and volt­
meter remain unchanged at 50 A and 220 V respectively?
Express the answer in joules and kilowatt-hours.
11.3. The reading of an electric meter is 0981 kWh. The
load consists of three bulbs of 100 W, two bulbs of 60 W,
and four bulbs of 15 W. What will the meter reading be in
30 days if the bulbs are switched on for 10 h every day?
What will the electricity bill be at a rate of 4 kopecks per
kilowatt-hour?
11.4. An electric motor driven by a current of 15 A at a
constant voltage of 220 V develops a power of 3 kW. Deter­
mine the efficiency of the motor and the cost of 8 h of elec­
tric energy, if it is charged at a rate of 4 kopecks per kWh.
11.5. A d.c. electric transmission line between the Volzh-
skaya hydroelectric plant and the Donbas is rated for a volt­
age of 800 kV with a nominal current of 1000 A. Determine
the transmitted power and the energy transferred per year
under these nominal' conditions.
11.6. Determine the electric energy expenditure over 8 h
of operation of two parallel-connected electric motors if the
current in one branch is 50 A and the supply voltage is
220 V.
11.7. How much energy is consumed by the motor of a
tram in one hour of continuous operation if the voltage across
the collector plates of the motor is 500 V and the current
in the winding of the motor is 130 A? Express the answer in
joules and kilowatt-hours.
11.8. Determine the power of the current in the circuit
shown in Fig. 52.
11.9. Five resistors are connected as shown in Fig. 53
and to a voltage supply U A b = 24 V. Determine the total
resistance of the circuit, the potential difference between
points C and D, the current in the fifth resistor, and the power
of the current in the circuit.
138 Ch. II. Fundamentals of Electrodynamics

11.10. An electric loader is driven by an accumulator bat­


tery consisting of 22 series-connected alkaline accumulators
with an emf of 1.1 V each. The power developed by the motor
during loading is 1.2 kW. Determine the current in the cir­
cuit if the voltage drop across the battery and in the wires
is 2.2 V.
11.11. The battery of an electromobile consists of 42 se­
ries-connected accumulators having an emf of 2.0 V each.

f t , = 7St fiz = SSl r /t2 = 6ft

Fig. 52 Fig. 53

Determine the total power of the battery if the current in


each of two parallel-connected electric motors is 50 A.
11.12. Determine the power developed by an underground
train consisting of six coaches if the motors in each coach
are connected in two parallel branches consisting of two se­
ries-connected motors. The supply voltage is 750 V and the
current in each branch is 150 A.
11.13. Determine the power developed by the motors of
an electric locomotive if they are connected into three
parallel branches of two series-connected motors, the
supply voltage is 3 kV, and the current in each branch is
300 A.
11.14. Determine the energy consumption in 8 hours if
four incandescent lamps are connected to a circuit at a volt­
age of 120 V (a) in parallel, (b) in series, and (c) in two
parallel groups containing two series-connected lamps. The
resistance of a lamp is 120 Q.
11.15. Determine the power consumed and the useful
power for the motor from Problem 10.89 if its efficiency is
90%.
11.16. Using the data of Problem 10.56, calculate the
power consumed by the motors of trolleybus 1 and the power
losses in the circuit.
§ 11. Work, Power, and the Thermal Effect of Current 139

11.17. A generator with an internal resistance of 0.01 Q


is connected to electric motors as shown in Fig. 54. The
voltmeter indicates 220 V and the ammeter 500 A. The
resistance of the leads is 0.05 Q. Determine the total power,
the power consumed by the fti-O .O S Q
electric motors, and the effic­
iency of the set-up.
11.18. Using the data of
Problem 10.61, determine the
power of the current in the
bulbs, and the energy expendi­
ture in ten hours of operation.
11.19. What must the cross- Fig. 54
sectional area of a copper wire
be to transmit a power of 1.0 kW at a voltage of 100 V over
a distance of 50 m so that voltage losses are less than 6 V?
11.20. Two identical resistors are connected to a circuit
at a constant voltage first in parallel and then in series. Is
the electric power the same in both cases?
11.21. The resistance of a glowing incandescent lamp is
360 £2. The supply voltage is 220 V. How many lamps are
connected in parallel in the circuit if the power consumed
by all the lamps is 2.15 kW? The resistance of the leads
should be neglected.
11.22. An electric loader lifts a load of 500 kg to a height
of 2 m at a constant velocity. The motor is driven by an
accumulator battery with an emf of 24 V at a current of
41 A. The efficiency of the loader is 80%. Determine the
velocity and duration of one operation, neglecting the in­
ternal resistance of the battery.
11.23. The motors of a tram moving uniformly along
horizontal rails develop a tractive force of 2 kN. The voltage
across the collector plates of the motors is 550 V and the
current in the circuit is 80 A. The efficiency is 80 %. Deter­
mine the velocity of the tram.
11.24. A tram starts to move at a constant acceleration
of 1 m/s2 for 8 s with a tractive force of 28 kN and an effic­
iency of 8 8 %. What is the current at the end of the eight
seconds if the supply voltage is 550 V?
11.25. Why is lead wire used in fuses?
11.26. The 8-A and 16-A fuses for the VAZ motor car have
the same length. What is the difference between the fuses?
140 Cli. II. Fundamentals of Electrodynamics

11.27. Given two hot plates one of which is rated for a


voltage of 127 V and the other for 220 V. Which coil is
thicker if the length and material of the coils are the same?
11.28. A carriage is heated with eight electric heaters
having a resistance of 275 Q each and connected in parallel.
The voltage across the terminals of the heaters is 550 V.
Determine the amount of heat given away by the heaters
during 18 h of operation, neglecting heat losses.
11.29. Two bulbs rated for 110 V are connected in series
and to a voltage supply of 220 V. Determine the voltage
across each bulb, the power, and the amount of heat liber­
ated by a bulb during 1 h if the powers of the bulbs are
(a) 60 W each, (b) 60 and 40 W, and (c) 60 and 100 W.
11.30. An electric soldering iron operates at a voltage of
220 V and a current of 0.22 A. How much tin at 293 K can
be melted by it per minute?
11.31. An electric samovar having a power of 600 W
boils 1.5 1 of water from 283 K in 20 min. Determine the
efficiency of the samovar and the cost of the energy if elec­
tricity is charged at a rate of 4 kopecks per kWh.
11.32. Two resistors, having resistances of 1 and 4 Q, are
connected in turn to a current source and are found to con­
sume the same power. Determine the internal resistance of
the current source.
11.33. Determine the emf and the internal resistance of
a current source if the power of an external circuit is 230 W
at a current of 10 A and 337.5 W at a current of 15 A.
11.34. An electric motor is connected to a voltage supply
of 220 V. The resistance of the motor winding is 1.8 Q and
the current in the motor is 12 A. Determine the power con­
sumed and the efficiency of the motor.
11.35. Water boils in an electric boiler 12 min after it
.has been switched on. The heating element consists of 4.5 m
of wire wound in a coil. What must be done to obtain boiling
water in 8 min? The energy losses should be neglected.
11.36. The heating element of an electric boiler has two
sections. If the two sections are connected in series, water
boils in the boiler in 27 min, while if connected in parallel
the water boils in 6 min. The resistance of one section is
40 Q. What is the resistance of the other section? Energy
losses should be neglected.
11.37. The distance between a generator with an emf of
§ 12. Electric Current in Electrolytes 141

240 V and an internal resistance of 0.1 Q and a consumer


is 50 in. How much copper is required for the cabling if a
power of 22 kW is rated for a voltage of 220 V?

§ 12. ELECTRIC CURRENT IN ELECTROLYTES

Basic Concepts and Formulas


Liquid conductors mainly include solutions of salts, alkalis,
and acids. The current carriers in liquid conductors aie the
ions formed by electrolytic dissociation. This is the decom­
position of neutral molecules of salts, alkalis, and acids
into positive and negative ions when they dissolve in water
or other solvent. The large permittivity of water (e = 8 1 )
and thermal motion lead to the decomposition of molecules.
The passage of a current through a liquid conductor
(electrolyte) is accompanied by a chemical transformation
of the substance and its deposition on the electrodes (this
is known as electrolysis).
Electrolysis is governed by Faraday’s two laws.
Faraday’s first law of electrolysis. The mass of a substance
liberated in electrolysis is proportional to the amount of
electricity passing through the electrolyte:
m - kQ,
where k is the electrochemical equivalent and is the amount
of substance liberated in the electrolysis by the passage of
1 C of electricity through the electrolyte.
Faraday’s second law of electrolysis. Electrochemical
equivalents are proportional to the ratio of the molar mass
to the valency of the substance:
. A/e
k2 Wj 1^2
where M 1 and M 2 are the molar masses, and nx and n2 are
the valencies.
It is important to remember that the charge passing
through an electrolyte to liberate 1 mol of a substance is
F = N Ae = 9.648456 x 104 C/mol, where F is the same
number for all electrolytes and is known as the Faraday con­
stant.
142 Ch. II. Fundamentals of Electrodynamics

Faraday’s generalized law is


_1_ M_
m= Q-
F n
The passage of a current through an electrolyte may cause
the electrodes to polarize, which induces a counter emf, i.e.
decreases the current through the circuit. In such a case,
problems can be solved with the help of Ohm’s law for a
subcircuit containing an emf:
I __ U %Pul
R
Worked Problems
Problem 68. A metal article is electrolytically plated with
a silver layer 20 pm thick. How long did the electrolysis
require for a current density of 2.5 X 10~3 A/cm2?
Given: h = 20 pm = 2 X 10“5 m is the thickness of the
silver layer, and j = 2.5 X 10“3 A/cm2 = 25 A/m2 is the
current density. From tables, we find the electrochemical
equivalent of silver, k = 1.118 X 10“tf kg/C, the density of
silver, p = 10.5 X 10s kg/m3, the valency of silver, n = 1,
the molar mass of silver, M = 108 x 10~3 kg/mol, and the
Faraday constant F = 9.65 X 104 C/mol.
Find: the time t of the electrolysis.
Solution. 1st method. We solve the problem using Fara­
day’s first law m = kit. This gives t = m/kl. The mass and
the current can be determined from the formulas m = pSh
and / = jS. Substituting these quantities into the formula
for time, we obtain
t p h t 10.5 X 103 kg/m3 X 20x10-* m _ 7[;m „
» l ~ 1.118 X 10- * kg/C X 25 A/m2 -
2nd method. If the electrochemical equivalent is unknown,
the problem can be solved using Faraday’s generalized law
m = —r —n It whence t = tM t tl - The mass and the current
can be determined as before: m = pV = pSh and I = jS.
This gives
t_ p hFn
Mj ’
. _ 10.5 X 103 kg/m3 X 20 X IQ-0 m X 9.65 X 104 C/mol _ 7500 g
108 x 10-3 kg/mol X 25 A/ma
§ 12. Electric Current in Electrolytes 143

Answer. The time required for silver plating is about 2.1 h.


Problem 69. During the electrolysis of silver nitrate solu­
tion, 12 g of silver took an hour to deposit on the cathode. The
voltage across the terminals of the electrolytic bath was
5.2 V, the resistance of the solution was 1.5 Q, and the po­
larization emf was 0.7 V. Determine the valency of silver
and the number of silver atoms liberated at the cathode.
Given-, t = 3600 s is the duration of the electrolysis, m =
1.2 X 10~2 kg is the mass of liberated silver, U = 5.2 V
is the voltage across the terminals of the bath, R = 1.5 Q
is the resistance of the electrolyte solution, and % = 0.7 V
is the polarization emf. From tables, we find the molar mass
of silver, M = 108 X 10“3 kg/mol, the Faraday constant
F = 9.65 x 104 C/mol, and the electron charge e = 1.6 X
io - 19 C.
Find: the valency n of silver and the number N of silver
atoms liberated at the cathode.
Solution. The valency of silver can be determined from
1 M MIt
Faraday’s generalized law m = y — It, whence n = -^pr ■
Since the polarization emf emerges in the circuit, the
current can be determined from Ohm’s law for a subcircuit
with an emf: I = (U — %)/R. Then the expression for the
valency becomes

_ M ({/—£) t 108 XlO-3 kg/mol (5.2 V- 0 .7 V) 3600 s ,


n~ mFR ’ ” — 1.2 X IO"2 kg X 9.65 XlO1 C/mol X 1.5 Q “ '

Since the valency of silver is unity, we can determine the


number of silver atoms by dividing the charge passing
through the electrolyte by the elementary charge: N = Q!e.
Since Q = It, we obtain

AT ( U- l ) t N (5.2 V —0.7 V) 3600 s


= 6.75 X 1022.
Re ~ 1.5 Q X 1 .6 x 10-1* C

Answer. Silver is monovalent, and 6.75 X 1022 silver


atoms were liberated at the cathode.
Problem 70. The current through an electrolytic bath
containing the solution of zinc sulphate (ZnS04) increases
linearly / = (2 + 0.02J). How much zinc will be liberated
at the cathode 5 min after the current starts changing?
144 Ch. II. Fundamentals of Electrodynamics

Given: I = (2 + 0.021) is the linear increase in current,


and t = 300 s is the duration of the electrolysis. From tables,
we find the electrochemical equivalent
of zinc, k = 3.4 X 10"7 kg/C.
Find: the mass m of the deposited
zinc.
Solution. We use Faraday’s first law
of electrolysis
m = kQ,
where Q is the amount of electricity
passing through the electrolyte. In
order to determine Q, we plot the cur­
rent versus time graph. The current
can be determined from the given
equation. At time t0 0, we have 7 0 — 2 A, while at
t = 300 s, we have / —- (2 + 0.02 X 300) A = 8 A. The
time variation of current is shown in Fig. 55.
From the graph, we see that the amount of electricity
passing through the electrolyte is equal to the area of the
hatched region (trapezium):

Q = I t = l ° ± L t , Q = (2 A + 8 A) 300 s = 1500 C.

The mass of the zinc is


m = 3.4 x 10- 7 kg/C x 1500 C - 51.0 X 10' 5 kg.

Answer. 510 mg of zinc will be deposited on the cathode.


Problem 71. Determine the power consumed during the
electrolysis of sulphuric acid solution if 150 mg of hydrogen
are liberated over 25 min, and the resistance of the electro­
lyte is 0.4 Q. The losses should be neglected.
Given: t 1500 s is the duration of the electrolysis, m —
0.15 x l0 “3 kg is the mass of the liberated hydrogen, R —-
0.4 Q is the resistance of the electrolyte. From tables, we
find the electrochemical equivalent of hydrogen, k —
1.044 x 10-8 kg/C.
Find: the power P consumed in the electrolysis.
Solution. In order to determine the power of the current,
we use the formula P = P R . The current can be determined
from Faraday’s first law of electrolysis: / = m/kt. Substi-
§ 12. Electric Current in,Electrolytes 145

tuting this quantity into the formula for power, we obtain


P P (0.15 x 10~3 kg)» 0.4 Q ..o -w
(*<)* ’ (1.044 X 10-o kg/C X 1500 s)* ¥’
Answer. The power consumed in the liberation of the
hydrogen is approximately equal to 37 W.

Questions and Problems


12.1. A solution of the salt NaCl is electrically neutral.
Can we state that there are no ions in the solution? Why?
12.2. Electrolytic capacitors are connected to a circuit,
with their polarity observed. Can the polarity be ignored?
Why?
12.3. Current reversal is used to polish the surface of a
metal electrolytically. What effect does it produce? Can
this method be used to sharpen cutting tools?
12.4. How much aluminium, silver, and copper will be
deposited on the cathode as a result of the passage of 1 C
of electricity through appropriate electrolytes? How many
electrons pass through each electrolyte thereby?
12.5. The electrolysis of copper sulphate solution yields
1 g of copper. How much aluminium can be obtained electro­
lytically by passing the same amount of electricity through
an appropriate electrolyte?
12.6. Two electrolytic baths are connected in series and
to a current source. The first bath contains a solution of
nickel sulphate N iS 0 4 while the second contains a solution
of chromium (II) chloride CrCl2. How much chromium will
be liberated in the second bath if 300 g of nickel are libe­
rated in the first bath?
12.7. Two series-connected electrolytic baths contain
copper sulphate CuS04 and copper chloride CuCl solutions
respectively. How much copper will be liberated in each
bath when 1 C of electricity is passed through them?
12.8. A student first calculated the electrochemical equiv­
alent of copper using the formula k = MIFn and then
experimentally determined it. What must the increase in the
mass of the cathode be after 15 min if copper sulphate solu­
tion is electrolyzed at a current of 1 A? The valency of
copper is 2.
10-0530
146 Ch. II. Fundamentals of Electrodynamics

12.9. To determine the electrochemical equivalent of


copper, a student electrolyzed a copper sulphate solution for
20 min at a current of 1.5 A. The cathode increased in mass
by 600 mg. What is the resultant electrochemical equiv­
alent? What are the absolute and relative errors of measure­
ment in comparison with the tabulated value?
12.10. A solution was electrolyzed for 20 min at a current
of 1.5 A. 594 mg were liberated at the cathode. What was
liberated?
12.11. During an electrolysis, 503 mg of a metal were
liberated at the cathode. The process lasted 5 min at a cur­
rent of 1.5 A. What was the metal and what is its valency?
12.12. What are the charges of mono-, bi-, and trivalent
ions?
12.13. The valencies of silver and gold are 1 and 3 re­
spectively. Determine their electrochemical equivalents.
12.14. How many silver atoms are liberated at the cathode
when silver nitrate solution is electrolyzed for 1 h at a cur­
rent of 1 A?
12.15. Two series-connected electrolytic baths contain
solutions of copper sulphate and auric chloride respectively.
As a result of the electrolysis, 2 g of copper was liberated
at the cathode. How much trivalent gold was liberated in
the other bath? How many copper and gold atoms were
deposited on the cathodes?
12.16. Copper is refined at a voltage of 0.3 V across the
bath terminals. How much copper is deposited on the cathode
over 1 h if the resistance of the electrolyte is 3 X 10"5 Q?
Copper is bivalent.
12.17. To obtain aluminium, a current of 50 A is passed
through molten cryolite with alumina at a voltage of 6 V.
Determine the energy consumption per ton of aluminium and
the resistance of the electrolyte.
12.18. In a copper refinery the cathodes are replaced after
10 days of continuous operation of electrolytic baths. The
amount of copper accumulated on each electrode over this
period is 71 kg. Determine the current density if the area
of a cathode is 0.9 m2.
12.19. During an electrolysis, 5 X 104 C of electricity
were passed through a ferric chloride (FeCl3) solution. How
much iron and chlorine are liberated in the process? At
which electrode is the chlorine liberated? Why?
§ 12. Electric Current in Electrolytes 147

12.20. How long does it take to gold-plate a watch case


electrolytically with a layer thickness of 12 pm at a current
density of 0.1 A/dm2?
12.21. The reflectivity of an automobile’s headlight is
increased by an electrolytic coating of a 10-pm thick silver
layer. How long must the electrolysis of a silver nitrate
solution with a current density of 0.3 A/dmz be to obtain
the required thickness of the layer?
12.22. A melt of aluminium salts is electrolyzed at a
voltage of 6.5 V. How much electricity is spent to obtain 1 t
of aluminium and how much does it cost at a rate of 2 kop­
ecks per kWh if the efficiency of the electrolyzer is
75 %?
12.23. How much aluminium is obtained by electrolysis
if 100 kWh of electricity has been spent? The electrolysis
is at a voltage of 6 V with an efficiency of 80%.
12.24. How long does it take for a copper anode 50 X
10 x 1 mm3 in size to completely dissolve if a copper sul­
phate solution is electrolyzed at a current of 0.3 A?
12.25. Electrolytic nickel plating of an article is carried
out at a current density of 0.8 A/dm2. Determine the rate of
growth of the nickel layer. The valency of nickel in the com­
pound is 2.
12.26. During the electrolysis of weakly acidified water,
0.5 1 of hydrogen are obtained at a pressure of 0.13 MPa
over 50 min. Determine the temperature of the hydrogen
if the current in the circuit is 1.6 A.
12.27. A silver nitrate solution was electrolyzed for
5 min, and the amount of silver deposited on the cathode was
336 mg. An ammeter in the circuit indicated 0.9 A. Was
this reading correct or has a correction to be made?
12.28. Given the electrochemical equivalent of oxygen,
determine the electrochemical equivalent of hydrogen,
sodium, and magnesium.
12.29. How much electric charge passes through a silver
nitrate solution in 20 s if the current increases from 1 to
4 A during the process? How much silver is deposited on the
cathode?
12.30. Determine the electric power spent if a weakly
acidified water is electrolyzed for 25 min liberating 0.5 g
of oxygen. The resistance of the electrolyte is 1.8 Q and does
not change with time.
10 *
148 Ch. II. Fundamentals of Electrodynamics

12.31. During an electrolysis of a silver nitrate solution,


the current in the bath varied thus: I = 0.2 + 6 X 10'3f.
How much silver was deposited on the cathode 300 s after
the current started to vary?
12.32. At what current density does the thickness of the
silver layer in a silver nitrate solution grow at a rate of
3 X 10~3 pm/s?
12.33. How many atoms of a monovalent metal are depos­
ited per square metre of cathode surface if an electrolysis
is carried out for 10 min at a current density of 5 A/m2?
12.34. During the electrolysis of a nickel sulphate solution,
2.19 g of nickel are deposited after 40 min on the surface
of the cathode. Determine the polarization emf if the voltage
across the bath terminals is 5 V and the resistance of the
solution is 1.4 Q.

§ 13. ELECTRIC CURRENT IN GASES AND IN VACUUM

Basic Concepts and Formulas


Under normal conditions, all gases are nonconducting. They
either do not contain charge carriers or the number of carriers
in them is small.
Heating or irradiating a gas ionizes it: its atoms lose a
valency electron and become positive ions. Some of the
electrons that escape the atoms recombine with neutral mol­
ecules to form negative ions. Thus, positive and negative
ions and electrons become the mobile charge carriers in the
gas.
In order to detach an electron from an atom, i.e. to ionize
it, the following work must be done:
= <pte,
where (p, is the ionization potential, which is different for
different gases. It should be kept in mind that 1 eV = 1.6 X
i o - 1B J.
For an impact ionization to emerge, an electron must
acquire a kinetic energy greater than or equal to the ioniza­
tion work
A
2 i-
§ 13. Electric Current in Gases and in Vacuum 149

An electron may acquire this energy at the expense of the


work done by electric field forces:
mv2l 2 = eEk,
when it travels a distance k between two collisions.
Positive ions also take part in ionization.
The passage of an electric current through a gas is known
as an electric discharge. There are several types of gas dis­
charge.
Conduction in vacuum is determined by the presence of
a source of charged particles. For example, in a vacuum tube,
electrons emitted from the surface of a heated cathode acquire
4 kinetic energy from the field, i.e.
mv2l2 = Ue,
where U is the voltage between the electrodes of the tube
and is known as the accelerating voltage.
The number of electrons emitted by the cathode depends
on its temperature. The saturation current density is given by
j = env,
where n and v are the number density and velocity of the
electrons respectively.

Worked Problems
Problem 72. Under normal conditions, a spark discharge
in air occurs at a field strength of 3 X 10® V/m. Determine
the energy required for an electron to ionize air molecules if
the mean free path of an electron is 5 pm. What must the
minimum velocity of an electron capable of ionizing air
molecules be?
Given: E = 3 X 10® V/m is the electric field strength and
k = 5 X 10"® m is the mean free path of an electron. From
tables, we find the electron charge e = 1.6 X 10' 19 C and
the electron mass m = 9.11 x 10"31 kg.
Find: the ionization energy W t and the minimum ve­
locity v required for the ionization.
Solution. The energy W, required to ionize air molecules
is received by an electron at the expense of the work done
by electric field forces: A = e (q>t — q>2), where the potential
difference can be expressed in terms of the electric field
150 Ch. II. Fundamentals of Electrodynamics

strength: <px — cp2 = Ek. Then the ionization energy is

Wl = A = eEk,
W, = 1.6 X 10- 19 C X 3 X 10" V/m x 5 x lO"6 m
= 2.4 X 10- 18 J.

The electron moving in the electric field acquires a ki­


netic energy mu4/2. The molecules may ionize when the
kinetic energy of the electron is higher than or equal to the
ionization energy. To determine the minimum velocity, we
can use the equality mi>2/2 = W x, from which v can be de­
termined:

2.3 x 10® m/s.

Answer. The ionization energy is 2.4 X 10~18 J, and the


velocity of the electrons is 2.3 x 10* m/s.
Problem 73. The separation between the electrodes in an
ionization chamber is 6.2 cm, and the area of each electrode
is 100 cm2. An ionizer produces 10® pairs of ions per second
in 1 cm3 of the chamber. The ion pairs have a mobility of
3.29 X 10~4 m2/(V-s). Assuming that the ions are mono­
valent, determine the saturation current and the field strength
in the chamber.
Given: S = 10~2 m2 is the surface area of an electrode,
I = 6.2 X 10~2 m is the separation between the electrodes,
n = 2 x 1015 m~3 is the number density of the ions, b =
3.29 X 10~4 m2/(V -s) is the mobility of the ions, and t = 1 s
is the time of ionization. From tables, we find the electron
charge e = 1.6 x 10~19 C.
Find: the saturation current / and the electric field
strength E in the chamber.
Solution. The charge carriers in gases are ions and elec­
trons. At the saturation current, all the charge carriers
reach the electrodes. The saturation current can be deter­
mined from the formula I = envS, where v = l/t. This
givesI

I — en — S .
§ 13. Electric Current in Gases and in Vacuum 151

Using the formula b = v/E for the mobility of charges, we


determine the held strength:

bt ’
Substituting in the numerical values, we obtain
/= r l.6 x 10"1* C x 2 x 10-*6 m~3 ?-* m 10"2m2 = 0.2 pA,
6.2 X 10“2 m
E = 3.29 X 10”4 m2/(V-s) x 1 s = 188 V/m.

Answer. The saturation current is 0.2 pA, and the electric


held strength is 188 V/m.

Questions and Problems


13.1. Under the action of an ionizer, a gas has become a
conductor. A charged electroscope placed nearby starts to
discharge rapidly. Why does the discharge cease after the
ionizer is removed?
13.2. Figure 56 shows a dependence of the current through
a gas on the applied voltage. What processes correspond to

Fig. 56 Fig. 57

different sections of the graph? Which section describes a


Self-sustained discharge?
13.3. What is the difference in the conduction of gases and
solutions?
13.4. What should be done to increase the saturation cur­
rent?
13.5. Is impact ionization possible at a low voltage if a
gas is at atmospheric pressure?
152 Ch. 11. Fundamentals of Electrodynamics

13.6. How can you explain that spark discharges occur


only intermittently?
13.7. Give examples when a corona discharge is (a) haim-
ful and (b) beneficial.
13.8. What type of discharge is observed in day-light
lamps? What particles are the charge carriers in this dis­
charge?
13.9. Why does impact ionization in rarefied gases become
more intense with decreasing voltage?
13.10. Why is the surface of the cathode in a vacuum tube
coated with a thin layer of metal (say, barium or strontium)?
13.11. Figure 57 shows the anode current versus anode
voltage for different temperatures of the filament of a diode.
How can the presence of the horizontal lines in the graph be
explained? What is the reason behind the increase in the
saturation current at higher temperatures?
13.12. How can an electron beam be controlled? How is it
controlled in the CRT tube of a television set?
13.13. A gas and a plasma are both electrically neutral
as a whole. What is the difference between them?
13.14. Determine the ionization potential for silver atoms
if an energy of 6.9 X 10~19 J is required to ionize them.
13.15. How will the velocity of the electrons in a CRT
change if the energy of an electron changes from 700 to
1000 eV as a result of a change in the voltage between the
anode and the cathode? What is the electron velocity in the
two cases?
13.16. For obtaining cathode rays, a voltage of 30 kV is
applied to the electrodes of a gas discharge tube. Determine
the maximum velocity of the electrons in the cathode beam.
13.17. A potential difference of 300 V is applied between
the cathode and the anode of a diode. Determine the velocity
of the electrons as they move in the tube if the separation
between the cathode and the anode is 10 mm. How long
do they move?
13.18. What must the minimum velocity of electrons be to
cause the impact ionization of cesium atoms, whofe work
function is 1.8 eV?
13.19. The ionization energy for a hydrogen atom is
13.5 eV. What must the minimum velocity of an electron be
to cause the impact ionization of the hydrogen atom?
13.20. On average, five ion pairs are formed per second
§ 14. Electric Current in Semiconductors 153

per cm3 of atmospheric air near the Earth’s surface due to


radioactivity of soil and cosmic radiation. The separation
between two electrodes is 8 cm. Determine the saturation
current density between the electrodes for singly charged
ions.
13.21. The work function of barium oxide electrons is
1.0 eV. What must the mean free path of the electrons in
a parallel-plate capacitor be for the electrons to ionize the
barium atoms? The field strength between the plates is
3 X 10s V/m. The field should be treated as uniform.
13.22. The saturation current in an air-filled tube is
2 X 10' 10 A at an electric field strength of 30 V/m between
the electrodes. The overall mobility of monovalent ions is
3.29 X 10~4 m2/(V-s), and the area of plane electrodes is
100 cm2. Determine the number density of the ions.
13.23. Determine the overall mobility of hydrogen ions
if the saturation current density is 2.8 X 10~10 A/m2 at a
field strength of 1.2 kV/m and a number density of ions of
104 cm-3.§

§ 14. ELECTRIC CURRENT IN SEMICONDUCTORS

Basic Concepts and Formulas


Semiconductors are materials having resistivity between
that of conductors and insulators; their resistivity decreases
with rising temperature and the presence of impurities.
Typical semiconductors are elements of group IV of the
Periodic Table, whose atoms have four valency electrons
in the outer shell (like germanium Ge and silicon Si). At low
temperatures, crystals of these elements do not contain free
electrons and are good insulators. The covalent bonds in
such crystals are ruptured with increasing temperature,
illuminance, or due to strong electric fields. Free electrons
appear, and intrinsic electron conduction emerges in (n-type)
semiconductors. Intrinsic hole conduction of (p-type) semi­
conductors is due to the displacement of holes.
Impurity conduction in semiconductors is due to the pres­
ence of “alien” group V elements (like arsenic or antimony)
or group III elements (like boron and aluminium). In the
former case, impurity electron conduction is created, and in
the latter, impurity hole conduction sets in. Thus, the
154 Ch. II. Fundamentals of Electrodynamics

introduction of impurities into pure semiconductors can


destroy the equilibrium between the p-type and n-type con­
ductions.
When p- and ra-type semiconductors are brought in con­
tact, a barrier layer emerges as a result of diffusion through
the p-n junction. The application of an external field to a
p-n junction may change its conduction and create the con­
ditions for unilateral conduction.
A semiconductor diode is a semiconductor with a p-n
junction. Its advantages over vacuum tube diodes include
small size, reliability, and efficiency.

Questions and Problems

14.1. Figure 58 shows the temperature dependences of


resistance for conductors and semiconductors. Which of
them corresponds to semiconduc­
tors?
14.2. What are the mobile charge
carriers in a pure semiconductor?
14.3. What is the ratio between
the number of holes and the num­
ber of free electrons in a pure semi­
conductor? Is this ratio preserved
for impurity conduction of semicon­
ductors?
Fig. 58 14.4. What will the type of con­
duction in germanium with trace
impurities of phosphorus or aluminium be?
14.5. How do the conductivities of germanium and silicon
vary with lowering temperature?
14.6. By what factor will the current density in a semi­
conductor change if the velocity of the electrons increases
from 0.5 to 0.75 m/s as a result of a temperature increase
from 0 to 175°C, while the electron number density increases
thereby from 1.3 x 1014 to 2.1 X 1018 m"3?
14.7. The velocity of directional motion of free electrons
in a semiconductor is 0.25 m/s for a given temperature.
Determine the mobility of the charges and their number den­
sity if the current density is 4 x 10-2 A/m2 for a field
strength of 100 V/m.
§ 15. Electromagnetism 155

14.8. What is a thermistor? Why are thermistors called


nonlinear resistances?
14.9. What is the difference between a thermistor and a
photoresistor?
14.10. What is a transistor? Which regions does the crystal
of a transistor contain?
14.11. The thickness of the base in a transistor is very
small (1-25 pm). Why?
14.12. What is the ratio between the emitter, base, and
collector currents?
14.13. What is the advantage of semiconductor devices
over vacuum tubes in radio engineering?

§ 15. ELECTROMAGNETISM

Basic Concepts and Formulas


A magnetic field is a special case of electromagnetic field,
being characterized by the action on a moving charged
particle of a force proportional to
the charge of the particle and its
velocity.
The shape of a magnetic field de­
pends on the shape of the current-
carrying conductor producing it.
For example, the magnetic field
formed around a straight current-
carrying conductor is graphically
represented by magnetic field lines
in the form of concentric rings in
the plane perpendicular to the di­
rection of the current (Fig. 59). The
direction of the magnetic field in
this case is given by Ampere’s (right- Fig. 59
hand screw) rule: rotation of the
head of a screw indicates the direc­
tion of the magnetic field lines if the motion of the screw
body coincides with the direction of the current.
The magnetic field of a current-carrying coil (solenoid)
is similar to the magnetic field of a permanent bar magnet.
The interaction between magnetic fields formed by par­
allel current-carrying conductors is determined by the
156 Ch. II. Fundamentals of Electrodynamics

formula
I
2na

where I x and I 2 are the currents in the conductors, I is the


length of wire over which the force is acting, a is the sepa­
ration between the conductors, and pm is the absolute per­
meability of the medium characterizing the dependence of the
force of interaction of current-carrying conductors on the
properties of the medium:
pm = HoH-
Here |i0 = 4n X 10-7 H/m is the magnetic constant, and
p is the relative permeability of the medium (see Table 21).
A magnetic field exerts a force FA (Ampere’s law) on a
current-carrying conductor:
Fa = BIl sin a.
If the conductor is perpendicular to magnetic field lines
(a = 90°), we have
Fa = BIl.
The proportionality factor B is called the magnetic induction
and is the force characteristic of the magnetic field. Magnetic
induction is a vector quantity. The SI
unit of magnetic induction is the tesla (T).
At any point in a uniform magnetic field, the
magnetic induction has the same magnitude
and direction. Therefore, such a field is
graphically represented by parallel straight
lines with uniform density.
The magnetic flux is equal to the number
of magnetic field lines piercing a surface of
area S if the magnetic induction vector coincides with the
normal to this surface:
<t = BS.
The SI unit of magnetic flux is the weber (Wb).
The magnetic properties of a current loop are characterized
by the magnetic moment P mag (Fig. 60):
P mag = IS.
§ 15. Electromagnetism 157

A current-carrying loop in a uniform magnetic field of


induction B experiences a rotational magnetic moment
(torque) M:
M — JSB sin a,
where a is the angle between vectors B and Pmag. For a =
90°, the torque w ill have the maximum value
M mas = I SB = P inagfl.
A closed current-carrying loop of length I is displaced
by a force FA through a distance b. Consequently, the
following work is done:
A = FAb = B llb.
But lb is the change in the area, AS, so
A = B I AS, or A = I AO.
The magnetic induction of a straight current-carrying
conductor is

= ‘2nT ’
where r is the shortest distance from the current-carrying
conductor to the point at which the induction B is being
determined.
The magnetic induction of the field produced by a circular
current is
^ = M-m •

The magnetic induction of the field in a solenoid is


n /©
B= — ’
where to is the number of turns and I is the length of the
solenoid.
The magnetic field inside a long solenoid is uniform.
Therefore, the magnetic flux in the solenoid is

The magnetic field is also characterized by the magnetic


field strength H which is related with the magnetic induction
158 Ch. II. Fundamentals of Electrodynamics

through the formula


B = pm//.
The force acting on an electric charge Q moving in a mag­
netic field is known as the Lorentz force F L:
Ft. = BvQ sin a,
where a is the angle between vectors B and v. The Lorentz
force is always perpendicular to the plane containing vectors
B and v and hence does no work. Without changing the mag­
nitude of the velocity of the charge, it only changes its d i­
rection and is responsible for centripetal acceleration. If
a = 90°, we have

i.e. a particle of mass m and charge Q moves in a circle under


the action of force FL.

Worked Problems
Problem 74. A direct current passes in the same direction
through two parallel wires separated by a distance of 30 cm .
The distance between the supports to which the wires are
fixed is 50 m. The current in the wires is 150 A. Determine
the magnitude and direction of the force with which the
wires interact.
Given: I x = 7a = / = 150 A is the current in each wire,
a = 0.3 m is the separation between the wires, and I =
50 m is the distance between the supports. From tables,
we find the magnetic constant p0 = 4n X 10-7 H/m and
the permeability of air, ji = 1.
Find: the force F between the wires (its magnitude and
direction).
Solution. Let us consider the distance between the supports
to be the active length of the wires. Noting that I » a,
we can assume that the wires are infinitely long so that the
following formula is applicable for the force of interaction:
1H
2na
p X 10-7 H/m X 1 (150 A)a 50 m _ f t 7 r H
F ~ 2n x 0.3 m ~~U' H
§ 15. Electromagnetism 159

In order to determine the direction of the force acting


between the wires, we shall analyze Fig. 61. The magnetic
field between the wires is weaker (the magnetic field lines
are directed against one another). In the outer region, the
magnetic field lines have the same direction, and the mag­
netic field is larger. Consequently,
wires carrying current in the same di­
rection must attract each other.
Answer. The force of attraction
between the wires is 0.75 N.
Problem 75. When a 0.5-m long
straight conductor carrying a cur­ Fig. 61
rent of 4 A is at right angles to a
uniform magnetic field, the field acts on it with a force of
2.8 N. What will the force exerted by the same field on the
conductor be if the angle between them is 30°?
Given: I = 0.5 m is the length of the conductor, I — 4 A
is the current in it, Ft = 2.8 N is the force corresportding
to 90°, and a x = 90° and a 2 = 30° are the angles between
the magnetic field lines and the current in the first ahd the
second case.
Find: the force F2 acting on the conductor.
Solution. Using Ampere’s law Fx = BIl sin Oj, we deter­
mine the magnetic induction of the uniform magnetic field:
2.8 H
= -ItI— B= = 1.4 T.
b sin----
at , 4 A x 0.5 m X 1
Knowing the magnetic induction, we can determine the
force exerted by the magnetic field on the current-carrying
conductor when the angle between them is 30°:
F2 = BIl sin a 2, F2 = 1.4 T x 4 A x 0.5m x 0.5 = 1.4 N.
Answer. The force acting on the conductor in the second
case is 1.4 N.
Problem 76. A 0.8-m long straight conductor moves in a
magnetic field of induction 5 X 10-2 T. The current in the
conductor is 15 A. The conductor is at an angle of 30° with
the magnetic induction vector. Find the force acting on the
conductor and the work done by the magnetic field to move
the conductor 1.8 m.
Given: B = 5 X 10~2 T is the magnetic induction of the
field, I = 0.8 m is the length of the conductor, / = 15 A
160 Ch. II. Fundamentals of Electrodynamics

is the current in the conductor, a = 30° is the angle be­


tween the direction of current I and vector B, and b = 1.8 m
is the distance by which the conductor is moved.
Find: the force F acting on the conductor and the work A
done by the magnetic held to move the conductor.
Solution. A current-carrying conductor in a magnetic
held is acted upon by the force
F = BIl sin a,
F = 5 x 10-2 T x 15 A x 0.8 m x 0.5 = 0.3 N.
Knowing the force acting on the conductor and the dis­
tance by which it is moved, we can hnd the work:
A = Fb, A - 0.3 N X 1.8 m - 0.54 J.
Answer. The magnetic held acts on the current-carrying
conductor with a force of 0.3 N and does a work of 0.54 J
to displace it.
Problem 77. A 30-cm long solenoid contains 3000 turns.
The diameter of each turn is 11 cm. Determine the magnetic
induction of the held inside the solenoid for a current of
1.5 A and the magnetic flux piercing each turn. What will
the change in the magnetic induction and in the magnetic
flux be if a carbon iron core is inserted into the solenoid?
Given: I = 0.3 m is the length of the solenoid, N = 3000
is the number of turns in it, d = 11 X 10~2 m is the di­
ameter of a turn, / = 1.5 A is the current; from tables,
we hnd the magnetic constant p0 = 4n X 10~7 H/m and
the permeability of carbon iron, p = 3000.
Find: the magnetic induction By of the held in the solenoid
without a core, the magnetic induction Bt of the held in
the solenoid with the core, and the corresponding magnetic
fluxes and <i>2.
Solution. Assuming that the solenoid is long enough for
the magnetic held in it to be uniform and directed along the
axis, the magnetic induction can be determined from the
formula

In the absence of a core, p = 1 (for ,ir). Then we have


1.5 AX3000
- 4n x 10-7 H/m = 1.9 X 10“2 T.
0.3 m
§ 15. Electromagnetism 161

In the presence of the core, the magnetic induction in­


creases by a factor of p:
B2 = 3000 x 1.9 x 10- 2 T = 57 T.
The magnetic flux can be determined from the formula
® = BS cos a.
In the problem under consideration, a = 0 and cos a = 1,
while S = nd2/4. This gives

*1 = « T '
^ _ 1.9 x 10~2 T x 3.14 x 121 x 10~4 m2
1 /i = 1 .8 x 10-'* Wb.
In the presence of the core,
<D2 = 3000 x 1.8 x 10-* Wb = 0.54 Wb.
Answer. The magnetic field induction in the solenoid
without a core is 1.9 X 10”2 T and 57 T with the core. The
magnetic fluxes in the solenoid are 1.8 X 10-4 and 0.54 Wb
respectively.
Problem 78. A current-carrying conductor is looped into
a circle of radius 10 cm. The magnetic moment of the current
loop becomes 0.314 A-m2. Determine the current in the loop
and the maximum torque exerted on it by a uniform magnetic
field of induction 5 X 10~a T.
Given: r = 0.1 m is the radius of the loop, p mag =
0.314 A-m2 is the magnetic moment of the loop, and B =
5 X 10~3 T is the magnetic induction of the field.
Find: the current / in the loop and the maximum torque
^fraas-
Solution. Knowing the magnetic moment of the loop, we
can determine the current in it: p mag — IS , where S = nr2,
which gives p mag = /n r 2, and hence
j __ P m a g 0.314 A-m 2
10 A.
jir2 3.14 x l 0 ~2 m2
The maximum torque can be determined from the formula
AIm&K — PmagB,
x = 0.314 A-m2 x 5 x 10"3 T ~ 1.6 x lO"3 N-m.
Answer. The current in the loop is 10 A, and the maximum
torque is approximately 1.6 X 10"3 N-m.
11-0530
162 Ch. II. Fundamentals of Electrodynamics

Problem 79. A proton that has acquired a velocity while


moving across a potential difference of 1 kV enters a uniform
magnetic held of induction 0.2 T at right angles to the mag­
netic held lines. Determine the radius of the circle in which
it will move and the period of its revolution.
Given: U = 1000 V is the accelerating potential difference,
B = 0.2 T is the magnetic induction of the held, a = 90°
is the angle between vectors B and v. From tables, we hnd
the mass of the proton mp = 1.67 X 10"27 kg and its charge
() = 1 . 6 x 10- 19 C.
Find: the radius r of the circle and the period T of revo­
lution of the proton.
Solution. An electric charge (in our case, the proton) mov­
ing in a magnetic held is acted upon by the Lorentz force
Fl = BvQ sin a, where a is the angle between vectors B
and v. Since tx = 90° and sin a = 1, we obtain FL = BvQ.
Since the Lorentz force is always normal to the plane con­
taining vectors B and v, it does no work, i.e. it does not
change the kinetic energy of the moving charge. The force
only changes the direction of the velocity. Hence we can
write BvQ = m pv2/r, whence
m pv
r = ~BQ~-
In order to hnd the proton velocity, we can use the energy
conservation law: the work done by the electric held U is
equal to the kinetic energy acquired by the proton: QU =
mpv2/2, whence
- l / 2 X 1 .6 X 10-1* C X 103 V
v = —V 1.67x10-” kg
= 4.4 x 105 m/s.
We can now determine the radius r of the circle:
1.67 x 10”27 kg x 4.4 X 106 m/s
~ 0.023 m.
0.2 T x 1 .6 x 10~1B C
Given the proton velocity and the radius of its orbit, we
can hnd its period:
rp 2nr
= ~ r *
T = 2 x 3.14 X 0.023 m ~ 0.033 x 10"6 s ot 0.3 ps.
4.4 x 10* m/s
§ 15. Electromagnetism 163

Answer. The proton moves in a circle of radius 0.023 m


with period of revolution 0.3 (.is.

Questions and Problems


15.1. A looped flexible conductor tends to expand into a
circle when a current is passed through it. Why?
15.2. Using a d.c. voltmeter and a magnetic needle on a
pivot, one can determine the side on which a generator is
located in a two-wire d.c. cable. How?

Fig. 62

Fig. 63

15.3. Determine the polarity of the electromagnets shown


in Fig. 62.
15.4. Determine the direction of the currents in the wind­
ings shown in Fig. 63.
15.5. Determine the polarity and the direction of the
magnetic held lines for the generator shown in Fig. 64.
15.6. Determine the direction of motion of current-carry­
ing conductors in the magnetic helds shown in Fig. 65.
15.7. What will happen to a permanent bar magnet if the
current in the solenoid has the direction indicated in Fig. 66?
15.8. What will the motion of the coil be relative to the
solenoid for the direction of the current indicated in Fig. 67?
15.9. What must the polarity of the magnet in Fig. 68
be for the current loop to rotate clockwise? Through what
angle will the loop be turned?
II*
164 Ch. II. Fundamentals of Electrodynamics

15.10. What is the force of the interaction between the


wires of a d.c. trolleybus line over 30 m if the separation
between the wires is 520 mm and the current in them is
200 A?
15.11. A d. c. transmission line is rated for a current of
150 A. What is the separation between two wires if they

interact with a force of 2.8 X 10”1 N over a distance of


50 m?
15.12. Two parallel conductors carrying the same current
are 25 cm apart. Determine the current in each wire if it
experiences a force of 2 mN per metre.
15.13. Two parallel wires are 6 cm apart. The currents in
the wires are 15 and 10 A. Over what segment of the wires
will a force of 1.4 mN act?
15.14. A 0.5-m long straight conductor carrying a current
of 5 A is in a uniform magnetic field of induction 0.16 T.
§ 15. Electromagnetism 165

Determine the forces acting on the conductor when the


angles between the direction of current in it and the magnetic
induction vector are 90°, 30°, and 0°.
15.15. Determine the magnetic induction of a uniform
magnetic field in which a 0.7-m long straight conductor
carrying a current of 10 A is acted upon by a force of 42 mN.
The angle between the direction of current and the magnetic
induction vector is 30°.
15.16. Using a strong horse-shoe magnet, one can deter­
mine whether an incandescent lamp is connected to an a.c.
or a d.c. source. How can this be done?
15.17. A force of 0.6 N acts on a 1-m long straight conduc­
tor placed in a uniform magnetic field at right angles to the

222 m

B
M /V

Fig. 69

magnetic field lines. The current in the conductor is 12 A.


What will the force exerted on this conductor be if the angle
between the direction of current in it and the magnetic
induction vector is 45°? Solve the problem using two different
methods.
15.18. A conductor M N made of a material with density p
and having cross section S is suspended on two weightless
unstretchable strings in a uniform magnetic field of induction
B (Fig. 69). At what current will the tension in the string
be zero? What must the direction of this current be?
15.19. The magnetic field strength 10 cm from a long cur­
rent-carrying straight conductor is 20 A/m. What force will
act on each metre of this wire if it is placed in a uniform
magnetic field with induction 2.5 T so that the angle between
the direction of the current and the magnetic induction
vector is 30°?
15.20. Determine the current that must be passed through
186 Ch. II. Fundamentals of Electrodynamics

a long straight wire to produce a magnetic held of the same


magnitude 1 m away as the magnetic held of the Earth near
its surface. The magnetic induction of the magnetic held of
the Earth should be taken to be 5.5 x 10~6 T.
15.21. Determine the magnetic held strength and the
magnetic induction of the held produced by a straight con­
ductor carrying a current of 7.8 A at a point 4.8 cm away.
15.22. A straight conductor carrying a current of 10 A
produces at a certain point a magnetic held of strength
40 A/m. Determine the magnetic induction at this point and
the distance between this point and the conductor. K
15.23. Two long current-carrying conductors are arranged
at right angles in the same plane (Fig. 70). Determine the
resultant magnetic induction of the held at points M and N
if 7X= 10 A, / , = 6 A, MO = NO = 5 cm, and MC =
NCi = 4 cm.
15.24. A current-carrying circular conductor produces a
magnetic held of strength 25 A/m at the centre. Determine
the magnetic induction of the held and the radius of the
loop if the current is 3.45 A.
15.25. The induction at the centre of a circular current
of radius 4 cm is 1.57 X 10' 4 T. Determine the magnetic
held strength at the centre of the loop and the current.
15.26. The current passing through a straight long con­
ductor produces a magnetic held of induction 0.8 x 10~4 T
4.4 cm away. The permeability of the medium is 1.1. De­
termine the current in the conductor and the magnetic held
strength 16 cm away.
15.27. Determine the magnetic moment of a circular loop
carrying a current of 10 A and having a radius of 6.0 cm.
15.28. The magnetic moment of a current-carrying wire
ring 15 cm in diameter is 4.2 x 10~a A-m*. Determine the
current in the ring and the magnetic held strength at its
centre.
15.29. A wire ring of radius 5.0 cm carrying a current of
6 X 10~* A is in a uniform magnetic held of induction
1.2 X 10-a T. The plane of the ring is parallel to the magnet­
ic held lines. Determine the maximum magnetic moment
exerted by the magnetic held on the ring.
15.30. A solenoid without a core is 100 cm long and con­
tains 600 turns. Determine the magnetic induction of the
held in the solenoid for a current of 0.4 A.
§ 15. Electromagnetism 167

15.31. The current in a solenoid whose diameter is small


in comparison with its length is 6.5 A. The solenoid is 65 cm
long and contains 750 turns. Determine the magnetic field
strength and the magnetic induction in the solenoid without
a core.
15.32. The magnetic induction of a very long solenoid
carrying a current of 3.0 A is 2.52 x 10~3 T. The solenoid
is wound tightly in one row and has no core. Determine the
diameter of the wire of which the solenoid is made.
15.33. Determine the permeability of soft steel in two
magnetic fields with strengths 1.5 and 5.0 kA/m respectively
(see graph 22). How does the permeability change with
increasing magnetic field strength during primary mag­
netization? Why?
15.34. Determine the magnetic induction in a nickel core
and the magnetic flux if the strength of the uniform magnetic
field in the core is 2.0 x 103 A/m, the cross-sectional area
of the core is 30 cm2, and the permeability is 200.
15.35. The winding of a solenoid without a core is made
of a wire whose diameter is 1.0 X 10~3 m. The turns have a
radius of 1.0 cm and are wound tightly. Determine the mag­
netic flux in the solenoid for a current of 2.0 A.
15.36. A rectangular frame made of a wire carrying a cur­
rent of 2.0 A is in a uniform magnetic field of induction
0.10 T. The size of the frame is 4 x 5 cm8. In a certain po­
sition, the magnetic flux through the frame is 0.80 x
10~4 Wb. Determine the maximum magnetic flux in the
frame when its plane is perpendicular to the magnetic field
lines. Determine the work done in rotating the frame to this
position.
15.37. The magnetic induction in the core of an electro­
magnet is 1.2 T and its cross-sectional area is 0.12 m2. De­
termine the magnetic flux through the core.
15.38. A steel rod having a cross-sectional area of 4.5 cm8
and a permeability of 160 is placed in a uniform magnetic
field of strength 7970 A/m so that the magnetic field lines
coincide with the normal to the cross section of the rod. De­
termine the magnetic flux piercing the rod.
15.39. The magnetic field strength in a solenoid with an
iron core is 1600 A/m. The cross-sectional area of the core is
10 cm8. Determine the magnetic induction of the field and
168 Ch. It. Fundamentals of Electrodynamics

the permeability of iron if the magnetic flux through the core


is 2 x 10-4 Wb.
15.40. An electron flies into a uniform magnetic held as
shown in Fig. 71. Determine the direction of the force acting
on the electron at the initial moment. What will its tra­
jectory be?
15.41. An electron moves in a uniform magnetic held at a
velocity of 1.0 X 104 km/s at right angles to the magnetic
induction vector. Determine the force
+ + + + + acting on the electron for a mag-
+ + + + + netic held strength of 150 A/m.
g p- v B 15.42. An electron flies into a uni-
+ + + + + form magnetic held with induction
+ + + + + 9.1 x 10~5 T. The electron velocity
is 1.9 X 107 m/s and at right angles
Fig. 71 to the magnetic induction vector. De­
termine the radius of the circle in
which the electron will move, the period, and frequency
of its revolution.
15.43. The Lorentz force exerted on an electron by crossed
electric and magnetic helds is determined by the formula
F l = eE + evB. What must the direction and magnitude
of the electron velocity he for it to move uniformly in a
straight line?
15.44. An electron flies into a uniform magnetic held
with induction 2.5 X 10~® T and moves in a circle of radius
40 cm. The electron velocity vector forms an angle of 90°
with the direction of the magnetic held. Determine the ki­
netic energy of the electron.
15.45. An electron having a velocity of 8.8 X 107 m/s
flies into a uniform magnetic held of induction 6.28 X
IQ-2 rp The angle between the velocity and magnetic induc­
tion vectors is 30°. Determine the radius and the lead of the
helical trajectory of the electron. Use the charge-to-mass
ratio of the electron.to the third significant digit. K
15.46. Two identical, singly charged ions fly at different
velocities into a uniform magnetic held. What will the
periods of their revolution be?
15.47. An electron and a singly charged ion fly at the
same velocity into a uniform magnetic held. What will the
periods of their revolution be?
§ 16. Electromagnetic Induction 169

§ 16. ELECTROMAGNETIC INDUCTION

Basic Concepts and Formulas


Electromagnetic induction consists in the emergence of an
induced emf and induced current in a closed loop if the
magnetic flux bounded by this loop varies with time:
„ AO
f1 “ ~aT •
Let us suppose that a rectangular frame lying in a plane
perpendicular to a magnetic field moves at a velocity v and
leaves the magnetic field. Then the magnetic flux piercing
the frame varies as
AO = —Blv to.
Therefore, the induced emf can be written in the form
S, = - B l v .
If vectors v and B form an angle a, we have = —Blv sin a.
An emf can also be induced in a stationary loop if the
magnetic induction of the field varies over time.
The direction of the current induced in a closed loop can
be determined using the right-hand rule or Lenz’s law.
According to Lenz’s law, the direction of the induced current
is such that the magnetic field of this current opposes any
change in the magnetic field inducing the current.
Self-inductance can be regarded as a special case of elec­
tromagnetic induction when a changing magnetic flux is
produced by a current as it varies. For example, when a cir­
cuit is closed, the self-inductance emf opposes (in accordance
with Lenz’s law) the increase in the current. When a circuit
is disconnected, the self-inductance emf opposes the decrease
in the current. For this reason, the induced current is di­
rected against the main current when the circuit is connected
and along the main current, when the circuit is disconnected.
The self-inductance emf is proportional to the rate of change
in the current in the circuit:
« _ _t A/

where L is the inductance of the circuit and is determined


by the size and shape of the conductor in it and by the mag-
170 Ch. 11. Fundamentals of Electrodynamics

netic properties of the medium containing the circuit. The


SI unit of inductance is the henry (H).
Since it is a component of electromagnetic held, the mag­
netic held has energy which is given by
1
W = ±-<bI = ±-LI* 2 L '

Worked Problems
Problem 80. A conductor AB is 0.6 m long and has a re­
sistance of 0.2 Q. It can move along a copper busbars CD
connected to a current source with an emf of 0.96 V and an
internal resistance of 0.1 fi
c ^ V (Fig. 72). The resistance of the
+ + + ++ + + + busbars is negligibly small. De­
+++ +++++ termine the current in the con­
y +++ — + +a ductor if it (a) is at rest, (b) moves
+++ ++++ + at a velocity of 0.5 m/s in a
++ + ++ + + + uniform magnetic held of induc­
tion 1.6 T directed along the nor­
c mal to the plane of the hgure
Fig. 72 away from us.
Given: 1= 0.6 m is the length
of the conductor, R = 0.2 S2 is
its resistance, % = 0.96 V is the emf of the current
source, r = 0.1 £2 is its internal resistance, v = 0.5 m/s
is the velocity of the conductor, and B = 1.6 T is the mag­
netic induction of the held.
Find: (a) the current I x in the stationary conductor and
(b) the current I 2 in the conductor moving in the magnetic
held.
Solution, (a) If the conductor is at rest, the current in it is
determined from Ohm’s law for a circuit:
8 j 0.96 V
/.= R + r » ~ 0 .2 Q + 0.1 Q
3.2 A.

(b) If the conductor moves at a velocity v in a uniform


magnetic held, an emf £, is induced in it. If the conductor
is a part of a closed circuit, then a current is induced in it
with a direction determined by the right-hand rule. In the
problem under consideration, the induced current is directed
§ 16. Electromagnetic Induction 171

against the current /j. Ohm’s law in this case has the form
T » -« i
12 R + r •

Since Si = B lv , we obtain
% —B l v
R + r •

Here we must note that the magnetic induction vector and


the velocity vector are at right angles to each other. Hence,
O.yCV —1 .6 T x 0 .ti m x 0 . 5 m/s
h 0.2 0 + 0.1 Q
= 1.6 A.

Answer. The current is 3.2 A in the stationary conductor,


and 1.6 A when the conductor moves in the magnetic field.
Problem 81. A circular coil 10 cm in
diameter is placed in a uniform magnetic
field with induction 0.12 T so that the
magnetic induction vector is normal to
the plane of the coil (Fig. 73). Determine
the number of turns in the coil if an
emf of 0.942 V is induced in it as a re­
sult of its rotation through 180° in
0.14 s.
Given: B = 0.12 T is the magnetic in­
duction of the field, d = 0.1 m is the diameter of a turn,
a = 180° is the angle of rotation of the coil, At = 0.14 s is
the time interval, and gj = 0.942 V is the induced emf.
Find: the number N of turns in the coil.
Solution. When the coil is rotated, the magnetic flux
bounded by the contour of the coil changes, and the emf
induced in it is

* * = --£ -* •
The problem statement indicates that before the coil is
rotated the angle between its normal and the magnetic
induction is a 0 = 0, and the magnetic flux through the coil
is O, = BS cos a„. After the rotation, the magnetic flux
becomes Oa = BS cos a, where a = 180°. The change in
the magnetic flux is 0 2 — O t = AO, AO = BS cos 180° —
BS cos 0°, cos 180° = —1 and cos 0° = 1. Therefore, AO =
\n Ch. II. Fundamentals of Electrodynamics

—BS — BS = —‘I BS. Since S = nd*l4, we obtain


Bnd-
AO= - 2
Substituting the change in the magnetic flux into the ex-
pression for emf, we obtain -- - ^ - .V. Then the number
of turns is
y 2C,A< 2x0.942 V x 0.14 s
JV “ B n d * ’ ~ 0.12 T x 3 . 1 4 x l 0 - s m 2 ~ / U '
Answer. The number of turns is 70.
Problem 82. Determine the inductance of a coil in which
the self-inductance emf induced by a change in the current
from 5 to 10 A in 0.1 s is 10 V. What is the change in the
energy of the magnetic held of the coil in this case?
Given: /j = 5 A is the initial current in the coil, / 2 =
10 A is the current after time At, At = 0.1 s is the time
during which the current changes, and Jfs - 10 V is the
self-inductance emf.
Find: the inductance L in the coil and the change AW
in the magnetic field energy of the coil.
Solution. We write the formula for the self-inductance emf
S s = —L ^ and then determine the inductance of the
coil:
r Ifs ^ r 10 V X 0 .1 S A 9 IT
L = —i T = -f^ 7 T ' L== 10 A — 5 A = ° - 2 H -
The magnetic field energies for currents / , and / 2 are
u / __ LI* rp _ L li
w i— 2 ’ 2— 2 '
The change in the energy is then
aw = w , - w l= - ^ — IL L = ±(11-1*),

AW = ° 3 H [(10 A)2 - (5 A)2] = 7.5 J.


Answer. The inductance of the coil is 0.2 H, the energy of
the magnetic field increases by 7.5 J with current.
Problem 83. A solenoid with a nickel core has 1000 turns
on 0.5 m, the cross-sectional area of a turn being 50 cm2.
Determine the magnetic flux in the solenoid and the magnet-
§ 16. Electromagnetic Induction 173

ic field energy if the current in the solenoid is 10 A and the


permeability of nickel is 200.
Given'. I = 0.5 m is the length of the solenoid, N = 1000
is the number of turns, S = 5 x 10~3 m2 is the cross-
sectional area of a turn, / = 10 A is the current, p = 200
is the permeability of nickel. From tables, we find the mag­
netic constant p0 =- 4n X 10-7 H/m.
Find: The magnetic flux <1>in the solenoid and the energy W
of the magnetic field of the solenoid.
Solution. The magnetic flux can be calculated from the
formula = BS. Since the magnetic induction of the field
IN
of a solenoid with a core is B = p0p — , we have
* I N c

0 = 4n x 10-7 H/m x 200 10 A x 1000 5 x 10"2 m2


0.5 m
= 2.5 x 10-2 Wb.
Given the magnetic flux and the current, we can determine
the energy of the magnetic field in the solenoid:
W = -|- 0 7 , W = — x 2.5 x 10-2 Wb x 10 A ~ 1.3 x 10-< J.
Answer. The magnetic flux in the solenoid is 2.5 X 10“'2 Wb,
and the energy of the magnetic field in the solenoid is 0.13 J .

Questions and Problems


16.1. Will an emf be induced in the conductors moving
as shown in Fig. 74?
i
(a ) W

J -'. 3 )

Fig. 74 Fig. 75

16.2. When the poles of a horse-shoe magnet are closed


with an armature (Fig. 75), the pointer of the galvanometer
is deflected. Why?
174 Ch. II. Fundamentals of Electrodynamics

16.3. Determine the direction of the emf induced in con­


ductors moving in uniform magnetic fields shown in Fig. 76.
16.4. A rectangular frame in a translatory motion (a)
enters a uniform magnetic field, (b) moves in it, (,c) leaves

la) M tb) M
n + *■

+ + +
is 8
+ + + +
/V
+■ + + +
Fig. 7ti

the field. The normal to the frame plane is directed along


the induction line. Is an emf induced in the frame under
these conditions? Why?
16.5. A rectangular frame rotates in a uniform magnetic
field about an axis parallel to the magnetic field lines. Is
an emf induced in this case?

lb) (0 id)

|A | W
cc:*

Fig. 77

16.6. Determine the direction of the current induced in a


frame (Fig. 77) rotating in a uniform magnetic field in the
direction indicated by the arrow.
16.7. Two identical steel rods are brought close to a cop­
per ring suspended in the vertical plane. In one case, the
ring is repelled from the rod. Why?
16.8. Determine the direction of the current .induced in a
solenoid if a bar magnet is moved upwards in it (Fig. 7.8)..
§ 16. Electromagnetic Induction 175

16.9. A bar magnet is caused to approach a copper ring as


shown in Fig. 79. Determine the direction of the current
induced in the ring.
16.10. A permanent bar magnet is falling through a cop­
per cylinder. Will the motion be free fall?
16.11. A conductor that has been folded in two moves in a
uniform magnetic field crossing magnetic field lines. Is an

Fig. 78 Fig. 79

emf induced in the conductor? What would a galvanometer


attached to the ends of the moving conductor indicate?
16.12. What is the value of the emf induced in a circuit
if the magnetic flux through it varies by 3.4 x 10"2 Wb/s?
16.13. A conductor of active length 15 cm moves at 10 m/s
at right angles to the lines of a uniform magnetic field with
induction 2.0 T. What current is induced in the conductor if
it is short-circuited? The resistance of the circuit is 0.5 Q.
16.14. A linear conductor of active length 0.7 m crosses
a uniform magnetic field at an angle of 30° and at 10 m/s.
Determine the magnetic induction of the field if the emf
induced in the conductor is 4.9 V.
16.15. Determine the emf induced at the ends of an aero­
plane's wings moving horizontally at 900 km/h if the wing
span is 36.5 m and the vertical component of the Earth’s
magnetic field is 40 A/m.
16.16. An electromagnet produces a magnetic field of
strength 4 x 10s A/m in air at its poles. Assuming that the
field is uniform, determine the minimum velocity of a con­
ductor of active length 10 cm required for an emf of 1 V
to be induced in it if the angle between the vectors of mag­
netic induction and velocity is 90°,
176 Ch. II. Fundamentals of Electrodynamics

16.17. A magnetic flux of 30 mWb piercing a closed circuit


decreases to zero in 1.5 X 10~2 s. Determine the average
emf and current induced in the circuit. The resistance of the
circuit is 4 Q.
16.18. A straight conductor moves at a velocity of 4 m/s
in a uniform magnetic field of induction 6 mT. The active
length of the conductor is 0.3 m. What is the angle between
the velocity and magnetic induction vectors if a potential
difference of 3.6 X 10“3 V is induced across the ends of the
conductor?
16.19. A solenoid contains 200 turns with a cross-sectional
area of 80 cm2. The magnetic induction in the solenoid
increases over 0.1 s from 2 to 6 T. Determine the average emf
induced in the solenoid winding.
16.20. What is the change in the magnetic flux in a coil
over 0.05 s if the coil contains 1000 turns, and the emf in­
duced in it is 120 V?
16.21. Determine the self-inductance emf in a coil of in­
ductance 0.5 H if the current in it decreases by 0.2 A over
lO- 1 s.
16.22. A voltage of 20 V is applied to a coil of inductance
1 H. How long does it take the current in the coil to reach
30 A?
16.23. The inductance of a circuit is 40 mH. What is the
self-inductance emf emerging in the circuit if the current
in it has changed by 0.2 A over 0.01 s? What is the change
in the magnetic flux in the circuit in this case?
16.24. The inductance of a circuit is 0.05 H. What is the
magnetic flux piercing the circuit if the current in it is 8 A?
16.25. Determine the inductance of a coil in which a self­
inductance emf of 0.5 V emerges as a result of a decrease
of 0.2 A in the current in 0.04 s.
16.26. How can we increase the inductance of a solenoid?
16.27. Why does a strong spark appear when a knife-
switch is used to disconnect a circuit containing a coil
with a core? How can this effect be eliminated?
16.28. A magnetic flux of 0.14 Wb is coupled with a cir­
cuit whose inductance is 0.02 H. Determine the current
passing in the circuit.
16.29. At what current in a coil of inductance 40 mH is
the magnetic field energy equal to 0.15 J?
16.30. Determine the energy stored in the magnetic field
§ 16. Electromagnetic Induction 177

of a coil of inductance 85 mH if the current passing through


the coil is 8 A.
16.31. A charge of 6 x 10~2 C for 0.01 s passes through
the cross-section of a coil of inductance 12 m il for a long
time. What are the magnetic energy and the magnetic flux in
the coil? What will the self-inductance emf emerging at the
moment the circuit is disconnected be if the magnetic flux
drops to zero in 0.05 s?
16.32. A circular loop of radius 5 cm is placed in a uniform
magnetic field of induction 1.2 x 10~2 T so that the normal
to the plane of the loop coincides with the direction of the
field. The resistance of the loop is 3.1 Q. How much elec­
tricity will pass through the loop if it is turned bv an angle
of 60° ?
16.33. One turn of insulated wire in the form of a planar
square frame with side I = 0.2 m is placed in a uniform
magnetic field perpendicular to the magnetic field lines.
Determine the current passing through the turn if the mag­
netic field starts to decrease at a constant rate of 0.1 T/s.
The resistance of the turn is 1 £2.

12-0530
Chapter III

Oscillations and Waves

§ 17. MECHANICAL VIBRATIONS AND WAVES.


SOUND AND ULTRASOUND

Basic Concepts and Formulas


A vibration (oscillation) is a periodical motion in which
a body (particle) passes through its equilibrium position
moving alternatively in opposite directions.
The time taken for a vibration to be completed is called
the period T, while the quantity reciprocal to the period is
known as the vibration frequency v:
v = l/7 \
The frequency is th number of vibrations that occur per
second. The unit of frequency is the hertz (Hz).
The most important type of vibration is the harmonic
vibration (harmonic motion) caused by a force proportional
to the displacement x. The displacement of a point under­
going a harmonic motion is given by the equation
x = A sin (cot + cpn),
where A is the amplitude of vibration, cot -}- cp0 = cp is the
phase, cp0 is the initial phase, and co is the circular frequency:
to = 2nv, or co = 2n/T.

The unit of circular frequency is the radian per second


(rad/s).
Remark. In the equation for a harmonic motion, A is the
maximum displacement (zrmax) and can be denoted by X.
By analogy, the amplitudes of velocity (vmax) and acce­
leration (amax) can be denoted by V and A respectively.
A simple pendulum (a particle suspended on a weightless
unstretchable string) has the following period for small
vibrations:
T--=2n V llg.
§ 17. Mechanical Vibrations and Waves 179

For a load of mass m vibrating on a spring with spring con­


stant k, the circular frequency and the period are given by
o>= | / k/m, T = 2ji ] / m/k.
A vibrating body has a potential and a kinetic energy.
For a load vibrating on a spring, the total energy of the
vibrations is given by

For an arbitrary displacement x, the energy of the load is

W-W' +W i - O f + X f .
If a particle vibrates in a medium, it causes neighbouring
particles to vibrate, and a wave propagates in the medium.
The velocity v and the wavelength k of the wave are related
thus
v = k/T, or v = kv.
The velocity of a wave does not depend on the frequency
of vibrations and only depends on the properties of the me­
dium. Therefore a transition to a different medium causes
the velocity and wavelength to change, while the frequency
remains unchanged.
Waves having frequencies from 16 to 20000 Hz can be
perceived by the human ear and are known as acoustic (sound)
waves. The velocity of sound is usually denoted by c.

Worked Problems
Problem 84. A boy rocks up and down on a board. The
time he takes to move from the upper position to the lower
position is 1.5 s. What is the frequency, circular frequency,
and period of the vibrations?
Given: t = 1.5 s is the time during which the boy moves
from the extreme upper to the lower position.
Find: the frequency v of the vibrations of the boy on the
board, the circular frequency to, and the period T of the
vibrations.
Solution. In this case, the period of the vibrations can be
defined as the time elapsed between two successive upper-
12*
180 Ch. III. Oscillations and Waves

most (or lowermost) positions of the boy. Thus,


T - 2t.
The frequency and the circular frequency can be deter­
mined from the formulas
v = 1IT, to = 2jiv.
Substituting in the numerical values, we obtain
7’ = 2 x 1.5 s = 3 s, v = — = 0.33s-i = 0.33H z,
u) = 2 x nrad X 0.33 s_I = 2.1 rad/s.
Answer. The frequency of the vibrations is 0.33 Hz, the
circular frequency is 2.1 rad/s, and the period is 3 s.
Problem 85. The coordinates of a point are defined by the
equation
x = 1.2 cos it (2t/3 -f- 1/4).
Determine the amplitude, circular frequency, frequency,
period, and the initial phase of the vibrations. Determine
the amplitudes of velocity and acceleration. What will the
phase be in 0.375 s after the beginning of motion?
Given: x = 1.2 cos n (2t/3 + 1/4) is the equation of mo­
tion of the vibrating point, t = 0.375 s is the time elapsed
after the beginning of vibrations.
Find: the amplitude X, the circular frequency to, the
frequency v, the period T, the initial phase q>0, the amplitude
V of velocity, the amplitude A of acceleration, and the
phase (p at the instant t.
Solution. Let us first transform the equation to a form
containing a sine instead of a cosine. We know from trigo­
nometry that cos a = sin (a + ji/2). Hence
x = 1.2 cos ji (2t/3 + 1/4)
= 1.2 sin [ji (2t/3 + 1/4) + ji/2]
= 1.2 sin [2ji</3 + ji/4 + n/2]
= 1.2 sin (2nt/3 + 3ji/4).
Comparing this equation with the equation of harmonic
motion x = X sin (at + <p0), we obtain X - 1.2 m, « =
2ji/3 rad/s = 2.1 rad/s, q>0 = 3n/4 rad = 2.36 rad.
§ 17. Mechanical Vibrations and Waves 181

The frequency, period, and phase can be determined from


the formulas
v - co/2ji, T = 1/v, <p = (at + <p0.
In order to find the amplitude of the velocity, we must
differentiate the equation of motion. We obtain
dx
-T7- =Xo)COs(a)7+q>0), or v = V cos (co£ + q>0).

Hence V = X<d.
To find the amplitude of the acceleration, we must dif­
ferentiate the velocity equation

- j - = - t'wsin (orf + cp0),


or
a = —A sin (wi + q>0).
Hence
A -- — Vto = — X to2.
Substituting in the numerical values, we obtain
2it rad/s
v = 0.33 Hz, 3 s,
3 X 2n rad
2n rad/s x 0.375 s rad = 3.14 rad,
3

F = 1 .2 m x - —-s -i = 2.5 m/s,

^ = -1.2m x s-2 = - 5.27 m/s2.

Answer. The amplitude of the vibrations is 1.2 m, the


circular frequency is 2.1 rad/s, the frequency is 0.33 Hz, the
period is 3 s, the initial phase is 2.36 rad, the velocity am­
plitude is 2.5 m/s, and the acceleration amplitude is
-5.27 m/s2. The phase in 0.375 s after the beginning of mo­
tion is 3.14 rad.
Problem 86. A load of mass 100 g is fixed on a spring (see
Kig. 85) with constant 100 N/m. It is displaced 3 cm from
the equilibrium position and receives a velocity of 10 cm/s.
What are the potential and kinetic energies of the load at
182 Ch. III. Oscillations and Waves

the initial moment? What is the total energy of the load?


Write the equation of its motion.
Given: m = 100 g = 0.1 kg is the mass of the load,
k = 100 N/m is the spring constant, x = 3 cm = 0.03 m
is the initial displacement of the load from the equilibrium
position, v = 10 cm/s = 0.1 m/s the initial velocity of the
load.
Find: the potential energy E p0 of the load at the initial
moment, the kinetic energy E k0 of the load at the initial
moment, the total energy E of the load and the equation of
motion.
Solution. The potential and kinetic energies of the load
can be determined from the formulas
kx2 m v 2
Epo ~ ’ E\to 2 •

The total energy of the load is


E = E p0 -)- E^q.

In order to write the equation of motion, we must find


the amplitude A, the circular frequency co, and the initial
phase of vibrations. It should be noted that the initial dis­
placement of the load is not the amplitude since in addi­
tion to the initial displacement the load receives a veloci­
ty. However, the total energy can be expressed in terms of
amplitude: E = kA2l2, whence
A = V 2E/k.

The circular frequency can be found from the formula


(0 = V k/m-

In order to determine the initial phase, we write the equa­


tion for harmonic motion in the general form:
x = A sin (o)t + <p0).
At time t — 0, the equation has the form x = A sin <p0,
whence
(p0 = arc sin (xiA).
§ 17. Mechanical Vibrations and Waves 183

Substituting in the numerical values, we obtain


Ev0 = 100 N/mX| x .^°~4 m* = 4 .5 x 10-2 J ,

E k0 = 0,1 kgX° '01 m2/s2 = 5 x 10“‘ J,


£ = 4.55 x 10”2 J,

-<-i/ ^ ^ F !?=3o,7x,o"2m-
<»- *” |N|^ 1 = 3lr> r;ui's >
Q
rp0= arc sin = arc sin 0.9945 = 1.4658 rad.
Answer. The potential and kinetic energies at the initial
moment are 4.5 x 10"2 J and 5 x 10"4 J respectively, the
total energy is 4.55 x 10~2 J. The equation of motion for
I he load has the form x = 3.017 x 10-2 x
'///////////A
sin(31.6f + 1.4658).
Problem 87. What will the change in the
period of oscillations of a pendulum be in a
lift moving upwards with an acceleration of
0.3g?
Given', a = 0.3g is the acceleration of the
lift.
Find: the period T of oscillations of the
pendulum. mg
Solution. Let us go over to a coordinate
system fixed to the lift. Then the pendulum Fig. 80
will experience the action of an inertial force
equal to ma and directed against the accelera-
lion a. To clarify this, let us consider the pendulum at rest.
It is acted upon by the tension F of the string and the force
of gravity mg. Now let the pendulum move with the accel­
eration a together with the lift (Fig. 80). We write Newton’s
second law
F — mg = ma, or F = m (g + a).
Tims, the weight of the load as if increases by the quantity
ma which is known as the inertial force. We must then replace
g in the formula for the period of oscillations by g + a
T = 2 n V U { g + a).
184 Ch. III. Oscillations and Waves

In the absence of acceleration, we have


T0 = 2n VTg.
Dividing these two equalities termwise, we obtain

T7 = V T+^ ’ or T ""T°^ 7T^ ’


Substituting in the numerical values, we obtain

V - n fa r - T k ~ & = 0 S 1 1 T ‘-
Answer. The period of oscillations of the pendulum will
decrease by a factor of 1.14.
Problem 88. Using vector diagrams, determine the ampli­
tude and phase of a vibration which is the resultant of three
harmonic vibrations = sin cot, x2 = 2 sin(co£ + jt/2),
and x3 = 2.5 sin (cot + n). Write its equation.
Given: xt = sin cot, x2 = 2 sin (cot + n/2), and x3 =
2.5 sin (cot ■+• n) are the component vibrations.
Find: the amplitude A and the initial phase <p0. Write
the equation of the resultant vibration.
Solution. From the given equations of harmonic motions,
we can determine the amplitudes and the initial phases:

Fig. 81
•i4j = 1 m, A 2 — 2 m, and A n = 2.5 m, cpni = 0, cpn2 =
ji/ 2 rad, and cp03 = it rad.
Using vector diagrams, we can sum vibrations of the same
frequency. In the case under consideration, this condi­
tion is satisfied.
For each vibration, a vector is plotted from the origin,
the length of the vector being equal to the amplitude, and
the slope equal to the initial phase. The composition of these
three vectors yields the amplitude and phase of the required
vibration. Figure 81a shows vectors with lengths j4lt
§ 17. Mechanical Vibrations and Waves 185

A 2, and A 3 and slopes 0, nl2, and ji. Let us first compose


the vectors lying on the same straight line (Fig. 816), and
then find the required vector using the parallelogram rule
(Fig. 81c).
Thus we get
A3 —At = 1.5 m,
A = V A l + ^ - A i ) 2, A = V * m2-f 2.25 m2 = 2.5 m,

sin a = -yTj-~ 0-8, a = 53°8',


cp= 180° - a = 126°52' = 0.705n.

Answer. The resultant vibration has an amplitude of


2.5 m and an initial phase of 0.705 rad, and is described by
the equation x = 2.5 sin (a>t + 0.705n).

Questions and Problems


17.1. What is the period of the vibrations of a particle
if it completes a single vibration in 5 s?
17.2. The vibration frequency of a particle is 2 Hz. How
many vibrations does it complete in 1 s?
17.3. A particle performs 60 complete vibrations in 2
min. What are the period and vibration frequency?
17.4. A load suspended on a spring vibrates at a frequency
o f 0 .4 Hz. Determine the circular frequency and the period
o f vibrations.
17.5. A ball is dropped on the floor. 0.5 s after the impact,
the ball reaches the upper point of its trajectory. Assuming
Ihat the impact of the ball against the floor is perfectly elas­
tic, determine the period and frequency of the motion.
17.6. Will the mode of the vibrations of a tuning fork
change if it is immersed in water?
17.7. An areometer is immersed in a liquid and then re
leased. In 1/8 s, it attains its equilibrium position. What
lire the areometer’s frequency and period of vibrations?
17.8. Figure 82 shows three pairs of oscillating pendulums.
What are their phases relative to each other?
17.9. The disc of a record player turns at 33 revolutions
per minute. Determine the circular frequency of the disc.
186 Ch. III. Oscillations and Waves

17.10. The angular velocity of the drive wheel of an au­


tomobile is 30 rad/s. What is the frequency and the period
of motion of the engine piston on direct gear?
to) (*) (c)

V-0 VO VO vo
Fig. 82

17.11. A merry-go-round turns through 90° in 0.5 s. De­


termine its frequency, period, and circular frequency.
17.12. The motion of a body is described by the equation
x = 4.25 sin (0.3i + 0.75).
What is the amplitude, circular frequency, initial phase,
and the period of oscillations? What is the phase at the in­
stant t = 0.5 s?
17.13. The equation of a harmonic motion has the form
x = 0.02 sin — t.
Determine the displacement of the body from the equilib­
rium position at tt = 0, t3 = 774, t3 -- 772, and <4 = 7 7712.
17.14. Using the conditions of Problem 17.13, determine
the phase of vibrations for tY = T, t 2 = 772, and t 3 =
774.
17.15. The position of a vibrating particle is determined
by the equation
x = 0.05 sin (at.
Determine the displacement of a particle if the vibration
phase is jt/4.
17.16. The equation of a harmonically vibrating particle
has the form
x = 1.2 sin n (- — ).
Determine the displacement of the particle at the instants
= 0, t 2 = 77724, and t 3 = 117724.
§ 17. Mechanical Vibrations and Waves 187

17.17. Using the conditions of Problem 17.16, determine


the coordinate of the particle at t1 = 177724, t 2 = 2T,
t3 = 3778. Plot the graph x = / (t).
17.18. The position of a vibrating body is described by the
formula x = 8 sin u>t. Determine the displacement of the
body if the vibration phase is 30°.
17.19. Determine the amplitude of the vibrations if the
displacement of a particle at a phase of 45° is 10 cm.
17.20. Using the graph of the vibrational motion of a
particle, shown in Fig. 83, determine the amplitude, fre­

x.mi
6
4
Z
0
-2 o.s t,s
-4

Fig. 83

quency, period, and the initial phase of the vibrations. Write


the equation of motion.
17.21. The law of vibration of a particle is represented
graphically in Fig. 83. Determine the displacement of the
particle at t = 5T/12.
17.22. Given the equations of harmonic motion of two
bodies
(a) xx = 2sin 0.5n (2< + 1),
x2 = 3 sin (nf + 1.5ji),
(b) xx = sin (tof + ji/2),
x2 = cos (Of.
Indicate the phase difference in their vibrations.
17.23. Using the equations of vibrations of two bodies,
determine their phase difference:

(b) Xj = 4 sin 1/4 (at -|- 7m), xz = 0.02 cos 1/3 (0.75o)t —3n).
188 Ch. III. Oscillations and Waves

17.24. Write the equation of a harmonic motion if


(a) to = n, Xl = ] / 3 for = 0, and x2 = 1for t2 = Tik.
(b) xt = 0 for t, = 1, x2 = y 3/2 for t 2 = 2, and x3 =
—3/2 for t 3 = 3.
17.25. Write the equation of a harmonic motion if
(a) A = 9 cm, v = 20 Hz,
(b) A = 5 m, v = 0.5 Hz, q>0 corresponds to 778,
(c) A = 1 m, T = 6 s, ip0 = — ji / 4 .
17.26. A body floating on the surface of a liquid is immersed
to a depth of 10 cm and then released. It starts to vibrate
at a frequency of 2 Hz/s. How long does it take the body to
move 5 cm from its equilibrium position? Determine the
time required for the body to traverse a distance of 5 cm from
the point of maximum deviation. What distance will the
body cover in 10 s? The vibrations should be assumed to be
undamped.
17.27. A pendulum consists of a ball suspended on a string.
What forces cause it to oscillate? What is the direction of
the restoring force? At what point is it at a maximum?
At a minimum?
17.28. A ball suspended on a string performs small oscil­
lations about the equilibrium position. How does the veloc­
ity of the ball vary during one period? Can the motion of
the ball be defined as uniformly variable? Why?
17.29. Two balls of the same size, one aluminium and one
lead, are suspended on strings of equal length. The balls are
deflected through the same small angle and released. Will
the periods of their oscillations be the same?
17.30. Will the two balls in Problem 17.29 come to a
halt at the same time? If not, which stops oscillating first?
17.31. At what position does a pendulum have the maxi­
mum and minimum acceleration and velocity?
17.32. Determine the restoring force acting on a simple
pendulum of mass 10 g for a deflection angle of 45°.
17.33. Determine the mass of a pendulum if the driving
force is equal to 1 N at a deflection angle of 30°.
17.34. Through what angle is a copper ball of diameter 2 cm
deflected if the restoring force acting on it is 0.183 N?
17.35. Why does a pendulum clock keep the correct time
only at a certain latitude?
§ 17. Mechanical Vibrations and Waves 189

17.36. The pendulum of a clock consists of a metal rod and


a load that can be fixed at any point along the rod. How will
the pace of the clock change with decreasing room tempera­
ture? How can the correct pace be restored?
17.37. What will the change in the period of oscillations
o f a pendulum be if its length is increased fourfold?
17.38. How should the length of the pendulum of a clock
he changed for it to keep correct time on the Moon?
17.39. Determine the oscillation periods of a 1-m long
pendulum in Moscow (g = 9.816 m/s2) at the North pole
(g = 9.832 m/s2), on the equator (g = 9.78 m/s2), and on the
Moon (g = 1.63 m/s2).
17.40. The pendulum mounted in the St. Isaac’s Cathe­
dral in Leningrad is 98 m long and completes 181.5 oscilla­
tions per hour. Using these data, determine the free fall
acceleration in Leningrad.
17.41. What will the speed of a pendulum clock be in (a)
a uniformly ascending lift? (b) a lift falling freely to the
ground? What effect will the phase of the pendulum’s oscil­
lations at the moment the lift starts falling have?
17.42. A ball of mass rn is suspended on a weightless
unstretchable string. The period of its oscillations is T„.
In addition to the force of gravity, a force F acts on the ball
along the vertical. What must the direction and magnitude
of this force (in comparison with the force of gravity) be
f o r the oscillation period to be (a) 27’0, (b) 0.87V
17.43. What must the force F be (see Problem 17.42) to
slop the pendulum’s oscillations?
17.44. A horizontal force F acts on a ball (see Problem
17.42). What must the magnitude of this force be (com­
pared to the force of gravity) for the oscillation period to be
(a) T = 0.946To, (b) T = 0.87V What will be the equilib­
rium position? Draw a diagram.
17.45. A pendulum consists of a metal ball of mass 10 g
and a silk thread. A charge of 10“®C is supplied to the ball,
and a horizontal electric field with strength 2 X 104 V/m
is applied. How will the mode of the pendulum’s oscilla­
tions change? What will the ratio of the oscillation periods
lio in the two cases?
17.46. A seconds pendulum (viz. a pendulum with an os­
cillation period of 1 s) is in a train moving with accelera-
190 Ch. III. Oscillations and Waves

tion a = 0.458#. Determine the period of oscillations of the


pendulum.
17.47. A pendulum the period of whose simple oscilla­
tions is 0.5 s is in a lift descending with acceleration a =
0.19#. What is the frequency of its oscillations?
17.48. A load is fixed on a spring as shown in Fig. 84.
When it is set vibrating along the vertical, the load is acted

A-------------------1 1n r v m n n fto l
A
' — [_ _ p O O tX J O U O « ^

Fig. 84 Fig. 85
upon by the elastic force of the spring and the force of grav­
ity. W ill the load’s oscillations be harmonic? What effect
does the force of gravity mg have?
17.49. One end of a spring with a constant of 50 N/m
is fixed, and a 1-kg load is suspended from the other end.
Determine the vibration frequency of such a pendulum.
17.50. The period of vibrations of a spring pendulum is
0.25 s. What is the spring constant if the mass of the load
is 200 g?
17.51. A body of mass 0.5 kg fixed to a spring stretches it
by 1 cm at rest. When it is displaced by 3 cm downwards and
released, it starts vibrating harmonically. Determine the
amplitude, circular frequency, period, and initial phase.
Write the equation of motion.
17.52. A body of mass 800 g is fixed to a spring with con­
stant 40 N/m and vibrates as shown in Fig. 85. The ampli­
tude of vibrations is 2 cm. What is the energy of vibrations?
Determine the maximum velocity and acceleration.
17.53. The motion of a body whose mass is 2 kg is de­
scribed by the equation
x = 0.8 sin ^nt + ).
Determine the energy of the vibrating body. How does the
energy depend on the initial phase?
17.54. A load of mass 1 kg is suspended from a spring
with constant 1000 N/m and placed in a rocket that is
§ 17. Mechanical Vibrations and Waves 191

launched with acceleration a = 2g. When the engine is


switched off, the load starts vibrating. What is the energy of
the vibrating load? What is its velocity when the load is
0.5 cm away from its equilibrium position? What are its
kinetic and potential energies at this point? Write the equa­
tion of motion, assuming that the free fall acceleration g
does not change during the flight.
17.55. The equation of motion of a particle has the form

x = 6 cos 0.2 sin Jt ——^ - cos ( n — O.ln j .

Write the equation in a more convenient form and determine


from it the amplitude, frequency, period, initial phase, and
circular frequency of the vibrations.
17.56. A watch with a spring and a pendulum clock are
mounted on a rocket being launched vertically upwards fly­
ing with an acceleration ax = g for 30 s, after which its
engines are switched off. When it is 980 m from the surface
of the Earth, the rocket begins slowing down and lands soft­
ly. What is the difference between the clock readings?
17.57. Plot the graphs of harmonic motions described by
the equations (1) x = 2 sin (at, (2) x = 3 sin (ati2, and
(3) x = 2 sin ((at -\- n).
17.58. Figure 86 shows the graphs of five vibrational mo­
tions. Determine the amplitudes of vibrations I I and V.
What is the amplitude of the resultant vibration? Compose
vibrations I I and V graphically.
17.59. Compose vibrations I and I I and / and V (Fig.
86) analytically and graphically. Determine the amplitude
of the resultant vibrations.
17.60. Using vector diagrams determine the amplitude of
the resultant vibrations (see Fig. 86) for (a) / and IV, (b)
II, IV, and V, and (c) I, III, and IV.
17.61. Using vector diagram, determine the phase shift
in the composition of vibrations I, III, and IV (see Fig.
86) .
17.62. Using vector diagrams, determine the amplitude
and phase of the vibration obtained by subtracting V from
IV (see Fig. 86).
17.63. Describe the motion of a particle participating in
a wave motion.
192 Ch. III. Oscillations and Waves

Fig. 86
§ 17. Mechanical Vibrations and Waves 193

17.64. Figure 87 shows the shape of a transverse wave.


The wave propagates to the right. Use arrows to indicate the
directions of the individual particle velocities.
17.65. Determine the directions in which the waves in
Figs. 88a and b propagate. The arrows indicate the direc­
tions of the velocities of the particles.
17.66. On what does the propagation velocity of a trans­
verse wave at the boundary between two liquid media de­
pend?
17.67. A wave propagates over the surface of water at a
velocity of 3 m/s. What will the velocities of the particles be
on the crest of the wave?
17.68. In southern countries, surfing (riding a hoard on the
crests of waves) is very popular by the sea. At what velocity

Fifj. 87 Fig. 88

does a wave carry a surfer if the wavelength is 25 m and the


wave particles vibrate with a period of 1.5 s?
17.69. In which medium is the velocity of sound higher:
air or iron? Can sound propagate in vacuum?
17.70. While opening a door, we sometimes hear a creak.
What is the origin of this sound? What is the role of the
door?
17.71. Which properties of a medium determine Ihe ve­
locity of sound? Explain by taking air and water as examples.
17.72. What does the velocity of sound in a given medium,
say, air depend on?
17.73. Why do people press an ear against the rails to
know whether a train is coming?
17.74. A meteorite strikes the surface of the Moon. How
long will it take for sensitive instruments on the Earth to
detect the sound of the explosion?
13 - 0 5 3 0
194 Ch. III. Oscillations and Waves

17.75. If a vibrating tuning fork is held against a table,


the sound becomes much louder. Why?
17.76. A vibrating tuning fork is first held in the hand and
then its end is brought in contact with a table. In which case
does the sound cease sooner and why?
17.77. The human ear has its greatest sensitivity in the
frequency range 1.5-3 kHz. Determine the wavelengths cor­
responding to this range if the velocity of sound is 340 m/s.
17.78. Sound propagates in air at 330 m/s for a vibration
frequency of 0.5 kHz. Determine the shortest distance in
the propagation direction between two points vibrating in
phase.
17.79. If sound waves propagate in a medium at 340 m/s
and a frequency of 500 Hz, what is the phase difference of
two particles of the medium 17 cm apart? The particles lie
on the line along which the wave propagates.
17.80. If we blow over the open end of the cap of a foun­
tain pen, a whistling sound is produced. Explain its origin.
17.81. The cap in Problem 17.80 is 5.5 cm long. What is
the frequency of the emerging sound?
17.82. The frequency of a tuning fork’s vibrations is
1.38 kHz. How long is the vibrating part of tiie tuning fork?
The velocity of sound is 332 m/s.
17.83. How will the frequency of the sound produced by a
tuning fork change with decreasing temperature?
17.84. The length of an acoustic wave of the same fre­
quency in air is a factor of 4.25 smaller than in water and
a factor of 10.7 smaller than in brick. Determine the velocity
of sound in water and in brick assuming that the velocity7 of
sound in air is 340 m/s.
17.85. What is the change in frequency, period of vibra­
tions, and wavelength of sound when it passes from air to
steel? The velocity of sound in steel is 5000 m/s.
17.86. Determine the depth of a sea if the response from
an echo sounder is received in 1.6 s, and the velocity of
sound in water is 1500 m/s.
17.87. Ultrasound is used to find defects in large bodies.
At what depth is a defect in an aluminium component de­
tected if the first reflected signal is received after 8 X 10"6s,
and the second, after 2 x 10~5 s? What is the height of the
component? The velocity of sound in aluminium is 510 m/s.
17.88. A person heard a clap of thunder 9 s after he saw the
§ 18. Alternating Current 195

flash of lightning. At what distance from the discharge was


he?
17.89. A thunderstorm takes place 4.5 km away from an
observer. What is the time interval between seeing the
lightning and hearing the thunder?
17.90. A ship surveying the bottom of the ocean moves at
.'Hi km/h. What is the percentage error in the measured depth
due to the motion of the ship? What is the error for a depth
of 3000 m?
§ 18. ALTERNATING CURRENT

Basic Concepts and Formulas


An alternating current is one that periodically changes its
direction in a circuit so that the current averaged over a pe­
riod T is zero.
Alternating current is produced by vibrational motion of
charge carriers, viz. electrons.
When a frame is rotated in a uniform magnetic field, an
emf is induced across its ends:
6 = $ max sin tot,
where e is the instantaneous emf, £ max is the maximum
(amplitude) emf, and a) is the angular velocity of rotation of
the frame and the circular frequency of alternating emf.
l he period and frequency are

The resistance R in which heat is liberated due to the pas­


sage of a current through it is known as ohmic resistance.
If the frame is connected to a load with an ohmic resistance,
an alternating electric current will pass through it, and an
alternating voltage will appear across the terminals:
i = / max sin tot, u = Umax sin tot.
Here u = iR.
An alternating current (a.c.) is equivalent to a direct cur­
rent of the same power. The effective values of alternating
voltage and current are
fj _ f^max j
J/2 ’
13*
196 Cli. III. Oscillations and Waves

An a.c. circuit containing a coil of inductance L has an


inductive reactance
X L — coL.
An a.c. circuit including a capacitor of capacitance C
has a capacitive reactance:
1
A c —-(iiC
The reactance of a circuit containing a capacitor, a re­
sistor, and an inductance coil in series is given by
X = X L - X c,
while the impedance is
z = V w + ( x L - x cy>.
The relation between Z, R, and X is presented graphically
by a triangle (Fig. 89), i.e. obeys the rules of geometric com­
position. The angle cp in the figure is
the phase difference between current
and voltage.
x*xL-xc Using the amplitude and effective
values, we can write Ohm’s law for
an alternating current, i.e.
Fig. 89
U„ = /maxZ, U = 1Z.
In the presence of a reactance, the current in the circuit
is not in phase with the voltage. If we assume that the ini­
tial phase of the current is zero, the voltage across the induc­
tive reactance leads the current by jx / 2 , while the voltage
across the capacitive reactance lags behind the current by
ji/2. Therefore, only in an ohmic resistance do the current
and voltage oscillate in phase. Hence, alternating currents
and voltages cannot be added algebraically, as was the case
for direct current. The composition is carried out using
vector diagrams in which the emf, voltage, and current are
depicted by vectors emerging from the origin and forming
with the abscissa axis an angle equal to the initial phase.
The average power liberated in an a.c. circuit is
p _ c o s ^ __ j y c o s ^

In electrical engineering, the concepts of active, reactive,


and total power are introduced. The active power P is asso-
§ 18. Alternating Current 197

ciated with the conversion of electrical energy into heat and


is given by
P = PR.
The unit of active power is the watt (W).
The reactive power Q is given by
Q = P X L or Q = P X C.
The unit of reactive power is the var (volt-ampere reactive).
The total power
S = PZ
is connected with P and Q through the power triangle
(Fig. 90). The unit of total power is volt-ampere (VA).
The active power is the average pow­
er liberated in the circuit. The reactive
power is associated with energy trans­
mission from the generator to a circuit
mid back (it is stored in inductance coils
and capacitors). The reactive power av­
eraged over a period is zero.
The active, reactive, and total power can be expressed in
terms of the effective current and voltage in the circuit:
P = I U cos <p, Q I U sin q>, S — IU.
Here cos cp is the power factor. It can be found as the ratio
of the resistance to the impedance or of the active power to
the total power:

Worked Problems
Problem 89. A resistor with resistance 20 £2, a coil with
inductance 0.0398 H, and a capacitor with capacitance
159 pF are connected in series to a generator. Determine the
voltage across the circuit components and in the entire cir­
cuit. What is the phase difference between the voltage and
the current? Determine the impedance of the circuit. What
will happen if the capacitive and inductive reactances are
equal? The frequency of the alternating current is 100 Hz
and the current is 2 A. Plot the vector diagram.
198 Ch. III. Oscillations and Waves

Given'. R = 20 Q is the resistance, L = 0.0398 H =


3.98 X 10"2 H is the inductance of the coil, C = 159 p,F =
1.59 X 10“4 F is the capacitance of the capacitor, / =
2 A is the effective value of the current, and v = 100 Hz
is the frequency of the alternating current.
Find,-, the voltages UR, UL, Uc , and U across the resis­
tor, the coil, the capacitor, and in the entire circuit respec­
tively, the phase difference cp between the voltage and the
current, and the impedance Z of the circuit.
Solution. The reactances of the coil and of the capacitor
can be determined from the formulas X L = oiL and X c =
l/o)C.
The impedance is given by
Z = V W + (Xl - X c)2= V & + (coL — l/(oC)2.
When the components of the circuit are connected in se­
ries, the same current passes through it, while the total vol­
tage is the sum of the voltages across the individual compo­
nents. The voltage across the resistor is
UR = IR,
and is in phase with the current. The voltage across the coil
is
UL = /coL,
and leads the current by n/2. The voltage across the ca­
pacitor is
Uc = / / (coC),
and lags behind the current by rt/2.
The total voltage can be found by constructing a vector
diagram. We choose the origin and draw a horizontal vector,
viz. the current axis relative to which the voltage vectors
will be plotted (Fig. 91a).
The vector of voltage UR across the resistor is plotted (from
zero) along the current axis since its phase angle is zero.
We now plot the vector of voltage UL across the inductance
coil. Positive angles are measured counterclockwise, so
the voltage vector will be plotted upwards at right angles
to the current axis.
The vector of the voltage Uc across the capacitor is plot­
ted downwards from zero at right angles to the current axis.
$ 18. Alternating Current 198

The geometric sum of the three vectors will give the total
voltage in the circuit. First we compose the vectors directed
along the vertical, and as a result obtain a voltage triangle
(Fig. 916) from which we can easily find the voltage and the
phase difference:
U x = UL —V c ,
U = V U r + U \ = I V R * + (cuL — l/coC)2,
mL —1/coC Ux
tan <p=
R Ur '
If a L = 1/toC, the reactance is zero. This phenomenon
is observed at the circular frequency w = to0 determined
fa)

from the formula = 1/(LC). Then the impedance of the


circuit attains its minimum value and is equal to the resis­
tance: Z = R, while the current attains its maximum value.
This phenomenon is known as voltage resonance, and to0
is the resonance circular frequency.
Substituting in the numerical values, we obtain
UR = 2 A x 20 Q --=40 V,
Ul = 2 A x 2xc x 100 s"1 x 0.0398 H = 50 V,
Uc = 2 A/(2it x 100 s-‘ x 159 x 10"4 F) = 20 V,
U = v (40 V)2 + (30 V)2 = 50 V,
X L — X c = 2ji x 100 s-‘ x 0.398 H - ( 2 n x 100 s' 1
x 1.59 x 10-4 F)“‘ ==15 Q,
tan cp= 15 S2/20 Q = 0.75, <p= 36°52\
U 50 V
z= V R 2+ ( x l Xc)z - / 2A
=-- 25 Q.
200 Ch. III. Oscillations and Waves

Answer. The voltages across the resistor, the inductance


coil, and the capacitor are 40, 50, and 20 V respectively, the
total voltage is 50 V, the phase difference is 36°52', the im-

Fig. 92 Fig. 93

pedance is 25 Q. When the inductive and capacitive reac­


tances are equal, resonance is observed.
Problem 90. A generator supplies an a.c. voltage of
80 V at a frequency of 20 Hz and is connected to an induc­
tance coil and a capacitor of 750 pF
connected in parallel. The resistance
of the coil is 1 Q and the induc­
tance is 0.1 H (Fig. 92).
Determine the currents in the
branches and the total current in
the circuit. What is the resonance
condition for the circuit? Determine
the impedance at resonance if
to 2L 2 > R 2.
Given: U = 80 V is the voltage
in the circuit, v = 2 0 Hz is the fre­
quency, C = 750 pF = 7.5xlO ~4 F
is the capacitance of the capacitor,
L = 0.1 H and R = 1 Q are the
inductance and the resistance of
the coil.
Find: I c, I L, and / , the currents in the branches and in the
entire circuit and the impedance Z at resonance.
Solution. Let us first consider the branch containing the
coil. We plot the vector diagram (Fig. 93) in the same way
as in Problem 89.
§ 18. Alternating Current 201

Using Fig. 93, we can find U and cpL:


U = IL + tan«pL = - ^ .

For the parallel connection (see Fig. 92), the voltage U


is the same in each branch, while the current is equal to the
sum of the currents in the parallel branches. The currents in
the branches have different phases, and hence they should
be composed geometrically, using vector diagrams. We plot
the voltage axis (Fig. 94). The current in the branch with
inductance has the value
T U
L 1//?a+ uaLa ’
and lags behind the voltage by cp^.
The current through the capacitor is
7C = UwC,
and leads the voltage by ji/2. The composition of these vec­
tors results in a vector 7 at an angle <p to the voltage axis.
The current 7 can be determined by cosine law:
72 = I I + Ic — 27/7C cos a.
We know from trigonometry that
tan q>L a)L
sin (pL =
Y 1+ tan* (pL YR* + <*'l * '
since tan <pr = oiL/Tf (see Fig. 93); we have
cos (p, = — 1 — = R — .
1/1 + tan2 <pL F f l a + o )U a
Since a = Ji/2 —q>L, cosa = sinq>L. Therefore,
n = PL+ Pc - 27l 7c — =^k= = - .

Angle cp can be determined from the equality of the projec­


tions of 7 and I L on the voltage axis:
7 cos <p= I L cos (pL, cos <p= cos q>L.

It is often more convenient to compose the vectors geo­


metrically in a different way. Only one is plotted from
202 Ch. III. Oscillations and Waves

the origin. The next vector is plotted from the end of the
preceding one. The resultant vector then connects the ori­
gin of the first vector to the end of the last one. For our prob­
lem, such a construction yields a current triangle (Fig. 95),
the calculations remaining the same.
If one of the parameters to, C, or L of the circuit is varied,
the total current and the phase angle <p will change. For

Fig. 95 Fig. 96

a certain relation between ©, C, and L, the phase difference


vanishes (Fig. 96). Then the current will have the mini­
mum value, and hence the impedance of the circuit, which
becomes equal to the resistance, increases. This phenome­
non is known as current resonance. The resonance condition
can be determined from Fig. 96:
IC — IL Sin
Substituting 7C, I L, and sin <pL into this equation, we
obtain

]/i?2+ a 1/R * + w*L*


The inductive reactance of inductance coils is usually
much higher than their resistance (o)L^> R), the more so
to2L2^> R 2. Neglecting R 2 in the last equality and
simplifying, we obtain the condition for current resonance:
i
(02 oj;
LC ’
Let us calculate the impedance of the circuit. It follows
from Fig. 96 that
U______________ R
I — I l COS (flL =
V R2+ u>%L2 | / f l 2 + “ o L 2
$ 18. Alternating Current 203

Since to2L2^»/?2, we obtain


f UR _ URC L
© gi2 ~
whence Z
1 ~ L RC •
Substituting in the numerical values, we obtain
r 80 V c or .
I* -= — — = 6.35 A,
/ ( I Q )H (2 n X 2 0 s-‘ x0.1 H)2
/ c = 80 V x 2n x 20 s"1 x 7.5 x 10' 4 F = 7.54 A,
2It x 20 s-1 X 0.1 H
cos a = = 0.997.
/ ( 1 Q)2+ (2ji x 20 s- 1 X 0.1 H)2
/ = V (6.35 A)2 + (7.54 A)2- 2 x 6.35 A x 7.54 A x 0.997
= 1.28 A.
The resonance frequency is
1 . -- --------
_____ 1 - 18.4 Hz.
r 2ji Y LC 2it y o . i H X 7.5 X 1Q-* F
The impedance is
0.1 H
Z 1 Q x 7.5x10-* F
= 133 Q.
Answer. The current through the coil and the capacitor is
(5.35 A and 7.54 A respectively, the total current is 1.28 A.
Resonance sets in at a frequency of 18.4 Hz, and the impe­
dance of the circuit is then 133 Q.
Problem 91. An electric bulb rated for 240 V and power
of 200 W, a coil with inductance 0.15 H, and a capacitor are
connected in series to a lighting circuit at a voltage of
220 V. What is the capacitance of the capacitor if the active
power liberated in the circuit is equal to half the total
power? Determine the current, the voltages across the com­
ponents of the circuit, the active, reactive and total powers,
and the power factor.
Given: U = 220 V is the voltage in the circuit, v = 50 Hz
is the frequency of the alternating current, U b = 240 V
and P b = 200 W are the rated voltage and power of the
bulb, L = 0.15 H is the inductance of the coil, and P =
0.5S is the relation between the active and total power.
Find: the capacitance C of the capacitor, the current I
in the circuit, the voltages UR, UL, and Uc across the bulb,
• oil, and capacitor respectively, the active power P, reac­
tive power Q, total power S, and the power factor cos cp.
204 Ch. III. Oscillations and Waves

Solution. The resistance of the bulb can be determined


from the formula
P b = U i/R , whence R = U y p b = ^ ^ - = m Q .
The power factor can be determined from the relation
P = 0.5S:
P 0.5S
cos cp= -n- S
0.5.
In order to determine the capacitance of the capacitor, we
must find the inductive and capacitive reactances:
X l = 2jivL, X L = 2n x 50 Hz x 0.15 H = 47.1 Q,
— = R = 0.5Z = 0 . 5 ^ R2 + (Xc - X h) \
Squaring the last equality, we obtain
4fl2 = R* + (x c - X L])2, 3R* = (Xc - X L) \
Hence
X C = R V 3 + X t , Xc = 288 Q 1 /3 + 4 7 .1 Q = 546 £2,

c - 2« x 50 h! x 546 fl = 5.83 X I0~* F = 5.83 (iF.


The impedance is
Z = 1/(288 Q)2+ (546 Q - 47.1 £2)2 = 576 Q.
Let us now find the current in the circuit
U_ 220 V
/ = 0.38 A.
Z ’ — 576 0
The voltage across all the components can be found as
follows:
UR = I R , Uc = I X C, UL = I X L,
UR = 0.38 A x 288 Q = 109.4 V,
Uc = 0.38 A x 546 Q = 207.5 V,
UL = 0.38 A x 47.1 Q = 17.9 V.
Let us now calculate the powers:
total power S = IU, S = 0.38 A x 220 V = 83.6 VA,
active power P = 0.5S, P = 4 1 .8 W,
§ 18. Altcrnatiug Curran 205
___________________ |/ o
reactive power Q = S sin y = S ^ 1 —cos2 <p= — S,

Q= 83.6 VA = 72.4 var.

Answer. The capacitance of the capacitor is 5.83 pF, the


current in the circuit is 0.38 A, the voltages across the bulb,
the inductance coil, and the capacitor are 109.4, 17.9, and
207.5 V respectively, the active, reactive, and total powers
are 41.8 W, 72.4 var, and 83.6 VA respectively, and the
power factor is 0.5.

Questions and Problems


18.1. The standard frequency of the alternating current
used in the USSR is 50 Hz. Determine the period of the os­
cillations. How many times per second does the direction of
the charge carriers change?
18.2. An ammeter connected to an a.c. circuit indicates
3 A. What is the amplitude of the current?
18.3. The breakdown voltage of a capacitor is 250 V. Can
it be connected to a lighting circuit at a voltage of 220 V?
18.4. The emf in an a.c. circuit varies as e = 250 x
sin lOOnf. Determine the effective value of the emf and the
circular frequency.
18.5. The time dependence of an alternating current is
described by the equation i = 90 sin (314* + ji/4). Deter­
mine the effective current and its phase, initial phase, and
frequency.
18.6. The instantaneous voltage is u = 179 sin cit. De­
termine the instantaneous voltage at times: 0, 0.0025, 0.005,
5/6, 4/3, 0.015, 0.0175, and 0.02 s. The frequency of the al­
ternating current is 50 Hz.
18.7. The instantaneous current at 1/3 of the period
is 2.6 A. What will the current be for a phase of (a) 1.5n
(b) 13ji/6?
18.8. A wire frame rotates uniformly in a uniform mag­
netic field about an axis perpendicular to the magnetic field
lines. In what position of the frame is the induced emf (a)
zero, (b) equal to the amplitude value?
18.9. A rectangular frame of length 10 cm and width 5 cm
rotates uniformly in a uniform magnetic Held of induction
206 Ch. III. Oscillations and Waves

0.02 T. The speed of rotation is 2865 rph. Determine the am­


plitude of the emf induced in the frame.
18.10. A rectangular metal frame of length 12 cm and
width 4 cm rotates uniformly in a uniform magnetic held.
What will the change in the emf induced in the frame be if,
without changing the speed of its
_______ i______ ^ rotation, it can be deformed so that
its length becomes 4 cm and width
12 cm?
18.11. A rectang
tates in a uniform magnetic held at a
constant speed about an axis per­
pendicular to the magnetic held
lines. The frame length is I = 14 cm
Fig. 97 and its width is d = 6 cm. How
will the emf induced in the frame
change if the frame is deformed such that it takes on the fol­
lowing dimensions: (a) I = 16 cm, d = 4 cm, (b) I = 10 cm,
d = 10 cm, and (c) I = 8 cm, d = 12 cm.
18.12. How many turns are wound on a frame having an
area of 367 cm2 and uniformly rotating in a uniform magne­
tic held with induction 0.115 T if a voltmeter connected to
its terminals indicates 90 V? The rotation period of the
frame is 2.5 x 10"2 s.
18.13. A frame containing 15 turns of area 200 cm2 rotates
uniformly in a uniform magnetic held at 10 rps. When
the frame is at an angle of 30° to the direction of the magnetic
held, the emf induced in the frame is 0.75 V. What is the mag­
netic held’s induction?
18.14. A wooden frame having an area of 150 cm2 and
containing ten turns of wire rotates uniformly in a uniform
magnetic held with induction 0.05 T. The position of the
frame at t — 0 is shown in Fig. 97. The frequency of the
induced alternating current is 10 Hz. Determine the ampli­
tude, circular frequency, the period, and the initial phase
of the induced emf. Write the equation for the instantaneous
emf.
18.15. A conductor with inductance 2.55 X 10-3 H is
switched from a circuit operating at a frequency of 50 Hz
to a circuit operating at a frequency of 300 Hz. Does its
impedance change? If it does, in what proportion? What is
its impedance in a d.c. circuit equal to?
§ 18. Alternating Current 207

18.16. A coil with inductance 8.42 X 10-3 H has a resis­


tance of 3 Q. What is its impedance in an a.c. circuit with
a current frequency of 50 Hz and in a d.c. circuit?
18.17. When a coil with inductance 0.3 H is connected to
n il.c. circuit at a voltage of 24 V, the current in it is 0.2 A.
Determine the impedance of the coil when it is connected to
.in a.c. circuit of frequency 50 Hz.
18.18. What will the impedance of a 100-pF capacitor be
in a d.c. circuit?
18.19. Determine the impedance of a 100-pF capacitor
ronnected to a.c. circuits with current frequencies 50 and
:!<>00 Hz.
18.20. A coil of inductance 0.15 H, a 500-Q resistor, and a
- fiF capacitor are connected in series to an a.c. circuit oper­
ating at a frequency of 400 Hz. Determine the impedance of
Ilie circuit.
18.21. Determine the impedance of the circuit connected
in the lighting mains if the circuit consists of a 500-p.F
nnil a 1000-p.F capacitors connected in series, a coil with
n resistance of 10 Q, and inductance 0.08 H.
18.22. A 530-p.F capacitor and a potentiometer with a
resistance of 5 Q connected in series are connected to the
mains which has a voltage of 220 V and a frequency of
50 Hz. Determine the current passing through the potentio­
meter.
18.23. A capacitor is connected to the mains which has a
voltage of 220 V and frequency of 50 Hz. The amplitude of
Ilie current in the circuit is 4.89 A. What is the capacitance
of the capacitor?
18.24. A coil of inductance 0.2 H is connected to an a.c.
i ircuit with a frequency of 100 Hz. The current in the coil
is 1.01 A. What is the voltage across the coil? What is the
mnplitude of the voltage?
18.25. A voltage of 80 V is supplied to a variable capaci­
tor and a coil with a core connected in series to it. The
i nrrent in the circuit was found to be 4 A. Determine the
voltage across the circuit elements if halving the capacitance
of the capacitor and the inductance of the coil increases the
i urrent to 20 A. What is the frequency if the initial in­
ductance is 1.91 X 10"2 H?
18.26. An a.c. circuit contains a resistor. Plot the ampli-
liule of the voltage across the resistor on a vector diagram
208 Ch. III. Oscillations and Waves

based on the current axis. Plot the graphs of the current and
voltage.
18.27. Plot the voltage vector diagram if an a.c. circuit
contains (a) a capacitor, (b) a solenoid. Plot the graphs for
the currents and the voltages.
18.28. Plot the vector diagram for the voltages of the a.c.
subcircuits operating at frequency 50 Hz and consisting of
the following components connected in series
(a) C = 1.33 x 103 pF, R = 2.4 Q,
(b) L = 3.18 x 10- 2 H, R = 17.3 Q.
18.29. Plot the graphs of voltage and current in the com­
ponents of the circuit in Problem 18.28.
18.30. Plot the vector diagrams of the voltage for the
following components when connected in series (a) L =
0.03 H and C = 2 x 10"‘ F, (b) L = 4.95 x 10' 2 H, C =
398 pF and R = 7.55 Q. The alternating voltage frequency
is 50 Hz.
18.31. Plot the voltage graphs for the components of the
a.c. circuit in Problem 18.30.
18.32. Using vector diagrams, prove the formula for the
reactance of a subcircuit consisting of a capacitor and an
inductance coil.
18.33. Using vector diagrams, prove the formulas for the
impedances of series-connected R, C and L in an alter­
nating current.
18.34. An a.c. circuit at voltage 120 V and frequency 50 Hz
is connected to series-connected (a) R =■ 60 Q, L =
0.255 H, (b) R = 3.8 Q and C = 2.27 X 10‘3 F, and (c)
L = 0.0764 H and C = 398 pF. De­
/ termine the impedance, current, and
voltages on the components of the
circuit. Plot the vector diagrams.
18.35. An a.c. vo
and frequency 50 Hz is applied to
Fig. 98 series-connected (a) R = 5 Q, L =
0.135 H, and C = 75 pF, (b)
R = 30 Q, L = 0.2 H, and C =
97 pF. Determine impedance, current, and voltage across
the components of the circuit. Plot the vector diagrams.
18.36. An a.c. generator of voltage 36 V (Fig. 98) is con­
nected in parallel to (a) R = 3 Q, X L = 4 Q, (b) R = 1 Q
X c = 2Q , and (c) X c = 2 Q, X L = 4 Q. The frequency of
§ 18. Alternating Current 209

llie alternating current is 50 Hz. Determine the current in


the branches, the total current, and cos <p. Plot the vector
diagrams.
18.37. Determine the current in the branches and the total
current in the circuit (see Problem 18.36) if (a) R = 6 Q,
A 'jr = 3 Q , and X c = 3 Q, (b) R = 9 £2, X L = 10 Q,
and X c = 4.8 Q, (c) R = 4 Q, X L = 2 Q, and X c = 6 Q.
Determine cos <p. Plot the vector diagrams (all the compo­
nents are connected in parallel).
18.38. A potentiometer and a solenoid with a core are
connected to an a.c. circuit of frequency 50 Hz. The phase
difference between the total voltage and the current is
30°. The resistance is increased by 100 Q. What must the
change in the inductance of the solenoid be for the phase
difference to remain unchanged? What is the voltage in the
circuit if the voltage across the solenoid is 60 V?
18.39. A resistor and an inductance coil with a neglig­
ibly low resistance are connected in series to an a.c. gen­
erator operating at frequency 40 Hz. The current in the
curcuit is 3 A and the phase difference between the voltage
and the current is 30°. Determine the resistance of the re­
sistor and the inductance of the coil (U = 6 0 V).
18.40. A potentiometer with resistance 8 Q is connected
in series to a capacitor with capacitance 398 pF and a coil
id inductance 0.0383 H. The frequency of the alternating cur­
rent is 50 Hz. Determine the voltage across the components
mid in the entire circuit and the phase difference between
Ilie current and voltage. The current in the circuit is 4 A.
I’lot the vector diagram.
18.41. A 0.00636-H coil is connected to a circuit at a
frequency 50 Hz. What must the capacitance of a capacitor
connected to the circuit be to obtain resonance?
18.42. Why are multipole generators used in hydroelectric
plants?
18.43. The rotor of a hydroelectric generator rotates at
150 rpm. How many magnetic pole pairs must the rotor
have in order to generate a current of frequency 50 Hz?
18.44. Determine the resonance frequency of the alter­
nating current in the circuit shown in Fig. 99 if the capaci­
tance of the capacitor is 2.5 X 10-5 F and the inductance
of the coil is 0.1 H.
18.45. The primary winding of a transformer is connected
I 'i- 0 5 3 0
210 Ch. III. Oscillations and Waves

to a d.c. circuit. Will the bulb connected to the secondary


winding glow?
18.46. The primary of a transformer is connected to an
a.c. circuit while the secondary is disconnected. Does the
transformer consume energy from the circuit?
18.47. Two neighbouring turns of a transformer’s sec­
ondary are short circuited. What will happen?
18.48. When a transformer is connected to an a.c. supply
with a voltage of 220 V, an emf of 110 V is induced in the
secondary winding. What is the trans­
formation ratio?
18.49. The transformation ratio of
a transformer is 10. What is the ratio
—O~ O- of the numbers of turns in each wind­
ing?
Fig. 99 18.50. An emf of 600 V is induced
in the secondary of a transformer con­
taining 1900 turns. How many turns of the primary wind­
ing are connected to a supply with voltage 220 V?
18.51. A transformer is connected to a supply with volt­
age 120 V. The primary winding contains 300 turns. What
must the number of turns in the secondary winding be for
a voltage of 6.4 V to be generated across it?
18.52. A voltage of 60 V at a frequency of 50 Hz is applied
to a circuit containing a coil with inductance 0.1 H and a
20-pF capacitor. What is the average power developed in
the circuit during one period? What are the reactive and
total powers?
18.53. An alternating current passes through a series-
connected resistor, capacitor, and coil whose resistances and
reactances are 2, 3, and 5.67 Q respectively. Determine the
active, reactive, and total power, the power factor (cos a),
and the voltage applied to the circuit if the current is 6 A.
18.54. The alternating voltage in a series-connected cir­
cuit with a current of 2 A is 90 V. The circuit consists of a
capacitor, a coil, and a resistor. The power factor is 0.6.
What are the active and the reactive powers? What are the
reactances and the resistance of the circuit’s components if
the reactance of the capacitor is twice that of the coil?
18.55. A source of an a.c. voltage u = 180 sin cat is
connected to an unbranched circuit containing a resistance
and two reactances, X L = 8 Q and X c = 12 Q. Deter-
$ 19. Electromagnetic Oscillations and Waves 211

mine the power factor, the active and reactive power if


the current in the circuit is 3 A.
18.56. An a.c. voltage of 100 V is applied to a 50-Q resis­
tor and capacitor with reactance 55 Q connected in paral­
lel. Determine the total, active and reactive powers, and
the power factor.
18.57. A source of a.c. voltage of 60 V and frequency
r>0 Hz is connected in parallel to a resistance of 15 Q and an
inductance coil with a negligible re­
sistance. The total power in the cir­ L C
cuit is 648 VA. What is the induc­
tance of the coil? What are the
active and reactive powers?
18.58. An alternating current
passes through a coil and a resistor
connected in parallel. The voltage
across the coil is 120 V, the current
in the resistor is 2 A, and total cur­ Fig. 100
rent is 3.5 A. Find the resistance of
the resistor and the coil. Determine the active, reactive and
total powers, and the power factor.
18.59. A coil, a capacitor, and a resistor are connected as
shown in Fig. 100. The voltage of the source is 40 V. Deter­
mine the power factor and the total, active, and reactive
powers if (a) X L = 15 Q, X c = 15 Q, R = 10 Q, (b) X l =
MQ, X c = 18 Q, and R = 13.3 Q.
18.60. A capacitor, a coil, and a resistor are connected in
parallel to an a.c. circuit of a voltage 220 V. Their reac­
tances and resistance are 8 ,12, and 6 Q respectively. Deter­
mine the power factor, and the active and reactive powers.
18.61. The power in two parallel branches of an a.c.
rircuit is 200 var and 346 W. The current in the unbranched
circuit is 8 A. What are the reactance and the resistance of
Ihc branches, the power factor, and the total power? What
is the impedance?
§ 19. ELECTROMAGNETIC OSCILLATIONS AND WAVES
Basic Concepts and Formulas
Any varying magnetic field generates an eddy electric field.
In turn, an electric field varying over time generates an eddy
magnetic field. This continuous transformation of the elec-
it*
212 Ch. III. Oscillalions and Waves

trie field’s energy into the energy of the magnetic field and
back is observed in an oscillatory circuit, viz. a circuit con­
taining a coil and a capacitor (Fig. 101). The resistance of
the oscillatory circuit must be low, otherwise the energy of
the electromagnetic field will be converted into heat, and
the oscillations in the circuit will soon cease.
Under these conditions, the period of the
natural electromagnetic oscillations is
T = 2n VL C,
and the frequency is
1 _ 1
V~ T ~ 2n Y"LC
An open oscillatory circuit emits waves of the wavelength
rK = cT, or >. = c/v,
where c is the velocity of electromagnetic waves equal to
the velocity of light in vacuum.
The velocity of propagation of the electromagnetic waves
depends on the properties of the medium, viz.

— = V pe, whence v = — ,
v K pe
vhere v is the velocity of propagation of the electromagnetic
waves in the medium, and p and e are the permeability and
permittivity of the medium. Since the permeabilities of all
dia- and paramagnetic media differ only slightly from unity,
we can assume that

While using this formula, it should be borne in mind that


the e known from electrostatics cannot always be used
since the oscillation frequency affects the permittivity.

Worked Problems
Problem 92. An oscillatory circuit consists of a coil with
inductance 20 pH and a capacitor whose capacitance can
be varied from 2 X 10"8 to 10-8 F. What is the wavelength
range of this circuit? Determine its frequency range.
§ 19. Electromagnetic Oscillations and Waves 213

Given: L = 2 X 10~8 H is the inductance of the coil,


C, = 2 X 10' 8 F and C2 = 10~8 F are the maximum and min-
i mum capacitances of the capacitor. From tables, we take
the velocity of light in vacuum c = 3 X 108 m/s.
Find: the maximum and minimum X2 wavelengths and
the boundary frequencies vt and v2 of the oscillatory circuit.
Solution. The wavelengths lie in the interval from Jt, =
(•/’, for capacitance C, to h2 = cT2 for capacitance C2.
The period of the circuit’s oscillations is determined by
the formulas
Ti = 2 n V L C l, T2=--2nV~LCl,
Tl = 2 x 3.14 V 2 x 10-3 H x 2 x 10~8 F = 3.97 x 10~6 s,
= 2 x 3.14 V 2 x 10-5 H x 10-« F = 2.81 x lO"8 s.
Then the wavelengths are
= 3 x 108 m/s x 3.97 x lO*8 s = 1191 m,
\ 2 = 3 x 108 m/s x 2.81 x 10-8 s = 843 m.
(iiven the periods, we can determine the frequencies:
= l/7 \, v2 = l/7 ’2,
vt = 252 kHz, v2 = 356 kHz.
Answer. The oscillatory circuit can operate in the wave­
length range 843-1191 m at frequencies 252-356 kHz.
Problem 93. The maximum voltage across the capacitor
of an oscillatory circuit is 120 V. Determine the maximum
current if the inductance of the coil is 0.005 H and the ca­
pacitance of the capacitor is 10-5 F. The resistance of the
circuit is assumed to be negligible.
Given: t/max = 120 V is the maximum voltage across the
capacitor, L = 0.005 H is the inductance of the coil, and
G = 10"8 F is the capacitance of the capacitor.
Find: the maximum current / max 1° the circuit.
Solution. The problem can be solved by two methods.
1st method. At resonance, the reactance of the circuit is
minimal, while the current is at its maximum. The resonance
frequency is__ oa0 = 1 / ] / ZC, / maI c = Umj X c , * c _ =
l / ( « . 9 = V L/C, / max c = Umaj v L / C = Um„ V C I L .
Argueing in the same way, we conclude that the current in
214 Ch. III. Oscillations and Waves

the inductance coil can be calculated from the same formu­


la.
Substituting in the numerical values, we obtain

2nd method. According to the energy conservation law, the


maximum energy of the magnetic field must be equal to the
maximum energy of the electric field: L/max/2 = CC/max/2.
Hence
^mai
max — ^maxV^ CIL.
Comparing the two solutions, we note that the second
method is much simpler.
Answer. In the absence of resistance, the maximum cur­
rent in the circuit is 5.37 A.

Questions and Problems


19.1. How will the frequency of an oscillatory circuit
change if the capacitance is reduced using a variable ca­
pacitor?
19.2. Will the insertion of a ferromagnetic core into the
coil of the oscillatory circuit affect the period of the elec­
tromagnetic oscillations in the circuit?
19.3. A transmitter operates at a frequency of 600 kHz.
What wavelength corresponds to this frequency?
19.4. An oscillatory circuit consists of a 200-pF capacitor
and a coil of inductance 20 mH. What are the frequency
and the period of natural oscillations of the circuit?
19.5. When a motor car enters a tunnel, the sound of its
radio is attenuated if it does not vanish altogether. Why?
19.6. The ionosphere reflects radiowaves. How then can
communication with spaceships by radio be realized?
19.7. An oscillatory circuit consists of a coil of inductance
2 mH and a capacitor whose capacitance can be varied fiom
10-9 F to 40 pF. What is the wavelength range for this cir­
cuit?
19.8. An oscillatory circuit consists of a coil of inductance
0.01 mH and a capacitor of capacitance 10~9 F. To what
wavelength is this circuit tuned? What frequency corre­
sponds to this wavelength?
§ Id. Electromagnetic Oscillations and Waves 215

19.9. An oscillatory circuit emits electromagnetic waves


of length 500 m. Determine the capacitance of the capaci­
tor in the circuit if its inductance is 1.5 mH.
19.10. What is the distance between an aeroplane and a
radar if a signal reflected from the plane was received
3 x 10“4 s after it had been
emitted?
19.11. How does the power
of electromagnetic radiation
depend on the oscillation fre­
quency of the emitting circuit?
19.12. Figure 102 shows the
circuit diagram of a crystal
receiver. What component
smooths the current pulses?
19.13. Programmes are per­
ceived from a crystal receiver
using earphones since the received signals are very weak.
How is a signal amplified in modern radios?
19.14. A radar emits electromagnetic waves of length
10 cm at a frequency of 2.25 GHz in a certain medium. What
is the propagation velocity of the waves in this medium and
what would their wavelength be in vacuum?
19.15. An oscillatory circuit containing a 1.5-p.F capaci­
tor should be tuned to a frequency of 1.5 kHz. What in­
ductance is required?
19.16. The time dependence of current in an oscillatory
circuit is described by the equation i = 0.06 sin 106nf.
Determine the frequency of the electromagnetic oscillations
and the inductance of the coil if the maximum energy of
the magnetic field is 1.8 X 10~4 J.
19.17. An oscillatory circuit tuned to a frequency of
20 MHz contains a coil with inductance 10"8 H and a paral­
lel-plate mica capacitor with a plate area of 20 cm2. Deter­
mine the thickness of the mica if its permittivity
is 6 .
19.18. The maximum current in an oscillatory circuit is
6.28 X 10"3 A and the maximum charge on the capacitor is
10"8 C. Determine the period of oscillations in the circuit.
To what wavelength is it tuned?
19.19. The maximum potential difference across the ca­
pacitor in an oscillatory circuit is 120 V. What will the
2 16 Ch. III. Oscillations and Waves

maximum current be if the capacitance of the capacitor is


9 pF and the coil has an inductance of 6 mH?
19.20. The time dependence of the current in an oscilla­
tory circuit is given by the equation i = 0.02 sin 500ti<.
The inductance of the circuit is 0.1 H. Determine the period
of electromagnetic oscillations, the capacitance of the cir­
cuit, and the maximum energies of the electric and magnetic
fields.
19.21. As the capacitance of a capacitor decreases by
100 pF, the resonance frequency of the oscillatory circuit
increases from 0.2 to 0.25 MHz. What is the inductance of
the circuit?
Chapter IV

Optics. Fundamentals
of the Special Theory of R elativity

§ 20. GEOMETRICAL OPTICS

Itasic Concepts and Formulas


To analyze the phenomena associated with the propagation
of visible radiation (light) in a homogeneous medium or.
through the boundary between two transparent media, we
introduce the concept of a light beam.
I n a homogeneous medium, light beams
propagate in a straight line, which
explains the formation of the umbra
nnd penumbra.
If a reflecting surface is placed in the
path of a light beam, light is reflected in Fig. 103
accordance with the following two laws:
1. The incident ray, the reflected ray, and the perpendicu­
lar to the reflecting surface at the point of incidence of the
ray lie in the same plane.
2. The angle of incidence e is equal to the angle of reflec­
tion e' (Fig. 103).
If the angles of incidence of two rays on a plane surface
arc the same, i.e. if the incident rays are parallel, they will
remain parallel after reflection. Diverging rays emitted by
a source of light remain divergent upon reflection (Fig.
104). In this case, the image formed at the intersection of
l lie continuations of the reflected rays will be virtual and
symmetric to the source about the mirror.
If a plane mirror is placed in the path of a converging
beam, the image formed by it will be real, the point of inter­
section of the continuations of the incident rays will be a vir­
tual image of the light source (Fig. 105), and S'O = SO.
Light rays incident on a spherical surface are reflected
from it according to the same laws. If the mirror (reflecting)
surface is the inner surface of the part of a sphere, the mir-
218 Ch. IV. Optics. Special Theory of Relativity

ror is called concave, and a mirror formed on the outer sur­


face of a sphere is called convex.
The straight line passing through the centre of curvature
C and point O (pole of a mirror) is known as the optical axis
of the mirror (Fig. 106).

Fig. 104 Fig. 105

A beam of rays incident on a concave mirror parallel to


the optical axis is reflected such that it intersects the axis at
a point F known as the focus. Concave mirrors have real fo-

Fig. 106

ci, while convex mirrors have virtual foci. The distance


FO is called the focal length and is denoted /:
FO = / = Rl 2,
where R is the radius of curvature of the mirror.
The formula for a spherical mirror has the form
§ 20. Geometrical Optics 219

Ilure a and a' are respectively the distances from the object
and from its image to the mirror. The minus sign indicates
that the mirror is convex, and the plus sign corresponds to
a concave mirror.
The linear magnification is given by

Here the minus sign in the denominator corresponds to a


concave mirror, and the plus sign to a convex mirror.
The image of an object formed by a spherical mirror is
constructed by using (1) an incident ray parallel to the op­
tical axis, (2) a ray passing through
the focus, and (3) a ray along the
radius of the curvature.
Light is refracted when a ray is
incident on the interface between
two different transparent media,
and obeys the following laws:
1. The incident and the refracted
rays lie in the same plane as the
perpendicular at the point of inci­ Fig. 107
dence to the interface between the
two media.
2. For two given media, the ratio of the sines of the angles
formed by the incident and refracted rays with the normal is
a constant and called the refractive index of the second me­
dium relative to the first one (Fig. 107):
sin e

In the case under consideration, the second medium is op­


tically denser than the first one. As a ray passes from the
firstr to the second medium, the angle of refraction is always
smaller than the angle of incidence. If a ray passes from an
optically denser medium to an optically rarer medium, it is
possible for the rays not to be refracted but reflected totally,
remaining in the same medium. This is called total inter­
nal reflection. It occurs when a ray is incident on the inter­
face at an angle larger than a critical angle, defined as the
220 Ch. IV. Optics. Special Theory of Relativity

angle for which the angle of refraction is equal to 90"":


sin Ecr 1 i
.
sin 90
= —
n
, sin err
c ‘
= —
n
.

The relative refractive index can be expressed in terms of


the velocities of light in two media:

?i2 , l = V l/V2-

The absolute refractive index is


n — civ,

where c is the velocity of light in vacuum.


Transparent media bounded by convex or concave surfaces
are called lenses. Convex lenses converge rays, while con­

cave lenses diverge them, if the refractive index of the lens


material is larger than the refractive index of the medium
surrounding it.
The principal optical axis of a lens is the line passing
through the centres of curvature of its two refracting sur­
faces (Fig. 108). The point 0 in a lens is called its optical cen­
tre. Rays passing through the optioal centre are not refracted.
Points F and F' are known as the foci of a lens. Any straight
line passing through the optical centre of a lens is called an
auxiliary optical axis.
In order to construct the image formed by a lens, three
rays are normally used: (1) a ray parallel to the principal
optical axis, (2) a ray passing through one of the foci, and
(3) a ray passing through the optical centre.
§ 20. Geometrical Optics 221

As is the case of spherical mirrors, the following formula


is applicable to lenses:

The sign rule here is the same as for mirrors.


The unit of optical power O = 1/f is the dioptre (D).
An image formed by a concave lens is always virtual,
while a convex lens forms a virtual image when the object
lies between the lens and a focus.
Lenses are widely used in optical instruments to magnify
images of objects (projectors, magnifiers, and microscopes)
or to increase the angle of view of examining objects a long
distance from the observer (binocular and telescopes).

Worked Problems

Problem 94. What will happen if a plane mirror is placed


in the path of a converging beam?
Solution. Let us examine the path of the rays shown in
Fig. 109. Rays 1, 2, and 3 are incident on the plane mirror

in a converging beam. After reflection, they will intersect


at a point S ’ which is a real image of point S. At this point
the rays would converge in the absence of the mirror. Using
222 Ch. IV. Optics. Special Theory of Relativity

the laws of reflection of light, we can prove the equality of


isosceles triangles M S'N and M SN, whence it follows that
S'O = SO.
Answer. A converging beam of rays will form a real and
symmetric image of a virtual bright point.
Problem 95. A concave spherical mirror forms a threefold
magnified image of an object. The distance from the object

Fig. 110

to the image is 2.6 m. What is the radius of curvature of the


mirror?
Given: p = 3 is the linear magnification of the image,
I = 2.6 m is the distance from the object to the image.
Find: the radius of curvature R of the mirror.
Solution. We know that R — 2f. Consequently, the solu­
tion reduces to determining the focal length of the spherical
mirror.
The image formed by a concave mirror is magnified in two
cases. Let us consider both.
1. The object lies between the focus and the centre of cur­
vature (Fig. 110). In order to construct the image A'B'
we shall use two rays, one of which is incident on the mir­
ror parallel to the optical axis and the other is directed along
the radius of curvature. After reflection, the rays will inter­
sect at point A '. Dropping the perpendicular from A' onto
the optical axis, we obtain point B '. We have a real image
of the object with linear magnification a' = {la.
The problem states that I = a' — a, and hence I =
Pa — a. Therefore, a = l/(p — 1) and a' = pi/(P — 1). We
substitute these values for a and a' into the formula for a
§ 20. Geometrical Optics 223

spherical mirror and determine the focal length:


1 1 1 1, 1 11 P - 1 | P - 1 1 _P*- l
ff4 ~
~ a„ +
^I a'
o' ’'
' f/4 ~ I •* pi ’ f ~ pi

Consequently, R = 2 x 0.975 m = 1.95 m.


2. The object lies between the mirror and the focus (Fig.
111). In order to construct the image, we shall use the same
rays as above. It can be seen from the figure that the image

Fig. i l l

of point A' is not on the intersection of the rays themselves


but on their continuations, i.e. the image is virtual. In the
formula for a spherical mirror, the quantity 1la' should be
taken with a minus sign. The subsequent reasoning is the
same as before, the only difference being that I = a -f- a .
The result will be the same.
Answer. The radius of curvature of the spherical mirror
is 1.95 m.
Problem 96. The radius of curvature of a convex spherical
mirror is 1.2 m. How far away from the mirror is an object
of height 12 cm if the distance between its virtual image and
the mirror is 0.35 m? What is the height of the image?
Given: R = 1.2 m is the radius of curvature of the mir­
ror, a' = 0.35 m is the distance between the virtual image
and the mirror, and h = 0.12 m is the height of the object.
Find: the distance a between the object and the mirror
and the height h’ of the image.
Solution. Considering that the focus and the image for a
convex mirror are virtual, we put the minus signs in front
224 Ch. IV. Optics. Special Theory of Relativity

of 1// and 1la in the formula:


a'f
" f-a ' •
0.35 m X 0.6 m
/ = -£- = 0.6 m , 0.6 m —0.35 m
■=0.84 m.

Let us determine the height of the image:

h' _ o' /i' _ ^ a '

h a ’ a 1

ni t —------T
0.12 m X 0.35 m _a »t a Ar
— ----------= 0.05 m, h —0.05 m.
0.84 m ’

Answer. The object is 0.84 m from the mirror, and the


image is 5 cm high.
Problem 97. The Sun forms an angle of 60° with the hori­
zon. Determine the length of the umbra at the bottom of an
opaque vessel illuminated by
sun light. The height of the
vessel is 25 cm. What will the
change in the length of the
umbra be when water is poured
into the vessel to a height of
20 cm (Fig. 112)?
Given: H = 25 cm is the
height of the vessel, <p = 60°
is the altitude of the Sun
above the horizon, h = 20 cm
is the height of the water column. From tables, we take the
refractive index for water, n = 1.33.
Find!: the length ^ of the umbra at the bottom of the empty
vessel and the change AI of the length of the umbra in the
vessel with water.
Solution. We use e to denote the angle formed by the di­
rection of incident rays with the vertical wall of the vessel.
It can be seen from the figure that the angle of incidence
e and the altitude of the Sun cp are related: e + cp = it/2,
e = 90°—60° - 30°.
The rays propagate in the empty vessel along a straight
line (along AM ), and the length I of the umbra can bo mien
§ 20. Geometrical Optics 225

lated from the right-angled triangle A M D :


- AD = H tan e, tan 30° = 0.577,
f! = 25 cm x 0.577 ~ 14.4 cm.
If the vessel contains water, the rays will be refracted at
the air-water interface since a light ray bends away from
the rarer medium into a denser medium. In this case, the
refracted ray is closer to the vertical, and the angle of re­
fraction e' is smaller than the angle of incidence e. The
length of the umbra in the vessel filled with water is
l2 = BD — BC + CD. But CD = ON.
1n the right-angled triangle OM N, we have ON = (H — h) x
tan e, ON = 5 cm x 0.577 ~ 2.9 cm. From the triangle
HOC, we find BC = h tan e'. In order to determine the an­
gle of refraction, we use the second law of refraction:
sin e/sin e' = n, sin e' = sin e/re, sin e' = 0.5/1.33 =
0.3759, and e' ~ 22°. From tables, we take tan 22° =
0.404. Consequently, BC = 20 cm X 0.404 = 8.1 cm.
The length of the umbra in the vessel with water is
l2 = BD = 8.1 cm -f 2.9 cm = 11 cm.
When the vessel is filled with water, the length of the
mnbra decreases by Al:
AI = 14.4 cm — 11 cm = 3.4 cm.
Answer. The length of the umbra in the empty vessel is
approximately 14.4 cm. The umbra in the vessel with water
is 3.4 cm shorter.
Problem 98. A light ray is incident at 45° on a glass slab.
The slab is 3 cm thick, and the refractive index of the glass
is 1.5. What will the displacement of the ray be as a result
of its passage through the slab? At what angle will the ray
emerge from the slab?
Given: e = 45° is the angle of incidence of the ray, h =
0.03 m is the slab thickness, and n = 1.5 is the refractive
index of the glass.
Find: the displacement 6 of the ray and the angle e't
at which the ray emerges from the slab.
Solution. Figure 113 shows that the displacement of the
ray is the shortest distance between the direction of the
1 5-0530
226 Ch. IV. Optics. Special Theory of Relativity

incident ray and the ray emerging from the slab: 6 = CD.
From the triangle ACD, we have 5 = AC sin (e — e'),
AC = h!cos e'. Consequently,
g _ h sin (e —e')
cos e'
In order to determine the displacement, we must find
the angle e'. Using the second law of refraction, we obtain
n = sin e/sin e', sin e' = sin e/n, sin e = ]A2/(2 X 1.5) =
0.47, and e' ~ 28°. This gives
«. 0.03 m X 0.29 n nn
6 = ----- g-gg----- ~ 0.0099 m ~ 9.9 mm.

Answer. After passing through the slab, the ray remains


parallel to the incident ray but is displaced by 9.9 mm-
t[ = e.
Problem 99. An object of height 6 cm is set at right angles
to the optical axis of a double-convex lens of optical power
5 D and 25 cm away from the
lens. Determine the focal
length of the lens, the position
of the image, the linear mag­
nification of the lens, and the
height of the image formed by
it.
Given: h = 0.06 m is the
height of the object, <J> = 5 D
is the optical power of the lens,
and a = 0.25 m is the dis­
tance between the object and
the lens.
Find: the focal length / of
the lens, the distance a' from the image to the lens, the
linear magnification p, and the height h' of the image.
Solution. Given the optical power, we can determine the
focal length of the lens:

^ = "O' T " D = 0-2 m'


Let us construct the image of the object (Fig. 114). By
choosing an appropriate scale, we can determine the re­
quired quantities from the diagram accurately enough.
§ 20. Geometrical Optics 227

Using the formula for a thin lens, we calculate a':


1^ fa 0.2 m X 0.25 m
= 1 m.
/ n I n* * ~t » 0.05 m

The linear magnification p can be determined from the


formula
Q 1m #
P“ n
T ’ 0.25 m —
Consequently, the height of the image will be
h' - pfe, h' = 4 x 0.06 m = 0.24 m.
Answer. The focal length of the lens is 20 cm, the image
is 1 in from the lens, the linear magnification of the lens is
L\. and the image is 24 cm high.

Fig. 114

Problem 100. An object 30 cm high stands vertically


80 cm from a lens with an optical power of —5D . Determine
the position of the image and its height.
Given: h = 0.3 m is the height of the object, a = 0.8 m
is the distance from the object to the lens, and <I> = —5 D
is the optical power of the lens.
Find: the distance a' from the lens to the image and the
height h' of the image.
Solution. The lens is concave since its optical power is
negative. Let us determine its focal length:
1
/ —0.2 m.
4) —5 D
15*
228 Ch. IV. Optics. Special Theory of Relativity

We construct the image of the object formed by the lens


(Fig. 115). The image is virtual. From the formula for a

thin lens, we determine the distance from the lens to the


image:
1_ 1 I 1 „» _ “/ 0.8 m ( —0.2 m)
a ' a' ’ a— f ’
- 0 .1 0 m .
/ 0.8 m + 0 .2 m
In order to determine the image height, we shall use the
formula for magnification:
10.16ml
h a
h' 0.3 m = 10.061in.
0.8 m
Answer. The distance from the lens to the image is —16 cm
(the minus sign indicates that the image is virtual) and it
is 6 cm high.
Problem 101. Given the principal optical axis M N, an
object AB, and the image A' of point A (Fig. 116). Deter-

A/

Fig. 116

mine the positions of the lens and its foci. Is the lens con­
verging or diverging? Construct the image of the object.
Solution. We draw a ray from A to A ’. It intersects the
principal optical axis at the optical centre of the lens. We
§ 20. Geometrical Optica 229

direct the second ray from point A parallel to the principal


optical axis. It is refracted by the lens, to hit A ', intersec­
ting the optical axis at a focus F' of the lens. We draw
a vertical plane through the focus, which is known as the
focal plane. In order to construct the image of the object,
we need only construct the image of B. We draw a ray from

li towards the lens at an arbitrary angle to the optical axis


Parallel to this ray, we draw an auxiliary optical axis, which
must intersect the ray at a point in the focal plane. Then
point B ’ is obtained where the ray intersects the principal op­
tical axis (Fig. 117).Connecting points A' and B ', we obtain
the image of the object.
Problem 102. A microscope has a magnification of 157.5.
The focal length of the objective is 0.3 cm. An object is
0.31 cm from the objective. Determine the magnification of
the eyepiece and its focal length. What is the length of the
microscope drawtube if the image is 26.25 cm from the eye­
piece?
Given: p = 157.5 is the magnification of the microscope,
/, = 0.3 cm is the focal length of the objective, at = 0.31 cm
is the distance from the object to the objective, and a\ =
26.25 cm is the distance from the eyepiece to the final image.
Find: the magnification p2 of the eyepiece, the focal
length / 2 of the eyepiece, and the length L of the drawtube.
Solution. The magnification of a microscope is deter­
mined by the magnifications P! and p2 of the objective
and eyepiece: p = PiP*.
In order to determine p2, we must know the magnifica­
tion Pj of the objective. Let us determine the position of
230 Ch. IV. Optics. Special Theory of Relativity

the image formed by the objective from the formula

fi 0.3 cm x 0-31 cm
a'i 0.31 cm — 0.3 cm
= 9.3 cm.
® i — /i

Remark. To avoid cumbersome numbers, we shall express


all quantities in centimetres:
9.3 cm
P i = ai — 30.
0.31 cm

We can now find the magnification of the eyepiece:


„ _ P 157.5
5.25.
“ 30

Eyepiece

Fig. 118

Let us determine the distance a2 from the eyepiece to the


image A[B[, which is the object relative to the eyepiece
(Fig. 118):
26.25 cm
a. 5 cm-
5.25
§ 20. Geometrical Optics 231

The eyepiece is arranged relative to the image A\B[ so


that the image is as if magnified. The image .4'#' w ill thus
he virtual, and while determining the focal length / 2 of the
eyepiece we must put the minus sign in front of 1/a2:
1 _ 1 1 1_ 1 1 f fi o
/a a 2 a r2 ’ f 2 5 cm 26.25 cm ’ ' 2 D Z C m'
The length of the drawtube of the microscope can be de­
termined from the formula
L = a[ -f o2, L = 9.3 cm + 5 cm = 14.3 cm.
Answer. The magnification of the eyepiece is 5.25, the
focal length of the eyepiece is 6.2 cm, and the length of the
drawtube is 14.3 cm.
Problem 103. A point is 35 cm along the optical axis from
a spherical concave mirror having a focal length 25 cm. At

what distance along the optical axis from the concave mir­
ror should a plane mirror be placed for the image it forms
to coincide with the point source?
Given: f = 0.25 m is the focal length of the mirror and
a = 0.35 m is the distance between the point source and
the mirror.
Find: the distance I between the plane and concave mir­
rors.
Solution. Without the plane mirror, the image of point S
would be formed at a point S[ (Fig. 119). Converging rays
are incident on the plane mirror, and hence point S j should
be treated as a virtual source of light, its image being real
(see Problem 94) and separated from the concave mirror by
232 Ch. IV. Optics. Special Theory of Relativity

a. The distance a' to S\ can be determined from the


formula
af 0.35 m X 0.25 m
a—f ’ 0.35 m —0.25 m
0.875 m.

Then the distance between the mirrors is


i i o — a i a nr . 0.875 m —0.35 m a a.
l = a-\---- ^— , f = 0.35m -|------------ ^---------- — 0.61m.

Answer. The point source and its image formed by the


plane mirror coincide when the distance between the mir­
rors is approximately 61 cm.

Questions and Problems

Reflection of light. Plane and spherical mirrors


20.1. A light ray is incident on a plane mirror. By what
angle will the reflected ray be deflected if the mirror is
turned through 20° ?
20.2. The angle of incidence of a light ray on a plane mir­
ror is decreased by 15°. How much will the angle between
the incident and the reflected ray be reduced?
20.3. Sun rays are incident at an angle of 24° to the hori­
zon. How can they be directed parallel to the horizon using
a plane mirror?
20.4. The Sun is 46° above the horizon. What must the
angle of incidence of the sun rays on a plane mirror be so
that the sun light is reflected (a) vertically downwards, (b)
vertically upwards? What will the angle between the reflected
ray and the mirror be? Draw the light paths on a diagram.
20.5. A parallel beam propagates horizontally from a pro­
jector. How should a plane mirror be arranged so that the
image of a slide is formed on the ceiling?
20.6. A mirror hangs vertically on a wall so that its up­
per edge is level with the person’s head. The length of the
mirror is 80 cm. What is the minimum height of the per­
son to still see himself full length?
20.7. A plane mirror is at 45° to the horizontal surface of a
table on which a book lies. In what plane will the image of
the book be formed?
$ 20. Geometrical Optics 233

20.8. A beam is incident on a plane mirror (Fig. 120).


Show graphically the light path. What type of image will
be formed of a lamp and where?
20.9. Two plane mirrors form an angle of 120°. The dis­
tance between the two images of a point source formed in

Fig. 120 Fig. 121

them is 20 cm. Determine the distance from the light


source to the point where the mirrors touch if it lies on the
bisector of the angle formed by the mirrors.
20.10. How should two plane mirrors be arranged for a
point source of light and its two images to lie at the ver­
tices of an equilateral triangle?
20.11. How many images of an object will be formed by
two plane mirrors placed (a) at right angles to each other,
(b) at an angle of 60°?
20.12. How many images will be formed by two mirrors
lying in parallel planes on both sides of an object?
20.13. How can we see the image of a large, building
formed by a small plane mirror?
20.14. On a sunny day, the focus of a concave mirror can
be determined just by using a ruler. How can this be done?
20.15. Determine the radius of curvature and the focal
length of a convex spherical mirror (Fig. 121) if MN = 3 cm
and AB = 30 cm.
20.16. A point source of light is on the axis of a spherical
mirror four focal lengths away from it. Construct its image.
20.17. The focal length of a concave spherical mirror is
0.6 m. At what distance from the pole must a point source
234 Ch. IV. Optics. Special Theory of Relativity

be located on the optical axis so that its image is formed at


the same point?
20.18. A concave spherical mirror forms a threefold mag­
nified inverted image of an object. What are the principal
focal length and the radius of curvature of the surface
of the mirror if the distance from the object to the image is
28 cm?
20.19. The focal length of a concave spherical mirror is
25 cm. Where should an object be placed so that its virtual
image is formed 1 m from the mirror’s pole?
20.20. When an object is 40 cm away from the pole of a
concave spherical mirror, the size of its image is equal to
that of the object. What is the focal length of the mirror?
20.21. A point source lies on the principal optical axis of
a concave spherical mirror, and its virtual image is 40 cm
away from the mirror. The radius of curvature of the mir­
ror is 1.2 m. Determine the distance from the source to the
pole. What is the magnification?
20.22. A concave spherical mirror forms a real image of
an object with magnification 3. When the object is moved a
further 5 cm away from the mirror, the magnification of the
new real image becomes 2. Determine the radius of curva­
ture of the mirror surface and the initial distance between the
object and the mirror.
20.23. A converging beam of solar rays is incident on a
concave spherical mirror whose radius of curvature is 0.8 m.
Determine the position of the point on the optical axis of
the mirror where the reflected rays intersect if the extensions
of the incident rays intersect the optical axis 40 cm from the
mirror’s pole.
20.24. How far from an object in front of a concave spheri­
cal mirror will a real image be formed if it is 42 cm from
the pole and the radius of curvature of the mirror is 48 cm?
20.25. Why are convex mirrors better than plane ones in
motor cars for viewing the road?
20.26. How far from a convex mirror does an object lie
if its image is formed 1 m away from the mirror? The focal
length of the mirror is 1.5 m.
20.27. When an object is 60 cm from the pole of a concave
spherical mirror, its image is formed with a magnification
P = —5. What does the minus sign of the magnification in­
dicate? What is the radius of curvature of the mirror?
$ 20. Geometrical Optics 235

20.28. Where will the image of an object be formed by a


convex spherical mirror with a 1.2-m radius of curvature
and what type of the image will it be if the object is 0.3 m
away from the mirror?
20.29. Converging rays are incident on a convex spherical
mirror so that their extensions intersect 30 cm behind the
mirror on the optical axis. The reflected rays form a di­
verging beam so that their extensions intersect the optical
axis 1.2 m from the mirror. Determine the focal length of
the mirror.
20.30. Two spherical mirrors (convex and concave) having
the same focal length of 36 cm are arranged so that their

Fig. 122

optical axes coincide (Fig. 122). The separation between the


mirrors is 1 m. At what distance from the concave mirror
should an object be placed so that its images formed by the
concave and convex mirrors are identical?

Refraction of light
20.31. What is the refractive index of flint glass if for a
light rays incident at an angle of 63° the angle of refraction is
29°40'?
20.32. A light ray is incident on the surface of water at an
angle of 50°. What is the angle of refraction in water?
20.33. At what angle should a light ray be incident on the
surface of crown glass so that the angle of refraction is 27°?
20.34. Determine the angle through which a light ray is
deflected as it passes from air to water at an angle of inci­
dence of (a) 30°, (b) 45°?
236 Ch. IV. Optics. Special Theory of Relativity

20.35. When a light ray is incident on a quartz plate at


an angle of 44°, the angle of refraction is 27°. For what angle
of incidence will the ray be refracted at an angle of 30°?
20.36. Why is it difficult to shoot a fish swimming in water?
20.37. A light ray is incident on an air-liquid interface at
45° and is refracted at 30°. What is the refractive index of
the liquid? For what angle of incidence will the angle be­
tween the reflected and refracted rays be 90°?
20.38. A light ray passes through the interface between
two transparent media. Under what condition will the angle
of refraction be equal to the angle of incidence?
20.39. Parallel beams of light pass from air to water. The
angle of incidence is 55°. Determine the angle of refraction
of the rays and the propagation velocity in water.
20.40. Determine the velocity of light in ice if, for an
angle of incidence of 61°, the angle of refraction is 42°.
20.41. As a light ray passes from air to a transparent liquid
at an angle of incidence of 69°, the angle of refraction is

Fig. 123 Fig. 124

38°30'. What is the velocity of light in the liquid? What si


the refractive index of the liquid?
20.42. Determine the angle of incidence of a light ray
on the surface of acetone if the angle between the reflected
and refracted rays is 120°.
20.43. The refractive index of rock crystal is 1.54. At
what angle must a light ray be incident from air to the
crystal so that the reflected ray is perpendicular to ihe
refracted ray?
20.44. A source of visible light at the bottom of an aquar­
ium filled with water emits rays incident on the water-air
interface (Fig. 123). Trace the light path afterwards.
20.45. Determine the critical angle of incidence for a
light ray passing from diamond to water.
§ 20. Geometrical Optics 237

20.46. Determine the refractive index of benzene if the


critical angle of incidence of rays on it is 42°.
20.47. For what angle of incidence of a light ray on a dia­
mond-water interface is total reflection observed?
20.48. If a bright metal ball is first blackened with soot
in a candle flame and then immersed in water, it appears
bright again. Why?
20.49. Can you explain why a test tube immersed at a
certain angle in a tumbler of water appears to have a mir­
ror surface for a certain viewing position?
20.50. The direction of an incident light ray is shown in
Fig. 124. The semicylinder is made of glass with refractive
index 1.8. What is the path of the ray afterwards?
20.51. A bulb is fixed at the bottom of an aquarium divided
into two parts by a partition (see Fig. 123). What is the
maximum height to which water may be poured if the bottom
at the opposite face is to be illuminated and the aqua­
rium is 60 cm long?
20.52. Solar rays are incident on the surface of water at
45°. How long is the shadow of a pole erected at the bottom
of a pond if the pole is vertical and completely immersed in
water? The length of the pole is 1.2 m.
20.53. Solve Problem 20.52 assuming that 0.2 m of the
pole is above the water surface.
20.54. A light ray is incident at 45° on a plate made of
crown glass with plane-parallel faces. Determine the thick­
ness of the plate if a ray emerging from it is displaced by
1.5 cm.
20.55. Determine the displacement of a light ray passing
through a 2.1-cm thick plate made of crown glass with plane-
parallel faces if the angle of incidence is 30°.
20.56. A plate with plane-parallel faces, having refrac­
tive index 1.8, rests on a plane mirror. A light ray is in­
cident on the upper face of the plate at 60°. Hovrfar from the
entry point will the ray emerge after the reflection by the
mirror if the plate is 6-cm thick?
20.57. Light rays passing through a plane-parallel plate
are displaced. Why do we not notice this through a window?
Under what conditions does the displacement become no­
ticeable?
20.58. A light ray is incident on a glass plate with plane-
parallel faces at 70°. The plate is 4-cm thick. The refractive
238 Ch. IV. Optica. Special Theory of Relativity

index of glass is 1.5. Determine (1) the displacement of the


ray and (2) the path length of the ray in the plate.
20.59. A light ray is incident at right angles on a right-
angled glass prism (Fig. 125). What will the path of the
ray be afterwards?
20.60. How must two right-angled prisms be arranged in a
periscope? Draw the light path in it.
20.61. - Two parallel rays 1 and 2 are incident on a glass
right-angled prism (Fig. 126). Trace the light path after­
wards.
20.62. A light beam is incident along the normal on a
lateral face of a prism with an angle of refraction of 30°

(frig. 127). Determine the angular displacement of the beam


as a result of its passage through the prism if its refractive
index is 1.8.
20.63. A ray incident on the lateral face of a glass prism
with an angle of refraction of 30° emerges from it at 30°.
The refractive index of glass is 1.5. Determine the angle of
incidence of the ray.
20.64. A ray is incident on a lateral face of a glass prism
along the normal and emerges from the prism at 25° from the
direction of the incident ray. The refractive index of glass
is 1.5. What is the angle of refraction of the prism?
20.65. The angle of refraction of a prism is 60°. A light ray
emerges from the prism at the same angle as it is incident
on it. The refractive index of the prism is 1.5. Determine the
angle by which the ray is deflected from its initial direction
as a result of its passage through the prism.
20.66. The angle of refraction of a prism is A . A light beam
is deflected by an angle 6 upon passing through the prism.
<j 20. Geometrical Optica 239

Determine the refractive index of the prism if the angle of


incidence of the ray is equal to the angle of refraction
as it emerges from the prism.

Lenses. Optical instruments


20.67. Determine the optical power of a double-convex
lens whose focal length is (a) 12.5 cm, (b) 50 cm.
20.68. Determine the optical powers of concave lenses
with focal lengths of —25 cm and —0.4 m.
20.69. Given lenses with optical powers of 4, —5, and
—2 D. Determine their focal lengths.
A

'r
Fig. 128

Screen^

V
Fig. 129

20.70. In what case will a double-convex lens be di­


verging?
20.71. Given an optical axis M N , a converging lens, its
foci, and a point source S. on the optical axis (Fig. 128).
Construct the image of the point source.
20.72. An opaque screen is placed between an object and
a converging lens (Fig. 129). What will happen to the im­
age? Use a diagram to explain your answer.
20.73. What will the paths of the rays be after refraction
in the lenses shown in Figs. 130a and b?
20.74. Two converging lenses should be placed in the
path of parallel rays so that the rays remain parallel after
240________ Ch. IV. Optics. Special Theory of Relativity

passing through both lenses. How should the lenses be ar­


ranged?
20.75. The focal length of a converging lens is 20 cm.
An object is 60 cm from the lens. Where will the image be
formed and what kind of image is it?

(a ) j \ lb) 't
F F ' F f

f i

Fig. 130

20.76. How far away will a converging lens with a focal


length of 10 cm form the images of objects located 5, 15,
20, and 25 cm from the lens?
20.77. Construct the images of the objects in Problem
20.76 on a 1 : 10 scale and answer the following questions:
(1) how does the height of
the image change with the
M-
f distance between the object
and the lens? (2) what type
B'\
of image is it when an ob­
ject is placed twice the fo­
Fig. 131 cal length from the lens? (3)
where should the object be
placed to obtain a virtual image?
20.78. When an object is placed 20 cm from a lens, the
image is the same size as the object. What are the focal
length and optical power of the lens?
20.79. Given an optical axis M N and the positions of an
object and its image (Fig. 131), determine graphically the
position of the lens (its optical centre O) and its foci. Is
it a converging or diverging lens? Is the image real or vir­
tual?
20.80. At what distance from a double-convex lens with
focal length 0.42 m is an object placed if its virtual image
is formed 56 cm from the object?
20.81. A candle is 2 m from a wall. When a converging
lens is placed 40 cm from the candle between the candle and
§ 20. Geometrical Optica 241

the wall, a sharp image of the candle is formed on the wall.


Determine the optical power of the lens and its magnifica­
tion.
20.82. A lens placed between a candle and a screen forms
a real triply magnified image of the candle on the screen.
When the lens is moved away from the candle by 0.8 m
without changing the position of the candle, a real image
one-third the size of the candle is formed on the screen. Deter­
mine the focal length of the lens.
20.83. An object 2 cm high is 15 cm from a double-con­
vex lens whose optical power is 10 D. Determine the height
of the image.
20.84. An object is 1.4/ (where / is the focal length) from
a converging lens with an optical power of 2 D. Where is
the image formed? What type of image is it?
20.85. An object is placed 0.8/ from a lens (see Problem
20.84). Where is the image formed? What type of image is
it?
20.86. The focal length of a double-concave lens is 12 cm.
What is the distance between the lens and an object if its
image is 8 cm away from the lens?
20.87. An object is 20 cm from a double-concave lens
with a focal length of 30 cm. Where is the image formed (rel­
ative to the lens) and how high is it if the object is 6 cm
high?
20.88. A virtual image is formed 8 cm from a diverging
lens with a focal length of 0.12 m. How far is the object from
its image?
20.89. Will the focal length of a glass lens change if it
is immersed in a medium with the same optical density as
the glass?
20.90. What will the magnification of a converging lens
with an optical power of 4 D be if an object is 1.4/ away
from it?
20.91. A virtual image one-fifth of the size of an object is
formed 6 cm from a diverging lens. Find the focal length and
optical power of the lens and construct the image of the
object formed by it.
20.92. The distance from a converging lens to an object
is thrice the focal length. Determine the magnification.
20.93. The optical power of a lens is 2.5 D. For a certain
position of an object relative to the lens, a real doubly mag-
1 C— 0 5 3 0
242 Ch. IV. Optics. Special Theory of Relativity

nified image is formed on a screen. What will the magnifica­


tion be if the object is moved 0.1 m closer to the
lens?
20.94. A converging lens forms a triply magnified real
image of an object. In order to obtain a virtual image with
the same magnification the lens is moved towards the object
by 10 cm. What are the focal length and the optical power of
the lens?
20.95. Two identical thin converging lenses brought in
contact so that their axes coincide are placed 12.5 cm from
an object. What is the optical power of the system and that
of a single lens if the real image formed by the system of
lenses is four times as large as the object?
20.96. A convex and a concave lens are brought in close
contact along their optical axes. The focal length of the
convex lens is 10 cm. When the system is placed at 40 cm
from an object, a sharp image of the object is formed on a
screen on the other side of the system. Determine the opti­
cal power of the concave lens if the distance I between the
object and the screen is 1.6 m.
20.97. The focal length of a projector’s objective is 15 cm.
The distance between the slide and the objective is 15.5 cm.
What is the linear magnification of the projector?
20.98. Determine the optical power of the projector’s
objective if it produces a 24-fold magnification when a
slide is placed 20.8 cm from the objective.
20.99. The objective of a projector has a focal length of
12.5 cm. How far should the screen be placed away from
the objective to obtain a twentyfold magnification?
20.100. The image of a 7 X 5 cm2 slide is projected onto
a 1.05 X 0.75 m2 screen using projector at its maximum
magnification. What is the optical power of the objective’s
lens if the screen is 4 m away?
20.101. What purpose do diaphragms serve in the objec­
tives of cameras?
20.102. The focal length of a camera’s objective is 50 mm.
The height of the image on the film of a building 80 m away
from the camera is 12 mm. What is the actual height of the
building?
20.103. The image on the frosted glass of a camera is
13.5 mm high when the object is 8.5 m away and 6 cm when
2 m away. Determine the optical power of the objective.
§ 21. Photometry 243

20.104. The focal length of a microscope’s objective is


5 mm and the drawtube is 16 cm long. Determine the magni­
fication of the eyepiece for normal sight if the microscope’s
total magnification is 200.
20.105. The focal length of a microscope’s objective is
0.4 cm. A sample is placed 0.1 mm away from the focus. The
magnification of the microscope is 400. Determine the focal
length of the eyepiece and the length of the drawtube if the
distance of best vision for an observer is 25 cm.

§ 21. PHOTOMETRY

Basic Concepts and Formulas


In photometry, the concept of a point source of visible ra­
diation (light) is employed. A fraction of this radiation cor­
responding to the wavelength range from 400 to 760 nm acts
on the retina of the eye and is perceived as light. A point
source is assumed to be consid­
erably smaller than the distance
over which its action is ana­
lyzed. Such a source is assumed to
emit uniformly in all directions.
An example of such a source is
a star. Fig. 132
The luminous flux <1) is the
power of visible radiation and is estimated by its action on
the retina of a normal eye. The unit of a luminous flux is
the lumen (lm).
The luminous intensity (candle power) / of a source is
defined as the ratio of the luminous flux propagating from
the source in a given direction within a small solid angle
to the magnitude of this angle:
, AO
1 ~ AQ •
The unit of solid angle is the steradian (sr). A steradian is
the solid angle bounded by a conic surface which cuts an area
of a sphere’s surface AS equal to the square of the radius
(Fig. 132), the apex of the solid angle being at the centre of
the sphere. The solid angle embracing the space around the
16*
244 Ch. IV. Optics. Special Theory of Relativity

point rests on the surface of the sphere and is equal to


4n R *
Q= R* = 4ji sr.
The total luminous flux emitted by a light source is
d>tot = IQ = 4jil .
The unit of luminous intensity is the candela (cd).
A luminous flux incident on the surface of a body is par­
tially reflected, reaches the eye retina, and causes the sen­
sation of ligh t—we see the things surrounding us.
The illuminance E is defined as the ratio of the luminous
flux Q incident on a small surface element to its area
E = CDIS.
The unit of illuminance is the lux (lx).
The illuminance of a surface by normal rays varies in in­
verse proportion to the squared distance from the light
source to the surface:
E 0 = H R 2.
If rays are incident on a surface at an angle, the illumi­
nance depends on the angle of incidence
E = E 0 cose, or £ = — cose.
The luminous intensities of two sources can be compared
using a photometer. The two sources are placed on either
side of the photometer at distances such that the illuminances
at the photometer are equal. By measuring the distances
from the light sources to the photometer and given the lumi­
nous intensity of one of the sources, we can determine the lu­
minous intensity of the other source:
E= h
R? »
Ii_ or i i _ m
Ri Is

Worked Problems
Problem 104. Two lamps with luminous intensities of
200 and 300 cd are suspended from 3-m high poles (Fig. 133).
The distance between the lamps is 4 m. Determine the illu­
minance on the ground at points A, B, and C.
§ 21. Photometry 245

Given: h = 3 m is the height of the lamps above the


ground, /j = 200 cd and / 2 = 300 cd are the luminous in-
tensities of the lamps, and I = 4 m
is the distance between them.
Find: the illuminances E A, E B,
and Ec at points A, B , and C.
Solution. At points A, B, and C,
the surface of the ground is illumi­
nated by both the lamps. Let us con­
sider each case separately. At
point A,
E a — E1A -f E 2A,
where ElA is the illuminance at
point A from the first lamp and E sA is that from the second
lamp. The light rays from the first lamp are incident along
the normal to the surface at point A, and hence
200 cd
E *1A = 9 m2
= 22.2 lx.

The illuminance produced at the same point by the second


lamp is due to the rays incident at an angle e (see Fig. 133).
Therefore, E iA = E 0 cos e, where E 0 is the illuminance of
the surface at point A due to rays incident along the nor­
mal, E0 = 12tr2. Since r = \^h2 I2, we have
E0 = • It can be seen from the figure that
h h
cos e = — = .. .
r l/fc* + Z*
The second lamp produces at point A an illuminance of

p 1 2 h p dOOcd X 3 m rj e) i
2A — As + /2 y j j + p ’ £’2A“ 25m2 X 5 m “

The total illuminance at point A is


E a = 22.2 lx + 7.2 lx = 29.4 lx.

The illuminance at the ground at point B is found in the


same way:
Eb — Eib -+■ E 2B'
246 Ch. IV. Optics. Special Theory of Relativity

The angle of incidence of the rays from the first lamp re­
mains the same, and
200 cd p p . 300 cd
Eb = cos e -j- hh*. » E b 25 m2 UD + 9 m2
38.1 lx.
r\
The illuminance at point C is
Ec — E lc + E2C.
Since point C is equidistant from the two lamps, the rays
from the lamps form the same angle with the normal.
Therefore,
COS ti
Ec = cos et + cos e„ or Ec (A H-
rI ri
3m
Here r\ = hz -f- (0.5/)2= 13 m2, and cos e, 0.83.
3.6 m
Finally, we have

Ec = (200 cd + 300 cd) = 31.9 lx.

Answer. The illuminance of the ground at points A , B,


and C is 29.4, 38.1, and 31.9 lx respectively.
Problem 105. Two incandescent lamps with luminous in­
tensities 25 and 225 cd are 1 m apart (Fig. 134). Where
should a screen be placed between
them for it to be equally illu-
r minated from both sides?
■o Given: / x = 25 cd and I t =
.[ 225 cd are the luminous inten­
sities of the first and second
Fig. 134 lamps and I = 1 m is the sep­
aration between them.
Find,', the distance from the first lamp to the screen.
Solution. We shall assume that the rays from the first
and second lamps are incident on the screen at right angles.
Then the illuminances of the screen by the lamps are

E, =

By hypothesis, E i = E2, and hence . But r2 =


ri ri
l —r.. Therefore, _ /« or — = ( i - 3rr —
1 r2 - (i _ ri)i » or I 2 ,) 1 *
$ 21. Photometry 247

We shall solve this equation for r,:

y it ‘ ri
v- r *> va = rt / r 2, i y j t= r t ( / / , + K r 2) ,

whence
1 m ]/25~cd n0(,
r. = ;-_ r— , r. = — — >. = 0.25 m.
l^ /i + V / j l/2 5 cd + /2 2 5 cd
Answer. The screen should be placed 0.25 m from the first
lamp.
Problem 106. A 25-cd point source of light is at the centre
of a spherical surface of radius 0.5 m. Determine the lumi­
nous flux incident on the inner surface of the sphere over an
area of 50 cm*.
Given: r = 0.5 m is the radius of the spherical surface,
I = 25 cd is the luminous intensity of the source, and S =
50 cm* = 5 x 10-3 m* is the area of the surface element.
<Find‘. the luminous flux <D.
Solution. In order to determine the luminous flux, we
must know the area of the surface on which it is incident
and its illuminance:
<D = ES.
The illuminance is produced by the rays incident along the
normal on the inner surface of the sphere. Therefore, E =
Hr2, and hence

( D = -rz
L 5 , <I>= 25cd 5 *™~3 sr = 0.5 1m.
0.25
Answer. A luminous flux of 0.5 lm is incident on a sur­
face element of area S.

Questions and Problems


21.1. Determine the total luminous flux emitted by a
source of luminous intensity 200 cd.
21.2. A luminous flux of 2 lm is uniformly distributed
within a solid angle of 0.5 sr. What is the luminous intens­
ity of the point source located at the apex of the angle?
21.3. A 15-cd point source of light is placed at the centre
of a hollow sphere of radius 30 cm. Determine the illumi-
248 Ch. IV. Optics. Special Theory of Relativity

nance of the inner surface of the sphere and the total lumi­
nous flux emitted by the source.
21.4. Determine the luminous flux passing through a sur­
face of area 20 cm2 located 5 m from a 100-cd point source
of light. The rays are incident normal to the surface.
21.5. The average illuminance in Leningrad during a
summer night (when the Sun does not sink deep below the
horizon) is 1 lx, while the illuminance during a moonlit
night is 0.1 lx. What are the luminous fluxes incident on the
Mars Field in Leningrad which covers an area of 0.1 km2?
21.6. An incandescent lamp (without a shade) of lumi­
nous intensity 25 cd is suspended 80 cm above a table. De­
termine the illuminance on the table.
21.7. The luminous intensity of the lamp in a photograph­
ic enlarger is 15 cd. Determine the illuminance on a piece
of photographic paper if the enlarger is 30 cm above it and
only 15% of the luminous flux is used.
21.8. An incandescent lamp in a room produces an illu­
minance of 28 lx at one wall and 7 lx on the opposite wall at
the same level. What is the ratio of the distances between
the lamp and the walls?
21.9. When will the illuminance under a lamp be higher:
for a luminous intensity of 120 cd 3 m away or for a lumi­
nous intensity of 25 cd 1.2 m away?
21.10. Parallel rays incident at an angle of 25° produce
an illuminance of 54 lx. At what angle of incidence will the
illuminance of the surface be 45 lx?
21.11. Before a sunset, sun light is incident on the sur­
face of the Earth at an angle of 81°. Compare the illumi­
nances produced on the surface of the Earth and on a
vertical wall facing the Sun.
21.12. The maximum illuminance that can be created by
sun light on the surface of the Earth is 108000lx. How far
from the Sun is the planet Mars when the maximum solar
illuminance on its surface is 48 000 lx?
21.13. Why does snow melt more quickly on sunlit
slopes than on sunlit horizontal areas?
21.14. When parallel rays are incident on an object alorg
the normal to its surface, the illuminance is 70 lx. What
will the illuminance of the surface be if the object is turned
so that the angle of incidence is 60°?
21.15. An electric bulb of luminous intensity 150 cd is
§ 21. Photometry 249

suspended above a round table of diameter 2 m. Determine


the maximum and minimum illuminances on the table if
the distance of the cent e of the table from the lamp is
1.5 m.
21.16. A lamp without a shade is suspended 1 m above
the centre of a table with diameter 1.2 m. Determine the
illuminance at the edge of the table if the total luminous
flux from the lamp is 650 lm.
21.17. A bulb without a shade is suspended 1 m above a
table. The distance between the bulb and a book lying at
the edge of the table is 2 m. What must luminous intensity
of the bulb be if the illuminance on the book is 25 lx?
21.18. The light from an electric bulb is incident on a
working space at an angle of 45° and produces an illuminance
of 141 lx. The luminous intensity of the bulb is 200 cd.
How far away is the bulb from the working space? How
high above the working space is it suspended?
21.19. Two bulbs with the same luminous intensity of
50cd are suspended 1 m above a table. The distance between
the bulbs is 140 cm. Determine the illuminance on the
table under each bulb.
21.20. Two lamps of 200 cd each are fixed to a pole 2 and
3 m from the ground respectively. Determine the illumi­
nance on the ground at 1 m from the foot of the pole.
21.21. A lamp is suspended at the top of a 10-m pole so
that the illuminance on the ground 10 m from the base of
the pole is 2.5 lx. What is the luminous intensity of Ihe
lamp?
21.22. Two lamps of 250 cd each are suspended from a
height of 4 m on poles 5 m apart. Determine the illuminance
on the ground midway the poles.
21.23. A lamp of 800 cd is suspended 10 m above the
ground. Over what area of the ground will the illuminance
be at least 1 lx?
21.24. Two incandescent lamps with luminous intensities
300 and 200 cd are suspended at a height of 3 m. The dis­
tance between the lamps is 4 m. Determine the illuminance
of the point on the ground between the lamps, where rays
from the first lamp are incident at 45°.
21.25. A lamp of luminous intensity 32 cd which was sus­
pended 1.2 m above the middle of a table is replaced by
another lamp whose luminous intensity is 90 cd. How high
250 Ch. IV. Optics. Special Theory of Relativity

must the second lamp be suspended if the illuminance at the


middle of the table is to remain unchanged?
21.26. A lamp suspended from a height of 6 m illuminates
a skating rink. How far from the lamp and from the centre
of the rink will the illuminance on the surface of ice be a
factor of 3.4 less than that at the centre?
21.27. A lamp with luminous intensity 25 cd is placed
15 cm to the left of a photometer. Another lamp is placed
45 cm to the right of the photometer, the illuminance on
both sides being the same. Determine the luminous intens­
ity of the second lamp.
21.28. Two lamps with luminous intensities 50 and
200 cd are 2.4 m apart. Where should an opaque screen be
placed between the lamps so that it is equally illuminated
on both sides?
21.29. The centre of a screen is illuminated by a light
source with a luminous intensity / that is placed a distance
Screen

T*<rDcd /'
O.Sm_ T . _
V/.
Fig. 135

I from the screen. Will the illuminance change if the lumi­


nous intensity and the distance from the light source are
both increased ra-fold?
21.30. A light source of 40 cd is placed between a screen
and a plane mirror 0.5 m from the screen (Fig. 135). The
distance between the screen and the mirror is 1.2 m. Deter­
mine the illuminance of the screen where the light ray is
incident along the normal. Assume that the mirror is per­
fectly reflecting.
21.31. A lamp used for printing photographs had lumi­
nous intensity 50 cd and was placed 1.2 m from the photo­
graph. The exposure time was 3 s. When the lamp burnt
out, it was replaced by another lamp of 40 cd placed 1 m
from the photograph. Determine the exposure time for the
new lamp.
§ 22. Wave Properties of Radiation 251

§ 22. PHENOMENA EXPLAINED BY THE WAVE PROPERTIES


OF RADIATION. INTERFERENCE. DIFFRACTION
Basic Concepts and Formulas
Optical radiation occupies a small interval in the electro­
magnetic wave spectrum and includes three regions, viz.
the ultraviolet, visible, and infrared wave bands.
Ultraviolet radiation corresponds to wavelengths from
about 5 to 400 nm and it can induce marked chemical re­
actions.
The radiation causing the sensation of light is known as
visible radiation, or just light. The lower boundary of the
spectrum of visible radiation lies between 380 and 400 nm,
and the upper boundary between 760 and 780 nm.
Infrared radiation corresponds to wavelengths from
780 nm (the upper boundary of visible radiation) to 1 mm.
It has a well-pronounced thermal effect.
Optical radiation is electromagnetic in nature. The wave­
length is given by
A. - cT, or k = ch ,
where c is the propagation velocity of electromagnetic
waves, T and v being their period and frequency.
As light waves pass from vacuum to a medium, the wave­
length changes (the frequency remaining unchanged):

where n is the absolute refractive index and c and v are re­


spectively the propagation velocities of electromagnetic
waves in vacuum and in a medium.
Phenomena which confirm the wave nature of visible ra­
diation include interference and diffraction.
The interference of light is the enhancement or attenua­
tion of light as a result of superposition of light waves. In
order to observe an interference pattern, the waves must
have the same wavelength and a constant phase difference,
i.e. they must be coherent.
Using Fresnel’s biprism, we can form two virtual images
S' and S " of the same source S, which emit coherent rays
(Fig. 136). When monochromatic radiation is incident on the
252 Ch. IV. Optics. Special Theory of Relativity

biprism (i.e. the radiation with a single frequency), bright


and dark fringes are formed on the screen. The maximum
brightness occurs when the optical path difference A is
equal to an even number of half-waves:
A = 2k — , where k = 1, 2, . . . .

If, however, one wave lags behind another by a half­


wavelength, i.e. the optical path difference is equal to an
odd number of half-waves, the
maximum attenuation of light
is observed:
A = (2 * -H ) A ,
where A= 0, 1, 2, . . . .
Interference can be observed
in thin films or for light pas­
sing through a system formed by a plano-convex lens and
a glass plate (Newton’s rings).
If we know the radius of curvature R of the lens and mea­
sure the radius rh of one of the dark rings (Fig. 137), we can

T
i
IX
I
I
I
I
I
~ ^ L 0 ! \ 0 j

IV 0
*
0
*
0
#

Fig. 137

determine the wavelength of the light illuminating a device


that yields Newton’s rings:
rh= VkkR,
where k = 0, 1, 2, . . . is the number of the dark ring. The
formula is valid when air, for which n = 1, occupies the
space between the plate and the lens.
§ 22. Wave Properties of Radiation 253

Diffraction is the bending of light round obstacles, and it


occurs when the size of the obstacle is commensurate with
the wavelength.
A diffraction grating is formed by alternating transparent
and opaque lines. The sum of the width of the line and of a
separation between two lines is known as the grating con­
stant (Fig. 138):
d = a -(- b.
where d is the grating constant, b is the width of a line, and
a is the distance between two lines.
The formula for a diffraction grating is
d sin ip = kk
where k is the order of a maximum. The brightest (zero-or­
der) maximum lies on the screen opposite the centre of the
grating.
The formula of a diffraction grating shows that one must
measure the angle <p to determine the wavelength using a
diffraction grating.
When white light is incident on a grating, a spectrum
known as the normal spectrum is formed on a screen. It
contains all the coloured bands (from red to violet).

Worked Problems
Problem 107. The wavelength of light in glass is 450 nm .
Light propagates in glass at a velocity of 1.8 X 10s km/s.
Determine the frequency of light, the absolute refractive
index of glass, and the wavelength of the light passing from
glass into vacuum.
Given: kg = 450 nm = 4.5 X 10~7 m is the wavelength of
light in glass, v = 1.8 X 105 km/s = 1.8 X 108 m/s is the
velocity of light in glass. From tables, we take the velocity
of light in vacuum, c = 3 X 108 m/s.
Find: the frequency v of light, the absolute refractive in­
dex n of glass, and the wavelength k of light in vacuum.
Solution. From the wavelength and the velocity of light
in glass, we can determine its frequency: kg = vh , whence
1.8 x 10®m/s = 4.0 x 1014 Hz = 400 THz.
4.5 X 10~7m
254 Ch. IV. Optics. Special Theory of Relativity

As light passes from one medium to another, its frequen­


cy remains unchanged, but its velocity and wavelength
change. Consequently, we can determine the wavelength of
light in vacuum:
c 3 X 10s m/s
X
v 4.0 X 10l* Hz
0.75 X 10'6 m.

The optical density of a medium is equal to the absolute


refractive index:
c 3 x 10* m/s ^
n
1.8 X 108 m/s
Answer. The frequency of light is 400 THz, the wavelength
in vacuum is 750 nm, and the absolute refractive index is
about 1.7.
Problem 108. Two coherent light sources emit light of
wavelength 550 nm which produces an interference pattern

on a screen (Fig. 139). The sources are 2.2 mm apart and


2.2 m from the screen. Determine whether the interference
at point O is constructive or destructive.
Given: X = 550 nm = 5.5 X 10-7 m is the wavelength of
light, I = 2.2 m is the shortest distance from the first
source to the screen, and d = 2.2 mm = 2.2 X 10-3 m is
the separation between the sources.
Find: the path difference A of the rays.
Solution. In order to answer the question, we must know
the path difference for the rays. The optical path difference
§ 22. Wave Properties of Radiation 255

is equal to their geometrical difference (since the rays pro­


pagate in the same medium, viz. air):
A = S 2D = S 20 — S tO, StO = I.
From the triangle S ,0 S 2, we determine SzO: Sz0 =
y W + l P = l Y i + (d/l)z. Since dll is much smaller than I,
we can use approximation { y \ ± a2 = 1 ± - y a 2) , whence
S , 0 > l [ l + i - ( 4 ) 2] . Then

There will be constructive interference at 0 if the path


difference contains integral number of waves, i.e. k = 1,
2, 3, . .
K A _ 1.1 x 10-6 m 0
K ~ X ~ 5.5 x 10-7 m

Answer. Constructive interference (bright fringe) occurs


at 0 .
Problem 109. A plano-convex lens with a radius of curva­
ture of 12 m is placed on a flat plate as shown in Fig. 140.
Monochromatic light is incident along the normal to the
plane face of the lens, and dark and bright rings are formed
in the reflected light. Determine the wavelength of the mo­
nochromatic light if the radius of the sixth dark ring is
7.2 x 10-3 m.
Given: R = 12 m is the radius of curvature of the lens,
k = 6 is the dark ring number, and ra = 7.2 X 10~3 m is
the radius of the sixth ring.
Find: the wavelength k of the monochromatic light.
Solution. The light wave incident on the plane surface of
the lens is partially reflected by the convex surface of the
lens and partially passes through the air gap d and then
reflected from the flat plate, the path difference being in­
creased by k/2 by the reflection. Thus, the second wave pas­
ses through the gap d twice, and hence the path difference
will be A = 2d + k/2.
256 Ch. IV. Optics. Special Theory of Relativity

For a dark fringe, we have A = —■(2k + 1 ) , or 2d +


A = Xk + , whence d —— .
From triangle ABC, we obtain R 2 = (R — d)2 + r\,
2Rd — d2 = r\. The quantity d2 can be neglected since d
is small. This gives 2Rd = rj:, whence d = r\!2R. Com­
paring the obtained expressions for d, we obtain

Xfc___r§_
2 2R ’

whence
X= (7.2 X 10~3 m)2 = 7.2 X lO'7 m.
kR ’ 6 x 12 m

Answer. The wavelength of light is 720 nm.


Problem 110. The grating constant of a diffraction grating
is 0.016 mm. The red line of the 2nd-order spectrum is 14.2 cm
from the middle line. The dis­
tance from the grating to a
screen is 1.5 m. Determine the
wavelength of the red light
and the width of the2nd-order
spectrum. The wavelength of
violet light is 4 X 10-7 m
(Fig. 141).
Given: d = 0.016 mm =
1.6 X 10~5 m is the grating con­
stant, h T = 14.2 cm = 1.42 X
10-1 m is the distance
between the red region of the
2nd-order spectrum and the
middle line, k = 2 is the order
of the spectrum, I = 1.5 m is the distance from the screen
to the diffraction grating, and i v = 4 X 10-7 m is the
wavelength of violet light.
Find: the wavelength k T of the red light and the width h
of the 2nd-order spectrum.
Solution. The wavelength of the red light can be deter­
mined from the formula for a diffraction grating: kkT=
d sin tp. Here the angle <p is very small so that we can
§ 22. Wave Properties of Radiation 257

put sin <p ~ tan cp. Then


« d tan ip • dhT
K = ----or
k kr’ r kl ’
1.6 X 10~6m X 1.42 X IQ-1 m
K= ;7.57 x lO"7 m.
2 X 1.5 m
In order to determine the spectral width, we must know
the distance from the middle line of the spectrum to the
violet region h v . This can be determined from the formula for
.. .. , kykl , 4x10”7m x2xl .5 m
a diffraction grating: h v = —— , h v = ------ 1 6 x l0 -6 m-----
= 7.5 X 10-2 m.
Thence
h = hT — h y = 14.2 X 10"2 m — 7.5 X 10-2 m
= 6.7 X 10-2 m.
Answer. The wavelength of the red light is approximately
equal to 760 nm and the width of the 2nd-order spectrum is
6.7 cm.

Questions and Problems


22.1. When would a bright spot be formed on a screen
where two beams from a coherent monochromatic light
source meet, and when would the spot be dark?
Screen

^Soooooooc 2

Fig. 142

22.2. Light from two coherent sources is incident at


points / and 2 on a screen (Fig. 142). Where will construc­
tive and destructive interference occur?
22.3. If crystals of common salt are placed in the flame
of a candle and then the flame is observed through a transpa-
17-0530
258 Ch. IV. Optics. Special Theory of Relativity

rent plate with parallel faces, alternating dark and yellow


fringes can be seen against the background of the flame. The
same pattern can be observed in reflected light (by placing
the plate behind the flame). How can this effect be ex­
plained?
22.4. Oil spots on the surface of sunlit water are rainbow
coloured. Why? Will the pattern change if the surface is
illuminated by monochromatic light?
22.5. Rays from two coherent light sources of wavelength
0.5 pm and with a path difference of 0.5 mm arrive at a
certain point in space. Will the interference be constructive
or destructive at this point?
22.6. Red light of wavelength 760 nm from two coherent
sources is incident on a screen, forming an interference pat­
tern of red and dark fringes. Determine the path difference
of the rays if four half-waves fit into it. What type of
fringe (red or dark) is formed for such a path difference?
22.7. A soap bubble displays all the colours of the rain­
bow in sun light. Why?
22.8. Determine the radius of the second dark Newton
ring in reflected light if a plano-convex lens with the radius
of curvature 8 m and a flat plate (Fig. 140) are illuminated
by a monochromatic light with a wavelength of 640 nm.
22.9. An instrument for observing Newton’s rings is il­
luminated by a monochromatic red light. The radius of the
third dark ring is found to be 2.8 mm. Determine the wave­
length of the red light if the radius of curvature of the plano­
convex lens is 4 m.
22.10. The air gap in an instrument for observing New­
ton’s rings (see F;g 140) is filled with water. What will the
change in the radii of the interference rings be?
22.11. What is the antireflection coating of objectives
based on? Why do objectives have a bluish-violet tinge in
reflected light?
22.12. The distance between two coherent monochromat­
ic point light sources is 1.5 cm. The sources are located at
36 m from a screen so that the line connecting them is paral­
lel to the plane of the screen. Determine the wavelength of
the light if the separation between adjacent interference
fringes is 1.8 mm.
22.13. A plano-convex lens with a radius of curvature of
8 m is put on a flat transparent plate. When the system is
§ 22. Wave Properties of Radiation 259

illuminated with green light from thallium (wavelength


536 nm), Newton’s rings are formed in the reflected light.
Determine the radius of the fifth dark ring.
22.14. The radius of the third dark Newton ring (see
Problem 22.13) is found to be 2.8 mm when the system is
illuminated by a monochromatic light. Determine the radius
of curvature of the plano-convex lens if the wavelength of
the monochromatic light is 720 nm.
22.15. If the light of a street lamp is viewed through one’s
eyelashes, rainbow rings appear around the lamp. Explain
this effect.
22.16. What is the difference between a diffraction spec­
trum and the spectrum obtained with a prism?
22.17. In a practical experiment on the wavelength of
light using a diffraction grating, the first diffraction maxi­
mum was observed on a screen 30 cm from the central line.
The grating constan t was 2 X 10~3 mm, and the distance
from the screen to the grating was 1.5 m. Using these data,
determine the wavelength.
22.18. Light from a gas-discharge tube is incident nor­
mally on a diffraction grating whose constant is 2 X 10"3 mm.
The orange line in the lst-order spectrum is seen at an angle
of 18°, and the blue line is seen at 14°. Determine the wave­
lengths of this light.
22.19. Spectra formed by a diffraction grating are pro­
jected on a screen 3 m away. Determine the wavelength of a
monochromatic light if the distance from the central line to
the lst-order spectrum is 22.8 cm and the grating constant
is 0.01 mm.
22.20. Monochromatic light corresponding to a sodium
line of wavelength 5.89 X 10"7 m is incident on a diffraction
grating. The angle at which this line is seen in the lst-order
spectrum is 17° 18'. Determine the grating constant. How
many lines does a centimetre of the grating contain?
22.21. Monochromatic light emitted by a mercury-vapour
lamp and having a wavelength of 579 nm is incident on a
diffraction grating with constant ? X 10-5 m and forms a
diffraction spectrum on a screen. The distance from the grat­
ing to the screen is 1.5 m. How far from the central line
will the coloured line in the lst-order spectrum be?
22.22. How many lines per millimetre will a diffraction
grating have for the green line (wavelength 500 nm) in the
17*
260 Ch. IV. Optics. Special Theory of Relativity

3rd-order spectrum to be observed at angle 48° 30'?


22.23. A monochromatic light of wavelength 500 nm is
incident normally on a diffraction grating containing 500
lines per millimetre. What is the highest order spectrum ob­
servable using this grating?

§ 23. RADIATION AND SPECTRA

Basic Concepts and Formulas


While light is a composite. We can prove this with a trigo­
nal prism. After passing through the prism, a ray of white
light splits into the coloured rays of the spectrum.
Dispersion is the dependence of the refractive index on the
wavelength of the transmitted light. The refractive index
rar for red light is smallest and for violet light n v, is
largest. This means that red light propagates faster in a
transparent medium than violet light: nr = c/vr, rav =
d v v. Since n r <Znv, clvT< i c / v v, or vT> v v.
The colour of a transparent body depends on the colour of
the light rays it transmits. Glass is green if it only transmits
the green light from white light.
The colour of an opaque body is determined by the colour
of the light it reflects.
All bodies when hot enough produce emission spectra:
1. Continuous spectra are emitted by heated liquids or
solids. These spectra are the same for all bodies and consist
of seven basic coloured bands which continuously merge into
each other.
2. Line spectra are obtained from heated gases or vapours.
Each chemical element has its own line spectrum which differs
from every other spectra in the number of lines, the colour,
and the position of the lines against the background of a
continuous spectrum.
3. Band spectra are obtained from molecules and consist
of a number of bands.
Absorption spectra appear when light passes through a me­
dium which can selectively absorb light and has a lower tem­
perature. The emergence of absorption spectra obeys Kirch-
hofl’s law: a substance mainly absorbs light with the wave­
lengths which it can emit.
§ 23. Radiation and Spectra 261

The thermal radiation of a body at a given temperature is


determined by its radiant exitance R e which equals the
ratio of the radiant flux emitted from a small surface ele­
ment to the area of this surface. Bodies can absorb radia­
tion incident on them. If a body completely absorbs the ra­
diant flux incident on it, it is called a blackbody. According
to the law on thermal radiation, a blackbody has the maxi­
mum radiant exitance.
Stefan-Boltzmann's law. The exitance of a blackbody eb is
proportional to the fourth power of its thermodynamic tem­
perature:
R e = oT4,
where a is the Stefan-Boltzmann constant.
Wien's displacement law. The product of the wavelength
corresponding to the maximum of radiant energy and the
thermodynamic temperature is constant for a blackbody:
= b,
where b is the Wien constant.
Consequently, as the temperature increases, the maximum
of the radiant energy is displaced towards shorter waves.

Worked Problems
Problem 111. Assuming that the temperature of the sur­
face of the Sun is approximately 6000 K, determine the
wavelength corresponding to the maximum energy, consid­
ering that the Sun is a blackbody.
Given: T = 6000 K is the temperature of the Sun’s sur­
face. From tables, we take the Wien constant: b = 2.89 x
10-3 m-K.
Find: the wavelength Xmax corresponding to the maxi­
mum energy.
Solution. The wavelength corresponding to the maximum
radiant energy in the spectrum of a blackbody at a given
temperature can be found from Wien’s displacement law
b --- kmaxT:
<i 2.89 X 10 ^ni'K / O 4 n —7 m
q w
"■max j i ^max — 6000"K —4.82 X 10 m.
Answer. The maximum of radiant energy corresponds to
the wavelength 482 nm.
202 Ch. IV. Optics. Special Theory of Relativity

Questions and Problems

23.1. Where is the velocity of light higher: in diamond or


in water?
23.2. What is the ratio between the velocities of light in
vacuum and in water? How long does it take for light to
travel 225 km in water?
23.3. One standard metre is as long as 1650763.73 wave­
lengths of 'the orange light emitted by krypton-86 atoms in
vacuum. What is the frequency of this radiation?
23.4. Determine the wavelengths for the red and violet
light at the edges of the visible spectrum if they correspond
to frequencies 3.95 X 1014 Hz and 7.5 X 1014 Hz.
23.5. What is the velocity of light in diamond if the
frequency 2.73 X 1014 Hz corresponds in it to a wavelength
of 450 nm?
23.6. The wavelength of blue light in vacuum is 500 nm.
Determine the wavelength corresponding to blue light in
water and its frequency.
23.7. Green light passes from air to water, its wavelength
decreasing thereby. What colour will be perceived by a diver
in the water?
23.8. What will the change in the wavelength of yellow
light of frequency 5.3 X 1014 Hz be as a result of its pas­
sage from glass to vacuum if its velocity in glass is 1.98 X
108 m/s?
23.9. Determine the velocities of light in a transparent
medium for extreme red (800 nm) and violet (400 nm) light
if the refractive indices in this medium for these wavelengths
are 1.62 and 1.67 respectively. What are the frequencies
and wavelengths of these types of light in the transparent
medium?
23.10. As light passes from water to vacuum, its wave
length increases by 0.120 pm. Determine the light’s wave­
lengths in vacuum and in water.
23.11. As light passes from vacuum to a transparent me­
dium its wavelength is reduced by a factor of 1.31. What is
the medium?
23.12. The word “light” is written in green on a sheet of
paper. Through which transparent medium would it be
impossible to read the word?
§ 24. Quantum Properties of Radiation 263

23.13. The star Sirius has a surface temperature of 104 K.


Determine the wavelength corresponding to the maximum
energy emitted by the star.
23.14. If one looks at a bright red object for a while and
then at a white wall, a green silhouette of the object will be
seen. How can this be explained?
23.15. What method was used to study the chemical
composition of the Sun?
23.16. The total energy emitted by the Sun per second is
about E = 4 X 1026 J. Assuming that the Sun is a black-
body, determine its surface temperature.
23.17. The spectral analysis of a nebula has revealed that
it has a continuous spectrum. What conclusion can be drawn
from this?
23.18. The Sun emits about 4 x 102a J of radiant energy
per second. Some data indicate that the radiant exitance of
the Sun has remained constant over the last 3 X 109 years
(1017 s). Determine the mass lost by the Sun per second in
the form of radiation and the mass that has been lost over
the last 3 X 10® years.
23.19. What is the melting point of tungsten if the wave­
length of the radiation corresponding to the radiant energy
maximum lies in the red spectral region of tungsten and is
equal to 784 nm ?§

§ 24. PHENOMENA EXPLAINED BY THE QUANTUM PROPERTIES


OF RADIATION. PHOTOELECTRIC EFFECT

Basic Concepts and Formulas


In addition to its wave properties, visible radiation also
exhibits particle properties. At low frequencies, the wave
properties of electromagnetic radiation are more pronounced,
while at very high frequencies its particle properties domi­
nate.
According to Planck’s theory, radiation is discrete. The
units of electromagnetic radiation are known as quanta, and
the particles carrying energy quanta are called photons.
The main characteristics of a photon are its energy e and
momentum p:
e = h \ = hc/X0,
264 Ch. IV. Optics. Special Theory of Relativity

where h is Planck's constant and k0 is the wavelength of


the radiation in vacuum.
Since photons, unlike other microparticles, move at the
velocity of light, the formula for momentum can be written
in the form
p = me.
According to Einstein’s formula, m = tic2, and hence
p = tic = hvlc or p = h/k0.
The mass of a photon is
m = hv/c2.
Comparing the formulas for energy, momentum, and mass,
we can arrive at the following conclusion: for a monochro­
matic light, all photons of frequency v have the same ener­
gy, momentum, and mass.
Photons at rest do not exist and therefore the rest mass of
a photon is zero.
The photoelectric effect arises during the interaction between
light and a substance. Electrons are detached from the
surface of a substance at the expense of the radiant energy
of photons to which the substance is exposed (the extrinsic
photoeffect).
The laws of the photoelectric effect are:
1. The saturation photoelectric current is proportional
to the luminous flux incident on the surface of the substance.
2. The maximum kinetic energy of the electrons liberated
from an irradiated surface does not depend on the radiation’s
intensity and is given by
2
fflUmax tt
— 2 — = eU,
where umax is the maximum velocity of the electrons, U the
minimum voltage at which no photoelectric current is ob­
served, and m and e are the mass and charge of the electron.
Einstein's relation for the photoelectric effect is

where A is the work function defined as the kinetic energy


which must be acquired by an electron in order to leave
the surface of the substance.
§ 24. Quantum Properties of Radiation 265

The maximum wavelength (the minimum frequency) at


which a photoeffect can be observed is known as the photo­
electric threshold for a given substance.
Equating the kinetic energy of an electron in Einstein's
formula to zero, we can determine the wavelength corre­
sponding to the photoelectric thresholds for different mate­
rials:
he . , he

The operation of photoelectric cells and photoresistors is


based on the application of intrinsic and extrinsic photo­
electric effects. The photoelectric effect is widely used in
various fields.

Worked Problems
Problem 112. A surface element of area 2 cm2 is illumi­
nated for 1 min by a radiation of energy 15 J. Determine the
pressure exerted by the radiation if the surface (a) com­
pletely absorbs the radiation and (b) completely reflects it.
Given: W = 15 J is the radiant energy, t = 60 s is the
time, S = 2 cm2 = 2 X 10~4 m2 is the area on which the ra­
diation is incident. From tables, we take the velocity of
light in vacuum c = 3 X 10® m/s.
Find: the pressure p, exerted by the radiation when it is
completely absorbed and the pressure p.2 when it is com­
pletely reflected.
Solution. The pressure arising as a result of the interac­
tion between radiation and the substance can be found from
the formula
p = - ^ ( 1 + p),

where W0 is the radiant energy per unit time per unit sur­
face area and p is the reflection coefficient.
(a) When the radiation is completely absorbed, p = 0
and

W 15 J
4.2 x 10-® Pa.
Pi Set 2x 10“4 m2 X 3 x 10® m /s x 60 s
X 10-«
266 Ch. IV. Optics. Special Theory of Relativity

(b) In the case of total reflection, p = l, and

P2 = ~ , Pi = 8.4 x 10-6 Pa.

Answer. The pressure produced by the radiation in the


first case is approximately 4.2 X 10~6 Pa, in the second
case, it is double this value.
Problem 113. The electron work function for zinc is
3.74 eV. Determine the radiation threshold of photoelectric
effect for zinc. Calculate the velocity of the electrons emit­
ted by zinc irradiated by ultraviolet light with a wavelength
of 200 nm.
Given: A = 3.74 eV = 3.74 X 1.6 x 10~19 J is the elec­
tron work function, X = 200 nm = 2 x 10~7 m is the
wavelength of incident radiation. From tables, we take
the velocity of light in vacuum c = 3 X 108 m/s, the
Planck constant h = 6.62 X 10~34 J-s, and the mass of
the electron, me = 9,1 x 10~31 kg.
Find: the radiation threshold Xth of the photoelectric
effect for zinc, the maximum electron velocity v.
Solution. The radiation threshold of the photoelectric
effect is the maximum wavelength which can cause the
photoeffect. In this case the kinetic energy of an electron
will be zero, and hence A = fevth = hc/Xtb. Thus

6.62 x 10”34 J -s x 3 X 10* m/s


^lh 3.74 X 1.6 X tO-19 J
- 3.32 x 10-7 m = 332 nm.

The energy hv of ultraviolet radiation photons incident on


the zinc plate is spent on the electron work function A and
on the kinetic energy imparted to the electron: = A +
— From this equation, we can determine the velocity
acquired by the electrons in photoeffect:

he . m ev2 t , / " 2 (he — XA)


~ ~ A= ~2 ’ U= V ’
§ 24. Quantum Properties of Radiation 267

v=

2 (6.62 x 10-31 J -s X 3 x 10s m/s —2 x 10-’ m x 3.74 X 1.6 X lO"1' J)


/= 9.3 X 10s m/s.
9.1 XlO-31 kg X 2 X 10-7 m

Answer. The maximum wavelength of the radiation which


may cause the photoeffect is 332 nm, the maximum velocity
of the knocked out electrons is 9.3 X 10s m/s.

Questions and Problems


24.1. What is the pressure exerted by light on 1 mm2 of
a black surface to which 500 J of radiant energy are trans­
ferred per second?
24.2. The solar pressure on the surface of the Earth is
4.7 X 10~4 Pa. Determine the energy of the radiation inci­
dent per second on a square metre of the Earth's surface
perpendicular to solar rays.
24.3. The number of X-ray photons of frequency 7 X
101B Hz incident per second on a square metre of a black sur­
face is 2.5 X 1015. What pressure does this radiation pro­
duce?
24.4. The yellow light of sodium vapour has a wavelength
of 530 nm. What is the energy of a quantum of this light in
joules and electronvolts?
24.5. What is the ratio of the energy of a photon corre­
sponding to the y-radiation of frequency 3 X 1021 Hz to the
energy of an X-ray photon of wavelength 2 x 10”10 m?
24.6. Determine the energy of a quantum corresponding
to wavelength 10~7 m (in joules and electronvolts).
24.7. How many photons make up 10-8 J of radiation at
wavelength 2 pm?
24.8. Determine the energy, mass, and momentum of an
ultraviolet photon whose wavelength is 360 nm.
24.9. Determine the energy, mass, and momentum of
X-ray photons of wavelength 4 x 10~u m.
24.10. A comet was observed in the sky after sunset. In
which direction was its tail pointing?
24.11. The photoelectric threshold for tungsten corre­
sponds to a wavelength of 405 nm. Determine the work func­
tion for tungsten.
268 Ch. IV. Optics. Special Theory of Relativity

24.12. The work function for cesium is 1.9 eV. Determine


the maximum wavelength of light at which a photoelectric
effect is observed.
24.13. Determine the maximum wavelength of light
which may cause the extrinsic photoeffect from nickel if the
work function for nickel is 4.5 eV.
24.14. The work function for platinum is 6.3 eV.
Will photoeffect be observed for a radiation of wavelength
10-7m?
24.15. Light of wavelength 500 nni is incident on the sur­
face of silver. Will the silver become charged or will it re­
main neutral? If it gets charged, what is the sign of
the charge? The photoelectric threshold for silver is
261 nm.
24.16. The maximum wavelength of radiation that can
produce a photoelectric effect in platinum is 234 nm. De­
termine the maximum kinetic energy acquired by the elec­
trons due to radiation of wavelength 200 nm.
24.17. What is the energy of electrons detached from the
surface of copper irradiated by light of frequency 6 X
1016 Hz if the work function for copper is 4.5 eV?
24.18. What is the velocity of the electrons knocked out
of sodium irradiated by light of wavelength 66 nm? The
work function for sodium is 4 x 10"1# J.
24.19. The photoelectric threshold for a certain metal is
690 nm. Determine the work function for this metal and
the maximum velocity acquired by its electrons due to ra­
diation of wavelength 190 nm.
24.20.1 What is the maximum velocity acquired by photo­
electrons knocked out of molybdenum by radiation of fre­
quency 3 x 1020 Hz? The work function for molybdenum
is 4.27 eV. Is the classical formula applicable in this
case?
24.21. If the surface of a metal is successively exposed to
radiation of wavelengths 350 and 540 nm, the maximum ve­
locities of the photoelectrons will differ by a factor of two.
Determine the metal’s work function.

1 It would be useful to reconsider this problem after studying


the section on special theory of relativity (§ 25).
§ 25. Special Theory of Relativity 269

§ 25. FUNDAMENTALS OF THE SPECIAL THEORY


OF RELATIVITY

Basic Concepts and Formulas


The special theory of relativity (STR) is based on two
postulates.
1. All physical processes in inertial frames proceed iden­
tically and do not depend on the choice of the reference
frame.
2. The velocity of electromagnetic waves in vacuum is the
same for all inertial frames. It does not depend either on
the velocity of the source or on the motion of the observer
(receiver of the light signal).
The postulates of the special theory of relativity are in
contradiction with the concepts of absolute time and space
formed in Newtonian classical mechanics. The STR implies
the following concepts.
The relativity of lengths (distances): the length I of a rod
(body) in a reference frame relative to which it moves will,
according to the STR, be less than the length l0 of the sta­
tionary rod:
i =i0y i - v*/c . 2

The transverse dimensions of a moving body will be the


same in all inertial reference frames.
The relativity of time intervals (relativistic time dilation):
the time t measured in a laboratory system in which an
observer is at rest and the intrinsic time x0 measured using
a clock moving together with the reference frame are relat­
ed as

___ :------ .

If the velocity of a moving reference frame is close to the


velocity of light, the moving clock will lag behind the sta­
tionary clock, i.e. time dilation will occur.
The mass of a body will depend on its velocity: the rest mass
m0 (intrinsic mass) and the mass m of the body moving at a
velocity close to that of light are connected through the
270 Ch. IV. Optics. Special Theory of Relativity

following relation:
m _ ___ _____
_ V i -V */c 2 '
Given that the mass depends on the velocity, we can write
the formula for the momentum of the body in the form
m^v
p = mv ■
V i —fa/e2 '
The relativistic law for velocity composition: the velocity
, of body M relative to a stationary observer in frame K
can be determined from the
formula
Vo "l + v
l + Dii>/e2 ’
where vt is the velocity of the
body M relative to the refer­
ence frame K ' , v is the veloc­
ity of the reference frame K'
relative to the stationary ref­
erence frame K (it is neces­
sary that the x-axis be the
same for the two reference
frames, Fig. 143).
If we assume that vx = v = c, the resultant velocity can­
not exceed the velocity c of light.
The equation relating mass and energy (Einstein’s mass-
energy relation): for any interconversion of matter into
energy, the change in energy is proportional to the change
in mass:
&E = c‘ Am.
Worked Problems
Problem 114. A rocket moves with a velocity equal to
0.6 of the velocity of light in vacuum relative to a stationary
pbserver. What is the change in the length of a 1-m long
steel ruler and the density of the steel of which it is made in
the rocket (in the direction of motion) with respect to the
observer? How much time will elapse according to the clock
of the stationary observer if six years have passed according
to the clock in the moving rocket?
§ 25. Special Theory of Relativity 271

Given: v = 0.6c is the velocity of the rocket relative to


the stationary observer, l0 = 1 m is the proper length of the
ruler, t0 = 6 years is the intrinsic time. From tables we
take the proper density of steel, p0 = 7.8 X 103 kg/m3.
Find: the length I of the ruler, the density p of steel, and
the time t.
Solution. The length of the ruler (along the trajectory of
motion) for the stationary observer, relative to whom the
rocket moves, can be determined from the formula

/ = /oy i - i ; * / c * t l = l m j / " 1 ---- — = 0.8 m.

The density of a substance is expressed in terms of mass


and volume: p = m/V, but V = IS = l0S \^ 1 — i;2/c2. In
this problem, the transverse dimensions do not change, and
hence
0 i , TTIq
P== V i - u 2/ c * l 0S V i —v 2 / c 2 ’ DUl 1 ^ S ~ = P°‘
Therefore
Po___ 7.8 x 103kg/m 3
(K64 1.2 x 104 kg/m3.
P = T —V2/c2 ’
The time for the stationary observer will be dilated and is
defined by the formula
6 years
= 7.5 years.
V 1— OF
Answer. The length of the ruler for the stationary observer
is 0.8 m, the density of steel is 1.2 x 104 kg/m3, and the
time period is 7.5 years.
Problem 115. The rest energy of a proton is approximately
938 MeV. Determine the rest mass of the proton and the
mass and velocity of protons to which a kinetic energy of
70 GeV has been imparted in an accelerator.
Given: E0 = 938 MeV is the rest energy of the proton,
E k = 70 GeV is the kinetic energy of a proton acquired in
the accelerator. From tables, we take the velocity of light
in vacuum c = 3 x 10® m/s.
Find: the rest mass mPo of the proton, the mass mp of the
proton after the acceleration, and the velocity v of the pro­
ton as a result of the acceleration.
272 Ch. IV. Optics. Special Theory of Relativity

Solution. We first recalculate the proton’s energy in SI


units (joules) 1 eV = 1.6 x 10~19C x I V = 1.6 x 10~19J,
1 MeV = 1.6 X 10~u J. Consequently, the rest energy is
E 0 = 938 x 1.6 x 10-13 J = 1.5 x 10-10 J.
The kinetic energy of the proton after acceleration is
Ek = 7 X 1010 X 1.6 X 10-19 J = 1.12 x 10-8 J.
In order to determine the rest mass of the proton, we shall
use Einstein’s relation E 0 = mP()c2:
h . 1.5 X 10-10 J
m mT
Po 9 X 10l* mVs9 = 1.67 x 10'27 kg.
The total energy of the proton can be determined from
the formula E = Ek -[- E0 = mpc2. Dividing the two sides of
the equality by E0 = mp cz, we obtain — — = — , or
0 m p0

— = -^ - + 1, whence
7>0 Eo

rn P = ( !■ + 1) 1.67 x 10"27 k g = 1.26 x 10-25 kg.

Having determined the mass of the proton after the accel­


eration, we can find its velocity:

_Tl« —
T . po ■■ V 1 - vVc* = , 1
p ]/l— Fflp

whence

V= c 1.67 xlO-27 kg ) = 0.99c.


/ * - ( 1.26xl0-« kg

Answer. The rest mass of the proton is 1.67 X 10~27 kg


(which corresponds to the tabulated value). The mass of the
proton after the acceleration is nearly 75 times larger than
its rest mass, and the velocity is equal to 0.99 of the ve­
locity of light.
Problem 116. Two rockets move towards each other each
at a velocity of 0.8c relative to a stationary observer. De-
§ 25. Special Theory of Relativity 273

termine the velocity with which the rockets approach each


other according to (a) classical mechanics and (b) the theory
of relativity.
Given: vx = v2 = 0.8c are the velocities of rockets relative
to the stationary observer on the Earth.
Find: the velocity uc\ at which the rockets approach each
other according to classical mechanics and ureI, the veloc­
ity according to relativity, as well as the difference Au in
these velocities.
Solution. According to classical mechanics, we have
ucl = Tj v2, ucl = 0.8c + 0.8c = 1.6c.
According to relativistic mechanics, we have
+ 1.6c
U,.ei i ^Tel — 0.64c2
0.976c.
1 Vl»2 1

The difference in these velocities is


Au = 1.6c — 0.976c = 0.624c.
These calculations show that the classical formula for
composing velocities cannot be used when bodies move at
velocities close to the velocity of light since this contradicts
the relativity’s postulate that the velocity of light in va­
cuum cannot be exceeded.

Questions and Problems


25.1. Compare the lengths of two metre rods moving in
their longitudinal directions with velocities 0.5c and 0.75c
relative to a stationary observer.
25.2. At what velocity of a body relative to a stationary
observer is its length equal to 0.8 of its proper length?
25.3. At what velocity should a body move relative to a
stationary observer for its size along the line of motion to
contract by a factor of two? Will the size change to an ob­
server moving together with the body?
25.4. A rocket moves at a velocity of 0.866c relative to a
stationary observer. The width of a plane rectangle arranged
along the line of the motion of the body is equal to half its
length. How will the rectangle appear to the stationary ob­
server?
18-0530
274 gh. IV. Optics. Special Theory of Relativity

25.5. The radius of an electron at rest is 2 x 10"13 cm.


What will the contraction in the electron’s radius be in the
direction of motion at 0.8 of the velocity of light?
25.6. The flight time measured in a rocket moving at a
velocity 0.96c is one year. How much time will have
elapsed for a terrestrial observer?
25.7. How much time will have elapsed on the Earth if
six years have passed in a rocket moving at a velocity of
2.4 x 10® m/s relative to the Earth?
25.8. The distance to the star in the Centaurus constella­
tion nearest the Sun is traversed by light in approximately
4.25 years. Express this distance in astronomical units and
in kilometres. Determine the flight time (by a clock in a
rocket) to this star and back in the rocket if the velocity of
the rocket relative to the observer is 0.99c, and 1 AU ~
150 million kilometres.
25.9. What will be the duration of a flight to a star by a
rocket flying at a velocity of 0.9c for a cosmonaut and for an
observer on the Earth if the distance to the star is 40 light
years?
25.10. One kilogram of water is heated by 80 K. What is
the increase in the mass of water?
25.11. What must the velocity of a particle be at which
its kinetic energy is equal to its rest energy?
25.12. What is the velocity of an electron if its mass is
four times its rest mass?
25.13. Determine the mass of an electron moving at veloc­
ities of 50% and 90% of the velocity of light.
25.14. What must the velocity of a proton be in an ac­
celerator for its mass to be 5% greater?
25.15. A body with a rest mass of 5 kg and density of
7.8 X 103 kg/m3 is in a rocket moving relative to a terres­
trial observer at a velocity of 2.4 x 10s km/s. Determine the
relativistic mass and density of the body.
25.16. A rocket moves at a velocity close to that of light
relative to a stationary observer. By what factor will the
mass and density of bodies in the rocket change for an ob­
server flying in it?
25.17. What must the velocity of a body be for its densi­
ty to be five times larger?
25.18. Express the rest energy of an electron (positron)
in megaelectronvolts. What will the radiant energy emitted
§ 25. Special Theory of Relativity 275

as a result of the annihilation of an electron and a positron


be?
25.19. Determine the momentum of an electron moving at
0.6 of the velocity of light.
25.20. Two aeroplanes fly towards each other at velocities
500 and 400 m/s. What is their relative velocity?
25.21. Two particles move towards each other at a ve­
locity (5/8)c each. What will their approach velocity be as
calculated using the classical and relativistic formulas?
Chapter V

Physics of Atomic Nucleus

§ 26. STRUCTURE OF ATOMIC NUCLEUS. ATOMIC ENERGY


AND ITS APPLICATION

Basic Concepts and Formulas


According to the nuclear model of atoms proposed by Ru­
therford and developed by Bohr, an atom consists of a pos­
itively charged nucleus and electrons revolving about it.
The nucleus contains protons and neutrons to which the
general term “nucleons” is applied.
A nucleus occupies a very small volume as compared to
the atom. If we assume that the atomic nucleus is a sphere,
the radii of the nuclei of different elements can be determined
by the formula
r = 1.4 x 10- UV A ,

where A is the mass number of an element.


The density of matter in a nucleus is of the order of 1.3 X
101T kg/ms. The mean density of nuclear matter is calcu­
lated from the formula
m A

P _ (473jii^ ’

where m is the mass of a nucleon and r is the radius of the


nucleus.
The positive charge of a nucleus is determined by the
product of the atomic number Z of the element in the Pe­
riodic Table and the elementary charge, and hence depends
on the number of protons in the nucleus.
The mass of a nucleus depends on the number of nucleons
in it. The unit of mass of an atom, viz. atomic mass unit
(amu), is equal to 1/12 of the mass of the carbon atom '£0.
The atomic mass unit can be expressed in SI units:1
1 amu = 1.66057 x 10~a7 kg.
§ 26. Structure of Atomic Nucleus 277

In order to determine the mass of an atom, it is usually


sufficient to use the mass number of the element:
m, = A x 1.66057 x lO"27 kg.
The nuclei of atoms of chemical elements are denoted by
zX , where X is the symbol of an element, A is the mass num­
ber, and Z is the atomic number of the element, which is
equal to the number of protons in the nuclei. The particles
are designated as follows: _[e is the electron, \e is the posi­
tron, *n is the neutron, \p is the proton (the nucleus of a
hydrogen atom is |H), and JHe is the a-particle.
The mass of an atomic nucleus is less than the sum of the
masses of the nucleons by a quantity known as the mass
defect:
Am = Zmp + Nmn — tox ,
where mx is the mass of the nucleus, mp is the mass of the
proton, mn the mass of the neutron. The mass ma of the atom
is taken from the Periodic Table. Then
mx = ma — Zmd.
Nucleons are confined in a nuclei by nuclear forces which
considerably exceed the electrostatic (repulsive) forces. For
this reason, when a nucleus is split, a certain amount of
energy is required to overcome the nuclear forces. When
nucleons combine to form a new nucleus, the energy known
as the binding energy is liberated.
The relation between the binding energy and mass defect
is given by Einstein’s relation
AE = Ame1.
A mass defect of 1 amu corresponds to a binding energy of
931.5 MeV.
Radioactivity (viz. the spontaneous disintegration of the
nuclei of some isotopes with the emission of a- and (1-par­
ticles and yrays) is a phenomenon that confirms the com­
pound structure of nuclear atoms.
Three principal types of radiation are emitted during ra­
dioactive decay (Fig. 144), a-radiation which consists of
helium nuclei jHe (the mass number after an a-decay decreases
by 4 and the charge by two units), ^-radiation which
consists of electrons whose velocities are close to the veloc-
278 Ch. V. Physics of Atomic Nucleus

ity of light (an electron is produced during P-decay as a


result of the transformation of a neutron into a proton, and
hence the positive charge of the nucleus increases by one, and
the mass number remains unchanged), and y-radiation which
accompanies a- and p-decay. The emission of a y-quantum
does not involve any change in the mass number or charge.
The rules for the nuclei formed during radioactive decays
are
for an a-decay, ^X, -► i Z z X 2+ JHe,
for a p-decay, ^X, l+ iX 2-f- _Je.

The law of radioactive decay states that the number AN of


nuclei undergoing u radioactive decay during the time in­
terval from t to t -(- AMs propor­
tional to the number N of nuclei
available at time t and to the length
of the time interval At:
AN = —XN At,
where X is a decay constant which
characterizes the rate of radioactive
decay for a given species of nuclei.
The minus sign indicates that the
number of nuclei decreases during
the decay process.
In order to characterize the sta­
bility of a nucleus, the concept of a half-life is introduced.
The half-life T1/2 is the time required for half the nuclei ca­
pable of decaying to do so:
„ In 2 0.693
“T

Worked Problems
Problem 117. The radioactive waste from nuclear power
plants contains the radioactive strontium isotope ” Sr which
has a half-life of 28 years. Over what period will the amount
of strontium decrease by a factor of four?
Given: ™Sr is the strontium isotope, and Tl/t = 28 years
is its half-life.
§ 26. Structure of Atomic Nucleus 279

Find: the time t during which the amount of strontium


decreases by a factor of four.
Solution. The amount of decaying nuclei as a function of
the half-life is represented by the curve in Fig. 145. We
conclude from this graph that the amount of strontium
nuclei will decrease to one quarter
of the initial value in 2 X 28 years =
56 years.
Problem 118. What fraction of
radioactive cesium *” Cs with a
half-life of 30 years will decay in a
year? Determine the decay constant.
Given: ‘jJ’Cs is the radioactive
cesium isotope, = 30 years is
the half-life, and t = 1 year is the
time.
Find: the fraction AN/ N0 of de­
cayed nuclei and the decay constant.
Solution: We use N 0 to denote the initial number of atoms
and N to denote the number of atoms remaining after time t.
Then we can write AN = N 0 — N, where AN is the number
of atoms that have decayed over the time t.
According to the law for radioactive decay, N = Ar0c(-X<),
e = 2.718... being the base of Napierian logarithm.
Assuming that the time t is small in comparison with the
half-life T, we can use the approximation

A N = - ^ - N 0t,
or
AN _ 0.093 AN 0.693 X 1 year
30 years
0.023.
N„ ~ T N0

The decay constant k characterizes the rate of the radio­


active decay and can be determined from the formula
, 0.693 , 0.693
~ T ' K ~ 30 x 365 X 24 x 3600 s 7.3 x 10"‘° s~‘.
Answer. The fraction of cesium nuclei that decays over a
year is approximately 2.3% , the decay constant is 7.3 x
1 0 -io g - i >
280 Ch. V. Physics of Atomic Nucleus

Problem 119. Determine the compositions of lithium nuc­


lei, hydrogen isotopes with mass numbers 1 and 2, and
uranium isotopes with mass numbers 235 and 238.
Given: JLi is the nucleus of a lithium atom, |H is the nu­
cleus of a hydrogen atom, *H is the nucleus of a heavy hy­
drogen atom (deuteron), and and 2“ U are the nuclei of
uranium isotopes.
Find: the number Z of protons and the number N of neu­
trons in the nuclei of the indicated elements.
Solution. The number of protons in a nucleus is deter­
mined by the charge number which is equal to the atomic num­
ber of the element in the Periodic Table. The charge number
is the subscript to the left of the element symbol. The su­
perscript is the mass number A that is equal to the number
of nucleons in the nucleus: A = Z + N. Therefore, the
number of neutrons is
N = A - Z.
For lithium ’Li, Z = 3 and N = 7 — 3 = 4.
For ordinary hydrogen (,'H), Z = 1, and for heavy hydro­
gen <JH), Z = 1, N = 1.
For 23‘U, Z = 92, N = 146.
For *“ U, Z = 92, N = 143.
In this problem, hydrogen and uranium are represented by
two isotopes. The isotopes of a chemical element have the
same number of protons and different number of neutrons.
Problem 120. The nucleus of the magnesium isotope with
a mass number 25 is bombarded by protons. The nucleus of
which element is formed if the nuclear reaction is accompa­
nied with the emission of a-particles?
Solution. In the Periodic Table, we find the atomic number
of magnesium (12) and write the nuclear reaction
” Mg + ; H - ^ X + ;He.
Since charge is conserved, the sum of the charge numbers on
the left-hand side must be equal to the sum of the charge
numbers on the right-hand side, viz. 1 2 - 1 - 1 = 2 + 2. Hence
the charge number Z of the unknown element is 11. The
eleventh site in the Periodic Table is occupied by sodium.
Since mass (the mass number) is conserved, the sums of
the mass numbers on the left- and right-hand sides must be
§ 26. Structure of Atomic Nucleus 281

equal (here this sum is 26). Consequently, we obtain a sodi­


um isotope with a mass number of 22.
In its final form, the reaction equation will be written as
“ Mg + JH “ Na + £He.
Problem 121. As a result of bombardment of aluminium
by a-particles, a new nucleus and a neutron are formed.
Write the nuclear reaction and identify the element whose
nucleus is formed.
Solution. We shall write the nuclear reaction. On the left-
hand side, we write the initial quantities, while the right-
hand side must contain the unknown nucleus and a neutron:
SAI-f- *He — zX + Jra.
Equating the charge and mass numbers on the left- and
right-hand sides (as in Problem 120), we conclude that the
new nucleus is that of phosphorus. In its final form, the reac­
tion is
S A i+ ;H e -> ;;p + ;» .
Problem 122. Determine the mass defect and the binding
energy of the nucleus of a nitrogen atom. What is the
binding energy per nucleon?
Given: “N is the nucleus of the nitrogen atom. From ta­
bles, we take the rest mass of a neutron mn = 1.00867 amu,
the mass of the nitrogen atom mN = 14.0067 amu, the mass
of a hydrogen atom mH = 1.00797 amu, the velocity of
light in vacuum c = 2.99792 X 108 m/s, and 1 amu =
1.66056 x 10-27 kg.
Find: the mass defect Am, the binding energy AE of the
nucleus of a nitrogen atom, and the binding energy per nu­
cleon AE/A.
Solution. The mass defect is the difference between the
sum of the rest masses of the free protons and the neutrons
constituting a nucleus on one hand and the mass of the
nucleus on the other, i.e.
Am = Zmp + Nmn — mx ,
where mx is the mass of the nucleus, mx = m,\| — Zme.
The expression for the mass defect can be considerably
simplified if instead of the sum of the masses of protons we
282 Ch. V. Physics of Atomic Nucleus

take the sum of the masses of hydrogen atoms:


Am = ZmH + Nmn — mN.
Substituting in the numerical values, we obtain
Am = 7 x 1.00797 amu + 7 x 1.00867 amu
— 14.0067 amu ~ 0.10978 amu,
Am = 0.10978 x 1.66 x 10'27 kg ~ 1.822348 x 10"28 kg.
The binding energy can be determined using Einstein’s
relation
AE = Ame2,
AE = 1.822 x 10-28 kg (2.9979 m/s)2 = 1.638 X 1 0 '11 J.
In atomic physics, the binding energy is expressed in me-
gaelectronvolts:
M eV -102.4 MeV.

Considering that 1 amu corresponds to the energy approx­


imately equal to 931.4 MeV, we can determine the binding
energy in a simpler way:
A£ = 0.10978 x 931.4 MeV ~ 102.25 MeV.
The results of calculations clearly differ insignificantly.
The binding energy per nucleon is
&E/A = 102 MeV/14 ~ 7.3 MeV/nucleon.
The result is in agreement with the tabulated data.
Answer. The mass defect is 0.1097 amu, the binding ener­
gy of the nitrogen nucleus is approximately 102 MeV, and
the binding energy per nucleon is 7 MeV.
Problem 123. The thermal power of each reactor installed
at the Leningrad nuclear power plant is 3200 MW. The elec­
tric power is 1000 MW. The mass of the uranium charge in
a reactor is 180 t. Determine the efficiency of the unit, the
mass of the uranium-235 consumed by the reactor over a
year of continuous operation at the total power. What
fraction of the total charge does the mass of the consumed
uranium constitute?
Given: P t = 3.2 X 10® W is the thermal power of the re
actor, Pc = 10® W is the electric power of the unit, t —
§ 26. Structure of Atomic Nucleus 283

1 year = 3.15 X 107 s is the time for which the nuclear fuel
consumption is being determined, m = 1.8 X 10s kg is the
mass of the uranium charge of a reactor. From tables, we take
the Avogadro constant N A = 6.02 x 1023 mol-1, the molar
mass of uranium M = 235 x 10~3 kg/mol, and the energy
liberated as a result of the fission of a 235U nucleus, E =
200 MeV.
Find: the efficiency r) of the power unit, the mass mt of
uranium consumed and the fraction m j m of the consumed
uranium.
Solution. The efficiency of a unit is defined as the ratio of
the electric power of the unit to the thermal power of a
reactor:
,, = ^ -100% , 7) = w 10096 = 31.3%.
In order to determine the mass of consumed uranium, we
must find the number of 235U atoms entering into the nuclear
reaction over the time of operation of the reactor. Since
about 200 MeV of energy is liberated during the fission of
a 23SU nucleus and that 190 MeV are converted into heat,
we can write N = P lt/El, where Ex = 190 MeV. Given the
number of atoms in the reaction and the mass of a uranium
atom, mv = M/ NA, we can determine the mass of con­
sumed uranium:
nr or
mv = mvN, M - Pit
j—,
235 X 10-3 kg -m o l"1 X 3.2 X 10» Wx 3.15x10’ s _ , of), .
~ G.02 x 10“ m o l'1 x 190 x 1.6 x 10"1* J 1<55Wl Kg'
Let us determine the fraction of the total charge con­
sumed over a year:
mi 1294 kg
m 0.007.
1.8 X 10* kg
Answer. The efficiency of the plant is about 31%, the
amount of uranium-235 consumed during a year of operation
is about 1300 kg, which is less than 1% of the total.

Questions and Problems


26.1. What is a-radiation? Why is it deflected less in a
magnetic field than ^-radiation?
26.2. What force deflects a- and fl-rays in a magnetic field?
284 Ch. V. Physics of Atomic Nucleus

26.3. Which of the three radiations (alpha, beta, and


gamma) is not deflected by either magnetic or electric
fields?
26.4. What is y-radiation? What is the difference be­
tween this radiation and X-rays?
26.5. How many electrons occupy the electron shell of a
neutral atom whose nucleus contains six protons and six
neutrons?
26.6. The atomic nucleus of any chemical element con­
sists of protons and neutrons. How can the emission of p-ra-
diation be explained?
26.7. Determine the half-life of radon if 1.75 X 10s out
of 10® atoms decay per day. What is the decay constant equal
to? (Use the approximate formula.)
26.8. The decay constants for bismuth-209 and poloni-
um-210 are 1.6 X 10'® s ' 1 and 5.8 X 10-8 s"1 respectively.
Determine their half-lives.
26.9. What fraction of the radioactive nuclei of ‘JC de­
cays over 100 years if its half-life is 5570 years?
26.10. It is known that 9.3 x 1018 out of 2.51 x 1019
available atoms of the sodium-24 isotope undergo p-decay.
The half-life is 14.8 h. Determine the decay time and con­
stant using the approximate formula (see Problem 118).
26.11. The uranium isotope 2jJ8U of 1-g mass emits 1.24 X
104 a-particles per second. Determine the half-life and decay
constant of the isotope.
26.12. Determine the composition of hydrogen JH, heli­
um 4He, aluminium ” A1, uranium SJ8U, and neptunium
28’Np nuclei. What can be said about the neutron content
of nuclei with increasing atomic number?
26.13. What is the difference between the nuclei of the
chlorine isotopes 3t5Cl and 32C1? How can you explain the
fact that chlorine has a relative atomic mass of 35.5 in the
Periodic Table?
26.14. The nucleus of which element contains 14 pro­
tons and the same number of neutrons? Which of the first
twenty elements in the Periodic Table have nuclei contain­
ing equal numbers of protons and neutrons?
26.15. Determine the charges of the lithium, copper, and
uranium-238 nuclei in coulombs.
26.16. One gram of radium emits 3.7 X 1010 a-particles
§ 26. Structure of Atomic Nucleus 285

per second. Determine the charge of this radiation in cou­


lombs.
26.17. Determine the radius and nuclear density of he­
lium and uranium-238 atoms.
26.18. Write the reaction for the direct transformation of
actinium-227 into francium-223. What type of a radioactive
decay is it?
26.19. What will happen to the uranium-237 isotope dur­
ing P-decay? What will the mass number of the new ele­
ment be? To which side of the Periodic Table will the nuc­
leus be shifted? Write the reaction equation.
26.20. A beryllium nucleus is formed by the reaction
between a lithium nucleus and a deuteron. What particle is
liberated in the process? Write the nuclear reaction equa­
tion.
26.21. The “age” of the objects discovered during an arche­
ological dig is determined from the isotopes of a certain ele­
ment the objects contain. Determine the charge and the
mass number and identify the element from the following
nuclear reaction:

■ J N + > -* 1 X + {p.
26.22. What are the resultant nuclei after the a- and
P-decay of xenon?
26.23. Two y-quanta are formed by the annihilation of
an electron and a positron. Assuming that the masses of the
electron and the positron are the same, determine the energy
of the y-radiation and its frequency.
26.24. The presence of explosives in the luggage of air
passengers can be detected using nuclear physics. An explo­
sive normally contains nitrogen isotopes with mass num­
bers 14 and 15. As a result of bombardment by neutrons,
nitrogen isotopes with mass numbers 15 and 16 are formed.
The latter isotope is radioactive and emits y-quanta that
can be detected. Write the equation of the nuclear reaction.
26.25. What must the energy of a y-quantum be for it to
be convertable into an electron-positron pair?
26.26. It is established that a proton is emitted during
the bombardment of an aluminium isotope by helium nuclei
with the formation of a new nucleus. Write the equation for
the nuclear reaction and identify the new nucleus.
286 Ch. V. Physics of Atomic Nucleus

26.27. When boron “ B captures a fast proton, three


almost identical tracks spreading in different directions are
formed in a Wilson cloud chamber where the process takes
place. What particles are responsible for these tracks?
26.28. Identify the particle denoted by the question
mark in the nuclear reaction 'Li + ? -►'JB + '0n.
26.29. What nuclei and particles are formed as a result of
the following reactions:
?;Pu + JHe
Ih + y -*>|h + ?
26.30. The transformation of phosphorus J“P into silicon
™Si is accompanied by the emission of a positron. WhaL
changes occur in the nucleus?
26.31. Determine the mass defect of a lithium nucleus in
atomic mass units and in kilograms.
26.32. Determine the mass defect of the boron nucleus
*°B in atomic mass units and in energy units.
26.33. Determine the mass defect and the binding energy
for the uranium-238 nucleus.
26.34. Analyze the following nuclear reactions and de­
termine whether the energy is liberated or evolved:
JHe + ®He
JBe + JH
|L i + JH
26.35. Determine the energy liberated in the nuclear re­
action ®Li + JH ->-,JHe + ®He (see Table 25).
26.36. In order to obtain 1 GW of electric power, 2 X
10® t of coal have to be burnt annually, which involves the
discharge of 8 X 103 t of ash and tens of thousands of tons
of sulphur dioxide into the atmosphere. How much uranium-
235 is required to obtain the same power for the same ef­
ficiency?
26.37. The fission of a uranium-235 atom into two frag­
ments is accompanied by the liberation of about 3 X 10"11 J
of energy. How much petrol has to be burnt to obtain the
same energy as that liberated in a nuclear reaction in which
1 g of uranium is consumed?
26.38. Calculate the energy liberated by the combustion
of 1120 t of A-l grade coal, of 376 t of petroleum, of
§ 26. Structure of Atomic Nucleus 287

5 X 105 m3 of natural gas and the energy generated by the


fission of 260 g of uranium-235.
26.39. The efficiency of the reactors installed at the Kolsk
and Rovensk nuclear power plants is 32%. How much ura­
nium 235 is consumed in a nuclear reactor per hour if its
electric power is 440 MW?
26.40. How much energy is liberated in the nuclear re­
actors of the nuclear-powered icebreaker Lenin if the daily
consumption of uranium-235 is 62 g?
26.41. The thermal reactors installed at atomic power
plants (operating on slow neutrons) do not utilize the nuclear
fuel very efficiently and cannot ensure the required scale of
production of nuclear power. The situation is different for
reactors on fast neutrons. Why?
26.42. Which of the particles listed below are stable: a
photon, an electron, a neutrino, a proton, a neutron, or a
n-meson?
26.43. The annihilation of a proton and an antiproton
generates y-radiation. Calculate the energy of the photons
if the masses of the proton and the anti proton are the same
and equal to 1.67 X 10~27 kg.
26.44. Controlled nuclear fusion would be a prodigious
way of obtaining energy. As much energy is liberated by the
fusion of the deuterium contained in one litre of ordinary
water as by combustion of 350 i of petrol. Calculate this
amount of energy.
26.45. If 5 X 104 kg of hydrogen are fused into 49 644 kg
of helium, how much energy is liberated?
Chapter VI

General Remarks on Astronomy

§ 27. FUNDAMENTALS OF ASTRONOMY

Basic Concepts and Formulas


Astronomy facilitates a deeper understanding of the physics
of the world and extends dialectical and materialistic ideas
about matter and various forms of its existence. The devel­
opment of cosmonautics has broadened the means for in­
vestigating the processes occurring in the Universe and the
way they influence the life on our planet.
The celestial sphere is defined as having an arbitrary ra­
dius with the centre at the point of observation, onto which

the celestial bodies are being projected. The celestial axis


P P X is the axis of the apparent rotation of the celestial
sphere. The points where the celestial sphere intersects the
celestial axis are known as celestial poles (Fig. 146).
The diurnal rotation of the celestial sphere makes it
possible to determine the constellations which set at a giv­
en geographical latitude and those which do not set. For
§ 27. Fundamentals of Astronomy 289

example, the constellation of Ursa Minor does not set at


the latitude of Moscow. The brightest star in this constella­
tion, Polaris, is very close to the north celestial pole. Since
the geographical latitude of an observation point can be
determined from the angular distance from the plane of the
horizon to a celestial pole, the geographical latitude in the
northern hemisphere can be determined from the altitude
of Polaris.
The ecliptic is the apparent annual path of the Sun on
the celestial sphere. The ecliptic intersects the celestial
equator at points known as vernal equinox and autumnal
Z e n iih
Z

N a d ir
Fig. 149

equinox (Fig. 147). The most remote points from the equa­
tor are passed by the Sun on the 22nd of June and on 22nd
of December, which are known as summer and winter sol­
stices respectively. The ecliptic passes through twelve con­
stellations of the Zodiac, namely Pisces, Aries, Taurus,
Gemini, Cancer, Leo, Virgo, Libra, Scorpio, Sagittarius,
Capricorn, and Aquarius.
The position of a star on the celestial sphere is deter­
mined by two coordinates: its declination 6 and right ascen­
sion a (Fig. 148).
The large circle of the celestial sphere passing through the
celestial poles and a given star is known as the declination
circle, while the angular distance from the celestial equator
to the star, measured along the declination circle is known
as the declination of the star. Declinations are positive to
19-0530
290 Ch. VI. General Remarks on Astronomy

the north of the equator and negative to the south of it.


The declination of the star is similar to the geographical
latitude. The right ascension is measured along the celestial
equator from the vernal equinox to the declination circle
passing through the given star. This coordinate is similar
to the geographical longitude and expressed in units of
time.
The passage of a star through the meridian is known as its
culmination. Each star passes through the meridian twice a
day. The apparent noon is the upper culmination of the cen­
tre of the solar disc, while the apparent midnight corresponds
to its lower culmination.
The altitude h of a star above the horizon at its upper cul­
mination can be determined from the formula (Fig. 149)
h = 90° — <p + 8,
where (p is the geographical latitude. In addition to the Sun
and planets, the Solar System contains asteroids (small
planets), comets, and meteoric dust.
Our star, the Sun, is an ordinary star in the Milky Way
(Galaxy). Its diameter is 1390000 km, i.e. 109 times the
diameter of the Earth and its mass is 333000 times that of
the Earth.
The hot gaseous sphere of the Sun consists of 85% of hy­
drogen and 13% of helium. Many other elements are present
in small amounts. The temperature at the centre of the Sun
is about 2 X 107 K and the pressure is of the order of 2 X
107 GPa. Under these conditions, fusion reactions take
place, which are the source of the solar energy.
The motion of celestial bodies and their gravitational
attraction obey the laws of physics.
Kepler's first law states that all planets revolve around
the Sun along elliptical orbits with the Sun as one of the
foci.
Kepler's second law: the radius vectors of a planet drawn
from the Sun sweep equal areas in equal times (Fig. 150).
Kepler's third law: the squares of the times taken by two
planets to describe their orbits are proportional to the cubes
of the semimajor axes of the orbits:
§ 27. Fundamentals of Astronomy 291

Newton refined Kepler’s third law for comparing the masses


of celestial bodies:
w i + w , r \2 _ RJi2
ma + m* T §4 flg4 ’
where m2 and m3, m4 are the masses of two pairs of celes­
tial bodies revolving one around the other, and T lJi ^34'
fl12, and R 3i are respectively the periods of revolution and
the mean distances between the bodies.
Newton's law of universal gravitation: all bodies in the
Universe are attracted to one another with a force propor­

tional to the product of their masses and inversely propor­


tional to the squared distance between their centres of mass:
p _^
r2 '
where G is the gravitational constant. The distances to ce­
lestial bodies are determined by parallax. The horizontal
parallax p is the angle at which the radius of the Earth per­
pendicular to the line of sight is seen from a celestial body
(Fig. 151):

19»
292 Ch. VI. General Remarks on Astronomy

The annual parallax of a star is the angle at which the ra­


dius of the Earth orbit perpendicular to the line of sight D
is seen from the star (Fig. 152):
D = -^ — .
sin p

The unit of length in astronomy is the astronomical unit


(AU), 1 AU = 149.6 X 106 km is the mean distance be­
tween the Sun and the Earth. When discussing stars and
galaxies, the units of length are the light year and parsec (pc),

1 pc = 206265 AU = 30.86 X 1012 km. One light year is


the distance traversed by light in one year: 1 light year =
0.3068 pc.
When solving some of the problems, the star chart should
be used. To do this, trace the circle on the flyleaf of the
book. It should be cut along the line corresponding to the
geographical latitude of the site of observation. By apply­
ing the circle to the star chart on the front flyleaf, you will
obtain the celestial map for the required month and hour of
observation. For example, to obtain the star chart at 10 pm,
on October 10, it is necessary that 10 pm in the cut circle
be matched with the mark for October 10 on the chart. In
the slot of the circle you will have all the celestial bodies
visible at that moment.

Worked Problems
Problem 124. Determine the ratio of the mass of the Sun
to the mass of the Earth if the period of revolution of the
Moon round the Earth is 27.2 days and the mean distance
from the Earth to the Moon is 384000 km.
§ 27. Fundamentals of Astronomy 293

Given: r Moon = 27.2 days is the period of revolution of


the Moon round the Earth, aMoon = 3.84 X 105 km is the
mean distance from the Earth to the Moon, and T’eartii =
365 days is the period of revolution of the Earth round the
Sun. From tables, we take the mean distance from the Earth
to the Sun aEarth = 1.5 X 108 km.
Find: the ratio between the mass of the Sun and the mass
of the Earth, Misun/^Earlh*
Solution. We shall use the formula for the relined ver-
„3
"Sun + mE arth ^ E arth “E arth
sion of Kepler’s third law:
mEarth ' mM ooir Moon “ Moon
Since the mass of the Earth relative to that of the Sun
and the mass of the Moon relative to the mass of the
Earth are negligibly small, we can rewrite the formula as
„3
Sun “ Earth
follows: msun/l arth = 4 ^ - Hence mc
mEarth^M oon “ Moon mEartli Moon
T2
1 Moon
7*2
1 Earth
Substituting in the numerical values, we obtain
mSun (1.5 XlO 8 km )3 (27.2 days)2 qqf)000
mEartb (3.84 X 10‘ km)3 (365 days)2
Answer. The ratio between the masses of the Sun and Earth
is about 330000.
Problem 125. Determine the mean distance between the
Earth and the Moon using the following data: (1) the hori­
zontal parallax of the Moon is p = 0.57', and (2) an elec­
tromagnetic signal sent to the Moon from the Earth returns
in 2.56 s. What is the mean velocity of motion of the Moon
round the Earth if the sidereal month is 27.3 days long?
Given: p = 0.57' is the horizontal parallax of the Moon,
t = 2.56 s is the time required for the electromagnetic sig­
nal to traverse twice the distance from the Earth to the
Moon, T = 27.3 days = 27.3 X 3600 x 24 s is the sidere­
al month. From tables, we take the mean radius of the
Earth ifEartt, = 6370 km and the velocity of light c =
3 x 105 km/s.
Find: the distance d from the Earth to the Moon and the
mean velocity v of the Moon in its orbit.
Solution. 1. The horizontal parallax of the Moon is defined
as the angle at which the radius of the Earth is seen from
294 Ch. VI. General Remark's on Astronomy

the Moon (see Fig. 151). Consequently, .


Here it is more convenient to express the parallax in degrees:
p = 0.57' = 0.95°. Then
360° X 6370 km
d= 0.95° X 6.28
^ 384380 km.
2. A radio signal sent to the Moon will be reflected by its
surface and return to the Earth. We know the time in which
the radio signal covers twice the distance from the Earth to
the Moon. Therefore,
ct 300000 km /s X 2.56 s
d ~ 384000 km.
T , d 2
In order to determine the mean orbital velocity of the
Moon around the Earth we use the formula v = 2ndlT, where
T is the sidereal month, i.e. the time taken by the Moon
to complete one revolution around the Earth relative to the
stars. Substituting the numerical values, we obtain
6.28 x 3.84 x 105 km
v ~ 1.02 km/s.
27.3 X 24 X 3600 s
Answer. The approximate distance between the Earth and
the Moon given by the two methods are (1) 384380 km and
(2) 384000 km. The mean orbital velocity of the Moon is
1.02 km/s.
Questions and Problems
27.1. How long does it take light to travel from the Sun
to the Earth? The distance from the Sun to the Earth is
149.6 x 10® km, and the velocity of light is 2.998 x
105 km/s.
27.2. Determine the distance to a Centauri (the nearest
star to the Solar System) if its light takes 4.25 years to
reach the Earth. Express the distance in kilometres and
parsecs.
27.3. The mean distance from the Sun to the farthest
planet Pluto is 40 AU. How long does it take light to cover
this distance?
27.4. It takes 2 X 10® years for light to travel from the
nearest to us galaxy in Andromeda. Express this distance
in parsecs.
27.5. List the brightest stars, whose magnitudes are ex-
§ 27. Fundamentals of Astronomy 295

pressed in negative numbers.


27.6. What constellation includes Sirius? What does its
negative declination indicate?
27.7. The star Sirius is about 8.4 X 1013 km from the
Earth. How long does it take its light to reach the Earth?
27.8. Light takes over 100000 years to traverse the larger
diameter of our Galaxy. Determine the approximate size of
the Galaxy in parsecs.
27.9. What is the brightest star in the northern celestial
hemisphere? What constellation does it belong to?
27.10. Name the points of the horizon lying on the celes­
tial meridian and celestial equator.
27.11. Name the equatorial constellation cut into two
unequal parts by another constellation.
27.12. Compile a list of the nonsetting constellations in
your locality by observation. Verify the correctness of your
observations with star chart.
27.13. Determine the position of the constellation Andro­
meda on the star chart at the moment of observation and
locate it in the sky. The nearest Galaxy to us is a misty spot
in the vicinity of this constellation. Observe this galaxy.
27.14. Does the Sun always rise exactly in the east and
set exactly in the west?
27.15. Which Zodiac constellations cannot be observed on
the north pole?
27.16. Where must an observer be to see the north celes­
tial pole at the zenith?
27.17. The angular distance of the celestial pole from
the zenith is 34° 15' for an observer at Moscow. What is the
geographical latitude of Moscow?
27.18. Determine the altitude of Polaris for your locality.
27.19. The geographical latitude of Leningrad is 59° 56'.
What is the angular distance between the zenith and the ce­
lestial pole for Leningrad?
27.20. Use the star chart to determine the constellation
which is located on the northern horizon at midnight on
May 15 at the latitude of Moscow.
Determine the constellation at that point and at the same
time at your latitude.
27.21. By how many degrees does the Sun move along the
ecliptic per day?
27.22. What are the declination and the right ascension
296 Ch. VI. General Remarks on Astronomy

of the Sun on the 22nd of March?


27.23. What are the declination and the right ascension
of the autumnal equinox?
27.24. What is the altitude of the Sun at noon in Moscow
at the summer solstice?
27.25. What will the angular diameter of the Earth be
for a cosmonaut on the Moon if the radius of the Earth is
6370 km and the distance from the Earth to the Moon is
384000 km?
27.26. The angular diameter of the Sun is 32'. Determine
the diameter of the Sun.
27.27. The maximum horizontal parallax of Mars is 23".
Calculate the minimum distance between Mars and the
Earth.
27.28. The annual parallax of Vega is 0.121". Calculate
the distance from the Earth to Vega in parsecs.
27.29. Calculate the period of revolution of Uranus
around the Sun if the mean distance between it and the Sun
is 19.19 AU.
27.30. Saturn orbits the Sun once in 29.46 years. Deter­
mine the mean distance between it and the Sun.
27.31. The period of revolution of Jupiter round the Sun
is 11.86 years. Determine the mean distance from it to the
Sun in astronomical units and in kilometres.
27.32. How will the Earth appear to an observer on the
Moon at the instant when full Moon is seen from the Earth?
27.33. Venus is sometimes called the morning star and
evening star. For what latitudes is this true?
27.34. What is the phase of Venus when it is observed as
a morning star?
27.35. The annual parallax of our closest star, aCentauri,
is 0.76". How long would it take for a spacecraft to fly there
at a velocity of 17 km/s?
27.36. The distance to Barnard’s star is 1.83 pc. What
is its annual parallax?
27.37. Why do the seasons not change on Venus?
27.38. Determine the mean density of solar matter. Take
the required data from tables.
27.39. Determine the linear velocity of the Vostok-1
spaceship, in which Yuri Gagarin orbited the Earth for the
first time, assuming that the orbit was circular and its mean
distance from the surface of the Earth was 251 km.
Appendices

1. Basic Physical Constants


Velocity of sound in air under normal
conditions c = 331.46 m/s
V elocity of light in vacuum c = 2.998 X 10® m/s
Gravitational constant G = 6.67 X 1 0 -n n -mVkg2
Free fall acceleration g = 9.807 m /s2
Avogadro constant N a = 6.022 X 1028 m ol-i
Molar volume of ideal gas under normal
conditions Fm = 22.4 X 10~3 ma/mol
Loschmidt number N h = 2.687 X 102‘ m -3
Molar gas constant R = 8.314 J/(m ol - K)
Boltzmann constant A- = 1.381 X 1 0 -23 J/K
Faraday constant F = 9.648 X 104 C/mol
Electric constant Fq = 8.85 X 10"12 F/m
Magnetic constant |i.0 = 4ji X 10"’ H/m =
1.257 X 10° H/m
Hydrogen atom mass mH = 1.673 X 1 0 -27 kg
Electron rest mass m . = 9.109 X 10~3i kg
Proton rest mass nip = 1.673 X 10-” kg
Neutron rest mass m i = 1.675 X 10-27 kg
Elementary charge € = 1.602 X 10-1“ C
Electron charge-to-mass ratio e ! m e = 1.759 X 10” C/kg
Ratio of proton and electron masses m . J m e = 1836.15
Atomic mass unit 1 amu = 1.660 X lO-27 kg
Planck constant h = 6.626 X 10-34 J -s
Stefan-Boltzmann constant o = 5.67 X 10-® W/(m2 -K4)
Wien constant b = 2.897 X 1 0 -3 m-K
Rydberg constant (for hydrogen) R x = 1.097 X 107 m -i
Triple point for water T = 273.16 K (/ = 0.01°C)

2. Density of Some Suhstances, p, kg/m 3

S o lid s (at 293 K)


Aluminium 2.7 X 103 Cast iron
Amber 1.1 X 103 gray 7.0 X 103
Rrass 8.5 X 103 white 7.5 X 1 0 3
Brick 1.5 X 103
298 Appendices

Concrete 2.2 X 103 N ichrome 8.3 X 103


Constantan 8.9 X 103 Nickel 8.9 X 103
Common salt 2.1 X 103 Nickeline 8.8 X 103
Copper 8.9 X 103 Oak 0.8 X to3
Cork 0.24 X 103 Paraffin 9.0 X 10*
Diamond 3.5 X 103 Platinum 21.5 X 103
Ebonite 1.2 X 103 Porcelain 2.3 X 103
Germanium 5.32 X 103 Rubber 0.94 X 103
Gold 19.3 X 103 Silver 10.5 X 103
Graphite 2.1 X io3 Snow 0.2 X 103
Ice (0°C) 0.9 X 103 Tin 7.3 X 103
Iridium 22.4 X io3 Tungsten 1.93 X 103
Iron, steel 7.8 X 103 Uranium 19.0 X 10s
Lead 11.4 X 103 Window glass 2.5 X 103
Manganin 8.5 X io3 Zinc 7.1 X 103
Mica 2.8 X 103

L iq u id s (at 293 K)
Acetone 0.8 X io3 Nitrobenzene 1.2 X 103
Aniline 1.02 X io3 Oil
Benzene 0.85 X IO3 castor (mineral) 0.92 X 103
Carbon bisulphide 1.26 X 103 transformer 0.89 X 103
Copper sulphate so­ vegetable 0.93 X 103
lution (saturated) 1.15 X 103 Petrol 0.7 X 103
Ethyl (methyl) al­ Petroleum 0.9 X 103
cohol 0.79 X 103 Turpentine 0.87 X 103
Ethyl ether 0.71 X 103 Water
Glycerine 1.26 X 103 at 277 K l.o x io3
Kerosene 0.8 x 103 heavy 1.06 X 103
Mercury (at 273 K) 13.6 X 103 sea 1.03 X 103

G a ses
(under normal conditions: p 0 = 1.013 X 105 Pa and T 0 = 273 K)
Acetylene 1.17 Hydrogen 0.09
Air 1.29 Krypton 3.74
Ammonia 0.77 Methane 0.72
Argon 1.78 Neon 0.9
Butane 0.6 Nitrogen 1.25
Carbon dioxide 1.98 Oxygen 1.43
Chlorine 3.21 Propane 2.01
Helium 0.18 Xenon 5.85

3. Specific Heat Capacity of Some Substances, c , J/(kg-K)


S o lid s

Aluminium 880 Cast iron 550


Brass 380 Cement 800
Brick 750 Concrete 920
Appendices 299

Copper 380 Platinum 125


Glass 840 Sand 970
Gold 125 Silver 250
Ice (snow) 2090 Sulphur 712
Iron, steel, nickel 460 Tin 250
Lead 120 Wood (spruce, pine) 2700
Naphthalene 1300 Zinc 400
Paraffin 3200
L iq u id s

Ethyl alcohol 2430 Machine oil 2100


Ethyl ether 2330 Mercury 125
Glycerine 2430 Transformer oil 2093
Iron 830 Turpentine 1760
Kerosene 2140 Water 4187

G a ses (at constant pressure)


Air 1000 Hydrogen 14300
Ammonia 2100 Nitrogen 1000
Carbon dioxide 880 Oxygen 920
Helium 5200 Water vapour 2130

4. Specific Heat of Combustion for Some Substances, q , J/kg

S o l i d f u e l

Brown coal 9.3 X 10s Coal


Charcoal 3.1 X 107 A-l grade 2.05 X 107
Chocks 1.5 X 107 A-2 grade 3.03 X 107
Coke 3.03 X 107 Donetsk 2.55 X 107
Firewood 8.3 X 106 Ekibastuz 1.63 X 107
Gun powder 3.0 X 10« Peat 1.5 X 107

L i q u i d f u e l

Diesel 4.2 X 107 Kerosene 4.4 X 107


Ethyl alcohol 2.7 X 107 Naphtha 4.33 X 107
Fuel oil 4.0 X 107 Petrol, petroleum 4.6 X 107

G a se o u s f u e l

(per cubic metre under normal conditions)


Blast-furnace gas 3.7 X 10" Producer gas 5.5 X 10c
Coke-oven gas 1.64 X 107 Town gas 2.1 X 107
Natural gas 3.55 X 107
300 Appendices

5. Boiling Point and Specific Latent Heat of Vaporization

Substance T, K f, °c r, /kg

Acetone 329.2 56.2 5.2 X 105


Air 81 -1 9 2 2.1 X 105
Ammonia 239.6 —33.4 1.37 X 108
Ethyl alcohol 351 78 8.57 X 106
Ethyl ether 308 35 3.52 X 10s
Freon-12 243.2 -2 9 .8 1.68 X 108
Iron 3023 2750 5.8 X 104
Mercury 630 357 2.85 X 108
Petrol 423 150 3.0 X 10s
Turpentine 433 160 2.94 X 106
Water
heavy 374.43 101.43 2.06 X 109
ordinary 373 100 2.26 X 10°

G. Pressure and Density of Saturated W ater Vapour


a t Various Temperatures

p x i o - 3, p x i o - 3,
t. °c p, kPa Ug/in3 i, °c p. kPa kg/m*

-1 0 0.260 2.14 16 1.813 13.6


-5 0.401 3.24 1 17 1.933 14.5
-4 0.437 3.51 18 2.066 15.4
-3 0.476 3.81 19 2.199 16.3
-2 0.517 4.13 20 2.333 17.3
-1 0.563 4.47 21 2.493 18.3
0 0.613 4.80 I 22 2.639 19.4
1 0.653 5.20 23 2.813 20.6
2 0.706 5.60 24 2.986 21.8
3 0.760 6.00 25 3.173 23.0
4 0.813 6.40 26 3.359 24.4
5 0.880 6.80 27 3.559 25.8
6 0.933 7.30 28 3.786 27.2
7 1.000 7.80 29 3.999 28.7
8 1.066 8.30 30 4.239 30.3
9 1.146 8.80 40 7.371 51.2
10 1.226 9.40 50 12.33 83.0
11 1.306 10.0 60 19.92 130.0
12 1.399 10.7 80 47.33 293
13 1.492 11.4 100 101.3 598
14 1.599 12.1 120 198.5 1123
15 1.706 12.8 160 618.0 3259
200 1554 7763
Appendices 301

7. Young’s modulus for Some Substances, £ , GPa


Aluminium 70 Copper 130
Brass 110 Iron 200
Brick 28 Lead 17
Cast iron 90 Steel 220
Concrete 20

8. Boiling Point and Specific Latent Heat of Vaporization


for Water under Various Pressures

(. °c p, MPa r, MJ/kg t , ce l>, MPa r, MJ/kg

10 0.001 2.47 197.4 1.47 1.95


100 0.1 2.26 346 15.7 0.9
151 0.49 2.11 347.15 22.1 0

9. Boiling Point and Critical Parameters for Some Substances


Critical parameters
Boiling point,
Substance t , °C temperature pressure
*cr- ^ pc r x l u \ Pa

Argon —186 -122.4 48


Ether 35 193.8 35.0
Ethyl alcohol 78 243.1 63
Helium —269 —267.9 2.25
Hydrogen —253 -2 4 1 12.8
Krypton —193 —63.62 54.27
Neon —246 —228.7 26.9
Nitrogen —196 -147.1 33.5
Oxygen —183 —118.4 49.7
Water 100 374.15 221.3
Xenon -1 0 8 18.76 57.64

10. Melting Point and Specific Latent Heat of Fusion for Some
Solids at Melting Point

Substance T,,,. K m ®c
t m t J/kg

Aluminium 932 659 3.8 X 105


Cast iron
gray 1423 1150 9.7 X 104
white 1473 1200 1.3 X 10s
302 Appendices

10. Melting Point and Specific Latent Heat of Fusion for Some
Solids at Melting Point (c o n t . )

Substance r m’ K m K J/kg

Copper 1356 1083 1.8 X 105


Gold 1337 1064 6.6 X 104
Iron 1803 1530 2.7 X 10‘
Lead 600 327 2.5 X 104
Mercury 234 -3 9 1.25 X 104
Naphthalene 353 80 1.51 X 10s
Silver 1233 960 8.8 X 104
Steel 1673 1400 2.1 X 10s
Sulphur 385.8 112.8 5.5 X 104
Tin 505 232 5.8 X 104
Tungsten 3683 3410 2.6 X 104
Water, ice 273 0 3.35 X 10s
Water, heavy 276.82 3.82 3.16 X 10‘
Wood’s metal* 341 68 3.2 X 104
Zinc 692 419 1.18 X 10s

* Wood’s metal with this melting point contains 50% bismuth, 25% lead,
12.5% tin, and 12.5% cadmium.

11. Surface Tension for Some Substances, o, N/m (at 293 K)


Acetone 0.024 Mercury 0.470
Castor oil 0.033 Milk 0.045
Copper sulphate solution 0.074 Petrol 0.029
Etner 0.017 Soap solution 0.040
Ethyl alcohol 0.022 Turpentine 0.027
Glycerine 0.059 Water 0.072
Kerosene 0.024

12. Coefficient of Linear Expansion for Some Solids, a , K -‘


Aluminium, duralumin 2.3 X 10-5 Gold 1.4 X 10-5
Brass 1.9 X 10 -1 Invar* 6 X 1 0 -’
Bronze 1.8 X 10 -6 Iron, steel 1.2 X 1 0 -5
Cast iron 1.0 X io-s Lead 2.9 X 1 0 -5
Concrete, cement (1-1.4) X 10-6 Nickel 1.28 X 1 0 -6
Copper 1.7 X 10 -5 Platinum 9 X 10-«
Ebonite 7.0 X 10 -6 Platinite 9 X 10-«
Glass Tin 2.1 X 10-5
quartz 6 X 10-’ Tungsten 4 X 10-«
window 9 X 10-" Zinc 2.9 X 10-s

Invar contains 64% iron and 3G% nickel.


Appendices 303

13. Coefficient of Volume Expansion for Some Liquids, {1, K-1


Acetone 1.2 X 10-3 Sulphuric acid 5.7 x io-4
Ethyl alcohol 1.1 X io-3 Transformer oil 6.0 x to-4
Ethyl ether 1.6 X 10-3 Water at
Glycerine 5.0 X 10-4 5-10°C 5.3 x io -5
Kerosene 1.0 x io-3 10-20°C 1.5 x io-«
Mercury 1.8 X 10"4 20-40°C 3.02 x io-«
Petrol 1.0 x io-3 40-60°C 4.58 x io-«
Petroleum 1.0 x io-3 60-80°C 5.87 X 10"4
80-100°C 7.02 x 10-4
14. P erm ittivity e for Some Substances
Air Paraffin 2.2
at 0.1 MPa 1.0006 Petrol 2.3
at 10 MPa 1.055 Porcelain 4-7
Amber 2.8 Rubber 2-3
Aniline 84 Rutile 130
Ebonite 2.7 Shellac 3.6
Epoxy resin 3.7 Sulphur 3.6-4.3
Glass 5-10 Transformer oil 2.2-2.5
Glycerine 39 Vacuum 1
Hydrogen 1.0003 Water 81
Ice at —18°C 3.2 Water at 0°C 88
Kerosene 2.0 Wax 5.8
Marble 8-9 Waxed paper 2.0
Mica 6-9
15. Resistivity of Some Materials, p, fl m
Aluminium 2.7 X 10-* Nickel 7.3 X 10-»
Brass 6.3 X 10-8 Nickeline 4.2 x io-7
Coal (4.0-5.0) X 10-5 Nichrome 1.05 X io-“
Constantan 4.7 X 10-’ Osmium 9.5 x io-8
Copper 1.68 X IO-8 Platinum 1.05 x io-7
Ferro-aluminium Rheotan 4.5 x io-7
high-resistance Silver 1.58 X IO-8
alloy 1.1 X 10-« Steel 1.2 x io-7
Gold 2.2 X IQ-® Tin 1.13 x io-7
Iron 9.9 X IO-8 Tungsten 5.3 X 10-8
Lead 2.07 X 10-7 Zinc 5.95 X 10-8
Manganin 3.9 X 10-’
Mercury 9.54 X 10-’

16. Temperature Resistance Coefficient for Some Substances, a , K -1


Cast iron 0.002 Nickeline 0.0001
Constantan 0.000005 Rheotan 0.0004
Manganin 0.000008 Steel 0.006
Nichrome and ferro-alumi- Tungsten 0.0050
nium high-resistance
alloy 0.0002
304 Appendices

17. Electrochemical Equivalent for Some Substances, k, kg/C*


Aluminium 9.32 X 10-8 Lead 1.074 X 10-«
Calcium 2.077 X 10-’ Magnesium 1.26 X io-7
Chlorine 3.67 X 10-’ Mercury 2.072 X 10-"
Chromium (bivalent) 2.79 X 10-7 Nickel
Copper bivalent 3.04 X io-7
bivalent 3.29 X 10-7 trivalent 2.03 X io-7
monovalent 6.6 X 10-7 Oxygen 8.29 X 10-8
Gold 6.81 X 10-7 Potassium 4.052 X 10-7
Hydrogen 1.045 X 10-8 Silver 1.118 X 10-«
Iron Sodium 2.383 X 10-’
bivalent 2.89 X 10-7 Zinc 3.388 X io-7
trivalent 1.93 X 10-7

18. Refractive Index for Some Materials, n

Acetone 1.36 Glycerine 1.47


Air 1.0003 Ice 1.31
Aniline 1.59 Methyl alcohol 1.33
Benzene 1.50 Quartz 1.54
Carbon bisulphide 1.63 Rock salt 1.54
Carbon tetrachloride 1.46 Sugar 1.56
Diamond 2.42 Sylvite 1.49
Ethyl alcohol 1.36 Turpentine 1.51
Glass Water 1.33
crown 1.5
flint 1.6-1.8

19. Mass of Some I««i«nes. amu*

Element Isotope Mass Element Isotope Mass

Hydrogen |H 1.00783 Carbon ,.1C 13.00335


?H 2.01410 Oxygen •JO 15.99491
»H 3.01605 Fluorine 191 ? 18.99843
Helium 2He 3.01603 Aluminium ?5A1 26.98153
JHe 4.00260 Phosphorus 29.97867
Lithium JLi ?*p
6.01513 Radon 2l«Rn 222.01922
JLi 7.01601
Beryllium JBe Radium 2sSRa 226.02435
8.00531
JBe 9.01219 Uranium 235.04299
Boron •JB 11.00930 Viu 238.05006
Carbon •|C 12.00000 Neptunium 2.337Np 237.04706
Plutonium 2j;pu 239.05122

* 1 am u is e q u a l to 1 /1 2 o f th e m ass of an ato m of :JC iso to p e .


Appendices 305

20. Psychrometric Table

Pry-bulb Difference between dry-bulb and wet -bulb temperatures


temperature

K •c 0 1 2 3 4 5 6 7 8 9 10 11

273 0 100 82 63 45 28 11
274 1 100 83 65 48 32 16
275 2 100 84 68 51 35 20
276 3 100 84 69 54 39 24 10
277 4 100 85 70 56 42 28 14
278 5 100 86 72 58 45 32 19 6
279 6 100 86 73 60 47 35 23 10
280 7 100 87 74 61 49 37 26 14
281 8 100 87 75 63 51 40 28 18 7
282 9 100 88 76 64 53 42 31 21 11
283 10 100 88 76 65 54 44 34 24 14 4
284 11 100 88 77 66 56 46 36 26 17 8
285 12 100 89 78 68 57 48 38 29 20 11
286 13 100 89 79 69 59 49 40 31 23 14 6
287 14 100 90 79 70 60 51 42 33 25 17 9
288 15 100 90 80 71 61 52 44 36 27 20 12 5
289 16 100 90 81 71 62 54 45 37 30 22 15 8
290 17 100 90 81 72 64 55 47 39 32 24 17 10
291 18 100 91 82 73 64 56 48 41 34 26 20 13
292 19 100 91 82 74 65 58 50 43 35 29 22 15
293 20 100 91 83 74 66 59 51 44 37 30 24 18
294 21 100 91 83 75 67 60 52 46 39 32 26 20
295 22 100 92 83 76 68 61 54 47 40 34 28 22
296 23 100 92 84 76 69 61 55 48 42 36 30 24
297 24 100 92 84 77 69 62 56 49 43 37 31 26
298 25 100 92 84 77 70 63 57 50 44 38 33 27
299 26 100 92 85 78 71 64 58 51 45 40 34 29
300 27 100 92 85 78 71 65 59 52 47 41 36 30
301 28 100 93 85 78 72 65 59 53 48 42 37 32
302 29 100 93 86 79 72 66 60 54 49 43 38 33
303 30 100 93 86 79 73 67 61 55 50 44 39 34

21. Permeability of Some Materials, p

P a r a m a g n e tie s (p > 1) D ia m a g n e tic s (p < 1)


Air 1.00000038 Bismuth 0.999824
Aluminium 1.000023 Copper 0.999990
Oxygen 1.0000019 Glass 0.999987
liquid 1.003400 Hydrogen 0.999999937
Tungsten 1.000176 Water 0.999991
:n -0 5 3 0
306 Appendices

F e r r o m a g n e tic s

(maximum permeability)
Cast iron 2000
Iron
carbon 3000
soft 8000
transformer 15000
Permalloy (Ni-Fe alloy for
transformer cores) 80000

R e m a r k . The maximum permeabilities of ferromagnetics are given


since the parameter depends on the external magnetic field strength.

22. Magnetic Induction as a Function of Magnetic Field Strength


for Soft Steel during Primary Magnetization

23. Some Astronomical Quantities (mean values)


Radius of the Garth 6.37 X 10“ m
Mass of the Garth 5.98 X 10M kg
Density of the Garth 5.52 X 103 kg/m;
Velocity of orbital motion of the Garth 106 km/h
Radius of the Sun 6.95 X 108 m
Mass of the Sun 1.98 X 1030 kg
Radius of the Moon 1.74 X 10® m
Mass of the Moon 7.33 X 102S kg
Distance between the centres of the Garth and
the Sun 1.49 X 1011 m
Distance between the centres of the Garth and
the Moon 3.84 X 10* m
Period of revolution of the Moon around the
Garth 27.3 days
Inclination of the ecliptic to the equator 23.5°
Appendices 307

24. Parameters of Some Bright Stars

Magni- Declina-
Star tude m Right ascension a tion 0

a Tauri (Aldebaran) 1.06 4 h 31 min 54 s +45°57'


B 8 8 8-0=8

P Orionis (Rigel) 0.34 5 h 12.1 min —8°15'


a Aurigae (Capella) 0.21 5 h 13 min +45°57'
a Orionis (Betelgeuse) 0.92 5 h 52.5 min + 7°24'
a Canis Majoris (Sirius) —1.58 6 h 42.9 min —16°39'
a Geminorum (Castor) 1.99 7 h 31.4 min +32°
a Lyrae (Vega) 0.14 18 li 35 min +38°41'
a Cygni (Deneb) 1.38 20 h 39 min + 45c06'

25. Binding Energy of Atomic Nuclei

N ucleus £ b . MeV E b/.4 , M e\

5H 2.2 1.1
?H 8.5 2.83
|He 7.7 2.57
$He 28.3 7.075
SLi 32.0 5.33
’Li 39.2 5.60
\Be 58.2 6.47
<“ B 64.7 6.47
‘ iB 76.2 6.93
>|C 92.2 7.68
97.1 7.47
•*N 104.7 7.47
io 127.6 7.975
‘| 0 131.8 7.75
?JAl 225.0 8.33
{JSi 255.2 8.51
so p 250.6 8.35
282? R n 1708.2 7.69
22| R a 1731.6 7.66
2bSu 1783.8 7.59
1801.7 7.57
«»Pu 1806.9 7.56

2 n*
308 Appendices

26. Prefixes Used with SI Units

f a c to r Prefix N otation

1 0 ‘" hexa H
IO*6 p eta P
1 0 1? te r a T
10* 8>8a G
10° m ega M
103 k ilo k
10- h ecto h
10i deed da
io - 1 deci d
io -3 c e n ti c
io -3 m i ll i III
io-° m ic r o p
io -9 uano 11
1 0-12 p ic o p
1 0 -1 2 le m to f
1 0 -1 * a tto U

27. Sines and Tangents of Angles from 0 to 90“


Angle
Sine Tangent
degrees radians

0 0 0.0000 0.0000
1 0.0175 0.0175 0.0175
2 0.0349 0.0349 0.0349
3 0.0524 0.0523 0.0524
4 0.0698 0.0698 0.0699
5 0.0873 0.0872 0.0875
6 0.1047 0.1045 0.1051
7 0.1222 0.1219 0.1228
8 0.1396 0.1392 0.1405
9 0.1571 0.1564 0.1584
10 0.1745 0.1736 0.1763
11 0.1920 0.1908 0 1944
12 0.2094 0.2079 0.2126
13 0.2269 0.2250 0.2309
14 0.2443 0.2419 0.2493
15 0.2618 0.2588 0.2679
16 0.2793 0.2756 0.2867
17 0.2967 0.2924 0.3057
18 0.3142 0.3090 0.3249
19 0.3316 0.3256 0.3443
20 0.3491 0.3420 0.3640
Appendices 309

27. Sines and Tangents of Angles from 0 to 90° (c o n t .)


Angle
Sine Tangent
degrees radians

21 0.3665 0.3584 0.3839


22 0.3840 0.3746 0.4040
23 0.4014 0.3907 0.4245
24 0.4189 0.4067 0.4452
25 0.4363 0.4226 0.4663
26 0.4538 0.4384 0.4877
27 0.4712 0.4540 0.5095
28 0.4887 0.4695 0.5317
29 0.5061 0.4848 0.5543
30 0.5236 0.5000 0.5774
31 0.5411 0.5150 0.6009
32 0.5585 0.5299 0.6249
33 0.5760 0.5446 0.6494
34 0.5934 0.5592 0.6745
35 0.6109 0.5736 0.7002
36 0.6283 0.5878 0.7265
37 0.6458 0.6018 0.67536
38 0.6632 0.6157 0.7813
39 0.6807 0.6293 - 0.8098
40 0.6981 0.6428 0.8391
41 0.7156 0.6561 0.8693
42 0.7330 0.6691 0.9004
43 0.7505 0.6820 0.9325
44 0.7679 0.6947 0.9657
45 0.7854 0.7071 1.0000
46 0.8029 0.7193 1.036
47 0.8203 0.7314 1.072
48 0.8378 0.7431 1.111
49 0.8552 0.7574 1.150
50 0.8727 0.7660 1.192
51 0.8901 0.7771 1.235
52 0.9076 0.7880 1.280
53 0.9250 0.7986 1.327
54 0.9425 0.8090 1.376
55 0.9599 0.8192 1.428
56 0.9774 0.9774 1.483
57 0.9948 0.8387 1.540
58 1.0123 0.8480 1.600
59 1.0297 0.8572 1.664
60 1.0472 0.8660 1.732
61 1.0647 0.8746 1.804
62 1.0821 0.8829 1.881
63 1.0996 0.8910 1.963
64 1.1170 0.8988 2.050
65 1.1345 0.9063 2.145
66 1.1519 0.9135 2.24C
310 Appendices

27. Sines and Tangents of Angles from 0 to 90° (e o n t .)


Angle
Sine Tangent
degrees radians

67 1.1694 0.9205 2.356


68 1.1868 0.9272 2.475
69 1.2043 0.9336 2.605
70 1.2217 0.9397 2.747
71 1.2392 0.9455 2.904
72 1.2566 0.9511 3.078
73 1.2741 0.9563 3.271
74 1.2915 0.9631 3.487
75 1.3090 0.9659 3.732
76 1.3265 0.9703 4.011
77 1.3439 0.9744 4.331
78 1.3614 0.9781 4.705
79 1.3788 0.9816 5.145
80 1.3963 0.9848 5.671
81 1.4137 0.9877 6.314
82 1.4312 0.9903 7.115
83 1.4486 0.9925 8.144
84 1.4661 0.9945 9.514
85 1.4835 0.9962 11.43
86 1.5010 0.9976 14.30
87 1.5184 0.9986 19.08
88 1.5359 0.9994 28.64
89 1.5533 0.9998 57.29
90 1.5708 1.000 00

28. Basic Formulas in Physics

Formula Quantities appearing in formulas Units

K in e m a tic s

r = r (t) r is the displacement of a particle m


t is the time interval s
X = X (t), y = y (t) x and y are the coordinates of a particle m
* = *(*) s is the distance covered by the particle m

U n i f o r m m o tio n

r — r0 = v( r — r„ is the increment of displacement m


X — x 0 = vxt x a and y 0 are the initial coordinates of ro
a particle
y — y„ = V
Appendices 311

28. Basic Formulas in Physics (e o n t . )

Formula Quantities appearing in formulas Units

|r | = x = s = v t s is the path length (the coordinate m


axis coincides with the direction of
displacement)
U n i f o r m l y v a r ia b le m o tio n

At
v v is the mean velocity of variable m/s
A t
motion
A r
v ■lim — v is the instantaneous velocity m/s
A <-0
In vector form
a = const a is the acceleration m/s2
v0 is the initial velocity m/s
v = v„ + at v is the instantaneous velocity m/s
In scalar form
vx = v 0 x + a x t vx and V y are the projections of velocity m/s
on the coordinate axes
Vv = ”00 + V
x — x 0 = v 0 x +, - “I**
± - a x and a y are the projections of accel- m/s*
eration on the coordinate axes
, a „ t 2
y - y0 = ”00 +
v* — v* = F a s 2
F r e e fa l l

v = g t g is the free fall acceleration m/s2


H is the height from which the body falls m
V* = 2g H v is the velocity at the end of the free m/s
fall

M o t i o n o f a b o d y th r o w n u p w a r d s

H is the maximum height of ascent m


312 Appendices

28. Basic Formulas in Physics (e o n t . )

Formula Quantities appearing in formulas Units

C u r v ilin e a r m o tio n

v = 2 n R f v is the linear velocity m/s


R is the radius of rotation m
/ is the rotational frequency Hz, s-1
to = 2 ji / o) is the angular velocity rad/s
to = v / R
a c = iN R a c is the centripetal acceleration m/s*
a c - - cD*fl
D y n a m i c s

p = m / V p is the density kg/m3


m is the mass
V is the volume mn
Newton’s second law
a = F / m is the (resultant) forceF N
F t = A m u is the impulse of forceF t N-s
m u is the momentum kg*m/s
Conservation of momentum
m v = const
Newton’s third law
TTt i Qj — TTt 2 ^2
P = m g P is the force of gravity N
Law of universal gravitation
p — Q F is the gravitational force N
ra
G is the gravitational constant N -ms/kgs
r is the distance between the centres m
of mass
F c = m v ^ lR F c is the centripetal force N
F c = n m 2R

F e l = k r is the elastic force


F e \ N
k is the spring constant N/m
F t = juV F f is the force of friction N
p is the friction coefficient
N is the force of normal pressure N
W o r k , p o w e r , e n e r g y

A = F,? cos a A is the mechanical work I


A = fv? for a = 0 a is the angle between the direction deg
of force and displacement
P = A/t P is the power W
P = F v P is the mean power W
u is the mean velocity m/s
Appendices 313

28. Basic Formulas in Physics (c o n t .)

Formula Quantities appearing in formulas Units

E y = mu8/ 2 £'k is the kinetic energy J


E p = mgh p is the potential energy
E J
_A useful X 100% t] is the efficiency
A spent
A useful is the useful work J
A spent is the spent work J
M o le c u la r P h y s ic s . H e a t

= a is the Avogadro constant


N mol-1
0 M
M is the molar mass kg/mol
no = Nk!Vm n0 is number density of molecules m~3
V m is the molar volume nrVmol
m„ = M I N a m0 is the mass of a molecule kg
V 0 is the volume of a molecule m*

3M
r* r0 is the radius of a molecule m
4npJVA
\ = v iz Xis the mean free path of a molecule m
y is the (arithmetic) mean velocity of m/s
molecules
z is the number of collisions per second s -1
d e ff is the effective diameter of a m
~\f 2jld-jtn0
molecule
2 KmB
5-»0 --- S— p is the pressure Pa
yripS is the root-mean-square velocity m/s
p = n 0k T is the Boltzmann constant
k J/K
T is the thermodynamic temperature K
E v = - r k T E y is the kinetic energy of translational J
motion of molecules
Equations of state of an ideal gas
R is the molar gas constant J/(mol-K)
rr-T T « r m / M = v is the amount of substance mol

plt Flt and 7\ are the parameters of


the first state of a gas
Pj, Fa, and T t are the parameters of
the second state of a gas
314 Appendices

28. Basic Formulas in Physics (c o n t . )

F o rm u la Q u a n titie s a p p earin g in fo rm u la s U n its

Dalton’s law
p is the pressure of a gas mixture Pa
P = Pi + P i + p [ and Pj are partial pressures Pa
Q —c m ^i) Q is the heat required to heat a body J
or given off as it cools
c is the specific heat J/(kg-Kj
Q = q m Q is the heat given off as fuel burns J
q is the specific heat of combustion J/kg
of fuel
m is the mass of burnt fuel kg
*1 = %UBCtul
v spent
100% T| is the efficiency of the heater %
^useful *3 the useful heat J
O a p e n t is the spent heat J
Q = r m Q is the heat required for vaporization J
or released during condensation
r is the specific latent heat of vapo- J/kg
rization
m is the mass of vapour kg
B = 100% B is the relative humidity of air %
P sat
pa is the absolute humidity of air kg/ms
P s a t is the saturated vapour density kg/m3
B = 100 % pa is the pressure of vapour contained Pa
P sat in air
P s a t is the saturated vapour pressure Pa
F_
a a is the surface tension N/m
I
F is the force of surface tension N
I is the length of the boundary of free m
surface
A
o= A is the work of molecular forces J
AS
AS is the decrease in the area of free m2
surface
2 a
h h is the height to which a wetting liquid m
Pgr rises (nonwetting liquid falls) in a
capillary
PL = 2 o /r r is the radius of a capillary m
pL is the Laplacian pressure for a Pa
spherical surface
Appendices 315

28. Basic Formulas in Physics (c o n t . )

Formula Quantities appearing in formulas Units

Q = k m Q is the heat required for fusion or J


liberated during crystallization
k is the specific latent heat of fusion J/kg

D e fo r m a tio n s

F_
a= S
a is the mechanical stress Pa
F is the force of elastic deformation N
S is the cross-sectional area of a de- ma
formed body
Hooke’s law
o = E — E is the Young’s modulus Pa
A I is the absolute deformation (elon- m
gation)
I is the initial length m
e= A l / l e is the strain
Ep= F A l / 2 E p is the potential energy of elastic J
deformation

T h e r m a l e x p a n s io n o f b o d ie s

a is the coefficient of linear expansion K -'


“ l0E T l0A T
l n is the length of a body at 273 K m
l = l 0 (i + a A T ) I is the length of the body at any tem­ ra
perature
AT1 is the change in temperature K
S = S 0 (1 + 2 a A T ) S 0 is the surface area of the body at m1
273 K
S is the surface area of the body at any m3
temperature
y __ y
ft= y ' —~ ~ y ~ \ f P *s coefficient of volume expansion K~l
V = V 0 (1 + PAY) V is the volume of the body at any tern- m3
perature
V „ is the volume of the body at 273 K m3
p= l P *s the density of a substance at kg/m3
any temperature
p0 is the density of the substance kg/m3
at 273 K
316 Appendices

28. Basic Formulas in Physics (c o n t.)

Formula Quantities appearing in formulas Units

E le c tr o s ta tic s

Charge conservation Q i, Q jj, . . Q n are electric charges C


law
@1 + ■ n is the number of electric charges in
+ Q n = coast a closed system
Coulomb’s law
1
F = 4ite„ I < ? i er2
1 1 <?2 is the force of interaction between N
F

point electric charges


Q u Q i are electric charges C
r is the separation between the charges m
e0 is the electric constant C/(N *m2)
e is the permittivity of the medium
E = F / Q t E is the electric field strength N/C
___Q Q t is a test charge in the field C
4:ie0er2
Q is the charge producing the field C
9 = W/Qt cp is the electric field potential V
Q
V 4n e0er W is the potential energy of the test J
charge
= 9i ~ 9i U is tne voltage V
= Q A is the work done by the electric J
field to move a charge from one point
in the field to another
U_
E =
d
E is the strength of a uniform field V/m,
N/C
U is the potential difference between two V
points in the field
d is the distance between the two points m
along the field line
C = Q /< p C is the capacitance F
C = Q / U

___ bbrS C is the capacitance of a (parallel-plate) F


L— d capacitor
S is the area of a plate m2
d is the thickness of a dielectric m
w= cuvz W is the energy of the capacitor J
Appendices 317

28. Basic Formulas in Physics (c o n t.)

Form ula Q uantities appearing in formulas Units

Electric current in metals

I = Q / t / is the electric current A


I = e n v S e is the charge of an electron (ion) C
I = e n u E S n is the number density of charge n r 3
carriers
v is the mean velocity of the directed m/s
motion of charge carriers
u —v/E u is the mobility of the charge carriers ms/(V -s)
1

i ~ ~ s
j is the current density A/m2
S is the cross-sectional area m2
i

r = p— is the resistance of the conductor


H Q
p is the resistivity of the conductor £2 -m
material
I is the conductor length m
S is the cross-sectional area of the m2
conductor
f i t = 770 (1 + aA T ) R t is the resistance of a conductor at any n
temperature
R a is the resistance of the conductor at Q
273 K
a is the temperature resistance coef­ K '1
ficient
Ohm’s law for a conductor
i|(t:

is the voltage across the conductor V


II

is the resistance of the conductor


R £2
Series connection of conductors
R e n = 77t + 77a 77jq is the equivalent resistance of con- £2
+ •^ + R n ductors
/? gq — R ^ n
for = R 2 = . . . n is the number of the conductors
= Rn 7?!, 7?j, . . R n are the resistances of Q
the conductors
Parallel connection of conductors
1 _ 1 | 1
R g q is the equivalent resistance £2
/feq 77, 77,

-
318 Appendices

28. Basic Formulas in Physics (cont.)

F orm u la Q u an tities appearing in form ulas Unit9

R t a = —— re is the number of resistors


^ re
for R x = R v R a, R 3, are the resis- Q
Ra = . . . = Rn tances
Ohm’s law for a closed circuit
g
/= p _i_ is the electromotive force of a cur-
% V
' rent source
R is the resistance of the external cir­ a
cuit
r is the resistance of the internal circuit Q
Ohm’s law for a subcircuit containing an emf
U _*
g is the counter emf V
1 ~ R
A = IU t A is the work done by electric current J
A = PRt
U*
t is the time s
a = -r - ‘
P = IU P is the power of an electric current W
P = PR
P = IP/R
P0 = I t P„ is the total power developed by a cur­ W
rent source
Pi = P„ - P r Pi is the power of the load in a closed w
circuit
Electric current in electrolytes
Faraday’s law
m = kQ or m = k it m is the mass of a substance deposited kg
on electrodes
Generalized law
1 M
m = —=--------- Q k is the electrochemical equivalent kg/C
t n
M is the molar mass kg/mol
F is the Faraday constant C/mol
re is the valency
Electromagnetism
F = — n — ----- F is the force of interaction between N
parallel current-carrying conductors
/ j and 1 2 are the currents in the conduc- A
tors
p. is the permeability
|i0 is the magnetic constant H/m
a is the separation between the con- m
ductors
Appendices 319

23. Basic Formulas in Physics (eo n t.)

Form ula Q uantities appearing In form ulas U n its

F x = B I l sin a F x is Ampere’s force (acting on a cur- N


rent-carrying conductor)
F A = B I l B is the magnetic induction T
for sin a = 1 I is the active length of a conductor m
a is the angle between the direction of deg
the current and the magnetic induction
vector
A = IB AS A is the work done in moving a cur- J
A = /AO rent-carrying conductor in a magnetic
field
A S is the change in the area embraced m*
by the current loop
AO is the change in the magnetic flux Wh
B SU = HoH ~2^r .B8tr is the magnetic induction of a T
straight current
r is the distance from the conductor to m
the point where the induction is
being determined
„ 1
^ c l r — MoM 2r B clr is the magnetic induction at the T
centre of a circular current
r is the radius of the circular current m
Rsol = M
B ^oM
-—Ia> 5 sni is the magnetic induction of the T
field in a solenoid
co is the number of turns
I is the length of the solenoid m
®9ol = ^ s n l ‘^ <I>S01 is the magnetic flux in the solenoid Wb
S is the cross-sectional area of the m5
solenoid
B = H is the magnetic field strength A/m
cL = B v e sin a F h is the Lorentz force (acting on a N
charged particle in a magnetic field)
e is the charge of a particle C
v is the velocity of tne particle m/s
a is the angle between vectors B and v deg

E l e c t r o m a g n e t i c i n d u c t i o n

AO
¥ = — % is the induced cmf V
AO is the change in the magnetic flux Wb
A t is the time interval over which the s
magnetic flux changes
320 Appendices

28. Basic Formulas in Physics (c o n t .)

Form ula Q u an tities appearing in form ulas U n its


ifso l------At $ S0l is the emf induced in a solenoid V
Sstr = B l v sin a $ str is the emf induced in a straight V
conductor during its motion
I is the active length of the conductor m
$ s is the self-inductance emf V
L is the inductance of the circuit H
L I 2
W= 2
W is the energy of the magnetic field J
of the.circuit

O s c i l l a t i o n s a n d W a v e s

T is the period of oscillations s


re is the number of complete oscillations
v = i / T v is the oscillation frequency Hz
co = 2 nlT = 2jiv ci> is the circular frequency rad/s
q> = cof cp is the phase of oscillations rad
tp = cot + <p0 <p0 is the initial phase rad
Equation of harmonic motion
x= A sin (cot + <p0) x is the displacement m
A is the amplitude of vibrations m
T is the period of oscillations of s
- * V t simple pendulum
I is the length of the pendulum m
g is the free fall acceleration m/s2
T = 2 k V m / k T is the period of elastic vibrations s
co is the circular frequency rad/s
k is the spring constant N/m
m is the mass of the load kg
X = v T X is the wavelength m
v is the velocity of propagation of wave m/s

A l t e r n a t i n g c u r r e n t

e = # 0 sin (cof + <p„) is the instantaneous emf in the circuit


e V
%0 is the amplitude of the emf V
u= U 0 sin (cot + <p0) u is the instantaneous voltage V
U 0 is the amplitude of the voltage V
Appendices 321

28. Basic Formulas in Physics (con(.)

Formula Q u an tities appearing in form ulas Units

i = 70 sin (tot + <p„) i is the instantaneous current A


/ 0 is the amplitude of the current A
I = IJV 2 I is the effective current A
U -- V J \ / 2 U is the effective voltage V
* = 8 0/ |/ 2 8 is the effective emf V
xAc ~ co1C ~ 2 n v C
1
is the capacitive reactance of the
X c Q
circuit
v is the alternating current frequency Hz
X L = 0 = 2n \ L X L is the inductive reactance of the n
______________ circuit
Z = Y R 2 + ( X —X c ) z Z is the impedance of the circuit with
l

series-connected components
R is the resistance of the circuit Q
cos cp = R t z cos <p is the power factor
p = U I cos <p P is the active power W
I and U are the effective current A, V
and voltage
Q = U l sin <p Q is the reactive power var
s = y p 2 + p; 5 is the total power VA
v t I t * ,

= l v T = k
k is the transformation ratio
V i
Ar1 and Ar2 are the numbers of turns in
the primary and secondary
U ! and U 2 are voltages across the pri- V
mary and secondary of the transformer

E l e c t r o m a g n e t i c o s c illa tio n s a n d w a v e s

T = 2 n Y L C T is the period of natural oscillations s


of an oscillatory circuit
X = c T X is the length of the electromagnetic m
waves
c is the velocity of light in vacuum m/s

G e o m e t r i c a l o p tic s

n = c /v n is the absolute refractive index


v is the velocity of light in a medium m/s
n«. i = a 'n ^ ' n«.j *s the relative refractive index
1 e e is the angle of incidence deg
e' is the angle of refraction deg

21-0530
322 Appendices

28. Basic Formulas in Physics (cont.)

Form ula Q uantities appearing in form ulas U n its

V1
n t, i — »v»2 u! is the velocity of light in the first m/s
medium
i>, is the velocity of light in the second m/s
medium
sin icr = 1I n ier is the critical angle of total internal deg
reflection
A A A
_L=±+_L / is th e focal length of a lens or m irror m
/ a ' a'
a is the distance from the object to the m
lens (mirror)
a' is the distance from the image to the m
lens (mirror)
4> = 1If is the optical power of a lens D
is the focal length of a m irror m
/= 4 f
R is the radius of curvature of the m
m irror
ft is the linear magnification
•■ 4
H is the height of the image m
h is the height of the object m

P h o to m e try
e

lm
O'

<I> is the luminous flux


II

Q is the luminous energy J


®tot = 4 it/ Otnt is the total luminous flux lm
I= 0>/<o I is the lum inous intensity cd
(D is the solid angle ST
E = <t>/S E is the illuminance lx
Laws of illum ination
E = I/r 2 r is the distance from a source of light m
to the surface being illum inated
E = E 0 cos a E 0 is the illum inance due to the normal lx
rays
or
E = — cos a a is the angle of incidence deg
r2
W a v e a n d q u a n tu m p ro p e r tie s o f r a d ia tio n
kX = d sin qp d is the grating constant m
<p is the angle to the diffraction maxi­ deg
mum
k is the spectral order
Appendices 323

28. Basic Formulas in Physics (c o nt .)

Form ula Q u an tities appearing in form ulas U nits

e = hv e is the energy of a photon J


h is the Planck constant J -s
e h
m is the mass of a photon kg
h
is the momentum of a photon kg-m /s
me=x me

— — — N hw p is the pressure of light Pa


P c e
N is the num ber of photons
/ is the radiation intensity J/(m 2-s)
E instein's equation
for photoelectric
efiect
kv-A + f A is the electron work function J

29. Some M athem atical Formulas


AIgebra
1. (a + b)2 = a2 -j- 2ab -f- 62,
(a — b)2 = a2 — 2ab + b2,
(a + b)3 = a3 + 3aa6 + 3a6a + b3,
(a — b)3 = a3 — 3alb -j- 3ab2 — b3,
a2 — b2 = (a — b) (a + 6),
a3 — b3 = (a — 6) ( a2 + ab + b2),
a3 + b3 = (a + 6) (a2 — ab + fc2).
2. The following inequality means th a t the arithm etic mean is
larger than or equal to the geometric mean
r
ajrb >
A
y^s, a> o, b> o.
The equality arises when a = b.
3. For a given quadratic equation
ax2 + bx + c = 0,

its two solutions can be found from the formula


— b ± V b2 — 4ac
2a
Si *
324 Appendices

or

*■•==-------------- 1-------------
The latter is more convenient when b is even.
For a reduced quadratic equation
I 2 -f- p x + q = 0,

its two solutions are given by

4. Suppose we are given a system of two equations in two un­


knowns:
f“i*-r l iy = cu
-p b t y —c..

In order to iind its solution, one unknown from one of the equ­
ations (say, the first) must be expressed in terms of the other unknown
( for instance, y in terms of x ) and substituted into the other equation:

Ci—All
y = — h— ’
Cj—
a t x + b t -S-r----- = c t .
bi
Solving this equation, we obtain

s ci&jj— c,6,
a l b 2 —a 2 b l

Substituting x into the expression for y , we find that

_ aica—a2ei
aib2—aa&l
The same method can be used to solve a syktem with a larger num ­
ber of equations: we express one unknown from one of the equations
of the system and substitute it into the remaining equations. This
reduces the number of equations and unknowns by one. Similarly, we
eliminate in turn the other unknowns until we are left with one equa­
tion in one unknown, which can be solved. The remaining unknowns
are determined in reverse order.
5. Some formulas for approximate calculations.
Appendices 325

If e < 1 (at least by a factor of 10), we have


1
= 1—e,
1+6

(l + e)2= l + 2e,
(1 —e)°-=l —2e,
(l + e)3= l + 3e,
(1 —e)s = 1 —3e,
j / l + e = l + - |- ,

^ = l - f
These formulas yield quite accurate results. For example, let us
calculate Y 1/3.96:

l/_ L
V 3.96 V 4 -0 .0 4 - 0.01
Y * {'-* ? ■ ) 2,,)
1 1 1
2 1 —0.005
2 1
( < - ™ )

= (1 + 0.005) = 0.5025.
A

G e o m e tr y

1. Given a right triangle with sides a and b and hypothenuse c.


Then
a2 + 6* = c2.
2. The area of a triangle is

s = T ah'
where h is the height dropped on side a from the opposite vertex.
The area of a rectangle with sides a and b is
S = ab.
The area of a trapezium with bases a and b and height h is

S = — (a + 6) h .
32G Appendices

The area of a circle of radius r is


S = nr".
3. The area of the surface and the volume of a sphere of radius r
are
S = 4nr: , V = \O n r 3.

The area of the lateral surface of a right cylinder with radius r


and length h is
S = 2nrh.
The total surface area and the volume of the right cylinder are
S = 2n r ( r + h ) , V = n r * h .

Trigonometry
1. The trigonometric functions of the sums and differences of
angles are
sin (a + P) = sin a cos p+ cos a sin P,
sin (a — P) = sin a cos p— cos a sin p,
cos (a + P) = cos a cos p— sin a sin P,
cos ( a — P) = cos a cos p+ sin a sin p.
2. The functions of double the angle are
sin 2 a = 2 sin a cos a
cos 2 a = cos2 a — sin 2 a .

3. The functions of half the angle are


„ . „a .
2 sin 2 — = 1 —cos a ,

CL
2 cos2 — = 1 + cos a.

4. The sum and the difference of the functions are

, . Q _ . a+p a —P
sin a + sin p = 2 sin — cos — »
* a+ P cl—P
sin a —sin p = 2 cos — sin — »
. a _ a+ p a —P
cos a + cos p = 2 cos — 5-*- cos — »
„ n . a + P . a —P
cos a — cos p = — 2 sin — 7 — ~ si n--------
9
Appendices 327

5. The trigonometric functions in terms of the tangent are


tan a 1
sin a = cos a = —
Y 1 + tan* o ’ V 1 + tan* a '

6. Cosine theorem: for an arbitrary triangle with sides a, ft, and c,


we have
a* = ft* + c* — 26c cos A ,
where A is the angle opposing side a .
The sine theorem:
a b l

sin A ~ sin B ~ sin C ’


where A , B , and C are the angles opposing sides a , 6, and c res­
pectively.
7. For e c 1, we have
sin e = e,
cos e = 1,
tan e = e.
8. Some derivatives and integrals:
(in)’= nin-1p

(pr) = —pzr>
_ j __
(V*)'
2V x ’
(sinx)' = cosx,
(cos x)' = —sin x,
x” d x =
»+l
dx
— = ln|x|,

^ s in x d x — —cosx,

^ cos x dx = sin i .
PERIODIC TABLE
«
V
o


o
Groups of
£E
a.• 1 ii 111 IV V

(H)
1 I

Li 3 Be 4 8 B 8 C 7 N
2 II 6.94,
Lithium
9.01218
2 Beryllium ?53 22 3a 2p’
2s* i _
10.81
Boron a
4 2p3
12.011
Carbon 2
5 2p3
14.0067
Nitrogen
Na 11 Mg 12
13 14 Al Si15 p
3 III 2296977
_
Sodium
i 24.305
3*1 a
2 3
39> 0 0 3p'
26.98154 «
. .
2 Magnesium a a Aluminium S
28085$ 5 , 3097376
0 3pJ
Silicon 3 Phosphorus
K 19, s, Ca 20 Sc 21 Ti
22 V 23
IV 39.098. 40.08 , 0 44 9559 9 47.90 3d,34s,3 £, 50.9415 , , ,1
4*1 0 4*a 0 M 'n ' a
Potassium ? Calcium 2 Scandium 3 Titanium a Vanadium a
4 Zn 1,3i
,29 CU £a3o 65.38 Ga 32 33 Qe As
V 0 69.72 M , 725, £ 74.9216
• M«4..63M ‘ 0 4pi a 4p» 0 4p3
a Copper S 4,1 Zinc 2 Gallium a Germanium 3 Arsenic
Rb 37,n Sr 38, Y 39 , Zr
4 91.22 40£^ 92Nb 9064
4i,i,•
t 10 88.9059
4d'5,» •
VI 65.467e 87.62
5s' 0 5»* 9 4d«5s’ S
Rubidium 2 Strontium 2 Yttrium a Zirconium5’ a Niobium 2
5 in
, 4' 107.866
Ag 448 IS
Cd
11241 IS
3 49 ,;50 118
H4.82 \l
Sn69 ,i5
iS
1 Sb 1217$
VII ii
0 s«>
2 Silver a Cadmium
• Sp>
0
2 5*9
Indium
*
3 5p3 .
Tin
*
a
5p* . ..
Antimony

Cs 55;ISBa La* 5 7 1 Hf 72j Ta 73-?


56j,a 138905s
VIII 1329054
09' 0
137.33
699 0
If 178.4, se7603la0 180947,54300* ISt
Sd'6j- e
Cesium 2 Barium 2 Lanthanum a Hafnium a Tantalum 3
6 »79 Au 5* 80 200.59 Tl 4 8 2 p b
IX ,e 196.9665 1
Hg « 4 81
204 3, IS 207 2 ”
0j
208 9804
•2 to3 Mercury a0 6pi Thallium
55d,°*1Gold I 60’ Lean a 8 6P3 «■
Bismuth
Fr 87 1 2260254 Ra 08 ,5 Ac** 891 Ku 104 s
105 ,?
33
7 X 1223]
Francium
II «
a Radium
33
793 ’® 12271 4 4 .^ 1 |26" surra-?;
2 Aciinium a Kurchatovium a
6d3703 ’•
a
* L A N T H A N ID E S
C e 58 S P r 59 s N d 601P m 611 S m 6 2 1E u ^ j G d 641
1 4 0 .9 0 7 7 '5 1 9 6 , IS
't o l ^ 4t36ss S ,442A Sr'S M45U , . ’s ,5° t , . S s - ' S 0 ■ s & w :
4 f'A s ?
C e r iu m a P r a s e o d y m iu m a N e o d y m i u m a P r o m e t h iu m a S a m a r iu m a E u r o p iu m a G a d o li n i u m a
k
** A C T IN ID E i
T h 9 0 4 P a 9i j U 92 j N p 93 J P
9
u 94
w
> A m 9 5 ,- C m 9 6 j
a*
2 3 2 .0 3 8 1 m 2 3 1 .0 3 5 9 « 2 3 7 0 4 8 2 « 1244] M (2 4 3 1 «
5 f36 d '7 s 3 S 5»46 4 '7 s 2 ‘I 51®7s? ’§ S f ' 79 * " ,2J M
TK
T h o r iu m *a P r o t a c t i n iu m a U ra n iu m a N e p tu n iu m : P lu to n iu m 1 A m e r ic iu m a C u r iu m a
OF THE ELEMENTS
elements
VI VII VIII
1 H 2 He
is’ 1.0079 , 400280
i$7
1 Hydrogen 2 Helium
8 0 9 F 10 Ne
159994 18998403 20.17«
6 ?p« ^ » rp» 6 ?P«
7 Oxygen 7 Fluorine 7 Neon
16 S 17 Cl 18 Ar
6 32 06 7 35453 e 39.94a
e 3p4 8 3pJ 6 3p®
3 Sulphur 2 Chlorine 2 Argon
Cr 24 Mn 25 Fe 26 Co 27 Ni 20
51996 t 13 54.9380 is 55.84, , 58.9332 ii
3ds4s' a 3ds4s7 8 3d6 4s? e 3d'4SJ I 58 7 0 3d»4s7 'i
Chromium 2 Manganese 2 Iron 2 Cobalt 2 Nickel 2

t 34 Se ,3 5 Br 9 36 Kr
ii 4 78.9a 18 79904 is 83.80
8 4p4 8 4p* 8 4p®
2 Selenium 2 Bromine 2 Krypton
Mo « Tc 4 3 j Ru 44 , Rh 45 Pd 46
95 94 ^ £ 98 9062 £ tOl.O? , 3 102 9055 H
40’’5s1 8 i 4d/S»* 8 4d®5s' c ,06'4 « , 5so'S
Molybdenum 3 Technetium 2 Ruthenium 2 Rhodium 2 Palladium 2

.5 2 Te , 5 3 |
ie Sp* , i2 7*o w 1269045 IS, s 54 Xe
13130
8 8 Sp' 8 5p®
2 Tellurium 2 Iodine 2 Xenon
w
m asj
74 i R e
” 186207
75 j O s
,0 190 2
76 i l r
ie 192.2?
77
,
i
,® Pt
1950s
78»
10
m Sd* $ » 7 5<JS6» 7 8 W*657 8 5d76s7 8 _ 5d*6 e
Tungsten
8
2 Rhenium 2 Osmium 2 Indium 7 Platinum 3 »7

Ji 84 P o »,|8 5 A t « 88 Rn
209 12101 ” Bp’ _ . 12221
8
3
to4 Polonium 2 p* 8 6
Astatine
8
2 Radon
A tom ic m oss^ Atomic n u m b e r Atomic mosses conlonn with rhe Iniemctionol Tibfe

O iirrib u rio n ol elections


,U 92j .. .
-D istrib u tio n
ol 1977
,$ +|the c tw o ty ol the lest vgnifuonr dig:!
cr i ] il il is set in jmcll type the numbers
ol e le c tro n s rn browns me the most numbers of the mnsl stnble
by unfilled ond fo llow ing-
U ranium 7 by levels isotopes Ihe nomes ond symbols ot elements m
co m p le te d su b le v e ls poreniheses ore not gerteroUy adopted

Tb65j Dy66 i Ho6 7 1 Er ea51 Tm69sYb70 =Lu 71l


41“ 1 41'W8
T erbium 2
™9
J & i '73X A ma&'A
1 6 2 .5 0
6S1
« 1 6 4 .9 3 0 4

O y sp ro siu m 2 H olm ium


2

2
1 6 7 .2 e

Erbium
4 ft t 6S 7 B
2 T hulium 2 Y tterbium 2 L u tetium 2

Bk97j Cf 98j Es 99J Fmtoos Md101,?INo)to2i (Lr)to3j


l23W-S ,25W ’i 1254i,-^’S
B erh eliu m 2
t2S6X A l25%rSl
C alifornium 7 E in stein iu m 2 F erm ium 2 Mendeievtum 7 (Hobeiium ] 2 (Lawrencium ) ?
Answers

1.1. Brownian movement and diffusion. 1.2. Molecular adhesive


forces in liquids are much stronger than in gases. 1.3. During painting,
the particles of a paint diffuse into pores (intermolecular spaces) of
the surface being painted. 1.4. In the region where the salt dissolves
a high salt concentration is created. As a result, diffusion takes place
to regions with a lower concentration. 1.5. The end faces of Johansson
blocks are highly polished; for this reason, molecular adhesive forces
come into play. 1.6. Gluing is based on molecular adhesive forces
and wetting. 1.7. About 316 mol. 1.8. 24 kg, 28 kg, and 4 kg. 1.9.
1.6 kg. 1.10 . 6.02 X 1029 and 6.02 X 1023. 1.11. About 2600 m3.
1.12. About 0.14 mol, 8.2 X 1022. 1.13. About 5.7 X 10-’ m3.
1.14. The mass of 1 m3 of brass is 8500 kg. 1.15. 105 mol. 1.16. 5.32 X
10-29 and 2.66 X lO"29 kg, 4.65 X 10-29 and 2.33 X 10*29 kg, 6.64 X
10-27 kg. 1.17. 16 X 10-3 kg/mol, 2.66 X 10’29 kg. 1.18. 1.46 X 1020.
1.19. About 1.54 X 1019. 1.20. The ratio is approximately 2 X 1014.
1.21. The ratio is 14.3 (considering that equal volumes of gases under
normal conditions contain the same number of molecules). 1.22.
7.3 X 10-29 kg, 1.97 kg/m3. 1.23. 9.9 X lO"29 kg. 1.24. 2 X
10-3 kg/mol. 1.25. 4.2 X 10-° m. 1.26. 1.26 X 10-25 kg, 1.01 X
10-28 m3, 5.8 X 10-10 m. 1.27. 9 X 109 m, 225 times. 1.28. 16 X
10~3 kg/mol. 1.29. 1.4 X 10-5/F 0, or 6.3%, where K0 is the molar
volume of the gas under normal conditions. 1.30. 4.0 X 10“8 m.
1.31. 1.12 X 10-4 s-1. 1.32. 550 m/s. 1.33. 6.5 X lO"8 m. 1.34. 9.3 X
10-8 m. 1.35. About 2 X lO"8 m. 1.36. 2.1 X lO"23 kg-m/s. 1.37.
About 3.9 X 10~8 m.

2.1. 470 m/s and 510 m/s for air and 450 m/s and 480 m/s for oxygen-
2.2. The mean kinetic energies of translational motion of helium and
neon are the same. 2.3. At 322 K. 2.4. 2.25. 2.5. About 631 and
8640 m/s. 2.6. 6.6 X 10~22 J, 1.2 X 10-19 J. 2.7. About 3860 K.
2.8. 1.25 X 104 J, 6.2 X 105 J. 2.9. 290 K. 2.10. 9.9 X 10"11 Pa,
high vacuum. 2.11. 1.45 X 1026. 2.12. About 6.5 X 10"21 J, 9.3 X
1029. 2.13. 1.3 X 10-9 Pa. 2.14. 1350 m/s, 6.07 X 10‘21 J. 2.15.
3.6 X 1012 molecules/cm3. 2.16. 105 Pa. 2.17. 1.3 X 1022. 2.18. 2.1 X
10-21 J. 2.19. 3.6 X 1021. 2.20. By 1.7%. 2.21. 4.46 X 1025, 148 g.
2.22. The thermodynamic temperature and the kinetic energy have
increased fourfold.
Answers 331

3.1. About 1.2 1. 3.2. About 64.4 kPa. 3.3. 323 K. 3.4. 614 1. 3.5.
1.82 m3. 3.6. 44 X 10~3 kg/mol, carbon dioxide. 3.7. 1.2 mol. 3.8.
About 2 kg. 3.9. 1.4 mol. 3.10. 4.7 X 10~3 m3. 3.11. 2.6 times.

0 T O T

Fig. 153

3.12. About 3.5 kPa. 3.13. 513 K. 3.14. 1.62 mol, 195 kPa. 3.15.
3 MPa. 3.16. 58 X 10"3 kg/inol, 2.2 X 1025. 3.17. 932 K .3.18. Hydro­
gen. 3.19. I l l kg. 3.20. 606 K. 3.21. 73.3 MPa. 3.22. About 10s Pa.
3.23. 1.2 kg/m3. 3.24. 0.46 kg/m3, 9.OX
1024 in-3, 444 m/s. 3.25. 28 X
lO-3 kg/mol, nitrogen. 3.26. 1.66 kg.
3.27. 0.7 MPa. 3.28. 2 X 1U5 Pa. 3.29.
See Fig. 153. 3.30. 53. 3.31. See Fig. 154;
(1) in joules, (2) isotherm 2 for the second
equation will lie further from the F-axis
since it corresponds to a higher temper­
ature. 3.32. 296, 963, and 17.6 g. 3.33.
See Fig. 155. 3.34. Isobar 1 corresponds
to the process occurring at a higher pres­
sure. An arbitrarily chosen temperature
T 1 corresponds to two values Ft and F2
of the volume, and hence to two values
p, and p 2 of pressure. But the product of Fig. 154
pressure and volume at a constant tem­
perature is constant, i.e. piFi = p 2F2. Therefore, volume V l on
isobar 1 corresponds to a higher pressure. 3.35. The gas leaks. 3.36.
8.5 1. 3.37. To 225 K. 3.38. By 82 K. 3.39. See Fig. 156. 3.40.
38.3 kN.
4.1. 250 J. 4.2. 380 J. 4.3. Aluminium, 1.9. 4.4. 4187 J, 460 J. 4.5.
The specific heat of copper is larger than that of aluminium, the spe­
cific heat of lead is the smallest of the three. 4.6. Copper weight will
be heated to a higher temperature. No, it does not. 4.7. The energy
required to heat a gas at constant pressure is spent both to increase
the internal energy of the gas and to do work of expansion. 4.8.
25.5 MJ. 4.9. 1 kg. 4.10. 1.73 X 108 J. 4.11. 1026 J. 4.12. 86 1.
332 Answers

4.26. 24 K. 4.27. 0.7 K. 4.28. 1.9 X 10® m3. 4.29. 225 MJ. 4.30.
187 K. 4.31. To about 78 m. 4.32. About 0.2 K. 4.33. 0.11 K. 4.34.
25. 4.35. 19 K. 4.36. 174 m/s. 4.37. 250 K. 4.38. 29%. 4.39. 30%.
4.40. 1029 kg. 4.41. About 24%. 4.42. About 17 kW. 4.43. About
550 km. 4.44. 4.28 kJ. 4.45. 26.4 k J . 4.46. 831 J. 4.47. 7462 J, 1662 I.
4.48. 48.1 kJ. 4.49. 32 X 10~3 kg/mol, oxygen. 4.50. 416 J. 4.51.

la) to 10
fin p h

k.
0 0 r
Fig. 155

106 J. 4.52. 1.8 kJ. 4.53. 8.31 J; no, it does not. 4.54. 0.31 kg. 4.55 .
Yes, it can, lor example, during adiabatic expansion or compression.
4.56. The pressure will change more during the adiabatic compression.
5.1. This can be done if the pressure is reduced somehow. 5.2. No,
they are not; the internal energy of steam is higher. 5.3. In such
vessels, evaporation takes place both from the water surface and through
the vessel walls, which contain pores to facilitate cooling. 5.4. Tne
pressure gauge indicates the excess pressure over the atmospheric
pressure. 5.5. The number density of molecules increases. 5.6. At
night, when the air temperature falls considerably, the water vapour
in the air saturates it and partially condenses to form water drops, i.e.
mist. 5.7. These substances have different boiling points. 5.8. Boiling
water quenches fire more quickly. 5.9. No, it is not. 5.10. The work
of expansion of the gas is done at the expense of a decrease in the
internal energy of the gas, as a result of which the temperature of the
released gas drops, ana the valve is covered by frost. 5.11. The differ­
ence between the humidities of air in the two climates determines
different evaporation rates from the skin. 5.12. No, it is not. In the
Answers 333

critical state, the specific latent heat of vaporization is zero for all
liquids. 5.13. 300 J. 5.14. The energy liberated during the conden­
sation of water vapour is about eight times more than that liberated
during the condensation of mercury va­
pour. 5.15. 11.3 MJ, 2.1 MJ. 5.16. 4.0
kJ. 5.17. 2.6 X 105 J. 5.18. 100 g. 5.19.
36.6°C. 5.20. 53 kg. 5.21. About 10°C.
5.22. 71.6°C. 5.23. About 3.3 kg, see
Fig. 157. 5.24. 84 kJ. 5.25. 2.25 X
10* J/kg, 0.01 X 106 J/kg, 0.44%. 5.26.
334 g. 5.27. 2.32 kJ. 5.28. 0.04 m3.
5.29. 17 min. 5.30. 51.8 kg. 5.31. 6.5 kg.
5.32. 35%. 5.33. 10*2 kg/m3, 78%.
5.34. 19.4 X 10-3 kg/m3, 22°C. 5.35.
17.3 X 10-3 kg. 5.36. 1°C. 5.37. At
about 7°C. 5.38. 9.4 mg, 65%. 5.39. To
284.5 K. 5.40. 9.4 X 10-3 kg/m3, 48%.
5.41. At 4°C. 5.42. 18°C. 5.43.
about 11 g. 5.44. 17°C, 13°C. 5.45. 19°C,
the relative humidity increases. 5.46.
0.4 m3. 5.47. It decreases to 54%. 5.48.
1.21 kg, 271 g. 5.49. 2.33 kPa. H i n t . p s = p J B , pa can be determined
from tne equation of state: pa = m R T / M V , p s = m R T / M V B . 5.50.
12.3 X 10~3 kg/m3, about 48%. H i n t . Solve the problem by using
the equation of state.

6.1. The surface tension changes: it decreases when soap is placed in


water and increases when sugar is placed. 6.2. Under the action of
the forces of surface tension, a pellet acquires the shape for which
the surface area is at a minimum. 6.3. Yes, it will. 6.4. Water is
a nonwetting liquid for a surface covered with grease. 6.5. See answer
to Problem 6.4. 6.6. This is done to create conditions in which the
solder is wetting. 6.7. A nonwetting liquid. 6.8. This is done in order
to prevent moisture from rising in tne capillaries of the building walls
6.9. The presence of capillaries in caked soil makes it possible for
water to rise closer to the surface. 6.10. See answer to Problem 6.9.
6.11. 9.6 X 10-3 N. 6.12. 8.4 X 10‘3 N. 6.13. 1.37 X 10-3 and
0.12 N .6.14. 1.56 X 10'3 N, towards the water. 6.15. 0.105 N. 6.16.
1.44 X 10-4 J. 6.17. 7.2 X 10~* J. 6.18. 3.15 X 10-* N/m. 6.19.
0.022 N/m, ethyl alcohol. 6.20. 2.9 cm. 6.21. 2.56 mg. 6.22. 15 mg.
6.23. In the test tube of pure water. 6.24. 1.14 X 103 kg/m3. 6.25.
0.7 cm.

7.1. Strong heating may recrystallize steel, which makes its mechan­
ical properties worse. 7.2. The shape of the single crystal will change.
7.3. Glass is brittle. 7.4. The melting point of lead is considerably
lower than the melting points of other metals. 7.5. Lead can be melted
by heating it in a hermetically sealed vessel with water. The temper­
ature of the water can then be raised considerably above the melting
point of lead. 7.6. In summer, the temperature of the layers of air
close to the surface of the Earth is above 0°C, and small ice crystals
formed in the upper cold layers of the atmosphere melt on their way
334 Answers

to the ground. Larger crystals have no time to melt and reach the
ground in the form of hail. 7.7. The melting point of the mixture of
snow and common salt is lower than (PC, and hence snow melts even
below CPC. 7.8. 0.4 cm. 7.9. 1.5 mm, 7.5 X 10~4. 7.10. 4 mm, about
1.4 J. 7.11. It will decrease by a factor of four. 7.12. 9 mm. 7.13.
3 X 107 Pa. 7.14. 1.07 X 10“ Pa. 7.15. 5 X lO '3 m. 7.16. 9.4 kN.
7.17. 25. 7.18. 7.5 cm2. 7.19. About 4.2 km. 7.20. 3 J. 7.21. 0.3 mm.

7.22. No, it will not. 7.23. Alcohol has a lower freezing point. 7.24.
About 159 kJ. 7.25. About 380 kJ. 7.26. 25.1°C. 7.27. =
P]A.] : pt^t = 0.67. 7.28. 320 K, see Fig. 158. 7.29. About 12 g, see
Fig. 159 . 7.30. 3.5 X 105 J/kg. 7.31. 19 kg. 7.32. 227 kg. 7.33. 17%.
7.34. About 80 kg. 7.35. 4.5 W. 7.36. 20% 7.37. 20 g. 7.38. 0.9 kg.
7.39. 0.8. 7.40. 3.35 X 10s J/kg. 7.41. 97 g.
8.1. The alloy has a very small coefficient of linear expansion and
it is used to ensure that the mechanism is accurate independently
of the thermal conditions. 8.2. The oval hole allows for linear expan­
sion during which the gap between the rails is reduced. 8.3. Different
coefficients of linear expansion would weaken the tube during its
operation (when it is heated); no, it cannot. 8.4. It will increase.
8.5. The sleeve should be heated. 8.6. The relation is approximate.
H i n t . We write the expression for a unit volume after heating a body
through I K : 1 + P = (1 + a)3. After raising the right-hand side
to the third power, we can neglect the terms 3a2 and a 3 in view of
their smallness, which gives p = 3a. 8.7. This is due to the relatively
small thermal expansion coefficient. 8.8. Thermal expansivity should
be taken into account. Its value for liquids is much larger than that
for the material of which the tanks are made. ,8.9. During heating,
stresses may emerge in teeth, which causes cracks in the enamel.
8.10. The strip will bend towards the metal with the smaller coefficient
of linear expansion; such strips are used in thermal relays used to
control temperatures automatically. 8.11. By 12 mm. 8.12. 10 K.
8.13. Tungsten. 8.14. 533.128 and 532.872 m. 8.15. About 903°C.
8.16. By 417 K. 8.17. To 391 K. 8.18. The aluminium component
is 330 m longer than the steel component. 8.19. 1.99 m. 8.20. 739.2 kN.
8.21. 15 K. 8.22. 79.2 kN. 8.23. 500°C. 8.24. About 4 kJ. 8.25.
37 kN. 8.26. 16.2 cm3. 8.27. This relation is approximate. 8.28.
Answers 335

0.306 m2. 8.29. 46 K. 8.30. 0.09 1, 0.087 1. 8.31. 0.08 1. 8.32. 1.85 X
103 kg/m3. 8.33. 400 K. 8.34. About 3.6 kg. 8.35. 299 K. 8.36. 9.48 1,
7.49 kg. 8.37. 9.49 1, 7.5 kg. 8.38. 57.8 m3, about 40.5 t. 8.39. 2.14 m3.
8.40. About 3.6 m3. 8.41. 1.8 X 10~4 K + 8.42. 42.5 kJ.
K
9.1. Two. 9.2. + 4 .8 X 10~19 C. 9.3. No, it cannot. 9.4. This can be
done if the end of the rod held in the hand is coated with an insulat­
ing material. 9.5. The sphere should be brought in contact with two
similar neutral spheres. 9.6. The static charge that may be formed as
the car moves is conducted to the earth through the chain. 9.7. Yes,
it can. 9.8. The surface charge density will increase. 9.9. At first
the ball will touch the electrically charged rod and then will be re­
pelled from it. 9.10. No, it will not. 9.11. 0, 2. 9.12. 1.44 mN, by
a factor of 81. 9.13. 2.1 x 10-" C. 9.14. 9 GN, 4.5 GN. 9.15. 95 km.
(a)

9.16. (1) 6-9, (2) in kerosene and wax paper, (3) to water. 9.17. Yes;
it will decrease by a factor of 3.2. 9.18. The drops will approach each
other with an initial acceleration of 3.4 m/s2, which will increase
further as the drops approach each other. Yes, it will. 9.19. 1.84 X
10~7 N. 9.20. 3.45 X 1011, about 0.01 N, ves, it will; see Figs. 160a
and b . 9.21. 2.25 X 10~4 N. 9.22. 5 X 10“7 C/m2. 9.23. 4 X 10-« C.
9.24. 4 x 10-7 and 2 X 10~7 C, 0.42 m. 9.25. 4.2 X 1042. 9.26. 1.77 X
10"u F/m, 5.2 X 10-7 C. 9.27. No, they do not. 9.28. This is done
for electrostatic shielding. 9.29. (a) yes, they can, (b) no, they cannot.
9.30. The field strength will be greatest where the curvature of the
surface is the largest, namely, at a point. For this reason, charge may
leak from the point, forming an “electric wind”. 9.31. See the answer
to Problem 9.30. 9.32. 0. 9.33. It means that the electric field acts
on a charge of one coulomb placed at this point with a force of 300 N.
9.34. 7 X 103 V/m. 9.35. 1.6 X 10‘5 N. 9.36. 3.2 X 10"14 N. 9.37.
3 X 104 V/m. 9.38. 2.2. 9.39. 1.44 X 105 V/m; the field strength will
be doubled. 9.40 . 3.75 X 104 V/m, 42 nC. 9.41. 104 electrons. 9.42.
Concentric spherical surfaces. 9.43. No, it will not. 9.44 . 9 X 103 V/m,
2.7 X 104 V/m. 9.45. 1.71 X 104 V/m. 9.46. About 3.6 X 105 V/m.
9.47. 1.13 kV/m. 9.48. (a) 0, (b) 0, (c) o/e0e. 9.49. 0.1 mg. 9.50.
9.93 m/s*. 9.51. 0, ]/ 2<?/(16jie0e/?2). 9.52. Due to electrostatic charg­
ing by induction. 9.53. About 5.9 X 105 C, 8.3 X 10s V. 9.54. 5.6 kV.
336 Answers

9.55. 38 V. 9.56. 3 X 10"4 J. 9.57. 6 kV, 6 kJ. 9.58. 1.44 X 10® V.


9.59. 2.4 X 10-® J, 150 V. 9.60. 2.56 kV, 4.1 X 10-1®J. 9.61. 3.5 X
104 V/m, 2.4 kV. 9.62. 36 V. 9.63. 1.8 kV. 9.64. 8 cm, 10'7 s. 9.65.
Yes, it can be changed by bringing another conductor close to the
charged one. 9.66. 2C . 9.67. 9 X 10° km, 7?sph//?Earth — 1400.
9.68. The capacitance, charge, and energy of the capacitor will in­
crease, while its voltage will remain unchanged. 9.69. 1.1 X 10~n F,
1.1 X 10~5 pF, 11 pF. 9.70. About711 pF. 9.71.4.5 cm, 3 X 10~7 C/m2.
9.72. The oxide film used as the dielectric in these capacitors is very
thin. 9.73. About 455 and 1786 V; the charge from the smaller sphere
will be transferred to the larger sphere until the potentials become
equal. 9.74. About 100 pF. 9.75. 1.8 mm. 9.76. 6.6 X lO*10 C. 9.77.
1.1 X 1 0 '7 C; it will decrease by a factor of six. 9.78. 6 mm. 9.79.
5 X 10-a F. 9.80. 3.8 X 10"s J. 9.81. (1) 1.09 pF, 2.2 X 10'2 J,
(2) 12 pF, 0.24 J. 9.82. 0. 6 pF. 9.83. 0.24 pF, 2.4 X 10'5 C. 9.84.
300 V. 9.85. 100 V, 10"2 C, 0.5 X 10~2 C. 9.86. 1 pF. 9.87. The ca­
pacitor with the higher capacitance. 9.88. 36 J, 13 kW. 9.89. 4.8 pF,
9.6 X 10~4 C. 9.90. In the second case. 9.91. C = ljl^°eri!; for
Ar
vacuum, 555 pF. 9.92. 0.215 J/m3.

10.1. 7.5 X 1016. 10.2 . 6.3 X 1021. 10.3. 103 A. 10.4. 3 X 1023 c m '3.
10.5. 1.1 X 10-4 m/s. 10.6. 3.6 X 10"2 V/m, 518 m. 10.7. 5.4 X
10-4 m2/(V -s). 10.8. 2.5 X 104 A/m2; yes, it is. 10.9. 2 A/m2. 10.10.
About 23 mm. 10.11. 6.2 A/mm2. 10.12. 0.054 0; it will be half.
10.13. Nichrome wire, 2.5 limes. 10.14. The current in the tungsten
wire is 1.87 times larger than that in the steel wire provided that
the internal resistance of the accumulator can be neglected in view
of it being small. 10.15. The resistance decreased by a factor of four.
10.16. 12 pF. 10.17. 1.2 A, about 42 Q. 10.18. 272 m. 10.19. 60 0.
10.20. 736 m. 10.21. 2.7 mA. 10.22. 3 V. 10.23. 2.7 V, 2.67 A/mm2.
10.24. 15 mm2. 10.25. 7.5 kN. 10.26. 0.017 0. 10.27. The resistance
of the nichrome wire is larger by a factor of 3.125. 10.28. Steel.
10.29. Will increase by 3.24 O for cast iron and by 0.34 £2 for the ferro-
aluminium high-resistance alloy. 10.30. The temperature resistance
coefficient must be as large as possible. 10.31. 230 O. 10.32. 25 K.
10.33. 6 0. 10.34. 250 K. 10.35. 558 V. 10.36. 146 mm2. 10.37. About
1260 kV. 10.38. / 2 > 1 1 , the resistance will increase; the current in
the circuit will decrease. The readings on ammeters A, and A t will
be the same. 10.39. 4 O. 10.40. 0.033 0. 10.41. 22 O, 10 A, t/, = 50 V,
U 2 = 70 V, U 2 = 100 V. 10.42. 200. 10.43. 240, 30. 10.44. 3.24,
2.12, 1.2, and 0.37 A. 10.45. 0.03Q. 10.46. Into two parts; in parallel.
10.47. 25 Q. 10.48. 44 0 , 5 A and 0.5 A. 10.49. 0.64 0 ,3 .2 ,1 .6 , and
0.2 A. 10.50. 1.8 0 and 1.2 0 . 10.51. 0.55 A, 0.275 A, 1.65 A, and
0.275 A. 10.52. 6 O, / , = 2 A, / 3 = f 4= l A. 10.53. 2.1 O, 21 V,
U l =12 V, I a = 7.5 A, U 3 = 9 V, / 2 = / 4 = / 5 = 2.5 A, f/2 = U . =
U h = 3 V. 10.54. The resistance of the cold filament is lower than
that of the hot filament. Consequently, the switch-on current is larg­
er than the switch-off current. 10.55. 4 0 , 3 A. 10.56. 508 V, 488 V,
475 V. 10.57. 27 V. 10.58. 4.5 0 , 2 0 , 1.5 0,0.67 0 , 0.46 0 ,0 . 10.59.
Answers 337

About 2.3 Q, 52.5 A, = 22.5 A, / 2 = 30 A, / , = / = / , = 7.5 A.


10.66. 20 A, / , = 5 A, / 2 = 15 A, I a = I t = 2.5 A, I b = / , = 10 A,
/ , = 7.5 A. 10.67. 2.5 X lO 3 Q. 10.68. 60 A. 10.69. 400 Q. 10.70.
10 kQ. 10.71. 250 Q, 225 V. 10.72. 17 m. 10.73. 5. 10.74. It measures
the voltage of the external circuit. 10.75. 1.5 V, 0.24 V. 10.76. 1.93 V,
0.25 Q. 10.77. 0.29 A, 1.02 V, 70%. 10.78. 230 V, 229 V. 10.79. 10 A.
10.80. 0.1 Q. 10.81. 10-* C. 10.82. 1.5 X 10'* C. 10.83. 12 V, 2 Q.
10.84. 147 V, 120 V. 10.85. 0.6 A, 3 A. 10.86. 225 V. 10.87. 236 V,
220 V. 10.88. 222 V, 21 mm2. 10.89. 535 V. 10.90. 230 V, 195 V.
10.91. 2 A, 540 V. 10.92. (a) 1.2 A, (b) 17.5 A, (c) 4.7 A. 10.93. 0.29 A,
0.8 A. 10.94. About 3.6 A. 10.95. 0.6 Q. 10.96. The internal resistance
of acid accumulators is very small, and therefore a large current emerg­
ing in a circuit may cause the plate to break down. 10.97. 2.5 A.
10.98. 1.25 A, 0.75 A. 10.99. / p a r s e r = 3. 10.100. They should be
connected in two parallel groups of three series-connected cells.
10.101. They should be connected in three parallel groups of four se­
ries-connected cells. 10.102. 1.3 V, the emf will be reduced by a factor
of seven. 10.103. Series connection. 10.104. It is more advantageous
when the external resistance is higher than the internal one. 10.105.
52 V, 50 V. 10.106. 3.42 V, 3.36 V. 10.107. 0.4 Q. 10.108. 0.77 A,
I . 26 V. 10.109. / = -------------------- , = 1 .2 4 A.
V t + 7? (ri + r2)

I I . 1. 18 C, 216 J. 11.2. 317 MJ, 88 kWh. 11.3. 1125 kWh, 5 rubles


76 kopecks. 11.4. 91%, 1 ruble 6 kopecks. 11.5. 800 MW, 7 X 10* kWh.
11.6. 176 kWh. 11.7. 234 MJ, 65 kWh. 11.8. 48 W. 11.9. 4 Q, U C D =
0, / , = 0, 144 W. 11.10. 54.5 A. 11.11. 8.4 kW. 11.12. 1.35 MW.
11.13. 2.7 MW. 11.14. (a) 13.8 MJ, (b) 8.64 X 10* J, (c) 3.46 MJ.
11.15. 82.5 kW, 74.25 kW. 11.16. 102 kW, 34 kW. 11.17. 125 kW,
110 kW, 0.88. 11.18. 550 W, 5.5 kWh. 11.19.2.8 mm2. 11.20. The
power is higher for the parallel connection. 11.21. 16. 11.22. 0.16 m/s,
12.5 s. 11.23. 63.4 km/h. 11.24. 462 A. 11.25. The melting point of
lead is relatively small (327°C). 11.26. The fuses have different cross-
sectional areas. 11.27. The coil of the 127-V hot plate is thicker.
11.28. 570 MJ. 11.29. (a) 110 V, 60 W, 0.22 MJ, (b) 88 V, 132 V; the
40-W bulb is under a voltage higher than the nominal value; 38.4 W,
57.5 W, 0.14 MJ, (c) 138 V, 82 V; the 60-W bulb is under a voltage
higher than the nominal value; 94 W, 56 W, 3.38 X 10* J, 0.2 MJ.
11.30. 26 g. 11.31. 79%, 0.8 kopecks. 11.32. 2 Q. 11.33. 24 V, 0.1 Q.
11.34. 2.64 kW, 90%. 11.35. The wire should be shortened to 3 m.
11.36. 20 or 80 Q. 11.37. 15 kg.
12.1. No, we cannot. 12.2. No, it cannot. Otherwise, the oxide film
would be destroyed by electrolysis. 12.3. Current reversal means that
its direction changes. When the article becomes an anode during the
current reversal, the uneven regions (protrusions) are dissolved more
vigorously and the edges or surfaces are smoothed over; yes, it can.
12.4. 9.32 X 10-9 kg, 1.118 X 10 6 kg, 0.33 X 10-" kg, 6.25 X 1018.
12.5. 283 mg. 12.6. 362 g. 12.7. 3.3 X 10-7 kg, 6.6 X 10'7 kg. 12.8.
2.96 X 10-* kg. 12.9. 3.33 X 10"7 kg/C, 0.004 X 10-" kg/C, 1.2%.
12.10. Copper. 12.11. Silver, 1. 12.12. 1.6 X 1 0 l* C, 3.2 X 10-»* C,
4.8 X 10-»* C. 12.13. 1.118 X 10-" kg/C, 6.8 X 1 0 7 kg/C. 12.14.
!iZ2 —0530
338 Answers

2.24 X 1022. 12.15. 4.13 X 10-3 kg, 1.88 X 1022, 1.26 X 1022. 12.16.
11.8 kg. 12.17. 64.5 GJ, 0.12 Q. 12.18. 278 A/m2. 12.19. 9.65, 18.4 g.
Iron is liberated at the cathode and chlorine at the anode. Positive
metal ions move to the cathode, while negative chlorine ions move
to the anode. 12.20. 9.4 h. 12.21. 52 min. 12.22. 2.6 X 104 kWh,
520 rubles. 12.23. 4.5 kg. 12.24. 12.5 h. 12.25. 2.7 X 10“9 m/s.
12.26. 312 K. 12.27. No, it was not. A correction of 0.1 A is required.
12.28. 1.045 X 10-8 kg/C, 2.38 X 10-? kg/C, 1.26 X 10'7 kg/C.
12.29. 50 C, 56 mg. 12.30. About 29 W. 12.31. 369 mg. H i n t . See
Fig. 161. The area of the trapezium is equal to the amount of elec­
tricity passing through the electrolyte from the moment when the
current starts changing. 12.32. About 28.2 A/m2. 12.33. 1.9 X 1022.
12.34. S Pol = U - £ R = 0.8 V.

13.1. As a result of recombination, charged particles are transformed


into neutral atoms, and the gas becomes an insulator. 13.2. O A is
the region of dynamic equilibrium between ionization and recombi­
nation (the region where Ohm’s law is ap­
proximately valid), A B is the region of the
saturation current (the current depends only
on the intensity of the ionizer), and B C is the
region of impact ionization (self-sustained
discharge). 13.3. (1) Charge carriers in a
gas are formed by an ionizer, while in solu­
tions they are formed due to the effect of
a solvent with thermal motion taken into ac­
count. (2) Gases exhibit electron and ion con­
ductivities, while solutions only have ion
conductivity. (3) Conductivity in a gas does
J00 t,S not obey Ohm's law. 13.4. The temperature
should be increased. 13.5. No, it is not. For
Fig. 161 a small mean free path, the electric field
strength should be high. 13.6. The large
current in the spark reduces the voltage across the electrodes,
which interrupts the discharge. 13.7. (a) Coronas formed around
high-voltage power lines; (b) the corona discharge can be used as a
filtre for the purification of flue gas. 13.8. Glow discharge; electrons,
gas ions, and mercury vapour. 13.9. When the gas is being rarefied,
the mean free path increases, and hence the energy W \ = e E k required
for ionization can be acquired at a lower electric field strength. 13.10.
The activation of the cathode (coating it with a layer of a more active
metal) reduces the work function of cathode electrons and saves energy.
13.11. The horizontal lines of the graph correspond to saturation cur­
rents for different temperatures of the filament. As the temperature
of the filament increases, the emission of electrons is increased. 13.12.
The electron beam can be controlled by using electric or magnetic
fields. In CRT tubes, this is done using parallel-plate capacitors.
Current-carrying coils on the neck of the tube are used to produce
magnetic fields. 13.13. In a gas under normal conditions, charge
carriers are absent, or their number is negligibly small. Plasma_ is
a mixture of electrically charged particles, such that the total negative
Answers 339

charge of the particles is equal in magnitude to the total positive


charge. The presence of charged particles in the plasma ensures its
electrical conductivity. This cannot be said of a gas. 13.14. 4.3 V.
13.15. It will increase by a factor of 1.2, 1.6 X 107 ra/s, 1.9 X
107 ra/s. 13.16. 1.03 X 10" m/s. 13.17. 1.03 X 107 m/s, 9.7 X 10"1®s.
13.18. 7.95 X 10s m/s. 13.19. 2.18 X 10® m/s. 13.20. 6.4 X 10"1®A/m2.
13.21. 3 pm. 13.22. 1.26 X 1013 n r 3. 13.23. 1.46 X 10-® ra2/(V.s).
14.1. Curve 1 . 14.2. Electrons and holes. 14.3. The number of holes
is equal to the number of electrons; no, it is not. 14.4. re-type (phos-
horus is pentavalent); p-type (aluminium is trivalent). 14.5. It will
S ecrease. 14.6. It will increase by a factor of 2.4 X 10*. 14.7. b =
2.5 X 10*3 m2/(V-s), re = 1018 m~3. 14.8. It is a semiconductor with
terminals to be connected to a circuit. For these thermistors, the
current does not depend linearly on the applied voltage. 14.9. Ther­
mistors are based on the temperature dependence of resistance, while
photoresistors make use of tne dependence of resistance on illumi­
nance. 14.10. A transistor is a semiconductor device in the form of
a crystal containing two p - n junctions. These regions are the base, the
collector, and the emitter. 14.11. If the thickness of the base is much
smaller than the mean free path, minority charge carriers getting into
it have no time to recombine. 14.12. / era = / b + / Coi- 14.13. The
application of semiconductor devices in radio engineering saves energy
and makes appliances more compact and durable.
15.1. If we mentally divide the current-carrying conductor into a
large number of elements, for each element there exists an element
in which the current has the opposite direction, and hence these two
elements repel each other. With such interaction between the elements,
the conductor will assume the circular shape. 15.2. Connecting the

Fig. 162 Fig. 163

voltmeter to points 1 and 2 (Fig. 162), we can determine the point


with a higher potential. Then we can bring a magnetic needle to one
of the conductors and determine the direction of current from the
deflection of the north pole. 15.3. See Fig. 163. 15.4. See Fig. 164.
15.5. See Fig. 165. 15.6. See Fig. 166. 15.7. The magnet will be re­
pelled by the solenoid and will rise. 15.8. The coil will be deflected
to the right. 15.9. For the indicated direction of the current, the loop
will be rotated clockwise if the north pole is on the right; through 90°.
15.10. 0.46 N. 15.11. 0.8 m. 15.12. 50 A. 15.13. 2.8 m. 15.14. 0.4 N,
0.2 N, 0. 15.15. 1.2 X 10~2 T. 15.16. We must bring the magnet
to the lamp. The filament in the a.c. circuit will oscillate and will
22 *
340 Answers

appear as blurred. 15.17. 0.42 N. 15.18. / = p S g / B \ from N to M .


15.19. 15.7 N. 15.20. 275 A. 15.21. 26 A/m, 3.25 X 10'5 T. 15.22.
5 X lO"5 T, 4 cm. 15.23. 7 X 10 5 and 1 X 10-5 T. 15.24. 3.14 X
10-» T, 6.9 cm. 15.25. 125 A/m, 10 A. 15.26. 16 A, 16 A/m. 15.27.
1.1 X 10-> A-m*.15.28. About 2.4 A, 16 A/m. 15.29. 5.7 X 10-«N-m.
15.30. 3 X IQ-4 T. 15.31. 7.5 X 103 A/m, 9.4 X 10'3 T . 15.32.

S N

Fig. 164

1.5 mm. 15.33. 800, 280. The permeability of steel decreases. When
the magnetization of a ferromagnetic attains saturation, the magnetic
induction increases only due to increasing magnetic field strength.
15.34. 0.5 T, 1.5 mWb. 15.35. 7.9 X 10~7 Wb. 15.36. 2 X 10~4 Wb,
2.4 X 10-4 J. 15.37. 144 mWb. 15.38. 7.2 X 10"4 Wb. 15.39. 0.2 T,

m (t>)

Ld Ld
-*■0 0*-

FI FI
Fig. 166

100. 15.40. At the initial moment, the force is directed vertically


downwards; a circle. 15.41. 3.0 X 10-l# N. 15.42. 1.2 m, 4.0 X 10~7 s,
2.5 MHz. 15.43. The velocity vector must be perpendicular to the
plane containing vectors E and B. By hypothesis, tne electron moves
uniformly in a straight line, and hence F L = 0, v = E / B . 15.44.
1.4 X 10-14 J. 15.45. 3.9 mm, 4.4 cm. 15.46. Tu = T la. 15.47.
Ti > re-
16.1. An emf will be induced in the case (a). 16.2. When the poles
of the magnet are connected (disconnected), the magnetic field in­
duction changes, which leads to the emergence of an induced current.
16.3. From N to M ; from M to N . 16.4. An emf is induced when the
frame enters the magnetic field or leaves it since in these cases the
magnetic Dux piercing the frame changes. 16.5. No. 16.6. The current
induced in the upper part of the frame is directed away from us in
the cases (a) and (d) and towards us in the cases (b) and (c). 16.7. The
Answers 341

second rod is nonmagnetized. 1G.8. The current through the galvano­


meter flows in the upward direction. 16.9. The induced current has the
counterclockwise direction. 16.10. When the magnet enters the cyl­
inder and emerges out of it, the magnetic field produced by the current
induced in the cylinder decelerates the motion. If the cylinder is
long, a = g in it. 16.11. Yes, it is. The galvanometer reading is 0
since the emf’s induced in each half of the conductor are equal in
magnitude and have opposite directions. 16.12. 3.4 X 10~2 V. 16.13.
6 A. 16.14. 1.4 T. 16.15. 0.46 V. 16.16. About 20 m/s. 16.17. 2 V,
0.5 A. 16.18. 30°. 16.19. 64 V. 16.20. 6 mWb. 16.21. 100 V. 16.22.
1.5 s. 16.23. 0.8 V, 8 mWb. 16.24. 0.4 Wb. 16.25. 0.1 H. 16.26.
A core made of a ferromagnetic must be inserted into the solenoid.
16.27. A self-inductance emf emerging during the disconnection of the
circuit has the same direction as the emf of the source, and their
joint action causes sparkling. A capacitor connected to the switch
eliminates this effect. 16.28. 7 A. 16.29. About 2.7 A. 16.30. About
2.7 J. 16.31. 0.216 J, 0.036 Wb, 0.72 V. 16.32. 1.5 X 10-s C. 16.33.
4 mA. H i n t . I = H R , * = —A<D/Ac, AO = A ( B l 2 ) . Since I 2
does not change, AO = l 2 A B , I = l 2 A B / ( R A t ) .

17.1. 5 s. 17.2. 2. 17.3. 2 s, 0.5 Hz. 17.4. 2.51 rad/s, 2.5 s. 17.5. 1 s,
1 Hz. H i n t . The time elapsed between the impact against the floor
and the moment the bounced ball attains the upper point is 772.
17.6. The frequency of vibrations will not change, but tne vibrations
will attenuate sooner. 17.7. 2 Hz, 0.5 s. 17.8. (a) In phase; (b) and
fc) in antiphase. 17.9. ca = 2n n / t = 3.46 rad/s. 17.10. 4.77 Hz,
0.21 s. 17.11.0.5 Hz, 2 s, 3.14 rad/s. 17.12. 4.25 m, 0.3 rad/s, 0.75 rad,
20.9 s, 0.9 rad. 17.13. 0, 2 cm, 0, —1 cm. 17.14. 2n rad, n rad, n/2 rad.
17.15.3.54 cm. 17.16. 0.85 m, 0.6 m, —0.6 m. 17.17. —1.04 m, 0.85 m,
0; see Fig. 167. 17.18. 4 m. 17.19. 14.2 cm. 17.20. 6 m, 2 Hz, 0.5 s,
—n/2 rad, x = 6 sin n (4< — 0.5). 17.21. 5.2 m. 17.22. n rad, 0.
17.23. (a) Acp = 3n/2 rad or Atp = —n/2 rad; (b) A<p = n/4 rad.
H i n t , (a) = 3nt/2 + 5n/2, = 3nt/2 + n/2 + n/2; Acp = (pt —
92. (I5) 9i = ca//4 + 7n/4, q>2 = cat/4 — n + n / 2 . 17.24. (a) x =
2 sin n (t + 1/3), J_b) x = / f s i n (5n/6) (t — 1). H i n t , (a) x = A X
sin (cat + <p0), l / 3 = A sin q>„, 1 = A sin [(2n / T ) (774) + cp0] = A X
sin (n/2 + <p0) = A cos tp0. Solving these equations together, we
obtain A = 2,_ cp0 = n/3. (b) 0 = A sin (ca + cp0), consequently,
ca = —9o, 1^3/2 = A sin ca, —3/2 - A sin2ca - 2A sin ca cos ca,
—3/2 = ]/3 cos ca, cos ca = —|/3 /2 , ca = 5n/6, A = y 3 . 17.25. (a) x =
0.09 sin 40nt, (b) x = 5 sin n (t + 1/4), (c) x = sin n (t/3 — 1/4).
17.26. 4.17 X 10-2 s, 8.33 X 10-2 s, 8 m. H i n t . The displacement
equal to half the amplitude corresponds to the phase n/6, or tt = 7712.
Knowing the period, we can determine the time t, = 0.0417 s. The
displacement equal to half the amplitude (from the point of maximum
deviation) corresponds to the change in phase by n/3, or t« = 776.
Hence ft = 0.0833 s. During 10 s, the body completes 20 vibrations,
and hence * = n x 4 A = 20 X 0.4 m = 8 m. 17.27. The force of
gravity does. The equilibrium position is. The force is at a maximum
in the extreme positions and at a minimum in the equilibrium position.
17.28. v = A ca cos (cat + cp0). The motion of the ball is not uniformly
342 Answers

variable since the restoring force varies all the time. 17.29. Yes, they
will. 17.30. The aluminium ball will come to a halt Arst. 17.31. The
acceleration for the amplitude value of the displacement is maximal
and in the equilibrium position is minimal. The velocity has the
maximum value in the equilibrium position. 17.32. F = m g sin a =
6.93 X 10-2 N. 17.33. 204 g. 17.34. 3 0 \ 17.35. The period of oscilla­
tions of a pendulum depends on the free fall acceleration which is
different at different latitudes. 17.3G. The length of the pendulum
becomes smaller with decreasing temperature and hence tne period
changes (the clock will be fast). The correct pace can be restored by
changing the length of the pendulum by displacing its load. 17.37.
The period will double. 17.38. The length of the pendulum should be
reduced by a factor of 6.05. H i n t . By nypothesis, the periods of the
pendulum on the Earth and on the Moon must be the same. Expressing
the periods by the formula T = 2 n y l / g for the conditions on the

Moon and on the Earth, we obtain (fEarth^Moon) (?Moon/?Earth)= 1.


^Earlh'^Moon = £Harth/(fMoon = 6.05, fMnon = /Earth/6.05. 17.39.
2.004 s, 2.003 s, 2.008 s, 4.92 s. 17.40. 9.82 m/s2. 17.41. (a) The clock
will keep correct time, (b) In the freely falling lift, weightlessness sets
in, and the restoring force vanishes. If the pendulum is in an extreme
position, no oscillation will take place, while in the equilibrium po­
sition the pendulum will uniformly rotate in the vertical plane. 17.42.
(a) The force must be directed upwards and equal to 0.75m g \ (b) the
force must be directed downwards and equal to 0.5625mg. H i n t . If
the period increases under the action of force F , the acceleration im-
parted by this force has the minus sign. T 0 = 2 n Y V g , 27'0 =
2« V V ( g — 8 i ) , 1/2 = y ( g — g j ) / g , g = 4 ( g — g j , gj = 0.75g.
17.43. G = m g , it is directed upwards. 17.44. (a) 0.5 m g , a = 0.464 rad
or a = 26°30\ (b) 1.2mg, a = 0.876 rad or a = 50°12'. S o l u t i o n .
The resultant force acting on the ball can be found from the right-
angled triangle (Fig. 168): F t = ma, ]/ m 2 g 2 + F 2 , whence a =
V g 2 + F2//™2. Solving this equation together with the two equations
for the periods of oscillations T 0 = 2 n Y l / g and k T 0 = 2 n Y U g \ <
where k = T / T p , we obtain F = m g j/l/(*« — 1). The equilibrium
Answers 343

position corresponds to tan a = F / ( m g ) . 17.45. T = 0.997'0 (see


solution of Problem 17.44). 17.46. 1.9 s (see solution to Problem 17.44).
17.47. 0.9 Hz (see solution to Problem 17.44). 17.48. Yes, they will
be harmonic; the force of gravity changes the position of equilibrium.
17.49. 1.13 Hz. 17.50. 126 N/m. 17.51. 0.03 m, 31.3 rad/s, 0.2 s,
—n/2 rad, x = 0.03 sin (31.31 — n/2). 17.52. 8 x 10~3 J. 0.14 m/s,
1 m/s2. 17.53. 6.32 J, the energy does
not depend on the initial phase. 17.54.
0.192 J, 0.6 m/s, 0.1795 J, 0.0125 J,
x = 1.96 X 10~2sin (31.61 — 0.5 ji).
H i n t . When the engine is switched off,
the spring is stretched to the amplitude ayg\
of the displacement. In this case,
m - 2 g = k A . 17.55. x = 3 cos 0.2 x
sin(nl —0.2ji), 3 cos0.2 m, 0.5 Hz, 2 s,
—0.2jt rad/s, ji rad/s. H i n t . By using
the formula for the sine of double
angle sin 2 a = 2 sin a cos a, the equa­
tion can be transformed to the conven­
tional form. 17.56. The pendulum clock
is 47.6 s slow. S o l u t i o n . The motion e,-9\
of the rocket can be divided into I a,
three parts (Fig. 169). On the first seg­ t,
ment, the acceleration is directed 7 7 7 7 7 7 7 7 7 7 ^7 7 7 7 7 7 7 7 7 ^7 7 7 "
upwards, and a, = g ; on the second
segment, which lasts until the engine Fig. 169
is switched off, the rocket falls free­
ly, and on the third segment of dece­
leration, the acceleration is directed upwards. Let us first determine
12 and 13. In order to find 12, we shall use the formula for a uniformly
accelerated motion: h = h 0 + v 0 t -f gl2/2. In the case under consid­
eration, h = H 2 , /»„ = //,, and i;0 = i;,. Taking into account the
sign of the acceleration and the quantities v, = gt, and H 1 = g t \ l 2,
we can write the equation H 2 = g t \ l 2 + gt,t2 — g t \ l 2, whence t 2 =
+ ]/2 ( t f — H 2 / g ) = 70 s. Let us now consider the third segment
of the path. Here v 2 = y, — g t 2 = g ( t , — t 2 ) = —392 m/s, a 3 =
— v \ I ( 2 H 2 ) = —8g , t 3 = v 2 / a 3 = 5 s. On the first segment, the period

of the pendulum clock is T = 2 j i l / 2 g = T J |/ 2. The clock is

A«i = ( |/ 2 — 1) = 12.4 s fast. On the second segment, the pendu­


lum clock has zero speed, and on the third segment it is fast: T =
T o/3, Atj = t 3 (3 — 1) = 10 s. In total, A t = At, + At 2 + A<3 =
12.4 s — 70 s + 10 s = —47.6 s. 17.57. See Fig. 170. 17.58. 2 m,
3 m, 5 m (Fig. 171). 17.59. x = 0, x = —sin wt, 0, —1 m (Fig. 172).
17.60. (a) 2 |/2 m , (b) 5.4 m, (c) 5.4 m (Fig. 173). 17.61. <p=arctan 0.4=
0.38 rad = 21°48' (Fig. 173c). 17.62. 3.6 m, 30°27' (Fig. 174).
17.63. The particle vibrates about its equilibrium position. 17.64.
See Fig. 175. 17.65. (a) To the right, (b) to the left. 17.66. The pro­
pagation velocity depends on surface tension. 17.67. 0. 17.68.
16.7 m/s. 17.69. The velocity of sound is higher in iron; no, it cannot.
17.70._ The sound emerges as a result of friction between metal sur­
faces in the hinge. Having a larger surface, the door augments the
344 Answers

Fig. 174 Fig. 175


Answers 345

vibrations. 17.71. Elastic forces. 17.72. The velocity of propagation


of sound depends on temperature and pressure. 17.73. An acoustic
wave propagating in the rail is partially reflected from the surface
of the rail inwards, and hence the intensity of the sound does not
attenuate as rapidly as in the spherical wave propagating in air.
17.74. The sound will not be detected. 17.75. The vibrations of the
tuning fork are transmitted through the surface of the table which
is much larger than the surface of the tuning fork. 17.76. The sound
from the tuning fork in contact with the table will die away sooner
since the energy of the tuning fork is spent on the vibration of the
table. 17.77. 22.7 cm-11.3 cm. 17.78. 0.66 m. 17.79. 0.5n rad. 17.80.
The air column in the cap will start vibrating. 17.81. 1545 Hz. 17.82.
6 cm. 17.83. The frequency of the sound will increase. 17.84. 1445 m/s,
3638 m/s. 17.85. The values of v and T will not change, while k will
increase by a factor of 14.7. 17.86. 1200 m. 17.87. 2.04 mm, 5.1 mm.
17.88. 3 km. 17.89. 13.2 s. 17.90. 2 X 10-s %, 6 cm.
18.1. 0.02 s, 100 times. 18.2. 4.24 A. 18.3. No, it cannot. 18.4. 177 V,
314 rad/s. 18.5. 63.8 A, (314/ + n/4) rad, n/4 rad, 50 Hz. 18.6. 0,
126.6 V, 179 V, 89.5 V. —155 V, —179 V, —126.6 V, 0. 18.7. —3 A,
1.5 A. 18.8. (a) If the plane of the frame is perpendicular to the mag­
netic field lines, (b) if tne plane of the frame is parallel to the magnetic
field lines. 18.9. 5 X 10~4 V. 18.10. The induced emf will not change.
18.11. (a) The induced emf decreases by a factor of 1.3125; (b) increases
by a factor of 1.19; (c) increases by a factor of 1.143. 18.12. 120.
18.13. 0.046 T. H i n t . The beginning of oscillations of the emf corre­
sponds to the position of the frame in which its plane is perpendicular
to the magnetic field lines. 18.14. 0 . 15ji V, 20n rad/s, 0.1 s, —n/4 rad,
e = 0.15n sin it (20/ —0.25). 18.15. The impedance of the conductor
increases sixfold; the impedance is equal to the resistance of the con­
ductor. 18.16. 4 R, 3 R. 18.17. 152.6 R. 18.18. The resistance will be
infinitely large. 18.19. 31.8 R, 0.53 R. 18.20. 531 R. 18.21. 18.5 R.
18.22. 28.2 A. 18.23. 50 (iF. 18.24. 127 V, 179 V. 18.25. U r . = 16 V,
U L = 96 V, 200 Hz. H i n t . The answer is obtained by solving the two
equations: X L — X c = U / I j and X L / 2 — 2 X C = t / // 2. 18.26. See
Fig. 176. 18.27. (a) See Figs. 177a and 6, (b) see Figs. 177c and d .
18.28. (a) See Fig. 178a, (b) see Fig. 178h . 18.29. (a) See Figs. 179a
and b , (b) see Fig. 179c. 18.30. (a) See Fig. 180a, (b) see Fig. 1806.
18.31. (a) See Fig. 181a, (b) see Figs. 1816 and c. 18.32. See Fig. 180a
to Problem 18.30. 18.33. See Fig. 1806 to Problem 18.30. 18.34. (a)
100 R, 1.2 A, 72 V, 96 V, see Fig. 182a, (b) 4 R, 30 A, 114 V, 42 V,
see Fig. 1826, (c) 16 R, 7.5 A, 180 V, 60 V, see Fig. 182c. 18.35. (a) 5 R,
44 A, 220 V, 1866 V, 1866 V, see Fig. 183a, (b) 42.4 R, 5.2 A, 156 V,
327 V, 171 V, see Fig. 1836. 18.36. (a) 12 A, 9 A, 15 A. 0.8, see
Fig. 184a, (b) 36'A, 18 A, 40.25 A, 0.894, see Fig. 1846, (c) 18 A, 9 A,
20 A, 0, see Fig. 1,84c. 18.37. (a) 6 A, 12 A, 12 A, 6 A, 1, see Fig. 185a,
(b) 4 A, 3.6 A, 7.5 A, 6 A, 0.67, sec Fig. 1856, (c) 9 A, 18 A, 6 A, 15 A,
0.6, see Fig. 185c. 18.38. The inductance should be increased by
0.184 H; 120 V/ H i n t . From Ihe condition — ^
V R2 + o>2/.s 2
,, . L 1
we obtain —= —- The equality of the phase differences leads to
23-0530
346 Answers

Fig. 177

Fig. 178
Answers 347

Fig. 182
348 Answers

the equation — ' = Hence


]/(/? + A/?)2 + (o2 ( L + AL)2 V R 2 + w2/,2.

AL = AR ~ = • 18-39- 17 3 Q> 0 04 H- 18-40- UR =


32 V, U L = 48 V, U c = 32 V, l/tot = 36 V, 26J34', see Fig. 186.
18.41. 1594 }iF. 18.42. Hydraulic turbines cannot ensure the required

Fig. 185

speed of rotation of the generator’s rotor. In order to obtain the stan­


dard frequency (50 Hz), multipole generators are used. An increase
in the number of pole pairs is equivalent to an increase in the speed
of rotation of the rotor. 18.43. 20. H i n t . The frequency v of altem at-
Answers 349

ing current and the speed of rotation n of the rotor are connected
through the relation v = p n , where p is the number of pole pairs.
18.44. 100 Hz. 18.45. No, it will not. 18.46. The transformer consu­
mes an insignificant amount of energy.
18.47. The huge current passing through „
a short-circuited turn leads to heating of 1*
the transformer and may damage it.
18.48. 2. 18.49. 10. 18.50. 696. 18.51.
16. 18.52. 0, 28.2 var, 28.2 VA. 18.53.
72 W, 96 var, 120 VA, 0.6, 20 V. 18.54.
108 W, 144 var, R = 27 ft, X L = 36 ft,
X c = 72 ft. 18.55. 0.995, 380 W, 36
var. 18.56. 270 VA, 200 W, 182 var,
0.74. H i n t . For the parallel connection,
the power factor can be determined by
two methods: cos <p = P / S or cos q) =
l / R _____

y \ i f t t _u i
i x q which follows from the
vector diagram for currents. 18.57. ..
0.019 H, 240 W, 602 var. 18.58. 60 ft, u c J
41.8 f t , 240 W, 344.7 var, 420 VA, 0.57. Pi„ , a A
18.59. (a) 1, 160 W, 0; (b) 0.6, 120 W, rig ' 180
160 var. 18.60. 0.97, 8067 W, 2020 var.
18.61. X = 12.5 ft, R = 7.2 ft, 0.865, 400 VA, 6.25 ft.
19.1. The frequency increases. 19.2. Yes, it will; the period will in­
crease. 19.3. 500 m. 19.4. 7.94 X 10* Hz, 1.26 X 10*6*s. 19.5. The
reflection of electromagnetic waves takes place. 19.6. Using ultra-
short waves penetrating through the ionosphere. 19.7. 53 and 2640 m.
19.8. About 200 m, 1.6 MHz. 19.9. 70 pF. 19.10. 45 km. 19.11. The
ower is proportional to the fourth power of frequency. 19.12. C|.
?9.13. The signal is amplified at the expense of the source of electric
energy. 19.14 . 2.25 X 109 m/s, 0.13 m. 19.15. 7.5 mH. 19.16. 0.5 MHz,
0.1 H. 19.17. 1.6 mm. 19.18. 10-s s, 300 m. 19.19. 4.65 A. 19.20.
4 X lO*34* s, 4 p,F, E m a g = £ el = 2 X 10-‘ J. 19.21. L =
= 2.3 mH.
4ji*ACvJv:|

20.1. By 40°. 20.2. By 30°. 20.3. The mirror should be placed on the
path of the rays at an angle of 78 or 12° to the horizontal. 20.4. (a) 68°,
22°; (b)22°, 68°; see Fig. 187. 20.5. The mirror should be placed at
an angle of 45° to the horizontal with the mirror surface facing up­
wards. 20.6. 160 cm. 20.7. In the vertical plane. 20.8. The image of
the lamp will be virtual and symmetric about the mirror, see Fig. 188.
20.9. 11.5 cm, see Fig. 189. 20.10. The mirrors should be placed at
right angles to the lateral sides of the triangle S , S S t at equal dis­
tances from the vertices, see Fig. 190. 20.11. (a) 3, (b) 5. 20.12. As
a result of multiple reflections hy the mirrors, a large number of images
will be formed. 20.13. By making the distance to the building large.
20.14. The rays emitted by the Sun can be treated as parallel, there­
fore, the point of their convergence will be the focus. 20.15. 39 cm,
350 Answers

Fig. 189

Fig. 192
Answers 351

19.5 cm. 20.16. See Fig. 191. 20.17. 1.2 m. 20.18. 10.5 cm, 21 cm.
20.19. 20 cm from the mirror. 20.20. 20 cm. 20.21. 24 cm, —1.7.
20.22. 60 cm, 40 cm. 20.23. 0.2 m from the mirror. 20.24. 14 cm.
20.25. The field of vision can be increased by using convex mirrors.

Fig. 195 Fig. 196

Fig. 197

20.26. 3 m. 20.27. 1.5 m. 20.28. —0.2 m, the image is virtual and


reduced. 20.29. 0.24 m, see Fig. 192. 20.30. 86 cm. 20.31. 1.8. 20.32.
35°. 20.33. 43°. 20.34. (a) About 8°, (b) 12°20'. 20.35. 50°. 20.36. The
image of the fish in water is virtual and displaced. 20.37. 1.41, 54°40'.
20.38. (1) For n, = n 2 , (2) when e = 0. 20.39. About 38°, 225000 km/s.
20.40. 2.3 X 105 km/s. 20.41. 2 X 10s km/s, 1.5. 20.42. 35°. 20.43.
57°. 20.44. See Fig. 193. 20.45. About 24°. 20.46. 1.5. 20.47. 33°20'.
20.48. Air bubbles on the surface of the blackened ball create the
352 Answer9

conditions of the total reflection at the water-air interface. 20.49.


The effect is explained by the total reflection. 20.50. See Fig. 194.
20.51. 26 cm. 20.52. 0.75 m, see Fig. 195a. 20.53. 82.5 cm, see Fig. 1956.
20.54. 4.5 cm. 20.55. 0.4 cm. 20.56. M N = 6.6 cm, see Fig. 196.
20.57. All points are displaced similarly; the displacement is notice-

>h !

! «
r

Fig. 201

able if the thickness of the glass is nonuniform. 20.58. 2.65 cm, 5 cm.
20.59. See Fig. 197. 20.60. See Fig. 198. 20.61. See Fig. 199. 20.62.
34°. 20.63. 17°. 20.64. 35°30'. 20.65. 38°. 20.66. n o s i n '4 —:

sin — . 20.67. 8 D, 2 D. 20.68. —4 D, —2.5 D. 20.69. 25 cm, —20 cm,


—50 cm. 20.70. It will be diverging if the transparent medium in
which the lens is placed is optically denser than the material of the
lens. 20.71. See Fig. 200. 20.72. The brightness of the image will be
reduced, see Fig. 201. 20.73. See Fig. 202. 20.74. They should be
arranged so that the foci of the lenses coincide. 20.75. 30 cm from the
lens, the image will be real and reduced by a factor of two. 20.76.
—10 cm, 30 cm, 20 cm, and about 17 cm. 20.77. (1) The height of the
image decreases, (2) the image is real, inverted, and full-sized, (3) the
object should be between the focus and the lens. 20.78. 10 cm, 10 D.
20.79. See Fig. 203. In order to determine the optical centre O , we
Answers 353

draw a ray from point A to A '. It will intersect the optical axis at
the optical centre. Then we draw another ray from point A parallel
to the optical axis, which after the refraction by the lens will pass
through the fo c u s/1' and will arrive at point A '. In this problem,
the converging lens produces a real image. 20.80. 24 cm. 20.81. About

3.1 D, 4. 20.82. 0.3 m. 20.83. 4 cm. 20.84. 1.75 m from the lens, the
image is magnified X2.5. 20.85. —2 m, the image is virtual, erect,
and magnified x 5 . 20.86. 24 cm. 20.87. —12 cm, 3.6 cm. 20.88.
16 cm. 20.89. Yes, it will, / = oo. 20.90. 2.5. 20.91. —7.5 cm, about
—13 D; see Fig. 204 . 20.92. 0.5 . 20.93. 4 . 20.94. 0.15 m, about 7 D.
20.95. 10 D, 5 D. H i n t . The optical power <I> of the system is the sum
of the optical powers of the lenses tnat are in contact and form the
354 Answers

system. 20.96. About 6.7 D, ^concave — d~(T~— ~d) ^convex-


20.97. 30. 20.98. 5 D. 20.99. 2.63 m. 20.100. 4 D. 20.101. For improv­
ing the sharpness of the picture. 20.102. 19 m. 20.103. About 8.9 D.
20.104. 7.25. 20.105. 2.5 cm, 16 cm.
21.1. 2.51 X 103 lm. 21.2. 2.4 cd. 21.3. 3.167 lx, 188.5 lm. 21.4.
8 X 10-3 lm. 21.5. 105 lm, 10® lm. 21.6. 39 lx. 21.7. 25 lx. 21.8. 2.
21.9. E 1 = 13.3 lx, E 2 = 17.4 lx, in the latter case the illuminance
is higher. 21.10. At about 41°. 21.11. The illuminance of the vertical
wall is 6.33 times higher. 21.12. 224 X 10® km. 21.13. Solar rays are
incident on the slopes at a smaller angle to their surface, consequently,
the same surface area absorbs a higher energy per unit time. 21.14.
35 lx. 21.15. 67 lx, about 38 lx. 21.16. 32.3 lx. 21.17. 200 cd. 21.18.
1 m, 0.71 m. 21.19. The illuminance under each bulb is the same and
equal to 60 lx. 21.20. 55 lx. 21.21. About 700 cd. 21.22. 19 lx. 21.23.
Over the area smaller than 942 m2. 21.24. About 31 lx. 21.25. 2 m.
21.26. About 9 and 6.7 m. 21.27. 225 cd. 21.28. 0.8 m from the lamp
with a lower luminous intensity. 21.29. Yes, it decreases by a factor
of n . 21.30. 123 lx. H i n t . The illuminance on the screen is determined
as the sum of the illuminances produced by the lamp and its image
formed by the plane mirror. 21.31. 2.6 s. H i n t . In order to obtain
photographs of the same quality, the following condition must be
fulfilled during printing: the same amount of energy must be supplied
to the photographic paper: 1 Y l = W 2 . Since W = O t = E S t , we
obtain E jStj = E 2 S t 2 . Using this equality and having determined
the illuminances, we calculate the time t 2 .

22.1. A bright spot (enhancement of light) is obtained when the optical


path difference of the waves is equal to an even number of half-waves.
A dark spot (attenuation of light) is obtained when the path difference
of the waves is equal to an odd number of half-waves. 22.2. Construc­
tive interference will occur at point 1 and destructive interference at
point 2 . 22.3. The effect is explained by interference. 22.4. The phe­
nomenon appears due to interference of light. Yes, it will; alternating
dark and bright fringes are formed, whose colour is determined by the
colour of rays incident on the surface of water. 22.5. The interference
will be constructive (bright fringe). 22.6. 1.52 X 10~® m, red. 22.7.
When, the soap film is illuminated by white light, interference occurs:
in various regions, depending on the film thickness, path difference
is created such that it causes enhancement of some wavelengths and
attenuation of others. 22.8. 3.2 mm. 22.9. 652 nm. 22.10. The radii
will decrease. 22.11. It is based on interference: the film coating the
objective is chosen so that the waves in the medium part of the spec­
trum are mainly attenuated. Red and violet rays are attenuated in­
significantly and produce a bluish-violet tinge. 22.12. X = d h /L =
750 nm, where d is the separation between the light sources, h is the
distance between two adjacent interference fringes and L is the dis­
tance from the screen to the line connecting the light’s sources. 22.13.
4.6 mm. 22.14. About 3.6 m. 22.15. This is explained by diffraction
of light. Eyelashes play the role of a diffraction grating. 22.16. Unlike
the spectrum obtained with a prism, a diffraction spectrum is uniform-
Answers 355

ly extended in all regions. Besides, spectra of several orders are obtain­


ed to the right and to the left of the central bright line. 22.17. 0.4 pm.
H i n t . For small angles, sines can be replaced by tangents; k k = d sin <p.
22.18. 018 nm, 484 nm. 22.19. 0.76 pm. 22.20. 2 X 10'6 m, 5 X
103 cm -1. 22.21. 4.34 cm. 22.22. 500. 22.23. 4. H i n t , k k = d sin q>,
k = d sin ip/X, the spectrum order will be maximum for sin tp = 1.

23.1. The velocity of light in water is higher. 23.2. 1.33, 1U'3 s.


23.3. 5 X 1014 Hz. 23.4. 759 nm, 0.4 pm. 23.5. About 1.23 X
108 m/s. 23.6. 380 nm, 6 X 1014 Hz. 23.7. Green, the colour of the
light is determined by the frequency of oscillations which does not
change as light propagates from one medium to another. 23.8. The
wavelength will increase by 19 nm. 23.9. 1.85 X 108 m/s, 1.8 X
108 m/s, 3.75 X 10u and 7.5 X 1014 Hz, 494 and 240 nm. 23.10.
About 605 nm and 485 nm. 23.11. Ice. 23.12. The word would not be
seen through a red medium since it does not transmit green light.
23.13. The ultraviolet radiation with a wavelength of about 290 nm
corresponds to the maximum energy. 23.14. Eyes begin to react to
complementary colours due to eye strain. 23.15. By investigating
solar spectrumby absorption lines (spectral analysis). 23.16. 6000 K.
H i n t . For determining the radiant exitance of the Sun, use the tabu­
lated value of the radius, e = E / ( 4 n R 2 ) . The temperature of the Sun's
surface is determined from Stefan-Boltzmann’s law e = g T 4. 23.17.
The nebula consists of stars. If it contained a gaseous substance, it
would have a line spectrum. 23.18. 4.4 X 10s kg, 4.4 X 1024 kg.
23.19. 3683 K. H i n t . Make use of Wien’s law.
24.1. About 1.67 Pa. 24.2. 141 kJ. 24.3. 3.86 X 10’7 Pa. 24.4. 3.75 X
10-i9 J, 2.34 eV. 24.5. 2000. 24.6. 2 X 10'18 J, 12.5 eV. 24.7. 1011.
24.8.5.52 X 10-*» J, 0.6 X 10-“ kg, 1.84 X 10~27 kg-m/s. 24.9.
5 X 10-18 J, 5.5 X 10-32 kg, 1.66 X 10-23 kg-m/s; 4 X 10'18 J,
4.4 X 10*“ kg, 1.33 X 10"26 kg-m/s. 24.10. Approximately to the
east. 24.11. 4.9 X 10-18 J. 24.12. 653 nm. 24.13. 273 nm. 24.14.
Yes, it will since X < Xth- 24.15. Silver will not be charged since
k > Xth . 24.16. 1.44 X 10-i» J. 24.17. About 3.93 X 10-*7 J. 24.18.
2.4 X 10* m/s. 24.19. 2.9 X 10-19 J, 1.3 X 10« m/s. 24.20. 2.85 X
108 m/s. H i n t . The electron velocity obtained indicates that the re­
lativistic formula has to be used. Since the electron work function
for molybdenum is negligible in comparison with the energy of a
photon, it can be neglected in calculations, and then the formula
—0 becomes h v = E 0 ( ——? —1 ) .
h \ = A - \ - E 0
- ] / l —v 2 / e ‘ / \ y i _ „2/c2 / ’
where £ 0=0.51 MeV is the rest energy of the electron. 24.21. 2.84 x
h e m v \ h e , m vl h e m o \
10-19 J. S o l u t i o n . ■= A - \
~ 2 » X. 2 J ~ 2 ~
lie m v h e m v \ h e m v I 1 1 )
A ,
A' I T — 2 2 X, 2 ■» hc ( Xs )
Smuj m v $ h e
1 \ A
h e h e i' 1 1
X, ) ’ A ^2 ~ \k ^1 xa 1

4- - r - h H r j -
356 Answers

25.1. AZ = 0.87 in — 0.66 m = 0.21 m. 25.2. v = 0.6c. 25.3. 2.61 X


10s m/s; no, it will not. 25.4. In the form of a square. 25.5. 1.2 X
10~13 cm, by a factor of 1.67. 25.6. About 3.57 years. 25.7. 10 years.
25.8. About 40.2 X 1012 km; about 2.68 X 105 AU; about 1.2 years.
25.9. 44.4 years for the observer and 19.4 years for the cosmonaut.
25.10. 3.73 X 10-13 kg. 25.11. 0.866c. 25.12. 0.968c. 25.13. 1.05 X
10-30 kg, 2.09 X 10-30 kg. 25.14. 9.2 X 107 m/s. 25.15. 8.33 kg,
2.2 X 104 kg/m3. 25.16. The mass and density will not change for the
observer in the rocket. 25.17. 0.89c. 25.18. 0.51 MeV, 1.02 MeV.
25.19. 2.05 X 10-22 kg-m/s. 25.20. 900 m/s. 25.21. 1.25c, which con­
tradicts the postulate of the theory of relativity on the impossibility
to exceed the velocity of light; ~0.9c.

26.1. It is the flow of doubly ionized helium atoms. Th;ir mass is


almost 8000 times as large as the electron mass. 26.2. The Lorentz
force does. 26.3. y-radiation. 26.4. y-radiation is electromag­
netic waves following X-rays in the electromagnetic spectrum and
differing from them in a higher frequency (of the order of 1020 Hz)
and hence in a higher energy. 26.5. 6. 26.6. An electron is born as
a result of conversion of a neutron into a proton. 26.7. About 4 days,
2 X 10~° s-1. 26.8. Tbi = 5 days, r p 0 = 138 days. 26.9. About
1.2%. 26.10. 7.9 h, 1.3 X 10'5 s '1. 26.11. 4.5 X 109 years, about
5 X 10'le S '1. 26.12. The atom of JH contains one proton and two
neutrons, helium atom has two protons and two neutrons, aluminium
atom has 13 protons and 14 neutrons, uranium atom has 92 protons
and 146 neutrons, neptunium atom has 93 protons and 144 neutrons.
The number of neutrons is increasing. 26.13. They differ in the number
of neutrons, tabulated chlorine consists of 75% chlorine with the mass
number of 35 and 25% chlorine with the mass number 37. 26.14. The
nucleus of silicon atom; the nuclei of helium, carbon, nitrogen, oxy­
gen, neon, magnesium, sulphur, and calcium. 26.15. 4.8 X 10'19 C,
4.64 X 10-18 C, 1.47 X 10'17 C. 26.16. 1.2 X 10'8 C. 26.17. flHe =
2.2 X 10'15 m, 7?u = 8.7 X 10'15 m, pHe = 1.44 X 1017 kg/m3,
Pu = 1.4 X 1017 kg/m3. 26.18. 227Ac -► 2« F r + 2He, a-decay. 26.19.
It will be converted into neptunium: 2j}JU -►2JJNp + _°,e. The mass
number does not change. Tne nucleus will be shifted to the right.
26.20. A neutron. JLi + 2H -►8Be + *n. 26.21. Z = 6, A = 14,
carbon isotope: ‘?N - f i n -*■ *JC + \ p . 26.22. It is an iodine atom ‘jrjl.
26.23. 1.02 MeV, 2.5 X 1020 Hz. 26.24. <$N + <?N + 2Jre -► ‘?N +
*§N + y. 26.25. The energy must be higher than the sum of the rest
energies of an electron and a positron, i.e. larger than 1.02 MeV.
26.26. ” A1 -f- gHe JJSi + Jp , the nucleus of silicon isotope. 26.27.
Identical tracks are left by identical particles, viz. nuclei of helium
atoms formed in the reaction ‘JB + \ p 3£He. 26.28. The missing
particle is an a-particle. 26.29. Curium 2««Cm, neutron. 26.30. A pro­
ton is converted into a neutron. 26.31. 0.12 amu or 1.99 X 10'28 kg.
26.32. 0.07 amu, about 65.2 MeV. 26.33. 1.97 amu or 3.27 X 10'27 kg,
1833 MeV or about 2.9 X 10'10 J. 26.34. (1) Energy is absorbed, (2)
liberated, (3) liberated. 26.35. 4.0 MeV or 6.4 X 10-13 J. 26.36.
770 kg. 26.37. 1.67 t. 26.38. In all cases, the energy is approximately
equal to 2 X 1013 J. 26.39. About 64 g. 26.40. 4.8 X 1012 J. 26.41.
In fast neutron reactors, natural- uranium is utilized with a higher
Answers 357

efficiency, and besides the nuclear fuel (plutonium) is reproduced.


26.42. Gamma-quantum, electron, neutrino, and proton. 26.43.
3 X 10-11 J. 26.44. About 1.13 X 10lu J. 26.45. About 3.2 X 1019 J.
27.1. 8 min 19 s. 27.2. 4.02 X 1013 km, 1.31 pc. 27.3. 5.54 h. 27.4.
6.2 X 105 pc. 27.5. Sirius, Canopus. 27.6. Canis Major; this constel­
lation is in the southern part of the celestial sphere. 27.7. 8.9 years.
27.8. 3.04 X 101 pc. 27.9. Vega; Lyra. 27.10. The north and south points
on the celestial meridian and points of east and west on the celestial
equator. 27.11. Serpens constellation. 27.12. The list of constellations
may change depending on geographical latitude. 27.13. See the answer
to Problem 27.12. 27.14. The Sun rises exactly at the east and sets
exactly at the west only during the vernal and autumnal equinoxes.
27.15. Aquarius, Capricorn, Sagittarius, Scorpius, and Libra. 27.16.
At the poles. 27.17. About 55°. 27.19.30°. 27.21. By 1°. 27.22.
0, 0. 27.23. 0, 12 h. 27.24. 57°42'. 27.25. 1.9°. 27.26. 1.392 X 106 km.
27.27. 5.71 X 107 km. 27.28. 8.2 pc. 27.29. 84.01 'years. 27.30.
9.539 AU. 27.31. 5.203 AU, 778 X 10s km. 27.32. In the form of a
narrow crescent. 27.33. This is valid for any latitude. 27.34. In the
first quarter. 27.35 . 7.5 X 104 years. 27.36. 0.545". 27.37. The incli­
nation of the Venus axis to the plane of its orbit is close to 90°. 27.38.
1400 kg/in3. 27.39. 7.7 X 103 m/s.
TO THE READER

Mir Publishers would be grateful for your comments


on the content, translation and design of this book.
We would also be pleased to receive any other sug­
gestions you may wish to make.
Our address is:
Mir Publishers
2 Pervy Rizhsky Pereulok
1-110, GSP, Moscow, 129820
USSR
Also from MIR Publishers

H IG H E R M A T H F O R B E G IN N E R S
(MOSTLY PHYSICISTS AND ENGINEERS)

Ya.B. Zeldovich, Mem. USSR Acad. Sc., and


I.M. Yaglom, D.Sc. (Phys.-Math.)
This took has been written as an introduction into higher mathe­
matics. Aside from such traditional topics as analytic geometry and
differential and integral calculus, the book introduces theLnotions of
power and trigonometric series, studies simple differential Equations,
and discusses some special topics in physics.
It is intended for high-school students, freshmen at universities
and technical colleges, and anyone wishing to brush up on, the
elements of higher mathematics.
A C O L L E C T IO N O F Q U E S T IO N S
A N D P R O B L E M S ;IN P H Y S I C S

L.A. Sena
The C o l l e c t i o n contains more than 400 questions and problems
covering all the sections of the physics course. All questions and
problems have detailed answers and solutions. For this reason the
two main sections of the book, Questions and Problems and Ans­
wers and Solutions, have identical headings and numbering: each
chapter in the first section has a corresponding chapter in the
second, and the numbering of answers corresponds to the number-1
ing of problems.
A special feature of the C o l l e c t i o n is the drawings and diagrams
for most of the questions and answers. The diagrams use a variety
of scales: linear, semilog, log-log, and quadratic.
Arrangement of the material in this C o l l e c t i o n corresponds to
the structure most commonly used in college physics textbooks. One
exception is the questions and problems involving the special
theory of relativity. These are placed in different chapters, starting
from the one dealing with mechanics.
The C o l l e c t i o n is intended for the self-instruction of students of
technical colleges.
O ve rlay C ircle f<

M pridian

Magnitude 1
>
ana smaller
• Magnitude 2
>r th e S tar C h a rt

Slar /▼» VernaL


dusters I equinox rj--«.n Constellation
= Nebulae A u tu m n a l ‘" 1— 1 bo und aries
------ equinox

You might also like